11

Réussis tes devoirs et examens dès maintenant avec Quizwiz!

Calcium Channel Blocker

"CA" Calan, Procardia, Cardizem Slow the HR and decrease BP (check HR and BP before). Inhibits mvmt of Calcium ions across membrane or cardiac and arterial muscle cells. Results in slowed impulse conduction, depression of myocardial constractility, dilation of coroanry arteries. SE: Constipation Procardia sometimes causes peripheral edema

Antiparkinson Agents Med Names

(-dopa) Artane, Cogentin, L-Dopa, Parlodel, Sinemet

for child w/ HIV who have a positive Mantoux test result, what is the minimum months of treatment?

12 months

black box warning

2004 FDA issued this. To be included at beginning of drug package inserts and drug information sheets. These warnings apply especially to children, who are at greater risk for suicidal ideation.

Antipsychotic dosage for older adults should be

25%-50% less than that of a normal adult dose

In the event that JA experiences a seizure, you would document all of the following EXCEPT the

Ability to stop the movements

Andrenergics Uses

COPD, Cardiac Arrest

Seasonal Affective Disorder (SAD)

Enhanced release of melatonin due to lower light levels

Colistin S is excreted via which of the following

Feces

Iron Preparations Med Names

Feosol (ferrous sulfate) Imferon

Seroquel

Indication: schizophrenia, depressive episodes Class: antipsychotic Labs: anemia labs, WBC, hepatic, blood glucose, cholesterol Side Effects: seizure, dizziness, weight gain, palpitations, hyperglycemia Precautions: mental status monitor, monitor BMI/weight, monitor BP, make sure med is taken

What is an obsession? EX?

It is a thought. EX: "I'm so bad, I made a mistake", feeling like someone close to you might die, or you might be harmed.

Pt takes drug highly protein bound, several days pass pt takes 2nd drug that is 90% protien bound, what happens to first drug?

More of the first drug is released from the protein and becomes more pharmacologically active.

Narcotics Med Names

Morphione, Codeine, Demerol, Dilaudid, Precodan (oxycodone)

Which of the folllowing drugs is a somatostatin analogue frequently prescribed for metastatic cancer related to severe diarrhea

Octreotide-sandostatin

Antihyperlipidemic: Med Names

Questran, Lipitor

Antitubercular Agents Uses

TB, to prevent disease in person exposed to organism

Pyridium Nursing Considerations

Taken with meals Urinary tract analgesic

The FDA madates to monitor/control the manufacture and marketing of drugs come from (what act)

The food,drug,and cosmetic act 1938

the nurse should be alert to counterfeit prescription drugs, the following are clues to counterfeit

Variation in labels Differed appearance of drug Different dose remained in applicator/vial etc

Electrolytes Uses

fluid and electrolyte imbalances, osteoporosis

Antifungals: Action

impairs cell membrane

Epoetin alfa (epogen, procrit) given to treat anemia in renal failure can be given IV or

subcutaneously

Insulin: Fast Acting Onset

.5-1h

Insulin: Combination Onset

.5h

What is the normal lithium level?

0.6-1.2

phentoin (Dilantin) is hung with what IV solution?

0.9% sodium chloride (NS)

Moexipril, an ACE inhibitor, is given

1 hour before meals

Food can hamper absorption w/ Norfloxacin (Noroxin) given for UTI, when should it be given?

1 hour before meals OR 2 hours after meals

173.) A nurse reviews the medication history of a client admitted to the hospital and notes that the client is taking leflunomide (Arava). During data collection, the nurse asks which question to determine medication effectiveness? 1. "Do you have any joint pain?" 2. "Are you having any diarrhea?" 3. "Do you have frequent headaches?" 4. "Are you experiencing heartburn?"

1. "Do you have any joint pain?" Rationale: Leflunomide is an immunosuppressive agent and has an anti-inflammatory action. The medication provides symptomatic relief of rheumatoid arthritis. Diarrhea can occur as a side effect of the medication. The other options are unrelated to medication effectiveness.

115.) A client received 20 units of NPH insulin subcutaneously at 8:00 AM. The nurse should check the client for a potential hypoglycemic reaction at what time? 1. 5:00 PM 2. 10:00 AM 3. 11:00 AM 4. 11:00 PM

1. 5:00 PM Rationale: NPH is intermediate-acting insulin. Its onset of action is 1 to 2½ hours, it peaks in 4 to 12 hours, and its duration of action is 24 hours. Hypoglycemic reactions most likely occur during peak time.

1. The nursery nurse is putting erythromycin ointment in the newborn's eyes to prevent infection. She places it in the following area of the eye: a. under the eyelid b. on the cornea. c. in the lower conjunctival sac d. by the optic disc.

1. C. The ointment is placed in the lower conjunctival sac so it will not scratch the eye itself and will get well distributed.

105.) A nurse is collecting data from a client and the client's spouse reports that the client is taking donepezil hydrochloride (Aricept). Which disorder would the nurse suspect that this client may have based on the use of this medication? 1. Dementia 2. Schizophrenia 3. Seizure disorder 4. Obsessive-compulsive disorder

1. Dementia Rationale: Donepezil hydrochloride is a cholinergic agent used in the treatment of mild to moderate dementia of the Alzheimer type. It enhances cholinergic functions by increasing the concentration of acetylcholine. It slows the progression of Alzheimer's disease. Options 2, 3, and 4 are incorrect.

Insulin: Fast Acting Duration

6-8h

9. The physician orders an intramuscular injection of Demerol for the postoperativepatient's pain. When preparing to draw up the medication, the nurse is careful to remove the correct vial from the narcotics cabinet. It is labeled a. simethicone. b. albuterol. c. meperidine. d. ibuprofen.

9. C. The generic name for Demerol is meperidine.

25.) A home care nurse visits a client recently diagnosed with diabetes mellitus who is taking Humulin NPH insulin daily. The client asks the nurse how to store the unopened vials of insulin. The nurse tells the client to: 1. Freeze the insulin. 2. Refrigerate the insulin. 3. Store the insulin in a dark, dry place. 4. Keep the insulin at room temperature.

2. Refrigerate the insulin. Rationale: Insulin in unopened vials should be stored under refrigeration until needed. Vials should not be frozen. When stored unopened under refrigeration, insulin can be used up to the expiration date on the vial. Options 1, 3, and 4 are incorrect.

168.) Colcrys (colchicine) is prescribed for a client with a diagnosis of gout. The nurse reviews the client's medical history in the health record, knowing that the medication would be contraindicated in which disorder? 1. Myxedema 2. Renal failure 3. Hypothyroidism 4. Diabetes mellitus

2. Renal failure Rationale: Colchicine is contraindicated in clients with severe gastrointestinal, renal, hepatic or cardiac disorders, or with blood dyscrasias. Clients with impaired renal function may exhibit myopathy and neuropathy manifested as generalized weakness. This medication should be used with caution in clients with impaired hepatic function, older clients, and debilitated clients. **Note that options 1, 3, and 4 are all endocrine-related disorders: Myxedema=Hypothyroidism**

238.) Ribavirin (Virazole) is prescribed for the hospitalized child with respiratory syncytial virus (RSV). The nurse prepares to administer this medication via which of the following routes? 1. Orally 2. Via face mask 3. Intravenously 4. Intramuscularly

2. Via face mask Rationale: Ribavirin is an antiviral respiratory medication used mainly in hospitalized children with severe RSV and in high-risk children. Administration is via hood, face mask, or oxygen tent. The medication is most effective if administered within the first 3 days of the infection.

A patient has come into the emergency room with her friend. Her friend states that she had been acting very strangely and confused. The friend states that the patient has diabetes and takes insulin. The nurse knows that signs and symptoms of hypoglycemia include: 1. slow pulse rate and low blood pressure. 2. irritability, anxiety, confusion, and dizziness. 3. flushing, anger, and forgetfulness. 4. sleepiness, edema, and sluggishness.

2. irritability, anxiety, confusion, and dizziness When blood sugar levels fall, hormones are activated to increase serum glucose. One of the hormones is epinephrine, which causes these symptoms.

what gauge is used for heparin or enoxaparin subc?

25-27

pt suffering from Ecoli infection- unable to swallow pills - an oral suspension of cephalexin 250mg ordered Usual dose is? How often is medication generally scheduled for?

250 -500 mg Q6h

1) A nurse is caring for a client with hyperparathyroidism and notes that the client's serum calcium level is 13 mg/dL. Which medication should the nurse prepare to administer as prescribed to the client? 1. Calcium chloride 2. Calcium gluconate 3. Calcitonin (Miacalcin) 4. Large doses of vitamin D

3. Calcitonin (Miacalcin) Rationale: The normal serum calcium level is 8.6 to 10.0 mg/dL. This client is experiencing hypercalcemia. Calcium gluconate and calcium chloride are medications used for the treatment of tetany, which occurs as a result of acute hypocalcemia. In hypercalcemia, large doses of vitamin D need to be avoided. Calcitonin, a thyroid hormone, decreases the plasma calcium level by inhibiting bone resorption and lowering the serum calcium concentration.

38.) An older client recently has been taking cimetidine (Tagamet). The nurse monitors the client for which most frequent central nervous system side effect of this medication? 1. Tremors 2. Dizziness 3. Confusion 4. Hallucinations

3. Confusion Rationale: Cimetidine is a histamine 2 (H2)-receptor antagonist. Older clients are especially susceptible to central nervous system side effects of cimetidine. The most frequent of these is confusion. Less common central nervous system side effects include headache, dizziness, drowsiness, and hallucinations.

154.) A nurse is reinforcing dietary instructions to a client who has been prescribed cyclosporine (Sandimmune). Which food item would the nurse instruct the client to avoid? 1. Red meats 2. Orange juice 3. Grapefruit juice 4. Green, leafy vegetables

3. Grapefruit juice Rationale: A compound present in grapefruit juice inhibits metabolism of cyclosporine. As a result, the consumption of grapefruit juice can raise cyclosporine levels by 50% to 100%, thereby greatly increasing the risk of toxicity. Grapefruit juice needs to be avoided. Red meats, orange juice, and green leafy vegetables are acceptable to consume.

97.) Amikacin (Amikin) is prescribed for a client with a bacterial infection. The client is instructed to contact the health care provider (HCP) immediately if which of the following occurs? 1. Nausea 2. Lethargy 3. Hearing loss 4. Muscle aches

3. Hearing loss Rationale: Amikacin (Amikin) is an aminoglycoside. Adverse effects of aminoglycosides include ototoxicity (hearing problems), confusion, disorientation, gastrointestinal irritation, palpitations, blood pressure changes, nephrotoxicity, and hypersensitivity. The nurse instructs the client to report hearing loss to the HCP immediately. Lethargy and muscle aches are not associated with the use of this medication. It is not necessary to contact the HCP immediately if nausea occurs. If nausea persists or results in vomiting, the HCP should be notified. **(most aminoglycoside medication names end in the letters -cin)**

241.) A client with a history of simple partial seizures is taking clorazepate (Tranxene), and asks the nurse if there is a risk of addiction. The nurse's response is based on the understanding that clorazepate: 1. Is not habit forming, either physically or psychologically 2. Leads to physical tolerance, but only after 10 or more years of therapy 3. Leads to physical and psychological dependence with prolonged high-dose therapy 4. Can result in psychological dependence only, because of the nature of the medication

3. Leads to physical and psychological dependence with prolonged high-dose therapy Rationale: Clorazepate is classified as an anticonvulsant, antianxiety agent, and sedative-hypnotic (benzodiazepine). One of the concerns with clorazepate therapy is that the medication can lead to physical or psychological dependence with prolonged therapy at high doses. For this reason, the amount of medication that is readily available to the client at any one time is restricted. **Eliminate options 2 and 4 first because of the closed-ended word "only"**

46.) A postoperative client has received a dose of naloxone hydrochloride for respiratory depression shortly after transfer to the nursing unit from the postanesthesia care unit. After administration of the medication, the nurse checks the client for: 1. Pupillary changes 2. Scattered lung wheezes 3. Sudden increase in pain 4. Sudden episodes of diarrhea

3. Sudden increase in pain Rationale: Naloxone hydrochloride is an antidote to opioids and may also be given to the postoperative client to treat respiratory depression. When given to the postoperative client for respiratory depression, it may also reverse the effects of analgesics. Therefore, the nurse must check the client for a sudden increase in the level of pain experienced. Options 1, 2, and 4 are not associated with this medication.

79.) Ibuprofen (Advil) is prescribed for a client. The nurse tells the client to take the medication: 1. With 8 oz of milk 2. In the morning after arising 3. 60 minutes before breakfast 4. At bedtime on an empty stomach

1. With 8 oz of milk Rationale: Ibuprofen is a nonsteroidal anti-inflammatory drug (NSAID). NSAIDs should be given with milk or food to prevent gastrointestinal irritation. Options 2, 3, and 4 are incorrect.

phenytoin therapeutic level

10-20 mcg/mL

1 mg to ng

1000 ng

19. Nurse Bryan knows that the age group that uses the most units of blood and blood products is: a. Premature infants. b. Children ages 1-20 years. c. Adults ages 21-64 years. d. The elderly above age 65 years.

19.Answer D. People older than 65 years use 43 percent of donated blood. This number is expected to increase as the population ages.

The client has been receiving spironolactone (Aldactone) 50 mg/day for heart failure. The nurse should closely monitor the client for which condition? a. Hypokalemia b. Hyperkalemia c. Hypoglycemia d. Hypermagnesemia

b. Hyperkalemia

Myxedema, which includes fatigue, general weakness, and muscle cramps, is a symptom of which endocrine disorder treated with levothyroxine (Synthroid)? a. Hyperthyroidism b. Hypothyroidism c. Cushing's syndrome d. Addison's disease

b. Hypothyroidism

A client is being changed from an injectable anticoagulant to an oral anticoagulant. Which anticoagulant does the nurse realize is administered orally? a. enoxaparin sodium (Lovenox) b. warfarin (Coumadin) c. bivalirudin (Angiomax) d. lepirudin (Refludan)

b. warfarin (Coumadin)

Elimination thru kidneys a drug must

be water soluble to pass thru.

157.) A client receiving nitrofurantoin (Macrodantin) calls the health care provider's office complaining of side effects related to the medication. Which side effect indicates the need to stop treatment with this medication? 1. Nausea 2. Diarrhea 3. Anorexia 4. Cough and chest pain

4. Cough and chest pain Rationale: Gastrointestinal (GI) effects are the most frequent adverse reactions to this medication and can be minimized by administering the medication with milk or meals. Pulmonary reactions, manifested as dyspnea, chest pain, chills, fever, cough, and the presence of alveolar infiltrates on the x-ray, would indicate the need to stop the treatment. These symptoms resolve in 2 to 4 days following discontinuation of this medication. **Eliminate options 1, 2, and 3 because they are similar GI-related side effects. Also, use the ABCs— airway, breathing, and circulation**

143.) A client has just taken a dose of trimethobenzamide (Tigan). The nurse plans to monitor this client for relief of: 1. Heartburn 2. Constipation 3. Abdominal pain 4. Nausea and vomiting

4. Nausea and vomiting Rationale: Trimethobenzamide is an antiemetic agent used in the treatment of nausea and vomiting. The other options are incorrect.

39.) The client with a gastric ulcer has a prescription for sucralfate (Carafate), 1 g by mouth four times daily. The nurse schedules the medication for which times? 1. With meals and at bedtime 2. Every 6 hours around the clock 3. One hour after meals and at bedtime 4. One hour before meals and at bedtime

4. One hour before meals and at bedtime Rationale: Sucralfate is a gastric protectant. The medication should be scheduled for administration 1 hour before meals and at bedtime. The medication is timed to allow it to form a protective coating over the ulcer before food intake stimulates gastric acid production and mechanical irritation. The other options are incorrect.

66.) Trimethoprim-sulfamethoxazole (TMP-SMZ) is prescribed for a client. A nurse should instruct the client to report which symptom if it developed during the course of this medication therapy? 1. Nausea 2. Diarrhea 3. Headache 4. Sore throat

4. Sore throat Rationale: Clients taking trimethoprim-sulfamethoxazole (TMP-SMZ) should be informed about early signs of blood disorders that can occur from this medication. These include sore throat, fever, and pallor, and the client should be instructed to notify the health care provider if these symptoms occur. The other options do not require health care provider notification.

67.) Phenazopyridine hydrochloride (Pyridium) is prescribed for a client for symptomatic relief of pain resulting from a lower urinary tract infection. The nurse reinforces to the client: 1. To take the medication at bedtime 2. To take the medication before meals 3. To discontinue the medication if a headache occurs 4. That a reddish orange discoloration of the urine may occur

4. That a reddish orange discoloration of the urine may occur Rationale: The nurse should instruct the client that a reddish-orange discoloration of urine may occur. The nurse also should instruct the client that this discoloration can stain fabric. The medication should be taken after meals to reduce the possibility of gastrointestinal upset. A headache is an occasional side effect of the medication and does not warrant discontinuation of the medication.

232.) A client in the mental health unit is administered haloperidol (Haldol). The nurse would check which of the following to determine medication effectiveness? 1. The client's vital signs 2. The client's nutritional intake 3. The physical safety of other unit clients 4. The client's orientation and delusional status

4. The client's orientation and delusional status Rationale: Haloperidol is used to treat clients exhibiting psychotic features. Therefore, to determine medication effectiveness, the nurse would check the client's orientation and delusional status. Vital signs are routine and not specific to this situation. The physical safety of other clients is not a direct assessment of this client. Monitoring nutritional intake is not related to this situation.

The nurse receives the patient's lab values throughout warfarin drug therapy. The expected therapeutic level is: 1. aPTT of three to four times the normal control value 2. aPTT one to two times the patient's baseline level 3. aPT one to two times the patient's last result 4. aPT one and half to two and half times the control value.

4. aPT one and half to two and half times the control values. Rationale: aPT is the coagulation study that monitors oral anticoagulant use, such as warfarin. As a result of one and half to two and half times the control value indicates adquate anticoagulation. aPTT is the coagulation study that monitors heparin use. aPT level of one would indicate a less than therapeutic level of anticoagulation.

1 pint to ml

480 ml

5. Some institutions will not infuse a fat emulsion, such as Intralipid, into central venous access devices (CVAD) because: a. Lipid residue may accumulate in the CVAD and occlude the catheter. b. If the catheter clogs, there is no treatment other than removal and replacement. c. Lipids are necessary only in the most extreme cases to prevent essential fatty acid (EFA) deficiency. d. Fat emulsions are very caustic.

5.Answer A. Occlusion occurs with slow infusion rates and concurrent administration of some medications. Lipid occlusions may be treated with 70 percent ethanol or with 0.1 mmol/mL NaOH. Lipids provide essential fatty acids. It is recommended that approximately 4 percent of daily calories be EFAs. A deficiency can quickly develop. Daily essential fatty acids are necessary for constant prostaglandin production. Lipids are almost isotonic with blood.

5. The nurse is aware that the patients who are allergic to intravenous contrast media are usually also allergic to which of the following products? a. Eggs b. Shellfish c. Soy d. acidic fruits

5.Answer B. Some types of contrast media contain iodine as an ingredient. Shellfish also contain significant amounts of iodine. Therefore, a patient who is allergic to iodine will exhibit an allergic response to both iodine containing contrast media and shellfish. These products do not contain iodine.

Dexamethasone (Decadron) is mixed with what IV solution?

50-100 mL of 0.9% sodium chloride (NS) OR 5% dextrose in water

Nitrofurantoin (macrodantin) given for acute UTI , what is the adult dosage for prophylaxis or recurrent UTI?

50-100m mg at bedtime

EA age 53 just returned from OR-48hrs prior meperdine demorol has been ordered for pain control The usual dose of meperidine postoperative is

50-100mg q4h PRN

9. Kent a new staff nurse asks her preceptor nurse how to obtain a blood sample from a patient with a portacath device. The preceptor nurse teaches the new staff nurse: a. The sample will be withdrawn into a syringe attached to the portacath needle and then placed into a vacutainer. b. Portacath devices are not used to obtain blood samples because of the risk of clot formation. c. The vacutainer will be attached to the portacath needle to obtain a direct sample. d. Any needle and syringe may be utilized to obtain the sample.

9.Answer A. A special portacath needle is used to access the portacath device. A syringe is attached and the sample is obtained. One of the primary reasons for insertion of a portacath device is the need for frequent or long-term blood sampling. A vacutainer will exert too much suction on the central line resulting in collapse of the line. Only special portacath needles should be used to access the portacath device.

b. Decrease the intravenous nitroglycerin by 10 mcg/min.

A client receiving intravenous nitroglycerin at 20 mcg/min complains of dizziness. Nursing assessment reveals a blood pressure of 85/40 mm Hg, heart rate of 110 beats/min, and respiratory rate of 16 breaths/min. What is the nurse's priority action? a. Assess the client's lung sounds. b. Decrease the intravenous nitroglycerin by 10 mcg/min. c. Stop the nitroglycerin infusion for 1 hour, and then restart. d. Recheck the client's vital signs in 15 minutes but continue the infusion.

c. Determine the client's history.

A client taking prazosin has a blood pressure of 140/90. The client is complaining of swollen feet. What is the nurse's best action? a. Hold the medication. b. Call the health care provider. c. Determine the client's history. d. Weigh the client.

a. Have the client lie down when taking a nitroglycerin sublingual tablet. b. Teach client to repeat taking a tablet in 5 minutes if chest pain persists. e. Warn client against ingesting alcohol while taking nitroglycerin.

A client who has angina is prescribed nitroglycerin. The nurse reviews which appropriate nursing interventions for nitroglycerin (Select all that apply.) a. Have the client lie down when taking a nitroglycerin sublingual tablet. b. Teach client to repeat taking a tablet in 5 minutes if chest pain persists. c. Apply Transderm-Nitro patch to a hairy area to protect skin from burning. d. Call the health care provider after taking 5 tablets if chest pain persists. e. Warn client against ingesting alcohol while taking nitroglycerin.

A postoperative patient has an epidural infusion of morphine sulfate (Astramorph). The patient's respiratory rate declines to 8 breaths/min. Which medication would the nurse anticipate administering? A) Naloxone (Narcan) B) Acetylcysteine (Mucomyst) C) Methylprednisolone (Solu-Medrol) D) Protamine sulfate

A) Naloxone (Narcan) Naloxone is a narcotic antagonist that can reverse the effects, both adverse and therapeutic, of opioid narcotic analgesics.

Accord, FDA drugs un which preg category are considered not to present risk to fetus

A- no risk to fetus

Antidysrhythmics Uses

A-Fib and flutter, tachycardia, PVCs

Cl taking high potency typical antipsychotics may develop adverse EPS reactions such as:

AKATHASIA

The teaching plans for EA taking meperidine would include

ALL OF ABOVE Not to use alcohol or CNS depressants while taking meperidine To report side effects Prevention of constipation

The nurse is aware that which group(s) of antihypertensive drugs are less effective in African-American clients? a. Diuretics b. Calcium channel blockers and vasodilators c. Beta blockers and ACE inhibitors d. Alpha blockers

c. Beta blockers and ACE inhibitors

The beta blocker acebutolol (Sectral) is prescribed for dysrhythmias. The nurse knows that what is the primary purpose of the drug? a. To increase the beta1 and beta2 receptors in the cardiac tissues b. To increase the flow of oxygen to the cardiac tissues c. To block the beta1-adrenergic receptors in the cardiac tissues d. To block the beta2-adrenergic receptors in the cardiac tissues

c. To block the beta1-adrenergic receptors in the cardiac tissues

Antidote for warfarin overdose a. protamine zinc insulin b. protamine sulfate c. vitamin K d. warfarin

c. vitamin K

sequence of the 4 processes of pharmocokinetocs

Absorb,Distribute,Metab,Excrete

Which of the followung drugs on overdose is known to cause live failure

Acetaminophen (tylenol) APAP

The Rn is developing a teaching plan for the CL which of the following is suggested to be included?

Actively involve CL Provide written instructions Consider use of variety media sources Provide for return demonstration

Teaching for a client receiving plasma protein fraction (Plasmanate) should include reporting which of the following possible adverse reactions?

Anaphylactic reaction

Cilostazol (Pletal) is being prescribed for a client with coronary artery disease. The nurse knows that which is the major purpose for antiplatelet drug therapy? a. To dissolve the blood clot b. To decrease tissue necrosis c. To inhibit hepatic synthesis of vitamin K d. To suppress platelet aggregation

d. To suppress platelet aggregation

what is used to treat a bladder spasm after prostatectomy?

Antispasmodic meds like Belladonna & opium (B&O) suppository OR propantheline bromide (Pro-Banthine)

ear drops best administered

At room temperature- maintains normal equilibrium

What is the antidote in a cholinergic crisis?

Atropine sulfate

Antidysrhythmics Med Names

Atropine, Lidocaine, Pronestyl, Quinidine, Isuprel

What is the gold preparation used to treat rheumatoid arthritis?

Auranofin (Ridaura)

While teaching the CL about taking new prescription of oral metronidazole the Rn should include which of the following sig elements

Avoid alcohol

Client taking triazolam(halcion) What are some important instructions for Rn to discuss

Avoid alcohol,smoking prevent rebound insomnia

Teaching about Metronidazole (Flagl)

darkening of the urine may occur and is harmless

mydriasis

dilation of the pupils

H2-Receptor Blockers Side Effects

dizziness, confustion, hypotension, impotence

when probenicid is administered with cefazolin or cefamandole which of the following results

drug action is increased

The nurse is working on a postoperative unit where pain management is part of routine care. Which statement below is the most helpful in guiding clinical practice in this setting? A) At least 30% of the U.S. population is prone to drug addiction and abuse. B) The development of opioid dependence is rare when opioids are used for acute pain. C) Morphine is a common drug of abuse in the general population. D) The use of PRN (as needed) dosing provides the most consistent pain relief without risk of addiction.

B) The development of opioid dependence is rare when opioids are used for acute pain.

Cromolyn sodium (Intal) is an inhaled nonsteroidal antiallerg agent & mast cell stabilizer; what is the major adverse effect?

Bronchospasm Cough Nasal congestion throat irritation wheezing

Solutions commonly used to irrigate the ear include the following SELECT ALL APPLY

Burrows Solution HYDROGEN PEROXIDE 3% ACETIC ACID

SE of Fluoxetine (Prozac)

CNS and GI dysfunction

which of the following statements are true regarding Ketorolac-Torodol SELECT ALL THAT APPLY

Can be administered Orally Can be administered IM Can be administered IV May be given only 5 days or LESS Incorrect answers begin w/ HAS Has an efficacy equal to morphine Has a usual dose of 15 mg IMq6h

universal Precautions require that you do all the following EXCEPT

Cap needles

Antibiotics: Aminoglycosides Nursing Considerations

Checking 8th cranial nerve (hearing) Check Renal Function (BUN) Take for 7-10 days Encourage Fluids (3,000ml/day when pushing)

Antineoplastic: Alkylating Agents Nurisng Considerations

Checking hematopoietic function Force fluids Good mouth care

Which of the following drug types is used in the treatement of myasthenia gravis

Cholinesterase Inhibitors

What health teaching plan for ampho B would include all except

Consume NO alcohol

The actions of centrally acting muscle relaxants include which of the following

Decrease pain and increase range of motion

What is the difference between onset and duration of delirium and dementia?

Delirium is acute onset and duration is days to weeks. Dementia is an insidious onset and the duration is months to years.

Which is NOT an effect of anticholinegic drugs

Diarhea Cholinergic Drugs dry -up!

When Cl refuses to take medication, RN must:

Document reason not taken

Anticonvulsants Nursing Considerations

Don't d/c abruptly Monitor I&O Caution with use of meds that lower seizure thresholds (ex. MOAIs, antipyscotic) No alcohol Urine is pink/reddish

When administering a skeletal muscle relaxant, the nurse observes the pt for the most common adverse reaction, which is

Drowsiness

Use of opiate related drugs for severe diarhea may cause which of the following SELECT ALL THAT APPLY

Drowsiness Constipation Paralytic Ilues

Which legislation decreases the time for approval of drugs used for treating aids cancer

Drug Regulation Act 1992

b. Dizziness c. Headache e. Ankle edema

During an admission assessment, the client states that she takes amlodipine (Norvasc). The nurse wishes to determine whether or not the client has any common side effects of a calcium channel blocker. The nurse asks the client if she has which signs and symptoms? (Select all that apply.) a. Insomnia b. Dizziness c. Headache d. Angioedema e. Ankle edema f. Hacking cough

Andrenergics Side Effects

Dysrhythmias, Tremors, Anticholinergic Effects (dry mouth, urinary retention)

What is ECT? What are interventions that are carried out for someone undergoing ECT?

Electroconvulsive therapy: planned controlled seizures to assist depression. The pt should be NPO, atropine to dry secretions (drooling) but not for a glaucoma patient, sleep 5-6 hours afterwards, and may have some memory loss. A Float nurse would be assigned to this patient.

Specific Nursing Interventions for RT would include all of the following EXCEPT

Encourage Fluids

Antivirals Nursing Considerations

Encourage fluids Not a cure, but relieves symptoms

When allopurinol-zyloprim is used for treating gout-the nurse

Encourages liberal fluid intake

most drug testing is US has which ethnic group for research participants

European Americans- trialed more often in europe prior to us.

What is the main concern with anorexia nervosa?

F&E imbalance and muscle weakness.

Cl taking nitrofurantoin should report which of the following to the health care provider

FEVER

what is given first: eye drops or eye ointment?

eye drops

The health teaching plan would include all the following EXCEPT:

finish Ab (from previous) False!!

pt taking Ticlopidine (Ticlid) will often develop neutropenia within

first 3 months of therapy

The nurse prepares nutritional teaching for a group of clients. The nurse recognizes that the client most at risk for ____________ anemia is the client diagnosed with Insufficient Dietary Intake.

Folate-deficiency

Biological activiy of a drug is determined by the

fit of the drug at the receptor site must fit or be recognized for drug to become active.

What should you do with an obsessive compulsive patient who is wringing their hands for hours and they are beginning to bleed?

Give him lotion instead of asking him to stop the ritual.

Antifungals: Nursing Considerations

Give w food monitor liver function good oral hygiene

Hypoglycemic Agent: Med Name

GlacGen (glucagon)

client taking zolpidem(ambien) for insomnia- RN prepares a care plan that includes monitoring pt with S/E Which is side effect?

Headache

Assessment Cl for treatment w antihelmintics includes which of the following

History food intake Collection stool determine if other household member have same s/s All above***

Insulin: Combination Names

Humulin 70/30

Diuretics Med Names

HydroDIURIL, Aldactone, Lasix, Mannitol

main adverse effect of Spironolactone (Aldactone), a diuretic

Hyperkalemia

The client receiving filgrastim (Neupogen) should be monitored for common adverse effects, which include:

Hypertension and skeletal pain.

Enalapril (Vasotec)

Indication: CHF, lower blood pressure, improve CO Class: ACE inhibitor Labs: BUN, Cr, Na, hepatic, urine output Side effects: hypotension, cough, dizziness Precautions: monitor blood pressure and pulse, monitor weight, monitor for fluid overload, monitor for cough

Lisinopril

Indication: CHF, lower blood pressure, improve CO Class: ACE inhibitor Labs: BUN, Cr, Na, hepatic, urine output Side effects: hypotension, cough, dizziness Precautions: monitor blood pressure and pulse, monitor weight, monitor for fluid overload, monitor for cough

Cyclobenzaprine (Flexrol)

Indication: acute painful muscle spasms Class: muscle relaxant Labs: N/A Side effects: dizziness, drowsiness Precautions: monitor pain and ROM

Calcitriol (Rocaltrol)

Indication: low blood calcium due to kidney dialysis, osteoporosis, hyperparathyroidism Class: Vitamin D Labs: Mg, Ca, phosphate Side effects: pancreatitis, dizziness, malasia Precuations: toxicity is hypocalcaemia assess for N/V, weakness, constipation, headache, bone pain, metallic taste, cardiac arrythmias Reversal: low Ca diet, stop drug

Hesperidins (Dmerol)

Indication: moderate to severe pain Class: opiod Labs: N/A Side Effects: seizure, confusion, hypotension, anaphylaxis, respiratory depression Precautions: assess pain, monitor vitals, monitor bowel function Reversal: Narcan

Sudafed

Indication: nasal congestion Class: adrenergic Labs: N/A Side Effects: seizure, anxiety, palpitations, anorexia, dizziness Precautions: assess congestion, monitor pulse, adequate fluid intake

Ceftazidime

Indication: pseudomonas, netropenia Class: anti-infective broad spectrum Labs: liver, WBC Side effects: seizure, C-Diff, rash, diarrhea, N/V, anaphylaxis Precuations: monitor for anaphylaxis, bowel function, assess infection, obtain culture

At team conference, Rn explains that the anti-inflammatory actions of salicylate are MOST LIKELY DUE TO:

Inhibition of prostaglandins

The mechanism for action of non-steroidal anti-inflammatory drugs include

Inhibition of synthesis of prostaglandins

What is neuroleptic malignant syndrome?

It is a rare but potentially fatal adverse reaction to antipsychotic medications that leads to autonomic dysfunction.

What is important to know about Tricyclics?

It takes 2-4 weeks before the drug effects kick in so teach relaxation techniques to utilize in the meantime. SE= Dry mouth, constipation (give med before bed, increase fluids, and high fiber diet (NO COLACE!!)). May also cause hypotension.

Diuretics

K+ and NA+ follows the walter --> electrolytes imbalances

-"tone"

K+ sparring diuretics

Antidiarrheals: Med Names

Kaopectate, Lomotil, Imodium, Paregoric

Erythropoietin regulates the process of red blood cell formation. The nurse understands that this mechanism is activated by a reduction of oxygen reaching the:

Kidneys.

What is an appropriate snack for someone on lithium who is going hiking?

Lemonade is good bc they need hydration but can't have caffeine. (No tea, coffee, etc.)

Bipolar Disorder: Med Names

Lithium (salt), Tegretol (mood stablizer), Depakote (mood stablizer)

Two advantages assoc w/ use of generic drugs include

Lower costs Same active Ingredients

Health teaching would include all following EXCEPT

Monitor daily weights

Glucocorticoids Nursing Considerations

Monitor fluid and electrolyte balance Don't dc abruptly Monitor for signs of infection

Thrombolytics Nursing Considerations

Monitor for bleeding Have amino caproic acid available - Antidote Check pulse, color, sensation of extremities Monitor EKG

Antidepressants: Tricyclics Nursing Considerations

Monitor for suicide takes 2-6 weeks to work take at bedtime monitor vital signs position slowly wean off medication no alcohol no sun- use sunscreen

RN interventions during antimaliral drug therapy include:

Monitoring urinary output/liver fxn

Your cl has a medication ordered to dilate his eyes such a medication belongs to which groups of drugs

Mydriatics

Misoprostol, gastric protectant, is given specifically to pts who use what drugs chronically?

NSAIDS

MAOI

Nardil is an ____. This class of drugs has many drug and food interactions that may cause a hypertensive crisis.

EQ is recieving isopto-eserine eye drops for treatment of glaucoma. You would assess EQ for possible side effects, including all of the following EXCEPT

Nausea

When teaching Cl taking Acyclovir about S/E that are assoc with this drug which of the following should the RN include

Nausea Headache Lethargy

the Rn should observe a Cl taking cephalosporin for which S/S

Nausea Vomiting Diarhea Itching

SubQ Needle size,Location,Angle of admin

Needle size 24-28g 5/8 inch Location Buttocks Abd back region arm Fatty areas upon body Angle 90*

-"nitr"

Nitrates angina vasodilation

Your cl. is taking a prostaglandin analogue and you assess for the most common adverse reaction, which is

Occular Hyperemia-Excess Blood

What is characteristic of the manic phase of bipolar disorder?

Onset is prior to age 30, Mood (ELEVATED, expansive, and irritable), Speech (loud, rapid, running, rhyming, clanging, vulgar), ? WEIGHT LOSS, GRANDIOSE (think they are GOD or can fly), DISTRACTED, HYPERACTIVE, delusions, decreased need for sleep, inappropriate, flight of ideas, begins suddenly and escalates over days.

For CL who are vomiting/Comatose medication administration is conraindicated via which route

Oral

Broad Spectrum penicillins may decrease the effectiveness of which of the following

Oral Contraceptives

A cl is taking Haldol 5 mg TID and C/O nearly falling down when he gets OOB the cl is likely experiencing

Orthostatic Hypo Tension

The LPN student observing a surgical procedure, which stage is pt most at risk:

Paralysis- Medulla regions responsible for- respiratory and cardiac effects.

What are extrapyramidal sx?

Parkinson's like symptoms (tremors, drooling, etc.)

A specific group of anticholinergics may be prescribed in teh early treatement of which of the following neuromuscular disorders

Parkinsonism

Phenytoin and birth control pills: what do you need to know?

Phenytoin can decrease the effectiveness of birth control pills

Which of the following statements is true about anticonvulsants

Phenytoin has been linked to cardiac defects

A Cl is recieving levoflaxacin in addition to oral anticoag what treatment should the RN anticipate admin if CL experiences an adv. effect as a result of his combo.

Phytonadione Vit K

What is the antidote for Warfarin (Coumadin)?

Phytonadione a.k.a Vitamin K

What are the S&S of amphetamine overdose?

RAPID RESPIRATIONS, restlessness, tremor, confusion, hallucinations, and panic.

What is characteristic of bulimia?

RECURRENT BINGE EATING, awareness of abnormal eating pattern, fear of not being able to stop eating voluntarily, depressed mood following binges, laxative misuse, calloused knuckles, and rotten teeth.

RT health teaching plan would include all of the following EXCEPT

Stay Busy to KEEP Mind off Nausea

After taking lithium carbonate 1200mg day for 5 days RT remains agitated and hyperactive todays plasma level was 0.8mEq/L and RT c/o feeling slow and having increased thirst. Your analysis is that the CL is:

Still manic w/o signs of toxicity

Thyroid Replacement Agents Med Names

Synthroid, Cytomel

Diuretics Nursing Considerations

Take with food or milk Take in am Monitor fluid and electrolytes

Antibiotics: Sulfonamides Nursing Considerations

Take with meals Encourage fluids Good mouth care

What is characteristic of a patient with PTSD?

Tense, HYPERVIGILANT, depressed, or emotionally distant. They may also suddenly believe that a traumatic event is recurring (a flashback) or experience survival guilt, intrusive memories, or psychic numbing.

8. The most important instructions a nurse can give a patient regarding the use of the antibiotic Ampicillian prescribed for her are to 1) call the physician if she has any breathing difficulties. 2) take it with meals so it doesn't cause an upset stomach. 3) take all of the medication prescribed even if the symptoms stop sooner. 4) not share the pills with anyone else.

The correct answer is (3). Frequently patients do not complete an entire course of antibiotic therapy, and the bacteria are not destroyed.

Cl. education for use of these drugs is important - The RN actions for these individuals include advising the CL and significant others that

The drug should be taken as prescribed and to consult the health care provider before discontinuing the medication

Serious Adv ganciclovir used for treatment of cytomegalovirus

Throbocytopenia

Atropine is frequently prescribed in all of the following situations EXCEPT

To treat urinary retention

The drug used to control an overactive bladder and contraindicated in cl with narrow-angle glaucoma is which of the following

Tolterodine Tartrate-Detrol

Which of the following drugs is most effective in treating glaucoma in african american cl.

Travaprost-Travatan

Potential Side effects/adverse reactions for a cl taking fluoxetine (prozac) include :SELECT ALL THAT APPlY

Tremors,Seizures,Insomnia,HA Incorrect = dysrhythmias

Atopine (Caridiac Use)

Tx Heart Blocks (increase HR) 4D's: Dry Mouth, Dry Stool, Dilated Pupils, Dilated Bladder (urinary retention)

Antipyretic Med Name

Tylenol

Phenelzine (Nardil) is a Monoamine oxidase inhibitor (MAOI) what type of foods need to be avoided bc it can cause hypertensive crisis?

Tyramine like yogurt, aged cheeses, smoked or processed meats, red wines, and fruits like avocados, raisins, or figs

The vascular phase of inflammation is associated w/

Vasodilation w/ increased permeability

Antineoplastic: Vinca Alkaloids Med Names

Velban (vinblastine), Oncovin (vincristine)

When administering a salicylate, the RN correctly administers the drug

W food or milk

Pt has been recieving Benzodiazepines for several yrs, what condition may this cause

W/D s/s if pt abruptly d/c

W severe diarhea, it is important to monitor all of the following EXCEPT

WBC

b. Headaches

When a client first takes a nitrate, the nurse expects which symptom that often occurs? a. Nausea and vomiting b. Headaches c. Stomach cramps d. Irregular pulse rate

a. Loss of appetite with slight bradycardia

Which assessment finding will alert the nurse to suspect early digitalis toxicity? a. Loss of appetite with slight bradycardia b. Blood pressure 90/60 mm Hg c. Heart rate 110 beats per minute d. Confusion and diarrhea

c. Risk for injury

Which nursing diagnosis would be possible for a client receiving intravenous heparin therapy? a. Potential for fluid volume excess b. Potential for pain c. Risk for injury d. Potential for body image disturbance

For best results it is recommended that tetracycline be taken

With Extra Fluid- reduce toxicity flush kidney On an empty stomach-inhibits drug rxn

The nurse knows that the client's cholesterol level should be within which range? a. 150 to 200 mg/dL b. 200 to 225 mg/dL c. 225 to 250 mg/dL d. Greater than 250 mg/dL

a. 150 to 200 mg/dL

Blood sugar is well controlled when Hemoglobin A1C is: a. Below 7% b. Between 12%-15% c. Less than 180 mg/dL d. Between 90 and 130 mg/dL

a. Below 7% A1c measures the percentage of hemoglobin that is glycated and determines average blood glucose during the two to three months prior to testing. Used as a diagnostic tool, A1C levels of 6.5% or higher on two tests indicate diabetes. A1C of 6% to 6.5% is considered prediabetes.

A resident of the nursing home has quite severe arthritis. When administering an analgesic to this elderly resident, the nurse should: a. Give the medication before the activity session in the day room. b. Give the medication when the resident states the pain is at 6 or higher on a 1-10 pain scale. c. Give the pain medication at mealtime. d. Make sure that the medication is not a narcotic.

a. Give the medication before the activity session in the day room.

Naloxone (Narcan) is administered to a client with severe respiratory depression and suspected drug overdose. After 20 minutes, the client remains unresponsive. The most likely explanation for this is: a.) The client did not use an opioid drug. b.) The dose of naloxone was inadequate. c.) The client is resistant to this drug. d.) The drug overdose is irreversible.

a.) The client did not use an opioid drug. If opioid antagonists (Naloxone) fail to reverse symptoms of respiratory depression quickly, the overdose was likely due to a non-opioid substance.

198.) Coal tar has been prescribed for a client with a diagnosis of psoriasis, and the nurse provides instructions to the client about the medication. Which statement by the client indicates a need for further instructions? 1. "The medication can cause phototoxicity." 2. "The medication has an unpleasant odor." 3. "The medication can stain the skin and hair." 4. "The medication can cause systemic effects."

4. "The medication can cause systemic effects." Rationale: Coal tar is used to treat psoriasis and other chronic disorders of the skin. It suppresses DNA synthesis, mitotic activity, and cell proliferation. It has an unpleasant odor, can frequently stain the skin and hair, and can cause phototoxicity. Systemic toxicity does not occur. **The name of the medication will assist in eliminating options 2 and 3**

226.) A client receiving lithium carbonate (Lithobid) complains of loose, watery stools and difficulty walking. The nurse would expect the serum lithium level to be which of the following? 1. 0.7 mEq/L 2. 1.0 mEq/L 3. 1.2 mEq/L 4. 1.7 mEq/L

4. 1.7 mEq/L Rationale: The therapeutic serum level of lithium ranges from 0.6 to 1.2 mEq/L. Serum lithium levels above the therapeutic level will produce signs of toxicity.

Antineoplastic: Alkylating Agents Action

interferes with rapidly reproducing DNA

Anticoagulant Coumadin Action

interferes with synthesis of vitamin 5-dependent clotting factors

Vincristine (Oncovin), anticancer med, major adverse effect?

peripheral neuropathy like weakness and sensory loss in the legs -decreased refleces -weakness -paresthesia -sensory loss

Antibiotics: Sulfonamides Side Effects

peripheral neuropathy, crystalluria, photosensitivity, GI upset, stomatitis

w/ Bleomycin, it is a priority to assess lung sound and pulmonary function studies bc

pneumonitis can manifest leasing to severe pulmonary fibrosis

Antithyroid Agents Action

reduce vasularity of thyroid, inbibits release of thyroid into circulation

Bipolar Disorder: Action

reduces catecholamine release

When administering a DMARD drug subQ the nurse should

rotate sites to minimize tissue irratation

Lindane is prescribed for scabies; what preexisiting condition is high risk for adverse effect of seizures?

seizure hx

Antidiarrheals: Actions

slows peristalsis, increases tone of sphincters

antivenin, polyvalent

snake bite antidote

antidote for Coumadin

vitamin K

45 yo pt admitted for treatment of Severe UTI- current meds include Septra and digoxin Usual dose of septa is?

160mg TMP/800mg SMZq12h

The analgesic drug to give to a child w/ a virus or the flu is

Acetaminophen(aspirin toxic children under age 8)

What is the response to someone experiencing a paranoid delusion?

Acknowledge what the person says but don't agree. Hone in on their feelings ("it must be very frightening"= EMPATHY)

Warfarin

Action: interferes with hepatic synthesis of vitamin K dependant clotting factors to prevent thromembolic events, extrinsic pathway Indications: prophylaxis and treatment of venous thrombis, pulmonary embolism, a-fib, and management of MI. Class-Therapeutic: anticoagulants Class-Pharmalogic: coumarins

Kefauver-Harris amendment was est. to improve safety by requiring which of the following in the literature

Adv./SE and Contraindications

Antibiotics General

Allergies, Super Infections, Organ Toxicity

Antineoplastic: Alkylating Agents Med Names

Cisplatin, Myleran, Cytoxan

Klonipin

Indication: seizures, panic disorders Class: anticonvulsant, benzo Labs: hepatic, renal, CBC Side effects: dizziness, lethargy, drowsiness, suicidal thoughts, behavioral changes Precautions: monitor vitals, physical dependence, monitor seizures, for long term use, assess mental status, Reversal: Flumazenil

Ketoalac

Indication: short term management not to exceed 5 days Class: NSAID Labs: hepatic, bleeding time, BUN, Cr, K Side Effects: drowsiness, anaphylaxis, GI bleeding, asthma, renal toxicity Precautions: rhinitis, asthma, and aspirin allergy increase risk for hypersensitivity, assess pain

Nystatin

Indication: skin fungus infections Class: antifungal Labs: N/A Side Effects: burning, itching Precautions: inspect affected area

Famotidine (Pepcid)

Indication: ulcers, GERD Class: H2 blockers Labs: CBC Side effects: confusion, arrythmias Precautions: assess for epigastria or abdominal pain, frank occult blood, gastric aspirate, or emesis

Bipolar Disorder: Uses

Manic episodes

Which following is are true about the drug rifapentine

Newest drug for treatment of TB Twice weekly dosing Taken in combo with another TB med-to avoid resistance

Protamine sulfate is the antidote for

heparin

Antitubercular Agents Side Effects

hepatitis, peripheral neuritis

Dantrolene sodium (Dantrium) major adverse effect?

hepatotoxic

Antineoplastic: Alkylating Agents Side Effects

hepatotoxicity, bone marrow suppression

Antifungals: Side Effects

hepatotoxicity, throbocytopenia, leukopenia, hemorrhage, pruritus

Antihypertensives: Alpha 1 Adrenergic Blockers Uses

hypertension, benign prostatic hypertrophy, reflex tachycardia

Antidiarrheals: Side Effects

sedation, anticholinergic effects

TCAs

severe case of depression Elavil anticholinergic effects decrease libido

Antidepressants: SSRIs Nursing Considerations

suicide precautions takes 4 weeks to fully work take in am turn urine pink

-"sulf"

sulfonamides suppress bacterial growth

signs of hepatitis

urine becomes very dark in color anorexia abdominal pain unusual tiredness weakness jaundice develops

Proton pump inhibitors have an ending of

"prazole"

which form is most rapidly absorbed in body

*Suspension, extended release others ( IV,SbL IM) Oral Rectal slow

Beta Blockers

-lol can cause bronchspasms airway is a consideration

Antihypertensives: Beta Blockers Med Names

-lol (beats)

Antibiotics: Aminoglycosides

-mycin

Antihypertensives: Alpha 1 Adrenergic Blockers Med Names

-osin

Benzodiazepines

-pams

Antihypertensives: Calcium Channel Blockers Med Names

-pine, Cardizem

Antihypertensives: Ace Inhibitors Med Names

-pril (think arteries)

excessive muscarinic stimulation signs

-salivation - sweating - involuntary urination - defecation - bradycardia - severe hypotension

Nalidixic acid (Neg Gram) given for UTI , what are normal adult dosage for this med?

1 g four times daily for a period of 1 week.

What is correct about schedule drugs that have potential for abuse

1 highest 3 moderate (4 and 5 not so much they are OTC) 1-3

1 L to kg

1 kg

before surgery the use of many herbal products should be discontinued

1-2 weeks before surgery

127.) The nurse provides medication instructions to an older hypertensive client who is taking 20 mg of lisinopril (Prinivil, Zestril) orally daily. The nurse evaluates the need for further teaching when the client states which of the following? 1. "I can skip a dose once a week." 2. "I need to change my position slowly." 3. "I take the pill after breakfast each day." 4. "If I get a bad headache, I should call my doctor immediately."

1. "I can skip a dose once a week." Rationale: Lisinopril is an antihypertensive angiotensin-converting enzyme (ACE) inhibitor. The usual dosage range is 20 to 40 mg per day. Adverse effects include headache, dizziness, fatigue, orthostatic hypotension, tachycardia, and angioedema. Specific client teaching points include taking one pill a day, not stopping the medication without consulting the health care provider (HCP), and monitoring for side effects and adverse reactions. The client should notify the HCP if side effects occur.

220.) A adult client with muscle spasms is taking an oral maintenance dose of baclofen (Lioresal). The nurse reviews the medication record, expecting that which dose should be prescribed? 1. 15 mg four times a day 2. 25 mg four times a day 3. 30 mg four times a day 4. 40 mg four times a day

1. 15 mg four times a day Rationale: Baclofen is dispensed in 10- and 20-mg tablets for oral use. Dosages are low initially and then gradually increased. Maintenance doses range from 15 to 20 mg administered three or four times a day.

142.) A health care provider has written a prescription for ranitidine (Zantac), once daily. The nurse should schedule the medication for which of the following times? 1. At bedtime 2. After lunch 3. With supper 4. Before breakfast

1. At bedtime Rationale: A single daily dose of ranitidine is usually scheduled to be given at bedtime. This allows for a prolonged effect, and the greatest protection of the gastric mucosa. **recall that ranitidine suppresses secretions of gastric acids**

202.) A nurse is collecting data from a client about medications being taken, and the client tells the nurse that he is taking herbal supplements for the treatment of varicose veins. The nurse understands that the client is most likely taking which of the following? 1. Bilberry 2. Ginseng 3. Feverfew 4. Evening primrose

1. Bilberry Rationale: Bilberry is an herbal supplement that has been used to treat varicose veins. This supplement has also been used to treat cataracts, retinopathy, diabetes mellitus, and peripheral vascular disease. Ginseng has been used to improve memory performance and decrease blood glucose levels in type 2 diabetes mellitus. Feverfew is used to prevent migraine headaches and to treat rheumatoid arthritis. Evening primrose is used to treat eczema and skin irritation.

116.) A nurse administers a dose of scopolamine (Transderm-Scop) to a postoperative client. The nurse tells the client to expect which of the following side effects of this medication? 1. Dry mouth 2. Diaphoresis 3. Excessive urination 4. Pupillary constriction

1. Dry mouth Rationale: Scopolamine is an anticholinergic medication for the prevention of nausea and vomiting that causes the frequent side effects of dry mouth, urinary retention, decreased sweating, and dilation of the pupils. The other options describe the opposite effects of cholinergic-blocking agents and therefore are incorrect.

93.) The client who is human immunodeficiency virus seropositive has been taking stavudine (d4t, Zerit). The nurse monitors which of the following most closely while the client is taking this medication? 1. Gait 2. Appetite 3. Level of consciousness 4. Hemoglobin and hematocrit blood levels

1. Gait Rationale: Stavudine (d4t, Zerit) is an antiretroviral used to manage human immunodeficiency virus infection in clients who do not respond to or who cannot tolerate conventional therapy. The medication can cause peripheral neuropathy, and the nurse should monitor the client's gait closely and ask the client about paresthesia. Options 2, 3, and 4 are unrelated to the use of the medication.

158.) A client with chronic renal failure is receiving epoetin alfa (Epogen, Procrit). Which laboratory result would indicate a therapeutic effect of the medication? 1. Hematocrit of 32% 2. Platelet count of 400,000 cells/mm3 3. White blood cell count of 6000 cells/mm3 4. Blood urea nitrogen (BUN) level of 15 mg/dL

1. Hematocrit of 32% Rationale: Epoetin alfa is used to reverse anemia associated with chronic renal failure. A therapeutic effect is seen when the hematocrit is between 30% and 33%. The laboratory tests noted in the other options are unrelated to the use of this medication.

Premarin

1. Hormone Replacement - Estrogen Tx Menopause Sxs SD - Vascular Headaches Periods continue 2. Prempro (comination estrogen and progesteron) No periods

235.) A tricyclic antidepressant is administered to a client daily. The nurse plans to monitor for the common side effects of the medication and includes which of the following in the plan of care? 1. Offer hard candy or gum periodically. 2. Offer a nutritious snack between meals. 3. Monitor the blood pressure every 2 hours. 4. Review the white blood cell (WBC) count results daily.

1. Offer hard candy or gum periodically. Rationale: Dry mouth is a common side effect of tricyclic antidepressants. Frequent mouth rinsing with water, sucking on hard candy, and chewing gum will alleviate this common side effect. It is not necessary to monitor the blood pressure every 2 hours. In addition, it is not necessary to check the WBC daily. Weight gain is a common side effect and frequent snacks will aggravate this problem.

171.) A nurse is preparing to administer furosemide (Lasix) to a client with a diagnosis of heart failure. The most important laboratory test result for the nurse to check before administering this medication is: 1. Potassium level 2. Creatinine level 3. Cholesterol level 4. Blood urea nitrogen

1. Potassium level Rationale: Furosemide is a loop diuretic. The medication causes a decrease in the client's electrolytes, especially potassium, sodium, and chloride. Administering furosemide to a client with low electrolyte levels could precipitate ventricular dysrhythmias. Options 2 and 4 reflect renal function. The cholesterol level is unrelated to the administration of this medication.

90.) A nurse is reviewing the record of a client who has been prescribed baclofen (Lioresal). Which of the following disorders, if noted in the client's history, would alert the nurse to contact the health care provider? 1. Seizure disorders 2. Hyperthyroidism 3. Diabetes mellitus 4. Coronary artery disease

1. Seizure disorders Rationale: Clients with seizure disorders may have a lowered seizure threshold when baclofen is administered. Concurrent therapy may require an increase in the anticonvulsive medication. The disorders in options 2, 3, and 4 are not a concern when the client is taking baclofen.

53.) Rifabutin (Mycobutin) is prescribed for a client with active Mycobacterium avium complex (MAC) disease and tuberculosis. The nurse monitors for which side effects of the medication? Select all that apply. 1. Signs of hepatitis 2. Flu-like syndrome 3. Low neutrophil count 4. Vitamin B6 deficiency 5. Ocular pain or blurred vision 6. Tingling and numbness of the fingers

1. Signs of hepatitis 2. Flu-like syndrome 3. Low neutrophil count 5. Ocular pain or blurred vision Rationale: Rifabutin (Mycobutin) may be prescribed for a client with active MAC disease and tuberculosis. It inhibits mycobacterial DNA-dependent RNA polymerase and suppresses protein synthesis. Side effects include rash, gastrointestinal disturbances, neutropenia (low neutrophil count), red-orange body secretions, uveitis (blurred vision and eye pain), myositis, arthralgia, hepatitis, chest pain with dyspnea, and flu-like syndrome. Vitamin B6 deficiency and numbness and tingling in the extremities are associated with the use of isoniazid (INH). Ethambutol (Myambutol) also causes peripheral neuritis.

110.) A client taking lithium carbonate (Lithobid) reports vomiting, abdominal pain, diarrhea, blurred vision, tinnitus, and tremors. The lithium level is checked as a part of the routine follow-up and the level is 3.0 mEq/L. The nurse knows that this level is: 1. Toxic 2. Normal 3. Slightly above normal 4. Excessively below normal

1. Toxic Rationale: The therapeutic serum level of lithium is 0.6 to 1.2 mEq/L. A level of 3 mEq/L indicates toxicity.

normal prothrombin time

10-13 seconds

Phenytoin (Dilantin) therapeutic serum level

10-20 mcg/mL

normal acetaminophen (Tyelnol) therapeutic serum level

10-20 mcg/mL

10. The nurse is administering an antibiotic to her pediatric patient. She checks the patient's armband and verifies the correct medication by checking the physician's order, medication kardex, and vial. Which of the following is not considered one of the five "rights" of drug administration? a. Right dose b. Right route c. Right frequency d. Right time

10. C. The five rights of medication administration are right drug, right dose, right route, right time, right patient. Frequency is not included.

SSRIs

1st line of defense against depression

Bethanechol chloride (Urecholine ) can cause nausea & vomiting when should the pt take the med

2 hours after meals

176.) A nurse notes that a client is taking lansoprazole (Prevacid). On data collection, the nurse asks which question to determine medication effectiveness? 1. "Has your appetite increased?" 2. "Are you experiencing any heartburn?" 3. "Do you have any problems with vision?" 4. "Do you experience any leg pain when walking?"

2. "Are you experiencing any heartburn?" Rationale: Lansoprazole is a gastric acid pump inhibitor used to treat gastric and duodenal ulcers, erosive esophagitis, and hypersecretory conditions. It also is used to treat gastroesophageal reflux disease (GERD). It is not used to treat visual problems, problems with appetite, or leg pain. **NOTE: "-zole" refers to gastric acid pump inhibitors**

29.) A client is taking Humulin NPH insulin daily every morning. The nurse reinforces instructions for the client and tells the client that the most likely time for a hypoglycemic reaction to occur is: 1. 2 to 4 hours after administration 2. 4 to 12 hours after administration 3. 16 to 18 hours after administration 4. 18 to 24 hours after administration

2. 4 to 12 hours after administration Rationale: Humulin NPH is an intermediate-acting insulin. The onset of action is 1.5 hours, it peaks in 4 to 12 hours, and its duration of action is 24 hours. Hypoglycemic reactions most likely occur during peak time.

88.) Dantrolene sodium (Dantrium) is prescribed for a client experiencing flexor spasms, and the client asks the nurse about the action of the medication. The nurse responds, knowing that the therapeutic action of this medication is which of the following? 1. Depresses spinal reflexes 2. Acts directly on the skeletal muscle to relieve spasticity 3. Acts within the spinal cord to suppress hyperactive reflexes 4. Acts on the central nervous system (CNS) to suppress spasms

2. Acts directly on the skeletal muscle to relieve spasticity Rationale: Dantrium acts directly on skeletal muscle to relieve muscle spasticity. The primary action is the suppression of calcium release from the sarcoplasmic reticulum. This in turn decreases the ability of the skeletal muscle to contract. **Options 1, 3, and 4 are all comparable or alike in that they address CNS suppression and the depression of reflexes. Therefore, eliminate these options.**

34.) A client with Crohn's disease is scheduled to receive an infusion of infliximab (Remicade). The nurse assisting in caring for the client should take which action to monitor the effectiveness of treatment? 1. Monitoring the leukocyte count for 2 days after the infusion 2. Checking the frequency and consistency of bowel movements 3. Checking serum liver enzyme levels before and after the infusion 4. Carrying out a Hematest on gastric fluids after the infusion is completed

2. Checking the frequency and consistency of bowel movements Rationale: The principal manifestations of Crohn's disease are diarrhea and abdominal pain. Infliximab (Remicade) is an immunomodulator that reduces the degree of inflammation in the colon, thereby reducing the diarrhea. Options 1, 3, and 4 are unrelated to this medication.

146.) A client has begun therapy with theophylline (Theo-24). The nurse tells the client to limit the intake of which of the following while taking this medication? 1. Oranges and pineapple 2. Coffee, cola, and chocolate 3. Oysters, lobster, and shrimp 4. Cottage cheese, cream cheese, and dairy creamers

2. Coffee, cola, and chocolate Rationale: Theophylline is a xanthine bronchodilator. The nurse teaches the client to limit the intake of xanthine-containing foods while taking this medication. These include coffee, cola, and chocolate.

136.) A nurse performs an admission assessment on a client who visits a health care clinic for the first time. The client tells the nurse that propylthiouracil (PTU) is taken daily. The nurse continues to collect data from the client, suspecting that the client has a history of: 1. Myxedema 2. Graves' disease 3. Addison's disease 4. Cushing's syndrome

2. Graves' disease Rationale: PTU inhibits thyroid hormone synthesis and is used to treat hyperthyroidism, or Graves' disease. Myxedema indicates hypothyroidism. Cushing's syndrome and Addison's disease are disorders related to adrenal function.

101.) Ketoconazole is prescribed for a client with a diagnosis of candidiasis. Select the interventions that the nurse includes when administering this medication. Select all that apply. 1. Restrict fluid intake. 2. Instruct the client to avoid alcohol. 3. Monitor hepatic and liver function studies. 4. Administer the medication with an antacid. 5. Instruct the client to avoid exposure to the sun. 6. Administer the medication on an empty stomach.

2. Instruct the client to avoid alcohol. 3. Monitor hepatic and liver function studies. 5. Instruct the client to avoid exposure to the sun. Rationale: Ketoconazole is an antifungal medication. It is administered with food (not on an empty stomach) and antacids are avoided for 2 hours after taking the medication to ensure absorption. The medication is hepatotoxic and the nurse monitors liver function studies. The client is instructed to avoid exposure to the sun because the medication increases photosensitivity. The client is also instructed to avoid alcohol. There is no reason for the client to restrict fluid intake. In fact, this could be harmful to the client.

65.) A nurse is reinforcing discharge instructions to a client receiving sulfisoxazole. Which of the following should be included in the list of instructions? 1. Restrict fluid intake. 2. Maintain a high fluid intake. 3. If the urine turns dark brown, call the health care provider (HCP) immediately. 4. Decrease the dosage when symptoms are improving to prevent an allergic response.

2. Maintain a high fluid intake. Rationale: Each dose of sulfisoxazole should be administered with a full glass of water, and the client should maintain a high fluid intake. The medication is more soluble in alkaline urine. The client should not be instructed to taper or discontinue the dose. Some forms of sulfisoxazole cause urine to turn dark brown or red. This does not indicate the need to notify the HCP.

227.) When teaching a client who is being started on imipramine hydrochloride (Tofranil), the nurse would inform the client that the desired effects of the medication may: 1. Start during the first week of administration 2. Not occur for 2 to 3 weeks of administration 3. Start during the second week of administration 4. Not occur until after a month of administration

2. Not occur for 2 to 3 weeks of administration Rationale: The therapeutic effects of administration of imipramine hydrochloride may not occur for 2 to 3 weeks after the antidepressant therapy has been initiated. Therefore options 1, 3, and 4 are incorrect.

228.) A client receiving an anxiolytic medication complains that he feels very "faint" when he tries to get out of bed in the morning. The nurse recognizes this complaint as a symptom of: 1. Cardiac dysrhythmias 2. Postural hypotension 3. Psychosomatic symptoms 4. Respiratory insufficiency

2. Postural hypotension Rationale: Anxiolytic medications can cause postural hypotension. The client needs to be taught to rise to a sitting position and get out of bed slowly because of this adverse effect related to the medication. Options 1, 3, and 4 are unrelated to the use of this medication.

40.) The client who chronically uses nonsteroidal anti-inflammatory drugs has been taking misoprostol (Cytotec). The nurse determines that the medication is having the intended therapeutic effect if which of the following is noted? 1. Resolved diarrhea 2. Relief of epigastric pain 3. Decreased platelet count 4. Decreased white blood cell count

2. Relief of epigastric pain Rationale: The client who chronically uses nonsteroidal anti-inflammatory drugs (NSAIDs) is prone to gastric mucosal injury. Misoprostol is a gastric protectant and is given specifically to prevent this occurrence. Diarrhea can be a side effect of the medication, but is not an intended effect. Options 3 and 4 are incorrect.

23.) A client who has been newly diagnosed with diabetes mellitus has been stabilized with daily insulin injections. Which information should the nurse teach when carrying out plans for discharge? 1. Keep insulin vials refrigerated at all times. 2. Rotate the insulin injection sites systematically. 3. Increase the amount of insulin before unusual exercise. 4. Monitor the urine acetone level to determine the insulin dosage.

2. Rotate the insulin injection sites systematically. Rationale: Insulin dosages should not be adjusted or increased before unusual exercise. If acetone is found in the urine, it may possibly indicate the need for additional insulin. To minimize the discomfort associated with insulin injections, the insulin should be administered at room temperature. Injection sites should be systematically rotated from one area to another. The client should be instructed to give injections in one area, about 1 inch apart, until the whole area has been used and then to change to another site. This prevents dramatic changes in daily insulin absorption.

1 kg to lb

2.2 lb

Insulin: Slow Acting Duration

24-36h

Withdrawl symptoms usually occur_____ hrs after the last opiod dose

24-48HRS

1 cup (8 oz) to ml

240 ml

Insulin: Intermediate Acting Onset

2h

Drugs in canada Schedule G moderate potential for abuse and require prescriptions for both initial allocation and refills. Drugs are similair to what US schedule

3 moderate potential

Ibuprofen is generally scheduled to be taken

3-4 times per day 8-900 mg max 3600

A full effective therapuetic response to anti-pyschotics usually takes

3-6 weeks

219.) A health care provider instructs a client with rheumatoid arthritis to take ibuprofen (Motrin). The nurse reinforces the instructions, knowing that the normal adult dose for this client is which of the following? 1. 100 mg orally twice a day 2. 200 mg orally twice a day 3. 400 mg orally three times a day 4. 1000 mg orally four times a day

3. 400 mg orally three times a day Rationale: For acute or chronic rheumatoid arthritis or osteoarthritis, the normal oral adult dose is 400 to 800 mg three or four times daily.

35.) The client has a PRN prescription for loperamide hydrochloride (Imodium). The nurse understands that this medication is used for which condition? 1. Constipation 2. Abdominal pain 3. An episode of diarrhea 4. Hematest-positive nasogastric tube drainage

3. An episode of diarrhea Rationale: Loperamide is an antidiarrheal agent. It is used to manage acute and also chronic diarrhea in conditions such as inflammatory bowel disease. Loperamide also can be used to reduce the volume of drainage from an ileostomy. It is not used for the conditions in options 1, 2, and 4.

139.) Prednisone is prescribed for a client with diabetes mellitus who is taking Humulin neutral protamine Hagedorn (NPH) insulin daily. Which of the following prescription changes does the nurse anticipate during therapy with the prednisone? 1. An additional dose of prednisone daily 2. A decreased amount of daily Humulin NPH insulin 3. An increased amount of daily Humulin NPH insulin 4. The addition of an oral hypoglycemic medication daily

3. An increased amount of daily Humulin NPH insulin Rationale: Glucocorticoids can elevate blood glucose levels. Clients with diabetes mellitus may need their dosages of insulin or oral hypoglycemic medications increased during glucocorticoid therapy. Therefore the other options are incorrect.

107.) A client receiving a tricyclic antidepressant arrives at the mental health clinic. Which observation indicates that the client is correctly following the medication plan? 1. Reports not going to work for this past week 2. Complains of not being able to "do anything" anymore 3. Arrives at the clinic neat and appropriate in appearance 4. Reports sleeping 12 hours per night and 3 to 4 hours during the day

3. Arrives at the clinic neat and appropriate in appearance Rationale: Depressed individuals will sleep for long periods, are not able to go to work, and feel as if they cannot "do anything." Once they have had some therapeutic effect from their medication, they will report resolution of many of these complaints as well as demonstrate an improvement in their appearance.

153.) A client is diagnosed with pulmonary embolism and is to be treated with streptokinase (Streptase). A nurse would report which priority data collection finding to the registered nurse before initiating this therapy? 1. Adventitious breath sounds 2. Temperature of 99.4° F orally 3. Blood pressure of 198/110 mm Hg 4. Respiratory rate of 28 breaths/min

3. Blood pressure of 198/110 mm Hg Rationale: Thrombolytic therapy is contraindicated in a number of preexisting conditions in which there is a risk of uncontrolled bleeding, similar to the case in anticoagulant therapy. Thrombolytic therapy also is contraindicated in severe uncontrolled hypertension because of the risk of cerebral hemorrhage. Therefore the nurse would report the results of the blood pressure to the registered nurse before initiating therapy. The findings in options 1, 2, and 4 may be present in the client with pulmonary embolism.

71.) After kidney transplantation, cyclosporine (Sand immune) is prescribed for a client. Which laboratory result would indicate an adverse effect from the use of this medication? 1. Decreased creatinine level 2. Decreased hemoglobin level 3. Elevated blood urea nitrogen level 4. Decreased white blood cell count

3. Elevated blood urea nitrogen level Rationale: Nephrotoxicity can occur from the use of cyclosporine (Sandimmune). Nephrotoxicity is evaluated by monitoring for elevated blood urea nitrogen (BUN) and serum creatinine levels. Cyclosporine is an immunosuppressant but does not depress the bone marrow.

60.) A nurse is planning to administer hydrochlorothiazide (HydroDIURIL) to a client. The nurse understands that which of the following are concerns related to the administration of this medication? 1. Hypouricemia, hyperkalemia 2. Increased risk of osteoporosis 3. Hypokalemia, hyperglycemia, sulfa allergy 4. Hyperkalemia, hypoglycemia, penicillin allergy

3. Hypokalemia, hyperglycemia, sulfa allergy Rationale: Thiazide diuretics such as hydrochlorothiazide are sulfa-based medications, and a client with a sulfa allergy is at risk for an allergic reaction. Also, clients are at risk for hypokalemia, hyperglycemia, hypercalcemia, hyperlipidemia, and hyperuricemia.

130.) The nurse is analyzing the laboratory results of a client with leukemia who has received a regimen of chemotherapy. Which laboratory value would the nurse specifically note as a result of the massive cell destruction that occurred from the chemotherapy? 1. Anemia 2. Decreased platelets 3. Increased uric acid level 4. Decreased leukocyte count

3. Increased uric acid level Rationale: Hyperuricemia is especially common following treatment for leukemias and lymphomas because chemotherapy results in a massive cell kill. Although options 1, 2, and 4 also may be noted, an increased uric acid level is related specifically to cell destruction.

5.) Mafenide acetate (Sulfamylon) is prescribed for the client with a burn injury. When applying the medication, the client complains of local discomfort and burning. Which of the following is the most appropriate nursing action? 1. Notifying the registered nurse 2. Discontinuing the medication 3. Informing the client that this is normal 4. Applying a thinner film than prescribed to the burn site

3. Informing the client that this is normal Rationale: Mafenide acetate is bacteriostatic for gram-negative and gram-positive organisms and is used to treat burns to reduce bacteria present in avascular tissues. The client should be informed that the medication will cause local discomfort and burning and that this is a normal reaction; therefore options 1, 2, and 4 are incorrect

52.) A client with tuberculosis is being started on antituberculosis therapy with isoniazid (INH). Before giving the client the first dose, a nurse ensures that which of the following baseline studies has been completed? 1. Electrolyte levels 2. Coagulation times 3. Liver enzyme levels 4. Serum creatinine level

3. Liver enzyme levels Rationale: INH therapy can cause an elevation of hepatic enzyme levels and hepatitis. Therefore, liver enzyme levels are monitored when therapy is initiated and during the first 3 months of therapy. They may be monitored longer in the client who is greater than age 50 or abuses alcohol.

210.) Dantrolene (Dantrium) is prescribed for a client with a spinal cord injury for discomfort resulting from spasticity. The nurse tells the client about the importance of follow-up and the need for which blood study? 1. Creatinine level 2. Sedimentation rate 3. Liver function studies 4. White blood cell count

3. Liver function studies Rationale: Dantrolene can cause liver damage, and the nurse should monitor liver function studies. Baseline liver function studies are done before therapy starts, and regular liver function studies are performed throughout therapy. Dantrolene is discontinued if no relief of spasticity is achieved in 6 weeks.

59.) A client is diagnosed with an acute myocardial infarction and is receiving tissue plasminogen activator, alteplase (Activase, tPA). Which action is a priority nursing intervention? 1. Monitor for renal failure. 2. Monitor psychosocial status. 3. Monitor for signs of bleeding. 4. Have heparin sodium available.

3. Monitor for signs of bleeding. Rationale: Tissue plasminogen activator is a thrombolytic. Hemorrhage is a complication of any type of thrombolytic medication. The client is monitored for bleeding. Monitoring for renal failure and monitoring the client's psychosocial status are important but are not the most critical interventions. Heparin is given after thrombolytic therapy, but the question is not asking about follow-up medications.

151.) A client is being treated for acute congestive heart failure with intravenously administered bumetanide. The vital signs are as follows: blood pressure, 100/60 mm Hg; pulse, 96 beats/min; and respirations, 24 breaths/min. After the initial dose, which of the following is the priority assessment? 1. Monitoring weight loss 2. Monitoring temperature 3. Monitoring blood pressure 4. Monitoring potassium level

3. Monitoring blood pressure Rationale: Bumetanide is a loop diuretic. Hypotension is a common side effect associated with the use of this medication. The other options also require assessment but are not the priority. **priority ABCs—airway, breathing, and circulation**

201.) A nurse is preparing to administer eardrops to an infant. The nurse plans to: 1. Pull up and back on the ear and direct the solution onto the eardrum. 2. Pull down and back on the ear and direct the solution onto the eardrum. 3. Pull down and back on the ear and direct the solution toward the wall of the canal. 4. Pull up and back on the ear lobe and direct the solution toward the wall of the canal.

3. Pull down and back on the ear and direct the solution toward the wall of the canal. Rationale: When administering eardrops to an infant, the nurse pulls the ear down and straight back. In the adult or a child older than 3 years, the ear is pulled up and back to straighten the auditory canal. The medication is administered by aiming it at the wall of the canal rather than directly onto the eardrum.

137.) A nurse is reinforcing instructions for a client regarding intranasal desmopressin acetate (DDAVP). The nurse tells the client that which of the following is a side effect of the medication? 1. Headache 2. Vulval pain 3. Runny nose 4. Flushed skin

3. Runny nose Rationale: Desmopressin administered by the intranasal route can cause a runny or stuffy nose. Headache, vulval pain, and flushed skin are side effects if the medication is administered by the intravenous (IV) route.

98.) The nurse is assigned to care for a client with cytomegalovirus retinitis and acquired immunodeficiency syndrome who is receiving foscarnet. The nurse should check the latest results of which of the following laboratory studies while the client is taking this medication? 1. CD4 cell count 2. Serum albumin 3. Serum creatinine 4. Lymphocyte count

3. Serum creatinine Rationale: Foscarnet is toxic to the kidneys. Serum creatinine is monitored before therapy, two to three times per week during induction therapy, and at least weekly during maintenance therapy. Foscarnet may also cause decreased levels of calcium, magnesium, phosphorus, and potassium. Thus these levels are also measured with the same frequency.

211.) A client with epilepsy is taking the prescribed dose of phenytoin (Dilantin) to control seizures. A phenytoin blood level is drawn, and the results reveal a level of 35 mcg/ml. Which of the following symptoms would be expected as a result of this laboratory result? 1. Nystagmus 2. Tachycardia 3. Slurred speech 4. No symptoms, because this is a normal therapeutic level

3. Slurred speech Rationale: The therapeutic phenytoin level is 10 to 20 mcg/mL. At a level higher than 20 mcg/mL, involuntary movements of the eyeballs (nystagmus) appear. At a level higher than 30 mcg/mL, ataxia and slurred speech occur.

245.) A client taking carbamazepine (Tegretol) asks the nurse what to do if he misses one dose. The nurse responds that the carbamazepine should be: 1. Withheld until the next scheduled dose 2. Withheld and the health care provider is notified immediately 3. Taken as long as it is not immediately before the next dose 4. Withheld until the next scheduled dose, which should then be doubled

3. Taken as long as it is not immediately before the next dose Rationale: Carbamazepine is an anticonvulsant that should be taken around the clock, precisely as directed. If a dose is omitted, the client should take the dose as soon as it is remembered, as long as it is not immediately before the next dose. The medication should not be double dosed. If more than one dose is omitted, the client should call the health care provider.

129.) Megestrol acetate (Megace), an antineoplastic medication, is prescribed for the client with metastatic endometrial carcinoma. The nurse reviews the client's history and contacts the registered nurse if which diagnosis is documented in the client's history? 1. Gout 2. Asthma 3. Thrombophlebitis 4. Myocardial infarction

3. Thrombophlebitis Rationale: Megestrol acetate (Megace) suppresses the release of luteinizing hormone from the anterior pituitary by inhibiting pituitary function and regressing tumor size. Megestrol is used with caution if the client has a history of thrombophlebitis. **megestrol acetate is a hormonal antagonist enzyme and that a side effect is thrombotic disorders**

22.) A nurse is caring for a client after thyroidectomy and notes that calcium gluconate is prescribed for the client. The nurse determines that this medication has been prescribed to: 1. Treat thyroid storm. 2. Prevent cardiac irritability. 3. Treat hypocalcemic tetany. 4. Stimulate the release of parathyroid hormone.

3. Treat hypocalcemic tetany. Rationale: Hypocalcemia can develop after thyroidectomy if the parathyroid glands are accidentally removed or injured during surgery. Manifestations develop 1 to 7 days after surgery. If the client develops numbness and tingling around the mouth, fingertips, or toes or muscle spasms or twitching, the health care provider is notified immediately. Calcium gluconate should be kept at the bedside.

27.) Sildenafil (Viagra) is prescribed to treat a client with erectile dysfunction. A nurse reviews the client's medical record and would question the prescription if which of the following is noted in the client's history? 1. Neuralgia 2. Insomnia 3. Use of nitroglycerin 4. Use of multivitamins

3. Use of nitroglycerin Rationale: Sildenafil (Viagra) enhances the vasodilating effect of nitric oxide in the corpus cavernosum of the penis, thus sustaining an erection. Because of the effect of the medication, it is contraindicated with concurrent use of organic nitrates and nitroglycerin. Sildenafil is not contraindicated with the use of vitamins. Neuralgia and insomnia are side effects of the medication.

81.) A client with trigeminal neuralgia is being treated with carbamazepine (Tegretol). Which laboratory result would indicate that the client is experiencing an adverse reaction to the medication? 1. Sodium level, 140 mEq/L 2. Uric acid level, 5.0 mg/dL 3. White blood cell count, 3000 cells/mm3 4. Blood urea nitrogen (BUN) level, 15 mg/dL

3. White blood cell count, 3000 cells/mm3 Rationale: Adverse effects of carbamazepine (Tegretol) appear as blood dyscrasias, including aplastic anemia, agranulocytosis, thrombocytopenia, leukopenia, cardiovascular disturbances, thrombophlebitis, dysrhythmias, and dermatological effects. Options 1, 2, and 4 identify normal laboratory values.

If the serum creatinine level rises above ____ mg/dL, the dose of Amphotericin B (Fungizone) should be reduced

3.5 mg/dL

Usual dose of Ibuprofen is

300-800mg per dose

1 soda (12 oz) to ml

360 ml

62.) A client is on nicotinic acid (niacin) for hyperlipidemia and the nurse provides instructions to the client about the medication. Which statement by the client would indicate an understanding of the instructions? 1. "It is not necessary to avoid the use of alcohol." 2. "The medication should be taken with meals to decrease flushing." 3. "Clay-colored stools are a common side effect and should not be of concern." 4. "Ibuprofen (Motrin) taken 30 minutes before the nicotinic acid should decrease the flushing."

4. "Ibuprofen (Motrin) taken 30 minutes before the nicotinic acid should decrease the flushing." Rationale: Flushing is a side effect of this medication. Aspirin or a nonsteroidal anti-inflammatory drug can be taken 30 minutes before taking the medication to decrease flushing. Alcohol consumption needs to be avoided because it will enhance this side effect. The medication should be taken with meals, this will decrease gastrointestinal upset. Taking the medication with meals has no effect on the flushing. Clay-colored stools are a sign of hepatic dysfunction and should be immediately reported to the health care provider (HCP).

218.) A film-coated form of diflunisal has been prescribed for a client for the treatment of chronic rheumatoid arthritis. The client calls the clinic nurse because of difficulty swallowing the tablets. Which initial instruction should the nurse provide to the client? 1. "Crush the tablets and mix them with food." 2. "Notify the health care provider for a medication change." 3. "Open the tablet and mix the contents with food." 4. "Swallow the tablets with large amounts of water or milk."

4. "Swallow the tablets with large amounts of water or milk." Rationale: Diflunisal may be given with water, milk, or meals. The tablets should not be crushed or broken open. Taking the medication with a large amount of water or milk should be tried before contacting the health care provider.

75.) A client with myasthenia gravis becomes increasingly weak. The health care provider prepares to identify whether the client is reacting to an overdose of the medication (cholinergic crisis) or increasing severity of the disease (myasthenic crisis). An injection of edrophonium (Enlon) is administered. Which of the following indicates that the client is in cholinergic crisis? 1. No change in the condition 2. Complaints of muscle spasms 3. An improvement of the weakness 4. A temporary worsening of the condition

4. A temporary worsening of the condition Rationale: An edrophonium (Enlon) injection, a cholinergic drug, makes the client in cholinergic crisis temporarily worse. This is known as a negative test. An improvement of weakness would occur if the client were experiencing myasthenia gravis. Options 1 and 2 would not occur in either crisis.

4.) The camp nurse asks the children preparing to swim in the lake if they have applied sunscreen. The nurse reminds the children that chemical sunscreens are most effective when applied: 1. Immediately before swimming 2. 15 minutes before exposure to the sun 3. Immediately before exposure to the sun 4. At least 30 minutes before exposure to the sun

4. At least 30 minutes before exposure to the sun Rationale: Sunscreens are most effective when applied at least 30 minutes before exposure to the sun so that they can penetrate the skin. All sunscreens should be reapplied after swimming or sweating.

4. The nurse is administering Augmentin to her patient with a sinus infection. Which is the best way for her to insure that she is giving it to the right patient? a. Call the patient by name b. Read the name of the patient on the patient's door c. Check the patient's wristband d. Check the patient's room number on the unit census list

4. C. The correct way to identify a patient before giving a medication is to check the name on the medication administration record with the patient's identification band. The nurse should also ask the patient to state their name. The name on the door or the census list are not sufficient proof of identification. Calling the patient by name is not as effective as having the patient state their name patients may not hear well or understand what the nurse is saying, and may respond to a name which is not their own.

19.) Tamoxifen is prescribed for the client with metastatic breast carcinoma. The nurse understands that the primary action of this medication is to: 1. Increase DNA and RNA synthesis. 2. Promote the biosynthesis of nucleic acids. 3. Increase estrogen concentration and estrogen response. 4. Compete with estradiol for binding to estrogen in tissues containing high concentrations of receptors.

4. Compete with estradiol for binding to estrogen in tissues containing high concentrations of receptors. Rationale: Tamoxifen is an antineoplastic medication that competes with estradiol for binding to estrogen in tissues containing high concentrations of receptors. Tamoxifen is used to treat metastatic breast carcinoma in women and men. Tamoxifen is also effective in delaying the recurrence of cancer following mastectomy. Tamoxifen reduces DNA synthesis and estrogen response.

94.) The client with acquired immunodeficiency syndrome has begun therapy with zidovudine (Retrovir, Azidothymidine, AZT, ZDV). The nurse carefully monitors which of the following laboratory results during treatment with this medication? 1. Blood culture 2. Blood glucose level 3. Blood urea nitrogen 4. Complete blood count

4. Complete blood count Rationale: A common side effect of therapy with zidovudine is leukopenia and anemia. The nurse monitors the complete blood count results for these changes. Options 1, 2, and 3 are unrelated to the use of this medication.

124.) A client with chronic renal failure is receiving ferrous sulfate (Feosol). The nurse monitors the client for which common side effect associated with this medication? 1. Diarrhea 2. Weakness 3. Headache 4. Constipation

4. Constipation Rationale: Feosol is an iron supplement used to treat anemia. Constipation is a frequent and uncomfortable side effect associated with the administration of oral iron supplements. Stool softeners are often prescribed to prevent constipation. **Focus on the name of the medication. Recalling that oral iron can cause constipation will easily direct you to the correct option.**

190.) A child is hospitalized with a diagnosis of lead poisoning. The nurse assisting in caring for the child would prepare to assist in administering which of the following medications? 1. Activated charcoal 2. Sodium bicarbonate 3. Syrup of ipecac syrup 4. Dimercaprol (BAL in Oil)

4. Dimercaprol (BAL in Oil) Rationale: Dimercaprol is a chelating agent that is administered to remove lead from the circulating blood and from some tissues and organs for excretion in the urine. Sodium bicarbonate may be used in salicylate poisoning. Syrup of ipecac is used in the hospital setting in poisonings to induce vomiting. Activated charcoal is used to decrease absorption in certain poisoning situations. Note that dimercaprol is prepared with peanut oil, and hence should be avoided by clients with known or suspected peanut allergy.

203.) A nurse is preparing to give the postcraniotomy client medication for incisional pain. The family asks the nurse why the client is receiving codeine sulfate and not "something stronger." In formulating a response, the nurse incorporates the understanding that codeine: 1. Is one of the strongest opioid analgesics available 2. Cannot lead to physical or psychological dependence 3. Does not cause gastrointestinal upset or constipation as do other opioids 4. Does not alter respirations or mask neurological signs as do other opioids

4. Does not alter respirations or mask neurological signs as do other opioids Rationale: Codeine sulfate is the opioid analgesic often used for clients after craniotomy. It is frequently combined with a nonopioid analgesic such as acetaminophen for added effect. It does not alter the respiratory rate or mask neurological signs as do other opioids. Side effects of codeine include gastrointestinal upset and constipation. The medication can lead to physical and psychological dependence with chronic use. It is not the strongest opioid analgesic available.

223.) A client with a psychotic disorder is being treated with haloperidol (Haldol). Which of the following would indicate the presence of a toxic effect of this medication? 1. Nausea 2. Hypotension 3. Blurred vision 4. Excessive salivation

4. Excessive salivation Rationale: Toxic effects include extrapyramidal symptoms (EPS) noted as marked drowsiness and lethargy, excessive salivation, and a fixed stare. Akathisia, acute dystonias, and tardive dyskinesia are also signs of toxicity. Hypotension, nausea, and blurred vision are occasional side effects.

231.) A client admitted to the hospital gives the nurse a bottle of clomipramine (Anafranil). The nurse notes that the medication has not been taken by the client in 2 months. What behaviors observed in the client would validate noncompliance with this medication? 1. Complaints of hunger 2. Complaints of insomnia 3. A pulse rate less than 60 beats per minute 4. Frequent handwashing with hot, soapy water

4. Frequent handwashing with hot, soapy water Rationale: Clomipramine is commonly used in the treatment of obsessive-compulsive disorder. Handwashing is a common obsessive-compulsive behavior. Weight gain is a common side effect of this medication. Tachycardia and sedation are side effects. Insomnia may occur but is seldom a side effect.

196.) A client has been prescribed amikacin (Amikin). Which of the following priority baseline functions should be monitored? 1. Apical pulse 2. Liver function 3. Blood pressure 4. Hearing acuity

4. Hearing acuity Rationale: Amikacin (Amikin) is an antibiotic. This medication can cause ototoxicity and nephrotoxicity; therefore, hearing acuity tests and kidney function studies should be performed before the initiation of therapy. Apical pulse, liver function studies, and blood pressure are not specifically related to the use of this medication.

212.) Mannitol (Osmitrol) is being administered to a client with increased intracranial pressure following a head injury. The nurse assisting in caring for the client knows that which of the following indicates the therapeutic action of this medication? 1. Prevents the filtration of sodium and water through the kidneys 2. Prevents the filtration of sodium and potassium through the kidneys 3. Decreases water loss by promoting the reabsorption of sodium and water in the loop of Henle 4. Induces diuresis by raising the osmotic pressure of glomerular filtrate, thereby inhibiting tubular reabsorption of water and solutes

4. Induces diuresis by raising the osmotic pressure of glomerular filtrate, thereby inhibiting tubular reabsorption of water and solutes Rationale: Mannitol is an osmotic diuretic that induces diuresis by raising the osmotic pressure of glomerular filtrate, thereby inhibiting tubular reabsorption of water and solutes. It is used to reduce intracranial pressure in the client with head trauma.

36.) The client has a PRN prescription for ondansetron (Zofran). For which condition should this medication be administered to the postoperative client? 1. Paralytic ileus 2. Incisional pain 3. Urinary retention 4. Nausea and vomiting

4. Nausea and vomiting Rationale: Ondansetron is an antiemetic used to treat postoperative nausea and vomiting, as well as nausea and vomiting associated with chemotherapy. The other options are incorrect.

144.) A client is taking docusate sodium (Colace). The nurse monitors which of the following to determine whether the client is having a therapeutic effect from this medication? 1. Abdominal pain 2. Reduction in steatorrhea 3. Hematest-negative stools 4. Regular bowel movements

4. Regular bowel movements Rationale: Docusate sodium is a stool softener that promotes the absorption of water into the stool, producing a softer consistency of stool. The intended effect is relief or prevention of constipation. The medication does not relieve abdominal pain, stop gastrointestinal (GI) bleeding, or decrease the amount of fat in the stools.

177.) A nurse is assisting in caring for a pregnant client who is receiving intravenous magnesium sulfate for the management of preeclampsia and notes that the client's deep tendon reflexes are absent. On the basis of this data, the nurse reports the finding and makes which determination? 1. The magnesium sulfate is effective. 2. The infusion rate needs to be increased. 3. The client is experiencing cerebral edema. 4. The client is experiencing magnesium toxicity.

4. The client is experiencing magnesium toxicity. Rationale: Magnesium toxicity can occur as a result of magnesium sulfate therapy. Signs of magnesium sulfate toxicity relate to the central nervous system depressant effects of the medication and include respiratory depression; loss of deep tendon reflexes; sudden decrease in fetal heart rate or maternal heart rate, or both; and sudden drop in blood pressure. Hyperreflexia indicates increased cerebral edema. An absence of reflexes indicates magnesium toxicity. The therapeutic serum level of magnesium for a client receiving magnesium sulfate ranges from 4 to 7.5 mEq/L (5 to 8 mg/dL).

The nurse's understanding of the clotting mechanism is important in administering anticoagulant drugs. the nurse understands that which of the following clotting factors are formed after injury to the vessels? 1. Fibrin, vitamin K 2. Thromboplastin, fibrinogen 3. Prothrombin, thrombin 4. Thrombin, fibrin

4. Thrombin, fibrin

160.) Meperidine hydrochloride (Demerol) is prescribed for the client with pain. Which of the following would the nurse monitor for as a side effect of this medication? 1. Diarrhea 2. Bradycardia 3. Hypertension 4. Urinary retention

4. Urinary retention Rationale: Meperidine hydrochloride (Demerol) is an opioid analgesic. Side effects of this medication include respiratory depression, orthostatic hypotension, tachycardia, drowsiness and mental clouding, constipation, and urinary retention.

140.) The client has a new prescription for metoclopramide (Reglan). On review of the chart, the nurse identifies that this medication can be safely administered with which condition? 1. Intestinal obstruction 2. Peptic ulcer with melena 3. Diverticulitis with perforation 4. Vomiting following cancer chemotherapy

4. Vomiting following cancer chemotherapy Rationale: Metoclopramide is a gastrointestinal (GI) stimulant and antiemetic. Because it is a GI stimulant, it is contraindicated with GI obstruction, hemorrhage, or perforation. It is used in the treatment of emesis after surgery, chemotherapy, and radiation.

180.) A health care provider (HCP) writes a prescription for digoxin (Lanoxin), 0.25 mg daily. The nurse teaches the client about the medication and tells the client that it is important to: 1. Count the radial and carotid pulses every morning. 2. Check the blood pressure every morning and evening. 3. Stop taking the medication if the pulse is higher than 100 beats per minute. 4. Withhold the medication and call the HCP if the pulse is less than 60 beats per minute.

4. Withhold the medication and call the HCP if the pulse is less than 60 beats per minute. Rationale: An important component of taking this medication is monitoring the pulse rate; however, it is not necessary for the client to take both the radial and carotid pulses. It is not necessary for the client to check the blood pressure every morning and evening because the medication does not directly affect blood pressure. It is most important for the client to know the guidelines related to withholding the medication and calling the HCP. The client should not stop taking a medication.

The nurse needs to know that codeine in cough syrup belongs to which schedule

5 Codeine in cough meds 3-codiene alone

If glucocorticoid is ordered with a broncho-dilator you need to wait how many min b/w admin medications

5 min

Nitrofurantoin (macrodantin) given for acute UTI , what is the adult dosage

50 mg every 6 hours

IV admin of phenytoin (Dilantin) is performed slowly and no faster than

50 mg/min **bc rapid admin can cause cardiovascular collapse

Insulin: Intermediate Acting Peak

6-12h

1 gr to mg

60 mg

The dosage of pyridostigmine bromide-mestinon is 600mg/day.How many tablets will the pt take if they are 60mg each.

600mg/60mg= 10 tablets

1 quart to ml

960 ml

Which of these characteristics should a nurse associate with a patient who has type 2 diabetes? (Select all that apply.) A) Exercise and diet may be sufficient treatment B) Is often obese with difficulty managing weight C) Prone to ketosis and ketoacidosis complications D) Genetics and strong familial links are causal factors E) Insulin resistance and inappropriate secretion

A B D E

c. Hypotension

A calcium channel blocker has been ordered for a client. Which condition in the client's history is a contraindication to this medication? a. Hypokalemia b. Dysrhythmias c. Hypotension d. Increased intracranial pressure

b. Administer protamine sulfate.

A client has been admitted through the emergency department and requires emergency surgery. The client has been receiving heparin. What nursing intervention is essential? a. Teach the client about the phenytoin. b. Administer protamine sulfate. c. Assess the INR before surgery. d. Administer vitamin K.

c. High-ceiling (loop) diuretic

A client has heart failure and is prescribed Lasix. The nurse is aware that furosemide (Lasix) is what kind of drug? a. Thiazide diuretic b. Osmotic diuretic c. High-ceiling (loop) diuretic d. Potassium-sparing diuretic

c. "I will increase dark-green, leafy vegetables in my diet."

A client is receiving warfarin (Coumadin) for a chronic condition. Which client statement requires immediate action by the nurse? a. "I will avoid contact sports." b. "I will take my medication in the early evening each day." c. "I will increase dark-green, leafy vegetables in my diet." d. "I will contact my health care provider if I develop excessive bruising."

c. Fish

A client taking spironolactone (Aldactone) has been taught about the therapy. Which menu selection indicates that the client understands teaching related to this medication? a. Apricots b. Bananas c. Fish d. Strawberries

c. Lungs clear.

A client with acute pulmonary edema receives furosemide (Lasix). What assessment finding indicates that the intervention is working? a. Potassium level decreased from 4.5 to 3.5 mEq/L. b. Crackles auscultated in the bases. c. Lungs clear. d. Output 30 mL/hr.

What is a delusion? Ex?

A false fixed belief. Ex: I am dating Madonna and she's waiting for me in a hotel room!

What is clang association? Ex?

A meaningless use of rhyming words. Ex: I read the bed and said the head then led Ted to the dead.

What is word salad? Ex?

A mix of words or phrases that have no meaning to the listener. Ex: Animals, fast, food, family, working, birds, loving, the net.

b. 0.5 to 2.0 ng/mL

A newly admitted client takes digoxin 0.25 mg/day. The nurse knows that which is the serum therapeutic range for digoxin? a. 0.1 to 1.5 ng/mL b. 0.5 to 2.0 ng/mL c. 1.0 to 2.5 ng/mL d. 2.0 to 4.0 ng/mL

A month or longer

A patient experiencing moderate depression is placed on sertraline (Zoloft). The nurse should counsel the patient to expect full effects from the drug in: a. 2-3 days b. 1 week c. A month or longer d. Within 24 hours after starting the drug

What is a neologism? Ex?

A word or phrase the patient constructs that's meaningless to everyone else. Ex: I bessaton the coaglese and vergified the rest.

A client in the trauma ICU is experiencing deep, throbbing pain. The nurse will provide medication for this pain because: A.) The pain is being transmitted over C fibers and the enkephalins will not be effective to control the pain. B.) The pain is being transmitted over A beta fibers and beta-endorphins will not be effective to control the pain. c.) The pain is being transmitted over delta fibers and dynorphins will not be effective to control the pain. d.) The pain is being transmitted over A beta fibers and the dynophins will not be effective to control the pain.

A.) The pain is being transmitted over C fibers and the enkephalins will not be effective to control the pain.

Cl taking bethanechol should report which of the following to the health care provider SELECT ALL THAT APPLY

ABDOMINAL DISCOMFORT INCREASED SALIVATION URGENCY ABDOMINAL CRAMPS Incorrect-HEADACHE

Attention Disorder Agents Uses

ADD, narcolepsy

-"sartan"

ARBs Angiotensin II receptor Blockers CHF HTN blocks aldosterone avoid suanas, hot tubs take at same time everyday

What are the guidelines for maintaining a therapeutic relationship with a schizophrenic patient?

Accept them as they are, monitor of hallucination/ delusions, assess their safety, give appropriate feedback on how interpret communication, SET LIMITS, and encourage them to learn about their mental illness.

Which of the following drugs would the RN MOST LIKELY deliver to a child w/ an elevated temperature

Acetaminophen (asprin is toxic to children under the age of 8)

Atypical Antipsychotic Agents Uses

Acute and chronic psychoses

Nursing Stratagies r/t administering NSAIDS include ALL of the following EXCEPT

Advising the Cl alcohol can be taken w/ NSAIDs

Drug interactions occur with cephalosporins and which of the following

Alcohol

Antacids

Aluminum - Constipation Magnesium - Diarrhea

suicidal ideation

Always ask pt about this before giving antidepressant. Children at greatest risk for this.

which drug resource is published annually and updated monthly?

American Hospital Formulary

Bronchodilators Med Names

Aminophylline, Brethine, Atrovent, Proventil, Primatene mist

Antifungals: Med Names

Amphotericin B, Nystatin

After a patient is discharged following an alcohol withdrawal episode and wants to stop drinking what is a good drug for him/her?

Antabuse- must be committed to not drinking or get severely sick. After the drug is d/c it is in the body for another 2 weeks.

Which of the following is NOT a property of anti-inflammatory agents

Anti-Hypertensive

Dilantin

Anti-convolsant Therapeutic level 10-20 (same as amminophylline) Toxicity > 20 Causes hyperplasia of gums - bleeding gums Give extra VIT. D

Primary ingredient in OTC drugs

Anti-histamines

Furadantin (nitrofurantoin) Nursing Considerations

Anti-infective Give w food or milk Monitor pulmonary status Take w/ cranberry juice

Psychomotor symptoms

Anticholinergic effects are common adverse effects of antidepressants such as imipramine (Tofranil). These effects may include: a. Psychomotor symptoms b. Tachycardia, hypertension, and increase in respiratory rate c. Tardive dyskinesias d. Blurred vision, dry mouth, and constipation

The health teach plan for a pt taking Tetracycline would NOT include which of the following?

Anticipate Urinary Urgency

SSRIs

Antidepressant drugs such as the ____ may not have full effects for a month or longer but some improvement in mood and depression should be noticeable after beginning therapy. DRUG OF CHOICE for depression.

Amphotericin B

Antifungal IVPB SLOW - 2+ hrs Hydrate pt - very toxic to kidneys, monitor BUN. Check Hearing Check K+ levels.

Acyclovir is effective against herpes virus was 1st introduced as an:

Antineoplastic

Antidepressants: SSRIs Side Effects

Anxiety, gi upset, change in appetite and bowel function, urinary retention

african americans

Are less responsive to Beta Blockers- Genetics

Antiplatelet Agents Med Names

Aspirin, Persantine, Plavix

Anticholinergics

Atropine GI - Slows motility, spasm Eyes - Dilates pupils *DO NOT GIVE TO GLAUCOME PTS* Heart - Increase HR Resp - bronchodilator (Atrovent)

The nurse is caring for a patient whose seizures are characterized by a 10- to 30-second loss of consciousness with mild symmetric eye blinking. Which seizure type does this most closely illustrate? A) Tonic-clonic B) Absence C) Atonic D) Myoclonic

B) Absence This scenario accurately describes absence seizures. Tonic-clonic seizures present with convulsions and muscle rigidity followed by muscle jerks. Patients may experience urinary incontinence and loss of consciousness. Atonic seizures cause sudden loss of muscle tone. Myoclonic seizures present with sudden muscle contractions that last but a second.

A patient admitted to the hospital with a diagnosis of pneumonia asks the nurse, "Why am I receiving codeine? I don't have any pain." The nurse's response is based on the knowledge that codeine also has which effect? A) Immunostimulant B) Antitussive C) Expectorant D) Immunosuppressant

B) Antitussive Codeine provides both analgesic and antitussive therapeutic effects.

The nurse is planning care for a patient receiving morphine sulfate (Duramorph) by means of a patient-controlled analgesia (PCA) pump. Which intervention may be required because of a potential adverse effect of this drug? A) Administer cough suppressant. B) Insert Foley catheter. C) Administer antidiarrheal. D) Monitor liver function tests.

B) Insert Foley catheter. Morphine can cause urinary hesitancy and urinary retention. If bladder distention or the inability to void is noted, the prescriber should be notified. Urinary catheterization may be required. Morphine acts as a cough suppressant and an antidiarrheal, so neither of those drugs would need to be administered to counteract an adverse effect of morphine. Liver toxicity is not a common adverse effect of morphine.

A patient taking Dilantin (phenytoin) for a seizure disorder is experiencing breakthrough seizures. A blood sample is taken to determine the serum drug level. Which of the following would indicate a sub-therapeutic level? A. 15 mcg/mL. B. 4 mcg/mL. C. 10 mcg/dL. D. 5 mcg/dL.

B. 4 mcg/mL. The therapeutic serum level for Dilantin is 10 - 20 mcg/mL. A level of 4 mcg/mL is sub-therapeutic and may be caused by patient non-compliance or increased metabolism of the drug. A leve of 15 mcg/mL is therapeutic. Choices C and D are expressed in mcg/dL, which is the incorrect unit of measurement.

Which of the following statements from a newly diagnosed client with diabetes indicates more instruction is needed? A.) i need to check my feet daily for sores B.) i need to store my insulin in the refrigerator C.) i can use my plastic insulin syringe more than once D.) i need to see my doctor for follow up exams

B.) i need to store my insulin in the refrigerator Insulin only needs to be stored in the refrigerator if it wont be used within 6 weeks, after being opened. It should be at room temperature when given to decrease pain and prevent lipodystrophy.

Which of the following changes in parasympathetic nervous system function are produced by the administration of antimuscular drugs? CHOOSE TWO

BRADYCARDIA, BRONCHODILATION

Amphoteracin B should monitor which lab results

BUN

Antitussive Agents Med Names

Benylin DM, Robitussin

Many cultural groups have lactose intolerances, S/S include:

Bloating

-"phylline"

Bronchodilator

Propranolol (Inderal) major adverse reaction?

Bronchospasm

A nurse administers naloxone (Narcan) to a postoperative patient experiencing respiratory sedation. What undesirable effect would the nurse anticipate after giving this medication? A) Drowsiness B) Tics and tremors C) Increased pain D) Nausea and vomiting

C) Increased pain Naloxone is a medication that reverses the effects of narcotics. Although the patient's respiratory status will improve after the administration of naloxone, pain will be more acute.

A patient has just been diagnosed with diabetes mellitus. His doctor has requested glucagon for emergency use at home. The nurse instructs the patient that the purpose of this drug is to treat: A. Hyperglycemia from insufficient insulin injection. B. Hyperglycemia from eating a large meal. C. Hypoglycemia from insulin overdose. D. Lipohypertrophy from inadequate insulin absorption.

C. Hypoglycemia from insulin overdose. Glucagon is for emergency use for insulin overdose. The patient will usually arouse within 20 minutes if unconscious. The family should also be instructed how to use the glucagon injection as well.

Before giving Ticlopidine (Ticlid) what baseline labs does a RN need?

CBC count

Before giving Carbamazepine (Tegretol), look at what lab results first?

CBC count **bc leukopenia, anemia, thrombocytopenia can occur

-"dipine"

Ca Channel blockers HTN angina dysrhythmias

glucagon, CaChl, Ca gluconate

Ca Channel blockers antidote

d. Constant, irritating cough

Captopril (Capoten) has been ordered for a client. The nurse teaches the client that ACE inhibitors have which common side effects? a. Nausea and vomiting b. Dizziness and headaches c. Upset stomach d. Constant, irritating cough

Cytoprotective Agents Med Names

Carafate

The drug of choice for seizure disorders that have not responded to other anticonvulsant drug therapy is

Carbamazepine

Antibiotics: Cephalosporins

Ceph-, Cef, Kef

Preperations that are helpful for loosening wax from the ear canal belong to which group

Ceruminolytics

Antipsychotic Agents Nruisng Considerations

Check CBC Monitor VS Avoid alcohol and caffeine

Antineoplastic: Hormonal Agents Nursing Considerations

Check CBC Monitor serum calcium

Antitubercular Agents Nursing Considerations

Check LFT Vit B6 given for peripheral neuritis Used in combination

Antihistamines: Med Names

Chlor-Trimeton, Benadryl, Phenergan

Antibiotics: Fluoroquinolones Med Names

Cipro

Centrally acting alpha 2 agonist

Clonidine, Guanfacine, Methyldopa works in the brain to reduce peripheral vascular resistance black sore tongue leukopenia take at bedtime

Atypical Antipsychotic Agents Med Names

Clozaril, Risperdal

Antigout Agents: Med Names

Colchicine, Probenecid, Allopurinol

Laxatives/Stool Softeners Nursing Considerations

Contraindicated w symptoms of acute abd Monitor fluid and electrolytes Chronic uses may cause hypokalemia Encourage fluid

Narcan

Counteracts Morphine - opioid narcotics.

SNRIs

Cymbalta caution with children and adolescents may cause suicidality decrease libido

18. A nurse is administering blood to a patient who has a low hemoglobin count. The patient asks how long to RBC's last in my body? The correct response is. A. The life span of RBC is 45 days. B. The life span of RBC is 60 days. C. The life span of RBC is 90 days. D. The life span of RBC is 120 days.

D. The life span of RBC is 120 days.

A client in the ICU who sustained a traumatic abdominal injury 1 week ago continues to complain of severe pain. The nurse notes his vital signs are normal. Which of the following would be appropriate for the nurse to do? a.) Encourage early return to ambulation. b.) Offer nonnarcotic analgesics for pain. c.) Utilize distraction d.) Provide the client with pain medication.

D.) Provide the client with pain medication.

Atypical Antipsychotics have a stronger affinity to

D4 Receptor, They block serotonin receptors

Which of the following requires intervention by the nurse caring for a client receiving Phenylephrine (Neosynephrine)?

Decreased urinary output

Antidiabetic Agents: Med Names

Diabinese, Orinase, Dymelor, Micronase

Carbonic Anhydrase Inhibitors Med Names

Diamox

you observe that JA is reciving adequate nutrition.Phenytoin may cause all of the following EXCEPT

Diaphoresis

Digoxin Antidote

Digibind

chloroquine increases effects of which of the following drugs

Digoxin Anti co Ags Neuormuscular Blocks

Anticonvulsants Med Names

Dilantin, Luminal, Depakote, Tegretol, Klonopin

Neuro problems r/t antihelmintics include which of the following

Dizziness headache weakness drowsiness

Which of the following symptoms would the RN expect in a patient experiencing salicylism

Dizziness,Tinnitus,Mental Confusion

Dilantin - Anticonvulsant

Don't give too fast - cardiac arrest NEVER mix with another med Urine Pink

Typical or traditional anti-pyschotics may cause extrpyramidal symptoms EPS or pseudoparkinsisms Which of the following symptoms is NOT Caused by EPS

Downward Eye movement

Preffered method to correct a charting error is to:

Draw single line thru error DTI

Sulfa Drugs

Drink lots of fluids Can cause blood dyscrasias - REPORT SORE THROAT, RASH IMMEDIATELY. UTI (Bactrim, sulfisoxazole) Ulcerative colitis (Sulfasalazine)

Which legislation ID those drugs that req a prescription and a new prescription for refills.

Durham Humphrey Amendment

When caring for a pt taking succinamide for seizure control, the nurse monitors the pt for blood dyscrasias. Which of the following symptoms would indicate that the pt may be developing a blood dyscrasia?

Dyscrasia- Abnormal Blood Count Levels Sore throat and General Malaise

With high Carbidopa-levodopa (Sinemet) levels what can occur?

Dyskinesia and impaired voluntary movement

Antidepressants: Tricyclics Med Names

Elavil, Tofranil, Norpamin

Cyclosporine (Sandimmune) given after kidney transplantation needs to be monitored for nephrotoxicity; what labs are monitored for signs of nephrotoxicity?

Elevated blood urea nitrogen Elevated creatinine levels

The drug that may be used as a substitute for penicillin is

Erythromycin

which of the following drugs are considered safer than barbiturates in the elderly?

Estazolam(ProSom) Temazepam(restoril) Triazolam(Halcion) All of which are sleep agents hypnotic/sedatives not barbiturates.

Antitubercular Agents Med Names

First Line -INH, Rifampin, Ethambutol, Streptomycin Second Line - PAS, PZA

What med is used to treat known or suspected ethylene glycol (antifreeze) intoxication?

Fomepizole (Antizol)

b. To increase the serum potassium level

For the client taking a diuretic, a combination such as triamterene and hydrochlorothiazide may be prescribed. The nurse realizes that this combination is ordered for which purpose? a. To decrease the serum potassium level b. To increase the serum potassium level c. To decrease the glucose level d. To increase the glucose level

Allupurinol (Zyloprim)

G - Gulp 3L fluid per day O - (NO) Organ Meats U - Output up to 2 L/day T - Teach

A common side effect of NSAID are

GI distress

most common SE of metformin (Glucophage) is

GI disturbances like - decreased appetite or anorexia -nausea -diarrhea

Antipyretic Side Effects

GI irritation, Liver dysfuction

Bipolar Disorder: Side Effects

GI upset, tremors, polydipsia, polyuria

factors that affect CL physiological responses to medications include

Genetics Diet Age

Which of the following drugs crosses the BBB the LEAST

Glycopyrrolate

Antigout Agents: Uses

Gout

Phenytoin is effective in the treatment of

Grand Mal Seizures

What are typical types of delusions that are less frightening than paranoid delusions?

Grandiose and religious

Abacavir is used to treat

HIV infection

Client recieved spinal anesthesia, what is most important for RN to monitor

HTN,Headaches

When applying medication topically, the Rn should use all of the following EXCEPT what to avoid contact with the medication

Hands

Haloperidol (haldol) is frequently used as an antipyschotic the RN should know that it

Has a sedative effect on agitated combatative persons

What should you do on admission for a person with anorexia nervosa?

Have them sign a contract to eat and retain fluid. Weigh each morning after void.

Pyridium Side Effects

Headache, Vertigo, Changes urine orange

If a patient in the hospital is talking about good memories from their life time what should be done?

Hospice should be called at this time.

most significant adverse effect of Epoetin alfa (Epogen, Procrit) is?

Hypertension

PH were to ingest drugs and or foods that interact which of the following is likely to occur

Hypertensive Crisis

Antithyroid Agents Uses

Hyperthroidism, myxedema

Hypoglycemic Agent: Uses

Hypoglycemia

Which of the following statements best indicates that an immuneocomprimised cl understand self app. of topical drug acyclovir

I need to touch the lesions and the open side of container with finger cot.

When would you be extremely concerned about a person with anorexia nervosa?

IF THEY BEGIN TO PURGE ALSO!

Which schedule accepted medical use

II thru V 2 thru 5

IM - Needle size,Location,Angle of admin

IM 20-22Needle 1-1 1/2 Locations Vastus lateralis Hip(ventrogluteal) deltoid no more 1 ml dorsogluteus Angle 90* or 45* dependant upon Fat content

Advise CL to report rxns to nausea/vomiting to healhcare provider is included in what part RN process

Implementation

instruct pt to aviod caffeine is what part of the RN process

Implementation

In institutions agencies controlled substances must be stored

In locked locations

What teaching intervention is appropriate for CL taking an Ab that causes diarhea secondary to elimination of normal intestinal flora

Include yogurt /buttermilk in diet

Effects of anticholinergic are to SELECT ALL THAT APPLY

Incorrect answers Increase Gi Motility Constrict Pupils CORRECT ANSWERS INCREASE HEART RATE DECREASE SALIVATION DECREASE MUSCLE RIDGITY RELAX BLADDER DETURSOR MUSCLE

Pt returns from surgery with spinal anethesia, to decrease the possibility of spinal headache you should:

Increase fluids Caffeine Position on flat bed

Advice for the CL taking anticholinergic drugs would include which of the following

Increase intake of foods and fluids high in fiber- prevents consitpation

health teaching for this medication include all of the following except:

Increased Caloric Intake

Which of the following is NOT and action of anticholinergics

Increased Gastrointestinal Peristalsis

A CL w benign prostatic hyperplasia BPH is recieving amantadine for influenza A the RN should monitor which of the following S/E

Increased Risk for Urinary retention

Drug- Food Interaction w/ acetaminophen include which of the following

Increased effect w/ caffeine and diflunisal (excedrin and migraines?) Assists with speedy interactions

Nursing care for EQ would include all of the following EXCEPT

Increased fluid intake

Vigabatrin is an anticonvulsant with qualities including which of the following

Inhibits enzyme that destroys GABA Adult dose is 1-4g/d in divided doses Used to treat complex partial seizures ALL OF ABOVE ARE CORRECT

Postpartum Depression

Intense mood changes associated with hormonal changes

NPH (Humalin N)

Intermediate Onset 1-2 hrs Cloudy

What is tardive dyskinesia?

Irreversible/permanent sticking out of the tongue, lip smacking, or leaning tree syndrome.

Which of the following is true regarding naloxone(NARCAN) SELECT ALL THAT APPLY

Is used to treat an overdose Has higher infinity to opiate receptor sites than opiods Is approved for use in neonates to reverse respiratory depression -NOT CORRECT ANSWERS Is only administered IV, May be given every 1 min to max of 10mg

Ibuprofen (motrin,advil,nuprin) is frequently taken anti inflammatory analgesic and atipyretic agent- Which of the following is true about Ibuprofen

It causes less GI upset that other NSAIDS

What is associative looseness? Ex?

It means a persons one thought isn't connected to the next thought. Ex: Blue lights, I go round and round, the grass is green.

When JA begins taking oral medication the nurse should know that the dose will most likely be

LOW- Start low increase as needed until therapuetic levels are reached

depression

Lack of energy, sleep disturbances, abnormal eating patterns ( OVER EATING) Feelings of despair, guilt, hopelessness Most common mental health disorder of elderly adults. Is associated w/ dysfunction of neurotransmitters in the brain connected with cognition and emotion.

Cardiac Glycosides Med Names

Lanoxin (digoxin)

During Interview the admission- you should obtain which following

Last PPD,Chest X ray results

Antianxiety Med Names

Librium, Xanax, Ativan, Vistaril, Equanil

Antidysrhythmics Side Effects

Lightheadedness, hypotension, bradycardia, urnary retention

Drugs that pass rapid the GI include:

Lipid soluble Non ionized

major site for drug metabolism

Liver

when CL taking INH frequent monitoring of which of the following is req.

Liver Enzymes

While taking acetaminophen, pts consume alcohol habitually are monitored by the RN for symptoms of toxicity, which includes:

Liver Tenderness

The following are among 8 groups of NSAIDS w/ which EXCEPTION

Macrolides- their own class

What are the two components of bipolar disorder?

Manic and Depressive

-"crome"

Mast Cells stabilizers anit-inflammatory

Before admin of medication, Rn should check a drug reference to obtain all EXCEPT

Maximum efficacy Therefore all of the following should be examined Protien bind effect Half life theuraputic ranges

Rn teaching client about zolpidem, what is important for Rn to teach

May lead to pyschological dependance

Hypoglycemic Agent: Nursing Considerations

May repeat in 15 min Give carbs orally to prevent secondary hypoglycemic reactions

What 2 ACE inhibitors are given 1 hour before meals?

Moexipril (Univasc) & Captopril (Capoten)

Andrenergics Nursing Considerations

Monitor B/P, Monitor Peripheral Pluses, Check Output

Bronchodilators Nursing Considerations

Monitor BP and HR When used with steroid inhaler, use bronchodilater first May aggravate diabetes

Atypical Antipsychotic Agents Nursing Considerations

Monitor Blood Work Chang position slowly Use sunscreen Monitor VS - Airway

Thyroid Replacement Agents Nursing Consideration

Monitor weight Take in am Monitor pulse and BP Enhances antideppresants and anticouglant stronger Decreases insulin and digoxin

Nursing interventions associated with administering urinary analgesics include all of the following EXCEPT

Monitoring Blood Pressure

Dobutamine (Dobutrex) is used to treat a client experiencing cardiogenic shock. Nursing intervention includes:

Monitoring for cardiac dysrhythmias.

A client with a diagnosis of cancer is receiving epoetin alfa (Epogen, Procrit) as part of the treatment regimen. The nurse evaluates the effectiveness of this drug by:

Monitoring the hematocrit and hemoglobin levels.

The primary care provider allows the pt to keep pilocarpine eye drops at the bedside and to self administer the eye drops four times daily. The nurse:

Must check with the pt to ensure the medication is used properly and at the righy time

Lithium

Must have salt or it will cause retention of this drug? patients should not avoid salt when taking this drug because it reduces its effects. salt is needed.

A pt w/ Myasthenia gravis becomes increasingly weaker and is given Edrophonium (Tensilon) and the symptoms improve.. what crisis is this?

Myasthenic crisis

Should an LPN be assigned to a person experiencing a panic attack?

NO!!!

The Rn assess a CL taking Gentamycin for S/E including:

Nausea Ototoxicity Photosensativity Thrombocytopenia

Antianginals Med Names

Nitro, Isordil

Clomipramine (Anafranil) is a tricyclic antidepressant used for

OCD

Mixed Opiod agonist-antagonists were developed in hope of decreasing

OPIOD ABUSE

CG 7 years old, reciving bactrim for an ear infection NURSING CARE FOR CG INCLUDES THE FOLLOWING SELECT ALL THAT APPLY

Obtain C&S ASSES for hematuria/oliguira monitor intake and output

-"ide"

Oral hypolglycemic

OxyContin

Oxycodone, Narcotic pain reliever, analgesic

What is characteristic of PICA?

PERSISTENT EATING OF NON-NUTRITIVE SUBSTANCES X 1 MONTH, infants (paint, plaster, cloth), older kids (bugs, rocks, sand), adults (chalk, starch, paper). STASH= Stuff is hidden, Things (hair, clay, starch), All non-food items, Sick-don't digest (iron deficiency and impaction are most common), and Have e.coli-ileus.

-"prazole"

PPI suppress gastric acid antiulcer don't give w/ food, 1-2 hrs between meals

Antiparkinson Agents Uses

Parkinson's Disease

Antidepressants: SSRIs Med Names

Paxil, Prozac, Zoloft

What drug might be given to a person experiencing a panic attack? Teaching?

Paxil. The pt should be taught that they shouldnt drive the 1st time they take it, it takes 2 weeks to work, and stress reduction techniques to use until medicine kicks in.

A specific adverse effects from nitrofurantoin is

Peripheral Neuropathy

Trimethadione and succinamides are effective drug therapy for

Petite Mal Seizures

Anxiety Is included in what part of the RN Process

Potential Rn Dx

J.W is taking Ibuprofen Ibuprofen is a rapid acting NSAID that inhibits prostaglandin synthesis the agent drug is classified as an

Propionic Acid Derivative

Client recieve conscious sedation, which drugs should the Rn expect to administer

Propofol(diprivan) Promotes natural sleep Midazolam(Versed) Promotes Sedative rxn. Some type combo of pain med and sedative

what is the antidote for heparin?

Protamine sulfate

Rifampin is being initiated prophylactically for CL who lives w family member who has haemophillus influenza meningitis what point should the RN include in client teaching about this med.

Protect undergarments becouse rifampin will change the urine to Orange red color

A RN instructs the Pt taking aspirin to avoid foods that contain salicylates becouse of the risk of adverse RXNS - which foods should the pt avoid?

Prunes, Tea

Cl taking cloroquin needs to know about which of the following possible side effects

Pruritus Fatigue Anorexia

The Rn closely monitors the CL taking lincosamides for which serious adv. effects.

Pseudomembranous Collitis

TCA antidepressants

Psychomotor symptoms, tachycardia, hypertension, increase in respiratory rate, and tardive dyskinesias are potential adverse effects of ______?

What are the signs of opiate overdose?

RESPIRATORY DEPRESSION --> respiratory arrest, circulatory depression, and unconsciousness-->death

The medication for a child w a virus should be one that prevents

REYES SYNDROME

TPN

RN Resp: check BG every 4hrs high risk for infection dressing change 48-72hrs IV tubing/fluid change every 24hrs substitute with 10% DW

134.) A nurse reinforces instructions to a client who is taking levothyroxine (Synthroid). The nurse tells the client to take the medication: 1. With food 2. At lunchtime 3. On an empty stomach 4. At bedtime with a snack

Rationale: Oral doses of levothyroxine (Synthroid) should be taken on an empty stomach to enhance absorption. Dosing should be done in the morning before breakfast. **Note that options 1, 2, and 4 are comparable or alike in that these options address administering the medication with food.**

What are illusions? Ex?

Real stimuli that has been misinterpreted. Ex: Thinking that someone waving at you is trying to stab you (see person just misinterpret).

What should you do immediately for someone experience a PTSD episode?

Reassure the pt that they are safe before he injures himself, you, or others. First try to orient them to their surroundings. Explain who you are, and tell them you're going to turn the lights on so that they can see they are in a hospital and not a jungle/etc.

The Rn enters CL room to find that pt heart rate 120 BP 70/50 he is flushed. Vacocin running IVPB- the Rn interprets this as severe adv. affects of red man syndrome- which should the Rn do?

Reduce the infusion to 10mg/min vanco- has peak/troph and should not be stopped immediatly!-must be weaned

Advantage of balanced anesthesia

Reduction of drugs to maintain desired state of anesthesia

Insulin: Fast Acting Names

Regular - can be given IV or Pump Humulin R

venlafaxine, Effexor

SNRI. an atypical antidepressant; off labeled use for hot flashes associated with menopause.

-"pram", -"ine"

SSRIs

Many subcategories are listed as atypical antidepressants which of the following groups is NOT classified as an atypical antidepressant

SSRIs

Order to recieve Demoral mg STAT is an example of what kind of order?

STAT order

Who is at risk for suicide as a result of depression? Evaluate risk?

SUBSTANCE ABUSERS, PEOPLE WHO LIVE ALONE, adolescents and elderly, recent crisis, chronic illness, previous attempts, identity crisis, hallucinating (voices telling them to harm themselves). (Use Sad Persons Scale to Evaluate).

Immunosuppressants Med Names

Sandimmune (cyclosporine)

The category for atypical antipsychotics is

Serotonin/Dopamine antagonist

Hypertensive Crisis

Severe Headache Palpitations Diaphoretic Stiff Neck

A Cl dx with Hsv3 Rn understands that illness is better known by which name

Shingles-an adult

It is important for RN to teach client when taking secoarbital(seconal)

Short acting treats insomnia- causes early awakening

Common urinary antiseptic drug-drug interactions include ALL of the following EXCEPT that

Sodium Bicarbonate inhibits the action of methanamine

Another name for local anesthesia is

Sodium Channel Blocker

Glucocorticoids Med Names

Solu-Cortef (hydrocortisone) Decadron, Prednisone

The two herbs that may be used for management of mild depression with health care providers approval are

St Johns Wart, Gingko

Diazepam is the drug of choice for the treatment of

Status Epilepticus

Andrenergics Actions

Stimulates Beta-2 Receptors in Lungs (causes brochdilation)

Zonisamide -Zonegran is contraindicated if the Cl. is allergic/sensitive to which of the following

Sulfonamides

condition that occurs when normal flora is distributed during Ab therapy AKA

Super Infection

Na bicarbonate

TCAs antidote

H2-Receptor Blockers Med Names

Tagamet, Zantac

Antibiotics: Tetracyclines Nursing Consideration

Take 1 h before or 2-3 after meals Do not take antacid, mile, iron SUNBLOCK - avoid sunlight Monitor renal function Note Expiration Date - Toxic after date

drug holiday

The ____ is to decrease the risk of dependence and to evaluate behavior.

c. To block the beta1-adrenergic receptors in the cardiac tissues

The beta blocker acebutolol (Sectral) is prescribed for dysrhythmias. The nurse knows that what is the primary purpose of the drug? a. To increase the beta1 and beta2 receptors in the cardiac tissues b. To increase the flow of oxygen to the cardiac tissues c. To block the beta1-adrenergic receptors in the cardiac tissues d. To block the beta2-adrenergic receptors in the cardiac tissues

a. Administer an additional dose of warfarin (Coumadin).

The client has an international normalized ratio (INR) value of 1.5. What action will the nurse take? a. Administer an additional dose of warfarin (Coumadin). b. Hold the next dose of warfarin (Coumadin). c. Increase the heparin drip rate. d. Administer protamine sulfate.

b. Rhabdomyolysis

The client is taking rosuvastatin (Crestor). What severe skeletal muscle adverse reaction should the nurse observe for? a. Myasthenia gravis b. Rhabdomyolysis c. Dyskinesia d. Agranulocytosis

6. The physician orders an intramuscular injection of Demerol for the post-operative patient's pain. When preparing to draw up the medication, the nurse is careful to remove the correct vial from the narcotics cabinet. It is labeled 1) simethicone. 2) albuterol. 3) meperidine. 4) ibuprofen.

The correct answer is (3). The generic name for Demerol is meperidine.

JA asks how long she will need these medications. The nurses answer is based on the understanding that

The medication are taken for a lifetime

a. Beta1 blocker

The nurse explains that which beta blocker category is preferred for treating hypertension? a. Beta1 blocker b. Beta2 blocker c. Beta1 and beta2 blockers d. Beta2 and beta3 blockers

d. Pulse below 60 beats/min and irregular rate

The nurse is assessing the client for possible evidence of digitalis toxicity. The nurse acknowledges that which is included in the signs and symptoms for digitalis toxicity? a. Pulse (heart) rate of 100 beats/min b. Pulse of 72 with an irregular rate c. Pulse greater than 60 beats/min and irregular rate d. Pulse below 60 beats/min and irregular rate

d. Chest pain

The nurse is monitoring a client during IV nitroglycerin infusion. Which assessment finding will cause the nurse to take action? a. Blood pressure 110/90 mm Hg b. Flushing c. Headache d. Chest pain

c. Bleeding may increase when taken with aspirin.

The nurse is teaching a client about clopidogrel (Plavix). What is important information to include? a. Constipation may occur. b. Hypotension may occur. c. Bleeding may increase when taken with aspirin. d. Normal dose is 25 mg tablet per day.

a. Impaction

The nurse would question an order for cholestyramine (Questran) if the client has which condition? a. Impaction b. Glaucoma c. Hepatic disease d. Renal disease

What are paranoid delusions?

The person thinks that someone is out to harm him/her.

What is characteristic of antisocial behavior?

They are like the troll under the bridge. They are unreliable, self-centered, poor judgement, conflicts with society, LACK OF REMORSE (love em and leave em), MANIPULATIVE, blames others, and splits the staff.

Antiemetics: Med Names

Tigan, Compazine, Reglan, Antivert, Dramine

Soon after admin of neuromuscular agent anesthesia intibates pt what is the purpose

To counteract the paralysis of the diaphragm the result of the meds, to ensure breathing for pt.

Librium and Valume

To prevent DT's in acute alcohol withdrwal Can produce physical dependency.

In excessive doses, acetylchoilesterase inhibitors can paradoxically potentiate a non-depolarizing neurumuscular blockade TRUE OR FALSE

True

An elderly Pt is reciving sulindac- the Rn is aware that older adults taking NSAIDS are at increased risks for:

ULCERS- Disease

Furadantin (nitrofurantoin) Uses

UTIs

Pyridium Uses

UTIs

Insulin: Slow Acting Names

Ultralente Humulin U

Pt complains to his RN about heartburn, nurse notes in the medication profile that an antacid has been ordered PRN, what will the RN do next

Verify last time med given, determine appropriateness to give next dose

Anti Impotence (Erectile Dysfunction) Med Names

Viagra, Levitra, Cialis

Antibiotics: Tetracyclines Med Names

Vibramycin, Panmycin

Antipyschotic drugs are NOT used treat

Violent behaviors

Cancer Drugs

Wear gloves, mask when preparing or giving. Can be absorbed throught skin

Which statement would be included when educating the patient taking trimethadione for seizures

Wear sunscreen and protective clothing when exposed to sunlight

The RN teaches a pt to self administer sumatriptan-immatrex when is this injection contraindicated?

Weekly prophylactic use- Expensive not used unless necassary

How would someone describe a schizophrenic?

Weird, strange, and different.

Antidepressants: Heterocyclics Med Names

Wellbutrin, Desyrel

Antidepressants: Heterocyclics Side Effects

Wellbutrin: agitation Desyrel: sedation

b. Apply the ointment to a nonhairy part of the upper torso.

What instruction should the nurse provide to the client who needs to apply nitroglycerin ointment? a. Use the fingers to spread the ointment evenly over a 3-inch area. b. Apply the ointment to a nonhairy part of the upper torso. c. Massage the ointment into the skin. d. Cover the application paper with ointment before use.

a. Perform all necessary venipunctures.

What intervention is essential before the nurse administers tenecteplase (TNKase)? a. Perform all necessary venipunctures. b. Administer aminocaproic acid (Amicar). c. Have the client void. d. Assess for allergies to iodine.

c. Apply the nitroglycerin patch for 14 hours and remove it for 10 hours at night.

What will the nurse instruct the client to do to prevent the development of tolerance to nitroglycerin? a. Apply the nitroglycerin patch every other day. b. Switch to sublingual nitroglycerin when the client's systolic blood pressure elevates to more than 140 mm Hg. c. Apply the nitroglycerin patch for 14 hours and remove it for 10 hours at night. d. Use the nitroglycerin patch for acute episodes of angina only.

c. Hydrochlorothiazide

What would cause the same client's electrolyte imbalance? a. High dose of digoxin b. Digoxin taken daily c. Hydrochlorothiazide d. Low dose of hydrochlorothiaizde

a. Decreased intracranial pressure

Which assessment indicates a therapeutic effect of mannitol (Osmitrol)? a. Decreased intracranial pressure b. Decreased potassium c. Increased urine osmolality d. Decreased serum osmolality

a. Client states that she has no chest pain.

Which client assessment would assist the nurse in evaluating therapeutic effects of a calcium channel blocker? a. Client states that she has no chest pain. b. Client states that the swelling in her feet is reduced. c. Client states the she does not feel dizzy. d. Client states that she feels stronger.

c. Assess potassium levels.

Which intervention will the nurse perform when monitoring a client receiving triamterene (Dyrenium)? a. Assess urinary output hourly. b. Monitor for side effect of hypoglycemia. c. Assess potassium levels. d. Monitor for Hypernatremia.

b. Fasting blood glucose level of 140 mg/dL

Which laboratory value will the nurse report to the health care provider as a potential adverse response to hydrochlorothiazide (HydroDIURIL)? a. Sodium level of 140 mEq/L b. Fasting blood glucose level of 140 mg/dL c. Calcium level of 9 mg/dL d. Chloride level of 100 mEq/L

Diarrhea and ataxia Hypotension and edema Slurred speech and muscle weakness

Which of the following symptoms would indicate to the nurse that a patient is experiencing lithium toxicity? (Select all that apply.) 1. Diarrhea and ataxia 2. Hypotension and edema 3. Hypertension and dehydration 4. Increased appetite, increased energy, and memory loss 5. Slurred speech and muscle weakness

b. "I will increase fiber in my diet."

Which statement indicates the client understands discharge instructions regarding cholestyramine (Questran)? a. "I will take Questran 1 hour before my other medications." b. "I will increase fiber in my diet." c. "I will weigh myself weekly." d. "I will have my blood pressure checked weekly."

SSRIs (ZLPP)

Zoloft-elderly Lexapro- Paxil-not for elderly Prozac-children antidepressant bruxism restless leg syndrome

Antivirals Med Names

Zovirax, AZT, Videx, Famvir, Cytovene

The client is also taking a diuretic that decreases her potassium level. The nurse expects that a low potassium level (hypokalemia) could have what effect on the digoxin? a. Increase the serum digoxin sensitivity level b. Decrease the serum digoxin sensitivity level c. Not have any effect on the serum digoxin sensitivity level d. Cause a low average serum digoxin sensitivity level

a. Increase the serum digoxin sensitivity level

When a client is taking ezetimibe (Zetia), she asks the nurse how it works. The nurse should explain that Zetia does what? a. Inhibits absorption of dietary cholesterol in the intestines. b. Binds with bile acids in the intestines to reduce LDL levels. c. Inhibits HMG-CoA reductase, which is necessary for cholesterol production in liver. d. Forms insoluble complexes and and reduces circulating cholesterol in blood.

a. Inhibits absorption of dietary cholesterol in the intestines.

Antidote for heparin overdose a. protamine sulfate b. vitamin K c. vitamin E d. cyanocobolamine

a. protamine sulfate

In the administration of hydrocortisone (Aeroseb-HC, Alphadern, Cetacort), it is vital that the nurse recognize that this drug might mask which symptoms? a.) Signs and symptoms of infection b.) Signs and symptoms of heart failure c.) Hearing loss d.) Skin infections

a.) Signs and symptoms of infection

Narcotics Action

acts on CNS receptor cells

Antipsychotic Agents Uses

acute and chronic psychoses

Thrombolytics Uses

acute pulmonary emboli, thrombosis, MI

What Rn action causes most med errors

administering unfamiliar medications

a pt being changed from oral to inhalation glucocorticoids could experience signs of

adrenal insufficiency

-"mycin"

aminoglycoside

Antibiotics: Penicillins Med Names

amoxicillin, ampicillin, augmentin

cholinergic crisis when there is

an overdose of Edrophoium (Tensilon) med

Omalizumab is an anti-inflammatory used for long term control of asthma; what major reaction can occur w/ admin of this med?

anaphylactic reactions

Antibiotics: Sulfonamides Actions

anatgonize essential component of folic acid synthesis

Atropine

antidysrhythmic increases firing of SA node & conduction through AV node

Furadantin (nitrofurantoin) Side Effects

asthma attacks, diarrhea

TCA's

atypical use of ____ is for the treatment of enuresis (bed wetting)

Antidepressants: Heterocyclics Nursing Considerations

avoid alcohol CNS depressants wean off slowly

The client is taking rosuvastatin (Crestor). What severe skeletal muscle adverse reaction should the nurse observe for? a. Myasthenia gravis b. Rhabdomyolysis c. Dyskinesia d. Agranulocytosis

b. Rhabdomyolysis

Which disease is characterized by increased body metabolism, tachycardia, increased body temperature, and anxiety, and treated with Prophylthiouracil (PTU)? a.) Hashimoto's thyroiditis b.) Graves' disease c.) Addison's disease d.) Cushing's syndrome

b.) Graves' disease

A client who is taking levothyroxine (Synthroid) begins to develop weight loss, diarrhea, and intolerance. The nurse should be aware that this might be an indication of what hormonal condition? a.) Addison's disease b.) Hyperthyroidism c.) Cushing's syndrome d.) Development of acromegaly

b.) Hyperthyroidism

Antibiotics: Macrolide Actions

binds to cell membrane and changes protein function

antivenin, equine

black widow antidote

Antihistamines: Action

block effects of histamine

Antihypertensives: Ace Inhibitors Actions

blocks ACE in lungs

Antihypertensives: Beta Blockers Actions

blocks beta-adrenergic receptors

Anticoagulant Heparin Action

blocks conversion of prothrobin to thrombin

Antipsychotic Agents Actions

blocks dopamine receptors in basal ganglia

during treatment of Tamoxifen given to breast cancer pt, monitior what

calcium cholesterol triglceride levels CBC platelet

Antidepressants: MAOIs Action

causes increased concentration of neurotransmitters

What is important to know about pt taking Rifampin (Ridadin) long term?

causes orange discoloration of sweat, tears, urine, feces.

Ca Channel Blocker

caution w/ PT taking digoxin/beta-blockers don't consume grapefruit juice IV give over 2-3 minutes

Antineoplastic: Hormonal Agents Actions

changes hormone input into sensitive cells

Antihypertensives: Beta Blockers Side Effects

changes in HR, hypotension, bronchospasms

atropine

cholinergic antidote

Theophylline is a methylxanthine bronchodilator and needs to limit xanthine containing foods like

coffee cola chocolate

Drug choice for treatment of chloroquine-resistant malaria is

combo of antimalarials

Cimetidine, histamine (H2) receptor antagonist, most common SE in older/elderly pt

confusion

Procainamide (Procanbid) given for cardiac dysrhythmia; what are signs of toxicity of Procainamide (Procanbid)?

confusion dizziness drowsiness decreased urination n/v tachydysrhythmias

Antiparkinson Agents Actions

converted to dopamine, stimulates postoynaptic dopamine receptors

A client is taking lovastatin (Mevacor). Which serum level is most important for the nurse to monitor? a. Blood urea nitrogen b. Complete blood count c. Cardiac enzymes d. Liver enzymes

d. Liver enzymes

A client is prescribed dalteparin (Fragmin). LMWH is administered via which route? a. Intravenously b. Intramuscularly c. Intradermally d. Subcutaneously

d. Subcutaneously

W/ Nalidixic acid, Coumadin should be

decreased

Pt has renal disorder creatinine clearance 40ml/min Her drug dose should be

decreased Normal creatinine is 0.6-1.2

Prazosin hydrochloride is an ANTI-hypertensive med , what manifestation determines a benefit of therapy

decreased BP

Desmopressin acetate is prescribed for diabetes insipidus what is the therapeutic response of the med?

decreased urinary output

Bronchodilators Action

decreases activity of phosphodiesterase

digibind

digoxin antidote

S/E of peptide include which of the following

dizziness,Neuro/Nephro toxicity Numbness/tingling

Beta Blockers

do not give in same line with furosemide

early signs of Digoxin toxicity

double vision loss of appetite or anorexia nausea/ vomiting diarrhea

-"afil"

erectile dysfunction

Antibiotics: Macrolide Med Names

erthromycin, Cleocin (clindamycin)

Major adverse effect of Alendronate (Fosamax)

esophageal irritation

Bone-Reabsorption Inhibitors: Biophosphonates Side Effects

esophagitis, arthralgia

ethanol

ethylene glycol antidote

Evaluate the effectiveness of the drug is included in what part of RN process

evaluation

disulfiram reaction

flushing palpitations nausea

S/E rxns of Amphotericin B include which of the following

flushing, hypotension, Hypokalemia, thrombophlebitis

Cytoprotective Agents Action

forms a barrier on ulcer surface

Sulfisoxazole needs to be given w/

full glass of water

Antihypertensives: Ace Inhibitors Nursing Considerations

give 1 hour before meals, or 3 hrs after Change positioning slowly Monitor VS

Antihistamines: Nursing Considerations

give w food use sunscreen avoid alcohol assess respirations

Antivirals Side Effects

headache, dizziness, GI symptoms

S/E of metronidazole may include

headache/ depression

Anti Impotence Side Effects

headaches, hypotension, priapism

Therapeutic effect of Epoetin alfa (Epogen, Procrit) ?? Epoetin alfa is used to reverse anemia associated w/ chronic renal failure

hematocrit between 30-33%

Cyclophosphamide for breast cancer needs to be monitored for

hematuria

Anticoagulant Heparin Side Effect

hematuria, tissue irritation

major complication of thrombolytic med?

hemorrhage

Antiplatelet Agents Side Effects

hemorrhage, throbocytopenia

Ribavirin given to a HOSPITALIZED child w/ respiratory syncytial virus (RSV) cannot be given orally, intramuscularly or subc, how is it given?

hood face mask oxygen tent

during treatment of Tamoxifen given to breast cancer pt, watch for

hypercalcemia

Asparaginase (Elspar), an antineoplastic med is contraindicated for pts w/

hypersensitivity pancreatitis hx of pancreatitis

serious SE of ACE inhibitor drugs

hypotension monitor BP

IV admin of Methocarbarnol (Robaxin) for multiple sclerosis can cause

hypotension bradycardia

Bethanechol chloride (Urecholine) is a direct acting muscarinic agonists (cholinergic medication) and can cause

hypotension vasodilation bradycardia excess salivation increased secretion of gastric acid abdominal cramps diarrhea

Myexdema indicates

hypothyroidism

Which following true regarding sulfonamides

increase anticoags effect of warfarin SJS is an adverse rxn Increased hypoglycemic effect w/ sulfonylureas

Iron Preparations Actions

increase availability of iron for hemoglobin

Epoetin alfa (Epogen, Procrit) can show a therapeutic effect in

increase in hematocrit will show in 2 weeks after therapy

nephrotoxicity

increased BUN increased creatinine

signs of developing pancreatitis

increased serum amylase increased serum triglycerides decreased serum calcium nausea vomiting abdominal pain radiating to the back

Attention Disorder Agents Action

increases level of catecholamines

Thyroid Replacement Agents Action

increases metabolic rate

Major SE of Etabercept (Enbrel) for rheumatoid arthritis

infection & pancytopenia; monitor WBC counts & platelets

Antibiotics: Tetracyclines Uses

infections, acne, prophylaxis for ophthalmia neonatorum

Alpha glucosidease inhibitors

inhibit carbohydrate digestion

Antidepressants: SSRIs Actions

inhibits CNS uptake of serotonin

deferoxamine mesylate

iron antidote

Fomepizole (Antizol) given for treatment of known or suspected ethylene glycol (antifreeze) intoxication is given w/ what type of IV fluid over a 30 min period?

isotonic 0.9% Normal saline 5% dextrose in water

signs of liver dysfunction

jaundice abdominal pain malaise

Cardiac Glycosides Uses

left-sided HF

Serotonin syndrome

may occur when taking another drug that affects reuptake of serotonin, causing sertonin to accumulate in the body. Can also be caused from SSRI with MAOI. Symptoms include confusion, anxiety, restlessness, hyperpyrexia, ataxia.

what process is used to eliminate med errors in healthcare enviroments as pts transition from one setting to another

medication reconcilliation records

-"phylline"

methylxanthines bronchodilator

Antibiotics: Penicillins Uses

moderate to severe infections, syphilis, gonococcus infections, Lyme disease

Narcotics Uses

moderate to severe pain, preoperative, postoperative

Antigout Agents: Nursing Consideration

monitor kidney stones Give w milk, food, antacids

Bipolar Disorder: Nursing Considerations

monitor serum levels give with meals increase fluid intake

Antihypertensives: Calcium Channel Blockers Nursing Considerations

monitor vs caution position change monitor edema (face swelling, SOB)

Antihypertensives: Angiotensin II Receptor Blockers Nursing Considerations

monitor vs do not chew or divide sustained release tablets

Statins

multiple drug interactions can prolong bleeding in patients taking coumadin

atropine sulfate

mushrooms, muscarinic antidote nerve gas antidote

signs of pancreatitis

n/v abdominal pain

Atropine sulfate causes mydriasis and cycloplegia and is contraindicated in

narrow angle glaucoma

Receptor located in different parts of the body may initiate a variety of responses dependant upon its anatomic site the recptor is:

nonspecific

peripheral neuritis signs

numbness tingling paresthesias in extremities

phenytoin level of 20 mcg/mL or higher the pt will show signs of

nystagmus (involuntar movements of the eyeballs)

which of the following is NOT a flouroquinalone w/ daily dosing

ofloxacin-floxacin

Mannitol

osmotic diuretic ICP Watch for headache

Pt w/ known cardiovascular disease is to recieve local anesthesia for large laceration, which will need to be closely monitored

palpatations

what tests need to be done w/ Asparaginase (Elspar), an antineoplastic med

pancreatic function tests

what major disease can develop due to toxicity of Didanosine (Videx) ?

pancreatitis

-"cillin"

penicillin

Antacids Uses

peptic ulcer, indigestion, reflex esophagitis

common SE of INH

peripheral neuritis

Adverse effect w/ Zalcitabine

peripheral neuropathy

Major SE of Vincristine (Oncovin, Vincasar) given to pt w/ ovarian cancer

peripheral neuropathy

what is given for potassium deficit

potassium chloride

Risk for injury: included in which Rn phase of nursing process

potential RN dx

Immunosuppressants Action

prevents production of T cells and their response to interleukin-2

Immunosuppressants Uses

prevents rejection for transplanted organs

Drug interaction occurs b/w acyclovir and which following drugs

probenicid

Major adverse effect of Asparaginase (Elspar), an antineoplastic med

prolonged blood clotting times

antidote for heparin

protamine sulfate

Today malaria is a common disease caused by

protozoa

Electrolytes Actions

replaces needed electrolytes

Your Cl is not wearing Id band what should be your first Rn Action

report find-have new id put on Cl

if signs of Iodism occur when taking potassium iodide solution (Lugol's solution), what should the nurse instruct the pt to do?

report symptom to MD

What is evidence based medicine

scientific research used for making clinical decisions Quality core measures used on basis-pts conditions Data voluntarily submitted by healthcare institutions providers

regular (Humalin R)

short-acting onset 0.5-1hr only one given via IV Clear

which statement about verbal order is correct

should only be used in ER situations

pts w/ DM taking insulin or oral hypoglycemics and taking glucocorticoid therapy will need

their insulin or oral hypoglycemic meds increased

valid indicator that measures the margin of safety of a drug is its

therapeutic index TD50/ED50

electroconvulsive therapy (ECT)

therapy for *severely depressed patients in which a brief electric current is sent through the brain. serious complications are seizure activity and anesthesia

-"zides"

thiazide diuretics

-"zide"

thiazide diuretics HTN Pulm. edema

repetitive transcranial magnetic stimulation (rTMS)

this treatment requires surgical implant of device. somatic treatment

Antibiotics: Sulfonamides Uses

ulcerative colitis, Crohn's, otitis media, UTIs

H2-Receptor Blockers Uses

ulcers, gastroesophageal reflux

Desmporessin (DDAVP)

used for DI reabsorbs of H2O in kidneys watch for fluid overload

Dopamine

vasoconstrictor 1st line of drug in sever hypotension

Antiplatelet Agents Uses

venous thrombosis, PE

Preffered site for Ztrack

ventrogluteal

Ticlopidine (Ticlid) can cause GI SE and should be given

with meals

oral steroids are given

with meals to decrease GI irritation

when are thrombolytics like streptokinase most effective?

within 4-6 hours after onset of symptoms

INH is hepatotoxic; what are signs of hepatitis

yellow skin yellow sclera

Ace Inhibitors

"PRIL" Captopril, Enalapril, Afosiopril Antihypertensive. Blocks ACE in lungs from converting angiotensin I to angiotensin II (powerful vasoconstrictor). Decreases BP, Decreased Aldosterone secretions, Sodium and fluid loss. Check BP before giving (hypotension) *Orthostatic Hypotension

32.) Desmopressin acetate (DDAVP) is prescribed for the treatment of diabetes insipidus. The nurse monitors the client after medication administration for which therapeutic response? 1. Decreased urinary output 2. Decreased blood pressure 3. Decreased peripheral edema 4. Decreased blood glucose level

1. Decreased urinary output Rationale: Desmopressin promotes renal conservation of water. The hormone carries out this action by acting on the collecting ducts of the kidney to increase their permeability to water, which results in increased water reabsorption. The therapeutic effect of this medication would be manifested by a decreased urine output. Options 2, 3, and 4 are unrelated to the effects of this medication.

109.) A client taking buspirone (BuSpar) for 1 month returns to the clinic for a follow-up visit. Which of the following would indicate medication effectiveness? 1. No rapid heartbeats or anxiety 2. No paranoid thought processes 3. No thought broadcasting or delusions 4. No reports of alcohol withdrawal symptoms

1. No rapid heartbeats or anxiety Rationale: Buspirone hydrochloride is not recommended for the treatment of drug or alcohol withdrawal, paranoid thought disorders, or schizophrenia (thought broadcasting or delusions). Buspirone hydrochloride is most often indicated for the treatment of anxiety and aggression.

Serum phenytoin levels should be monitored to determine if the blood serum level is within the therapuetic range, thus avoiding toxic levels.The therapuetic range of phenytoin is

10-20 mcg/mL

normal adult dosage for Bethanechol chloride (Urecholine) for post-op bladder spasm

10-50 mg three to four times a day

10. The most serious adverse effect of Alprostadil (Prostin VR pediatric injection) administration in neonates is: a. Apnea. b. Bleeding tendencies. c. Hypotension. d. Pyrexia.

10.Answer A. All items are adverse reactions of the drug. However, apnea appearing during the first hour of drug infusion occurs in 10-12 percent of neonates with congenital heart defects. Clinicians deciding to utilize alprostadil must be prepared to intubate and mechanically ventilate the infant. Careful monitoring for apnea or respiratory depression is mandatory. In some institutions, elective intubation occurs prior to initiation of the medication.

10. What is the purpose of "tunneling" (inserting the catheter 2-4 inches under the skin) when the surgeon inserts a Hickman central catheter device? Tunneling: a. Increases the patient's comfort level. b. Decreases the risk of infection. c. Prevents the patient's clothes from having contact with the catheter d. Makes the catheter less visible to other people.

10.Answer B. The actual access to the subclavian vein is still just under the clavicle, but by tunneling the distal portion of the catheter several inches under the skin the risk of migratory infection is reduces compared to a catheter that enters the subclavian vein directly and is not tunneled. The catheter is tunneled to prevent infection.

16. A client with advanced cirrhosis of the liver is not tolerating protein well, as eveidenced by abnormal laboratory values. The nurse anticipates that which of the following medications will be prescribed for the client? a. lactulose (Chronulac) b. ethacrynic acid (Edecrin) c. folic acid (Folvite) d. thiamine (Vitamin B1)

16. A. The client with cirrhosis has impaired ability to metabolize protein because of liver dysfunction. Administration of lactulose aids in the clearance of ammonia via the gastrointestinal (GI) tract. Ethacrynic acid is a diuretic. Folic acid and thiamine are vitamins, which may be used in clients with liver disease as supplemental therapy.

17. A female client tells the clinic nurse that her skin is very dry and irritated. Which product would the nurse suggest that the client apply to the dry skin? a. glycerin emollient b. aspercreame c. myoflex d. acetic acid solution

17. A. Glycerin is an emollient that is used for dry, cracked, and irritated skin. Aspercreame and Myoflex are used to treat muscular aches. Acetic acid solution is used for irrigating, cleansing, and packing wounds infected by Pseudomonas aeruginosa.

17. Estrogen antagonists are used to treat estrogen hormone-dependent cancer, such as breast carcinoma. Androgen antagonists block testosterone stimulation of androgen-dependent cancers. An example of an androgen-dependent cancer would be: a. Prostate cancer. b. Thyroid cancer. c. Renal carcinoma. d. neuroblastoma.

17.Answer A. Prostate tissue is stimulated by androgens and suppressed by estrogens. Androgen antagonists will block testosterone stimulation of prostate carcinoma cells. The types of cancer in options 2-4 are not androgen dependent.

Insulin: Intermediate Acting Duration

18-26h

19. Auranofin (Ridaura) is prescribed for a client with rheumatoid arthritis, and the nurse monitors the client for signs of an adverse effect related to the medication. Which of the following indicates an adverse effect? a. nausea b. diarrhea c. anorexia d. proteinuria

19. D. Auranofin (Ridaura) is a gold preparation that is used as an antirheumatic. Gold toxicity is an adverse effect and is evidenced by decreased hemoglobin, leukopenia, reduced granulocyte counts, proteinuria, hematuria, stomatitis, glomerulonephritis, nephrotic syndrome, or cholestatic jaundice. Anorexia, nausea, and diarrhea are frequent side effects of the medication.

20. A client has been taking benzonatate (Tessalon) as ordered. The nurse tells the client that this medication should do which of the following? a. take away nausea and vomiting b. calm the persistent cough c. decrease anxiety level d. increase comfort level

20. B. Benzonatate is a locally acting antitussive. Its effectiveness is measured by the degree to which it decreases the intensity and frequency of cough, without eliminating the cough reflex

Understanding RT need for hydration you encourage him to have at least______mL of fluid daily

2000mL

Zonisimide 200 mg is prescribed. The drug is availble in 100mg tablet- the Rn administers

200mg/100mg= 2Tablets

RT 50 Y/O receiving prochlorperazine-compazine for nausea and vomiting. The usual dose of compazine varies with form and includes all of the following EXCEPT

25mg Q3H PRN

29. Central venous access devices (CVADs) are frequently utilized to administer chemotherapy. What is a distinct advantage of using the CVAD for chemotherapeutic agent administration? a. CVADs are less expensive than a peripheral IV. b. Once a week administration is possible. c. Caustic agents in small veins can be avoided. d. The patient or his family can administer the drug at home.

29.Answer C. Many chemotherapeutic drugs are vesicants (highly active corrosive materials that can produce tissue damage even in low concentrations). Extravasations of a vesicant can result in significant tissue necrosis. Administration into a large vein is optimal. CVADs are more expensive than a peripheral IV. Dosing depends on the drug. IV chemotherapeutic agents are not administered at home. They are given in an outpatient or clinic setting if not given during hospitalization.

233.) Diphenhydramine hydrochloride (Benadryl) is used in the treatment of allergic rhinitis for a hospitalized client with a chronic psychotic disorder. The client asks the nurse why the medication is being discontinued before hospital discharge. The nurse responds, knowing that: 1. Allergic symptoms are short in duration. 2. This medication promotes long-term extrapyramidal symptoms. 3. Addictive properties are enhanced in the presence of psychotropic medications. 4. Poor compliance causes this medication to fail to reach its therapeutic blood level.

3. Addictive properties are enhanced in the presence of psychotropic medications. Rationale: The addictive properties of diphenhydramine hydrochloride are enhanced when used with psychotropic medications. Allergic symptoms may not be short term and will occur if allergens are present in the environment. Poor compliance may be a problem with psychotic clients but is not the subject of the question. Diphenhydramine hydrochloride may be used for extrapyramidal symptoms and mild medication-induced movement disorders.

69.) A nurse who is administering bethanechol chloride (Urecholine) is monitoring for acute toxicity associated with the medication. The nurse checks the client for which sign of toxicity? 1. Dry skin 2. Dry mouth 3. Bradycardia 4. Signs of dehydration

3. Bradycardia Rationale: Toxicity (overdose) produces manifestations of excessive muscarinic stimulation such as salivation, sweating, involuntary urination and defecation, bradycardia, and severe hypotension. Treatment includes supportive measures and the administration of atropine sulfate subcutaneously or intravenously.

148.) A client is taking cetirizine hydrochloride (Zyrtec). The nurse checks for which of the following side effects of this medication? 1. Diarrhea 2. Excitability 3. Drowsiness 4. Excess salivation

3. Drowsiness Rationale: A frequent side effect of cetirizine hydrochloride (Zyrtec), an antihistamine, is drowsiness or sedation. Others include blurred vision, hypertension (and sometimes hypotension), dry mouth, constipation, urinary retention, and sweating.

230.) A client is placed on chloral hydrate (Somnote) for short-term treatment. Which nursing action indicates an understanding of the major side effect of this medication? 1. Monitoring neurological signs every 2 hours 2. Monitoring the blood pressure every 4 hours 3. Instructing the client to call for ambulation assistance 4. Lowering the bed and clearing a path to the bathroom at bedtime

3. Instructing the client to call for ambulation assistance Rationale: Chloral hydrate (a sedative-hypnotic) causes sedation and impairment of motor coordination; therefore, safety measures need to be implemented. The client is instructed to call for assistance with ambulation. Options 1 and 2 are not specifically associated with the use of this medication. Although option 4 is an appropriate nursing intervention, it is most important to instruct the client to call for assistance with ambulation.

30. Epinephrine is administered to a female patient. The nurse should expect this agent to rapidly affect: a. Adrenergic receptors. b. Muscarinic receptors. c. Cholinergic receptors. d. Nicotinic receptors.

30.Answer A. Epinephrine (adrenaline) rapidly affects both alpha and beta adrenergic receptors eliciting a sympathetic (fight or flight) response. Muscarinic receptors are cholinergic receptors and are primarily located at parasympathetic junctions. Cholinergic receptors respond to acetylcholine stimulation. Cholinergic receptors include muscarinic and nicotinic receptors. Nicotinic receptors are cholinergic receptors activated by nicotine and found in autonomic ganglia and somatic neuromuscular junctions.

118.) A nurse is caring for an older client with a diagnosis of myasthenia gravis and has reinforced self-care instructions. Which statement by the client indicates that further teaching is necessary? 1. "I rest each afternoon after my walk." 2. "I cough and deep breathe many times during the day." 3. "If I get abdominal cramps and diarrhea, I should call my doctor." 4. "I can change the time of my medication on the mornings that I feel strong."

4. "I can change the time of my medication on the mornings that I feel strong." Rationale: The client with myasthenia gravis should be taught that timing of anticholinesterase medication is critical. It is important to instruct the client to administer the medication on time to maintain a chemical balance at the neuromuscular junction. If not given on time, the client may become too weak to swallow. Options 1, 2, and 3 include the necessary information that the client needs to understand to maintain health with this neurological degenerative disease.

54.) A nurse reinforces discharge instructions to a postoperative client who is taking warfarin sodium (Coumadin). Which statement, if made by the client, reflects the need for further teaching? 1. "I will take my pills every day at the same time." 2. "I will be certain to avoid alcohol consumption." 3. "I have already called my family to pick up a Medic-Alert bracelet." 4. "I will take Ecotrin (enteric-coated aspirin) for my headaches because it is coated."

4. "I will take Ecotrin (enteric-coated aspirin) for my headaches because it is coated." Rationale: Ecotrin is an aspirin-containing product and should be avoided. Alcohol consumption should be avoided by a client taking warfarin sodium. Taking prescribed medication at the same time each day increases client compliance. The Medic-Alert bracelet provides health care personnel emergency information.

56.) Heparin sodium is prescribed for the client. The nurse expects that the health care provider will prescribe which of the following to monitor for a therapeutic effect of the medication? 1. Hematocrit level 2. Hemoglobin level 3. Prothrombin time (PT) 4. Activated partial thromboplastin time (aPTT)

4. Activated partial thromboplastin time (aPTT) Rationale: The PT will assess for the therapeutic effect of warfarin sodium (Coumadin) and the aPTT will assess the therapeutic effect of heparin sodium. Heparin sodium doses are determined based on these laboratory results. The hemoglobin and hematocrit values assess red blood cell concentrations.

217.) A health care provider prescribes auranofin (Ridaura) for a client with rheumatoid arthritis. Which of the following would indicate to the nurse that the client is experiencing toxicity related to the medication? 1. Joint pain 2. Constipation 3. Ringing in the ears 4. Complaints of a metallic taste in the mouth

4. Complaints of a metallic taste in the mouth Rationale: Ridaura is the one gold preparation that is given orally rather than by injection. Gastrointestinal reactions including diarrhea, abdominal pain, nausea, and loss of appetite are common early in therapy, but these usually subside in the first 3 months of therapy. Early symptoms of toxicity include a rash, purple blotches, pruritus, mouth lesions, and a metallic taste in the mouth.

The PCP prescribes ethosuximide syrup 500 mg for a pt with abscence seizures.The drug is available in a strength of 250mg/5mL the Rn administers.

500mg/250mg x 5 mL = 10mL

A pt is prescribed fentanyl sublimaze 50mcg IM 30 min before surgery.The Rn has a vial w dose 0.05 mg/dl the RN calculate and administers how much medication

50mcg /3=0.05 0.05/0.05=1mcg

JA 24 old has a seizure disorder and is going to start taking phenytoin -the drug of choice to control seizure activity JA will initially recieve IV phenytoin which should be administered at a maximum rate of

50mg/minute

6. Mr. Jessie Ray, a newly admitted patient, has a seizure disorder which is being treated with medication. Which of the following drugs would the nurse question if ordered for him? a. Phenobarbitol, 150 mg hs b. Amitriptylene (Elavil), 10 mg QID. c. Valproic acid (Depakote), 150 mg BID d. Phenytoin (Dilantin), 100 mg TID

6. B. Elavil is an antidepressant that lowers the seizure threshold, so would not be appropriate for this patient. The other medications are anti-seizure drugs.

7. Mrs. Jane Gately has been dealing with uterine cancer for several months. Pain management is the primary focus of her current admission to your oncology unit. Her vital signs on admission are BP 110/64, pulse 78, respirations 18, and temperature 99.2 F. Morphine sulfate 6mg IV, q 4 hours, prn has been ordered. During your assessment after lunch, your findings are: BP 92/60, pulse 66, respirations 10, and temperature 98.8. Mrs. Gately is crying and tells you she is still experiencing severe pain. Your action should be to a. give her the next ordered dose of MS. b. give her a back rub, put on some light music, and dim the lights in the room. c. report your findings to the RN, requesting an alternate medication order be obtained from the physician. d. call her daughter to come and sit with her.

7. C. Morphine sulfate depresses the respiratory center. When the rate is less than 10, the MD should be notified.

8. Drugs can cause adverse events in a patient. Bone marrow toxicity is one of the most frequent types of drug-induced toxicity. The most serious form of bone marrow toxicity is: a. aplastic anemia. b. thrombocytosis. c. leukocytosis. d. granulocytosis.

8.Answer A. Aplastic anemia is the result of a hypersensitivity reaction and is often irreversible. It leads to pancytopenia, a severe decrease in all cell types: red blood cells, white blood cells, and platelets. A reduced number of red blood cells causes hemoglobin to drop. A reduced number of white blood cells make the patient susceptible to infection. And, a reduced number of platelets cause the blood not to clot as easily. Treatment for mild cases is supportive. Transfusions may be necessary. Severe cases require a bone marrow transplant. Option 2 is an elevated platelet count. Option 3 is an elevated white count. Option 4 is an elevated granulocyte count. A granulocyte is a type of white blood cell.

9. Serious adverse effects of oral contraceptives include: a. Increase in skin oil followed by acne. b. Headache and dizziness. c. Early or mid-cycle bleeding. d. Thromboembolic complications.

9.Answer D. Oral contraceptives have been associated with an increased risk of stroke, myocardial infarction, and deep vein thrombosis. These risks are increased in women who smoke. Increased skin oil and acne are effects of progestin excess. Headache and dizziness are effects of estrogen excess. Early or mid-cycle bleeding are effects of estrogen deficiency.

c. Rapid IV bolus of Adenosine (Adenocard)

A client is admitted to the emergency department with paroxysmal supraventricular tachycardia. What intervention is the nurse's priority? a. Administration of digoxin IV push b. Administration of oxygen, 2 lpm c. Rapid IV bolus of Adenosine (Adenocard) d. Instructing client to "bear down"

c. Have the client increase fluids and fiber in his diet.

A nurse is caring for a client taking cholestyramine (Questran). The client is complaining of constipation. What will the nurse do? a. Call the health care provider to change the medication. b. Tell the client to skip a dose of the medication. c. Have the client increase fluids and fiber in his diet. d. Administer an enema to the client.

a. Call the health care provider to switch the medication.

A nurse is caring for a client who is taking an angiotensin-converting enzyme inhibitor and develops a dry, nonproductive cough. What is the nurse's priority action? a. Call the health care provider to switch the medication. b. Assess the client for other symptoms of upper respiratory infection. c. Instruct the client to take antitussive medication until the symptoms subside. d. Tell the client that the cough will subside in a few days.

A teaching plan for a patient who is taking lispro (Humalog) should include which of these instructions by the nurse? A) "Inject this insulin with your first bite of food because it is very fast acting." B) "The duration of action for this insulin is about 8 to 10 hours, so you'll need a snack." C) "This insulin needs to be mixed with regular insulin to enhance the effects." D) "To achieve tight glycemic control, this is the only type of insulin you'll need."

A) "Inject this insulin with your first bite of food because it is very fast acting." Lispro is a rapid-acting insulin and has an onset of 15 to 30 minutes with a peak action of about 2 hours, not 8 to 10 hours. Because of its rapid onset, it is administered immediately before a meal or with meals to control blood glucose rise after meals. Lispro insulin must be combined with intermediate- or long-acting insulin not regular insulin, which is also a short-duration insulin, for glucose control between meals and at night. To achieve tight glycemic control, patients must combine different types of insulin based on duration of action.

A client has been newly diagnosed with hypothyroidism and will take levothyroxine (Synthroid) 50 mcg/day by mouth. As part of the teaching plan, the nurse emphasizes that this medication: A) Should be taken in the morning B) May decrease the client's energy level C) Must be stored in a dark container D) Will decrease the client's heart rate

A) Should be taken in the morning

The nurse is aware that an overdose of fluphenazine would require which of the following treatments

Activated charcoal administration

Collistin S given Im special considerations

Adding 1 % lido to medication-prevent pain when admin

Glucocorticoids Uses

Addison's disease, Crohn's disease, COPD, leukemias

Rn should

Admin medications and extra Fl. Monitor Urinary Output Observe allergic response

What treatment should you implement for delirium tremens?

Administer D5 1/2 NS at 100cc/hr, now adding thiamine 100 mg, folate 1 mg, and multivitamins to the IV fluids once daily. (the B vitamins help with red blood cell formation and thiamine helps prevent wernicke-korsakoff syndrome). Place them in a quiet/private room, leaving the light on at all times, and side rails up on bed. Administering a sedative (chlordiazepoxide or benzodiazepines) will help to calm the patient down.

Antineoplastic: Antitumor Med Names

Adriamycin, Bleomycin, Actinomycin D

CL is taking levoflaxacin(levaquin Rn knows that which is true regarding this drug

Adv affects include dysrythmias

JT should be told NOT to take Valium w/

Alcohol, becouse it can cause CNS depression/Respiratory distress

A nurse is caring for a cancer patient receiving subcutaneous morphine sulfate for pain. Which of the following nursing actions is most important in the care of this patient? A. Monitor urine output. B. Monitor respiratory rate. C. Monitor heart rate. D. Monitor temperature.

Answer: B Morphine sulfate can suppress respiration and respiratory reflexes, such as cough. Patients should be monitored regularly for these effects to avoid respiratory compromise. Morphine sulfate does not significantly affect urine output, heart rate, or body temperature.

What should you do with someone who is admitted with obsessive compulsive disorder?

Ask what makes them anxious. Don't interfere with rituals unless harming themselves or others. and interact in a positive way if need to influence.

The cl is taking large doses of aspirin for an arthritic condition the RN needs to know ALL the following except:

Aspirin can lower blood sugar in cl. w/ diabetes, causing hypoglycemia

how can flushing that is caused by Nicotinic acid (Niacin) given for hyperlipidemia be prevented?

Aspirin or NSAID like Ibuprofen (Motrin) taken 30 min before med

Nursing assessment of a client receiving serum albumin for treatment of shock should include:

Assessing lung sounds.

Obtains CL weight to be used for future comparison- included in what phase RN process

Assessment Collect date- baseline data

When performing a Tensilon test, the RN obtains Edrophonium (Tensilon) and what antidote?

Atropine sulfate

The nurse is teaching a patient who is newly diagnosed with epilepsy about her disease. Which statement made by the nurse best describes the goals of antiepilepsy medication therapy? A) "With proper treatment we can completely eliminate your seizures." B) "Our goal is to reduce your seizures to an extent that helps you live a normal life." C) "Epilepsy medication does not reduce seizures in most patients." D) "These drugs will help control your seizures until you have surgery."

B) "Our goal is to reduce your seizures to an extent that helps you live a normal life." Epilepsy is treated successfully with medication in a majority of patients. However, the dosages needed to completely eliminate seizures may cause intolerable side effects. Neurosurgery is indicated only for patients in whom medication therapy is unsuccessful.

The nurse is preparing to give ethosuximide (Zarontin). The nurse understands that this drug is only indicated for which seizure type? A) Tonic-clonic B) Absence C) Simple partial D) Complex partial

B) Absence Absence seizures are the only indication for ethosuximide. The drug effectively eliminates absence seizures in approximately 60% of patients and effectively controls 80% to 90% of cases.

Following heparin treatment for a pulmonary embolism, a client is being discharged with a prescription for warfarin (Coumadin). In conducting discharge teaching, the nurse advises the client to have which diagnostic test monitored regularly after discharge? A) Perfusion scan B) Prothrombin Time (PT/INR) C) Activated partial thromboplastin (APTT) D) Serum Coumadin level (SCL

B) Prothrombin Time (PT/INR)

Untreated hyperglycemia may lead to all of the following complications except: a. Hyperosmolar syndrome b Vitiligo c. Diabetic ketoacidosis d. Coma

B. Excessively high blood sugar or prolonged hyperglycemia can cause diabetic ketoacidosis, the condition in which the body breaks down fat for energy and ketones spill into the urine. Diabetic hyperosmolar syndrome occurs when blood sugar is excessively high and available insulin is ineffective. In this case, the body cannot use glucose or fat for energy and glucose is excreted in the urine. Without immediate medical attention, both conditions may result in coma or death.

glucagon (high dose)

Beta blocker antidote

The cholinergic drug used primarily to increase urination is

Bethanechol chloride- URECHOLINE

A patient is taking glipizide (Glucotrol) and a beta-adrenergic medication. A nurse is teaching hypoglycemia awareness and should tell the patient that which of these symptoms may not occur? A) Vomiting B) Muscle cramps C) Tachycardia D) Chills

C) Tachycardia Glipizide is a sulfonylurea oral hypoglycemic medication that acts to promote insulin release from the pancreas. Beta-adrenergic blockers can mask early signs of sympathetic system responses (most importantly, tachycardia) to hypoglycemia, which is the most common adverse effect of glipizide. Vomiting, muscle cramps, and chills are not symptoms of activation of the sympathetic nervous system that arise when glucose levels fall.

Miotics

CONSTRICT (Pilo-Carpine) For glaucoma - increased outflow of aqueaous humor

Bronchodilators Uses

COPD, preterm labor

Antibiotics: Penicillins Nursing Considerations

Check for hypersensitivity Give 1-2 h before or 2-3 h after meals Cross allergy to cephalosporins

Antiplatelet Agents Nursing Considerations

Check for signs of bleeding Give with food or milk

Potassium Chloride

Check labs before giving, Never give IV PUSH Give with FOOD (GI distresss)) Extreme caution if pt. receiving potassium-sparing diuretic. NOT for pts with RENAL DISEASE.

When you calculate the dose for cardiac med, the drug dose is LARGE the best initial action is to:

Check you calculation

Which of the following drugs/food changes the action of INH

Cheese(high in thymine) antacids

What should the Rn include when teaching about gentamycin

Cl should report Hearing loss Cl must use sunscreen Cl will be monitored for Ulcers and Vaginitis Cl should increase Fl intake

Becouse of the effects of cholinergic blocking drugs on intestinal motility, the nurse must monitor the pt taking these drugs for the development of

Constipation

Antibiotics: Fluoroquinolones Nursing Consideration

Culture and Sensity before starting therapy Encourage Fluids (3000ml.day) Take 1 h before or 2 h after meals

A patient screened for diabetes at a clinic has a fasting plasma glucose level of 120 mg/dl (6.7 mmol/L). The nurse will plan to teach the patient about a. use of low doses of regular insulin. b. self-monitoring of blood glucose. c. oral hypoglycemic medications. d. maintenance of a healthy weight.

D Rationale: The patient's impaired fasting glucose indicates prediabetes and the patient should be counseled about lifestyle changes to prevent the development of type 2 diabetes. The patient with prediabetes does not require insulin or the oral hypoglycemics for glucose control and does not need to self-monitor blood glucose.

A patient with hyperthyroidism is taking propylthiouracil (PTU). The nurse will monitor the patient for: A) gingival hyperplasia and lycopenemia. B) dyspnea and a dry cough. C) blurred vision and nystagmus. D) fever and sore throat.

D) fever and sore throat.

Heparin

Dilution: undiluted Continuous: 25,000 units with 250-500 ml of 0.9% NaCl or D5W Rate: over at least 1 min Continuous: adjust to maintain therapeutic aPTT, infuse approx 1,000 units/hr Concentration: varies Continuous: 50-100 units/ml Compatibility: y-site many Action: potentiates the inhibitory effect of ant thrombin. In low doses prevents the conversion of prothrombin to thrombin. Higher doses neutralize thrombin. Indications: prophylaxis and treatment for various thromboembolic disorders. Class-Therapeutic: anticoagulant Class-Pharmalogic: antithrombotics

Newly Dx diabetic pt taking oral hypoglycemic- his currando recomended that he drinks sabila tea 3x day- as home health Rn you should

Do not discourage Encourage his practice to drink tea- ensure he follows regimen with other meds- hypoglycemic meds

neuroliptic malignant syndrome

Dyspnea or tachypnea tachycardia irregular pulse rate fever BP changes increased sweating loss of bladder control skeletal muscle rigidty

The pt taking solifenacin -vesicare for an overactive bladder c/o dry mouth. the nurse should

Encourage the pt to take frequent sips of water

What is dystonia?

Face spasms, or awkward twisting of the head/body.

Which of the following drugs are effective in combating herpes simplex viruses HSV-1/2

Familclovir Ganciclovir Valacyclovir All of above**

Cholinergic blocking drugs are contraindicated in pt's w/

Glaucoma

Direct - Acting Vasodilator

Hydralazine (apresoline) Relaxes smooth muscle of blood vessels, lowing peripheral esistance. Teach pt to check pulse d/t change in C.O. If HR <60 Contact physician.

Atrovant (Ipratropium Bromide)

Indication: COPD, cold, cough, allergies Class: bronchodialator Labs: N/A Side Effects: headache, palpitations Precautions: monitor respiratory status

Oxybutyn

Indication: Neurogenic bladder, increase bladder capacity Class: anticholinergic Labs: N/A Side Effects: drowsiness, constipation, urinary retention Precautions: monitor voiding pattern, assess for bladder distention

Ciprofloxacin

Indication: UTI, gonorrhea, sinus infection, e-coli Class: anti-infective Labs: liver, WBC, glucose, cholesterol, Ca Side effects: seizure, C-Diff, rash, diarrhea, N/V, anaphylaxis, arrythmias, hyperglycemia Precuations: monitor for anaphylaxis, bowel function, assess infection, obtain culture

Cephalonia (Keflex)

Indication: UTI, sore throat, e-coli, pneumonia, wounds Class: anti-infective Labs: liver, WBC Side effects: seizure, C-Diff, rash, diarrhea, N/V, anaphylaxis Precuations: monitor for anaphylaxis, bowel function, assess infection, obtain culture

Fluconazole

Indication: UTI, yeast infection Class: antifungal Labs: liver, renal Side effects: hypokalemia, hepatotoxicity Precautions: assess infection, renal dysfunction need dose adjustment

Clonazepam

Indication: seizures, panic disorders, anxiety Class: benzodiazepine Labs: CBC, liver Side effects: suicide, mood changes, lethargy, ataxia Precuations: monitor levels, suicide, monitor and record seizure activity Reversal: Flumanazil

What resources provides basis for standards in drug strength and composition throughout the world?

International Pharmacopeia

Enteric Coated tablets absorbed where

Intestines

Most common side effect for helminthiasis is in the

Intestines

Major adverse effect w/ Potassium iodide solution (Lugol's solution)?

Iodism

The Rn knows that which of the following drugs modify the action of tetracycline

Iron Antacids Warfarin Milk Products

What is a concern for pregnant women with PICA?

Iron deficiency anemia (ALL AGE GROUPS). Tx= foods high in iron (legumes, veggies, raisins). Give with vitamin C to help absorb.

Ferrous Sulfate

Iron supplement do not take with food, milk, antacid take with VitC High fiber diet to decrease constipation

Dependancy S/S that can occur when taking benzodiazepines and are abruptly stopped include:

Irratibility and nervousness

What is behavior modification? What should you reward with? What should you not reward with?

It is changing behaviors by reward the good and ignoring the bad. You should reward with your time (story,play). You should not reward with food.

Dietary counseling to meet calcium requirements include encouragement to eat

Leafy greens

One of your pt medications is in a liquid form you pour the medication with the container at eye level and read the meniscus at what point

Lower part curve

What are antipsychotics that commonly cause the 6 side effects listed previously?

LoxaPINE, clozaPINE/ Risperidone, Haldol/Haloperidol, Phenothiazine (DRUG CLASS)- ThroaZINE, Prolixin, ChlorpromaZINE, CompaZINE.

What are the two drugs to know for depression?

MAOIs and Tricyclics

Antacids Med Names

MOM, Maalox, Amphojel

When administering phenobarbital to an elderly pt, the nurse should monitor the pt for unusual effects of the drug such as:

Marked Excitement

Antidepressants: MAOIs Med Names

Marplan, Parnate, Nardil

Antihypertensives: Beta Blockers Nursing Considerations

Masks signs of shock and hypoglycemia Take with meals Do not dc abruptly

Which of the following drugs are commonly used in the treatment of Roundworms/pinworms

Mebendazole

long term therapy with anti-helmintic is req with all the following drugs except

Mebendazole

lithium (Eskalith)

Medication use for Mania symptoms of bipolar

What is the medical treatment for depression?

Meds (MAOIs, Tricyclics) and ECT

Cl with dx of intestinal ambeisas develops n/v/d flushing/tachy/hypotension palpatation- pt states consumption of several alcoholic beverages Rn should obtain drug history for which drug.

Metronidazole- ambeiasis

Ca gluconate

Mg Sulfate antidote

Anticoagulant Coumadin Nursing Considerations

Monitor PT Normal 9-12 sec Therapeutic level 1.5 times control Antagonist- Vit K Monitor for bleeding Give PO

Antidysrhythmics Nursing Consideration

Monitor VS and Cardiac Rhythm

Anticoagulant Heparin Nursing Considerations

Monitor clotting time or PTT Normal 20-45 sec Therapeutic level 1.5-2.5 times control Antagonist- protmaine sulfate Give SC or IV - Do NOT aspirate or Massage!

Electrolytes Nursing Considerations

Monitor fluid and electrolyte levels

Carbonic Anhydrase Inhibitors Nursing Considerations

Monitor for systemic effects

Antiparkinson Agents Nursing Consideration

Monitor for urinary retention Large doeses of Vit B6 reverse effects Avoid use of CNS depressants

Rn assess CL recieving Gentamycin for which of the following toxicities

Nephrotoxicity Ototoxicity

The client experiences shock following a spinal cord injury. This type of shock is classified as:

Neurogenic.

The nurse administers filgrastim (Neupogen) to the client. The nurse explains that this drug is used in the treatment of:

Neutropenia, or neutropenia secondary to chemotherapy.

The rn acknowledges which RN interventions for client taking cipro

Obtain C&S Tell Cl to avoid taking Cipro w/ antacids Monitor the CL for hearing loss Encourage Fl to prevent crystalluria

a pt with toothache is unable to make it to dentist, wishes to utilize natural alternative. Pt most likely to benefit from

Oil Cloves- Common for Dentistry natural analgesic/Antiseptic

Drugs that narrow therapeutic ranges such as digoxin require plasma/serum levels to be monitored to avoid drug toxicity

Periodic interval

Antibiotics: Aminoglycosides Uses

Pseudomonas, E. coli

Your cl is taking a carbonic anhydrase inhibitor You will assess for which side effect

RENAL CALICULI

Side effects of opioid analgesics/ agonists include SELECT ALL THAT APPLY

SEDATION CONSTIPATION NAUSEA/VOMITING RESPIRATORY DEPRESSION DOES NOT INCLUDE HTN,URINARY FREQUENCY

What is an important thing that needs to be done with schizophrenics and why?

SET LIMITS with them because otherwise they won't manage their hygiene properly. (Poor hygiene skills). They may also break rules or be unintentionally rude (inform them of the mistake and help them regain control).

What interventions should be implemented for someone at risk for suicide?

Safety Precautions: Remove harmful objects, Close observation, 1 on 1, and Written contract.

Narcotics Nursing Considerations

Safety precautions Avoid alcohol Monitor VS Narcan

Your client abuses drugs- LSD Herroin which belong to what schedule

Scedule 1

If your goal is to decrease salivation AND promote sedation for a pt. perioperatively, which of the following is the BEST choice

Scopolamine

Antianxiety Side Effects

Sedatiosn, confustion, hepatic dysfunction

J.W(ibuprofen) teaching plan would include all the following EXCEPT

Suggesting a decreased fluid intake

a. Coronary thrombosis b. Acute myocardial infarction c. Deep vein thrombosis (DVT) d. Cerebrovascular accident (CVA) (stroke) e. Venous disorders

When a newly admitted client is placed on heparin, the nurse acknowledges that heparin is effective for preventing new clot formation in clients who have which disorder(s)? (Select all that apply.) a. Coronary thrombosis b. Acute myocardial infarction c. Deep vein thrombosis (DVT) d. Cerebrovascular accident (CVA) (stroke) e. Venous disorders

c. The client who has stopped taking a beta blocker due to cost.

Which client will the nurse assess first? a. The client who has been on beta blockers for 1 day. b. The client who is on a beta blocker and a thiazide diuretic. c. The client who has stopped taking a beta blocker due to cost. d. The client who is taking a beta blocker and Lasix (furosemide).

a. Alteration in cardiac output related to effects on the sympathetic nervous system

Which is a priority nursing diagnosis for a client taking an antihypertensive medication? a. Alteration in cardiac output related to effects on the sympathetic nervous system b. Knowledge deficit related to medication regimen c. Fatigue related to side effects of medication d. Alteration in comfort related to nonproductive cough

d. "I should stir the powder in as small an amount of fluid as possible to maintain potency of the medication."

Which statement indicates to the nurse that the client needs further medication instruction about colestipol (Colestid)? a. "The medication may cause constipation, so I will increase fluid and fiber in my diet." b. "I should take this medication 1 hour after or 4 hours before my other medications." c. "I might need to take fat-soluble vitamins to supplement my diet." d. "I should stir the powder in as small an amount of fluid as possible to maintain potency of the medication."

you read in the chart that your pt is allergic to one of his prescribed meds your first action is to

Withhold the medication call the provider

A nurse is monitoring a client newly diagnosed with diabetes mellitus for signs of complications. Which of the following, if exhibited in the client, would indicate hyperglycemia and warrant physician notification? a) polyuria b) diaphoresis c) hypertension d) increased pulse rate

a) polyuria Classic symptoms of hyperglycemia include polydipsia, polyuria, and polyphagia. Options B, C, and D are not signs of hyperglycemia.

A client who received heparin begins to bleed, and the physician calls for the antidote. The nurse knows that which is the antidote for heparin? a. protamine sulfate b. vitamin K c. aminocaproic acid d. vitamin C

a. protamine sulfate

Antianxiety Actions

affect neurotransmitters (CNS depressent)

Serum half life of drug is the time required

after absorption for 1/2 the drug to be eliminated

neuroliptic malignant syndrome occurs in what meds

antipsychotic

-"vir"

antiviral HIV

A client is taking warfarin 5 mg/day for atrial fibrillation. The client's international normalized ratio (INR) is 3.8. The nurse would consider the INR to be what? a. Within normal range b. Elevated INR range c. Low INR range d. Low average INR range

b. Elevated INR range

A client is taking hydrochlorothiazide 50 mg/day and digoxin 0.25 mg/day. What type of electrolyte imbalance does the nurse expect to occur? a. Hypocalcemia b. Hypokalemia c. Hyperkalemia d. Hypermagnesemia

b. Hypokalemia

Two days after surgery, an elderly client refuses a PRN dose of analgesic dose for fear of becoming "hooked." The nurse should respond by stating that: a. It is impossible to become hooked on PRN narcotics. b. Short-term use of narcotics is not likely to cause a person to become dependent on them. c. Side effects that occur in the elderly mean that medications will be discontinued as soon as possible. d. The elderly are least likely to become dependent on narcotics.

b. Short-term use of narcotics is not likely to cause a person to become dependent on them.

-"pam", -"lam"

benzodiazepine anxiety sedative anti-convulsent

The anxiolytic alprazolam xanax is from which drug group

benzodiazepines

Cardiac Glycosides Side Effects

bradycardia, nausea, vomiting, visual disturbances

Antitussive Agents Side Effects

bradycardia, respiratory depression, drowsiness, dizziness, anticholinergic effects

signs of Iodism

brassy taste burning sensation in the mouth soreness of the gums & teeth

Which of the following diabetes drugs acts by decreasing the amount of glucose produced by the liver? a. Sulfonylureas b. Meglitinides c. Biguanides d. Alpha-glucosidase inhibitors

c. Biguanides Biguanides, such as metformin, lower blood glucose by reducing the amount of glucose produced by the liver. Sulfonylureas and Meglitinides stimulate the beta cells of the pancreas to produce more insulin. Alpha-glucosidase inhibitors block the breakdown of starches and some sugars, which helps to reduce blood glucose levels

The nurse is teaching a client about clopidogrel (Plavix). What is important information to include? a. Constipation may occur. b. Hypotension may occur. c. Bleeding may increase when taken with aspirin. d. Normal dose is 25 mg tablet per day.

c. Bleeding may increase when taken with aspirin.

The nurse acknowledges that beta blockers are as effective as antianginals because they do what? a. Increase oxygen to the systemic circulation. b. Maintain heart rate and blood pressure. c. Decrease heart rate and decrease myocardial contractility. d. Decrease heart rate and increase myocardial contractility.

c. Decrease heart rate and decrease myocardial contractility.

A client is admitted to the emergency department with an acute myocardial infarction. Which drug category does the nurse expect to be given to the client early for the prevention of tissue necrosis following blood clot blockage in a coronary or cerebral artery? a. Anticoagulant agent b. Antiplatelet agent c. Thrombolytic agent d. Low-molecular-weight heparin (LMWH)

c. Thrombolytic agent

A classic drug interaction, greatly involving an increased bleeding time, involves warfarin and a. vitamin B-6 b. acetaminophen c. acetylsalicylic acid d. all of the above

c. acetylsalicylic acid

Fosphenytoin (Cerebyx) a. is a controversial agent for depression b. is used to control tremors due to Parkinsonism c. can be administered intravenously d. is ineffective after 5 days of therapy

c. can be administered intravenously

A client who incurred an arm injury describes his pain as "sharp and localized to the lower arm." The nurse recognizes that this type of pain would be relieved best by administration of which type of medication? a.) Muscle relaxant b.) Acetaminophen c.) Narcotic analgesics d.) Ice packs

c.) Narcotic analgesics Injury to tissues produces nociceptor pain, which usually responds to conventional analgesic pain medications such as opiates or NSAIDS.

A client in the ICU tells the nurse he is experiencing severe pain. Prior to administering a narcotic analgesic to this client, the nurse will conduct a pain assessment to include: a.) Pain b.) Nociception c.) Pain behaviors d.) Suffering

c.) pain behavior There is a theory that addresses pain as having four facets: nociception, pain, suffering, and pain behaviors. Of these four facets, only the fourth, pain behavior, can be observed. This nurse will only be able to assess the client's pain behavior in the pain assessment.

Major SE of Aluminum hydroxide (Amphojel), an antacid, is

constipation

Antidiarrheals: Side Effects

constipation, anticholinergic effects

Iron Preparations Side Effects

constipation, dark stools, tachycardia

A physician has prescribed propylthiouracil (PTU) for a client with hyperthyroidism and the nurse develops a plan of care for the client. A priority nursing assessment to be included in the plan regarding this medication is to assess for: a) relief of pain b) signs of renal toxicity c) signs and symptoms of hyperglycemia d) signs and symptoms of hypothyroidism

d) signs and symptoms of hypothyroidism Excessive dosing with propylthiouracil (PTU) may convert the client from a hyperthyroid state to a hypothyroid state. If this occurs, the dosage should be reduced. Temporary administration of thyroid hormone may be required. Propylthiouracil is not used for pain and does not cause hyperglycemia or renal toxicity.

The nurse administers morphine sulfate 4 mg IV to a client for treatment of severe pain. Which of the following assessments requires immediate nursing interventions? a.) Blood pressure 110/70 b.) The client is drowsy. c.) Pain is unrelieved in 15 minutes. d.) Respiratory rate 10/minute

d.) Respiratory rate 10/minute Opioids activate mu and kappa receptors that can cause profound respiratory depression. Respiratory rate should remain above 12. The BP is not significantly low. Drowsiness is an expected effect of morphine. Unrelieved pain warrants further assessment, but not as immediately as do decreased respirations.

Ethambutol (Myambutol) causes optic neuritis which does what

decreases visual acuity & the ability to discriminate between the colors red and green.

Laxatives/Stool Softeners Side Effects

diarrhea, dependence, abdominal cramps, hypermagnesemia

Anticholinergics Actions

dilates pupil, causes bronchodialtion adn decreased secreations, decreases mobility and GI secreations

Antibiotics: Tetracyclines Side Effects

discoloration of primary teeth if taken by the mother during pregnancy, glossitis, rash, phototoxic reaction

Thrombolytics Action

dissolves or lyses blood clots

Glimepiride (Amaryl) and alcohol can cause what reaction

disulfiram like reaction

Antiparkinson Agents Side Effects

dizziness, ataxia, atropine like effects

Diuretics Side Effects

dizziness, orth hypotension, leukopenia

Narcotics Side Effects

dizziness, sedation, respiratory depression, hypotension, constipation

Antidiarrheals: Nursing Considerations

do not use with abdominal pain monitor for urinary retention give 2h before or 3h after other medications

all signs of digoxin toxicity, a cardiac glycoside

double vision loss of appetite nausea bradycardia difficulty reading green or yellow vision seeing spots halos confusion vomiting diarrhea decreased libido impotence

earl signs of water intoxification (overhydration, hyponatremia)

drowsiness listelessness headache

all signs of water intoxification (overhydration, hyponatremia)

drowsiness listelessness headache decreased urination rapid weight gain confusion seizures coma

Antihistamines: Side Effects

drowsiness, dry mouth, photosensitivity, bronchospasm

Atypical Antipsychotic Agents Side Effects

extrapyramidal, anticholinergic, sedatives, ortho hypotension

Hypothyroidism signs

fatigue cold intolerance excessively dry skin

Antipyretic Uses

fever

serotonin syndrome

fever, muscle stiffness ALOC hallucinations

MAOI major adverse effect

hypertensive crisis

Antidepressants: MAOIs Side Effects

hypertensive crisis when taken with tyramine-containing foods (cheese, liver, beer, wine), photosensitivity

Excessive doses of levothyroxine (Synthroid) produce signs of

hyperthyroidism

Propylthiouracil (PTU) is used to treat

hyperthyroidism or Grave's disease

what is common following treatment for leukemias & lymphomas bc chemotherapy results in massive cell kill

hyperuricemia or increased uric acid level

what is the systemic manifestation in a pt w/ salicylism

hyperventilation

Mafenide acetate (Sulfamylon) needs to be monitored for what systemic effect?

hyperventilation -d/c med for 1-2 days

Repaglinife (Prandin) common SE

hypoglycemia

glucagon

hypoglycemic agents antidote

Antihypertensives: Calcium Channel Blockers Side Effects

hypotension, renal failure, angioedema

Antianginals Side Effects

hypotension, tachycardia, headache, dizziness

common SE of Bumetanide (Bumex)

hypotension; thus monitor BP

Thyroid Replacement Agents Uses

hypothyroidism, Graves' disease

After Cl w influenza makes which of the following statements: RN concludes that the Cl appropriatly understands principle of self admin of prescribed oral Ab.

i will continue to take the Ab as ordered even though i no longer have S/S

Tacrolimus (Prograf) is used in caution in pts w/

immunosuppressed renal function impairment hepatic function impairment pancreatic function impairment

Terbutaline is a bronchodilator and is used w/ caution in

impaired cardiac function DM HTN Hyperthryroidism Hx of seizures

Streptomycin -major thing to look for?

impaired sense of hearing

instruct CL not to d/c meds abrubtly Is included in what part of the RN Process

implementation

Antihypertensives: Calcium Channel Blockers Actions

inhibits movement of calcium across cell membrane

NSAIDS Action

inhibits prostaglandin synthesis

Antibiotics: Tetracyclines Actions

inhibits protein syntheis

Antibiotics: Aminoglycosides Action

inhibits protein synthesis in gram-negative bacteria

Diuretics Actions

inhibits reabsorption of sodium and water, blocks effects of aldosterone

Bone-Reabsorption Inhibitors: Biophosphonates Action

inhibits resorption of bone

Antidepressants: Tricyclics Actions

inhibits reuptake of neurotransmitters

Antibiotics: Cephalosporins Actions

inhibits synthesis of bacterial cell wall

Antibiotics: Penicillins Actions

inhibits synthesis of cell wall

Antidysrhythmics Action

interfere with electrical exciablility of heart

Antineoplastic: Antitumor Action

interferes with DNA and RNA sythesis

Antibiotics: Fluoroquinolones Actions

interferes with DNA replication in gram-negative bacteria

Atypical Antipsychotic Agents Actions

interferes with binding of dopamine in the brain

Antineoplastic: Vinca Alkaloids Action

interferes with cell division

Antiplatelet Agents Actions

interferes with platelet aggregation

Dantrolene (Dantrium) is a skeletal muscle relaxant and can cause

liver damage; thus monitor liver function studies

before starting INH, the nurse gathers a baseline of what lab?

liver enzyme levels

Before giving Valproic acid (Depakene), look at what results first?

liver function tests

Low protien level in blood decreases the # of protien binding sites and causes an increase in amount of free drug in the plasma, potential result in drug overdose. Examples of what can cause low protein levels include:

malnutrition, Elderly Both do not obtain suffecient nutrients.

Antineoplastic: Antimetabolites Side Effects

nausea, vomiting, oral ulcerations, bone marrow suppression, alopecia

Intrdermal Needle size,Location,Angle of admin

needle size 5/8 Location usually forearm angle 10-15*

Kanamycin sulfate (Kantrex) given for respiratory tract infection, what are the adverse reactions?

nephrotoxicity irreversible ototoxicity neurotoxicity

Thyroid Replacement Agents Side Effects

nervousness, tachycardia, weight loss

Antacids Action

neutralize gastric acids

Cholestyramine (Questran) is a bile acid sequestrant used to lower the cholestrol level and pt compliance is low due to

poor taste and patability

Bleomycin is an antineoplastic med that can cause interstitial pneumonitis which can progress to_____ thus monitor ____ function studies as well as hematological, hepatic, & renal function tests.

pulmonary fibrosis; pulmonary

Which organ is destroyed when administering radioactive I-131? a.) Pituitary gland b.) Adrenals c.) Parathyroid d.) Hypothalamus

radioactive I-131 destroys the thyroid because the thyroid cells are the only cells in the body that can take up iodine

Auranofin (Ridaura) is a gold preparation that is given orally for rheumatoid arthritis, what are the early signs & symptoms of toxic reaction of Auranofin (Ridaura) ?

rash purple blotches pruritus mouth lesions metallic taste in mouth

Antivirals Uses

recurrent HSV, HIV infections

Succinycholine(Anectine) neuromuscular block agent,given during surgery expected response would be

relaxation muscles- paralysis

cycloplegia

relaxation of the ciliary muscles

Antianginals Action

relaxes smooth muscle, decreases venous return

Succimer (Chemet) given for lead poisoning specifically for Chelatin therapy,what needs to be monitored and what lab?

renal function bc med is excreted by kidneys monitor BUN level

Amphotericin B (Fungizone) major adverse reaction

renal impairment monitor kidney function; watch for elevated serum creatinine

SE of Meperidine hydrochloride are

respiratory depression orthostatic hypotension tachycardia drowsiness mental clouding constipation urinary retention

Anticonvulsants Uses

seizures

Glucocorticoids Action

stimulates formation of glucose, alters immune response

Hypoglycemic Agent: Action

stimulates liver to change glycogen to glucose

Antineoplastic: Vinca Alkaloids Side Effects

stomatitis, alopecia, loss of deep tendon reflexes, bone marrow suppression

the injectable form of Bethanechol chloride (Urecholine) is given

subc

Before giving a thiazide diuretic like hydrochlorothiazide ask if they are allergic to?

sulfa

Drug used to treat seborrheic dermatitis

sulfacetamide sodium-isopto cetamide

Antitussive Agents Actions

suppress cough reflex by inhibiting cough reflex in medulla, decrease viscosity of secretions

Glucocorticoids Side Effects

susceptible to infection and masks infection, insomnia, hypoglycemia, hypokalemia, psychoses, depression, stunted growth

Hyperthyoidism signs

tachycardia chest pain tremors nervousness insomnia hyperthermia heat intolerance sweating

Isoproterenol hydrochloride is a adrenergic bronchodilator ; what are the SE

tachycardia hypertension chest pain dysrhythmias nervousness restlessness headache increased pulse increased BP

Bronchodilators Side Effects

tachycardia, dysrhythmias, palpitations, anticholinergic effects

Captopril

take 1hr before meals

which of the following drugs/foods are known to change the action of keflex

uricosurics

major SE of Baclofen (Lioresal)

urinary retention

Cyclobenzaprine hydrochloride (Flexeril) for muscle spasms has anticholinergic effects and is used in caution w/

urinary retention glaucoma increased intraocular pressure

Bethanechol chloride (Urecholine) can be hazardous to pts w/

urinary tract obstruction weakness of the bladder wall bc elevation of pressure within the urinary tract could rupture the bladder in pts w/ these conditions

Injection site well defined by bony prominence landmarks is:

ventogluteal -hip

SE of peripheral neuritis from INH can be minimized w/

vitamin B6

Antidiarrheals: Uses

vomiting

Aluminum hydroxide (Amphojel), an antacid, is given

w/ meals

phytonadione (Vit K)

warfarin antidote

Local anesthesia indicated for which procedures

Dental Diagnostic Suturing lacerations

Each time the pt requests an opioid analgesic , the RN must

Determine the exact location and intensity of the pain

Antacids Side Effects

constipation, diarrhea, acid rebound

Laxatives/Stool Softeners Uses

constipation, preparation for sx and procedures

Cytoprotective Agents Side Effects

constipation, vertigo

179.) A nurse provides medication instructions to a client who had a kidney transplant about therapy with cyclosporine (Sandimmune). Which statement by the client indicates a need for further instruction? 1. "I need to obtain a yearly influenza vaccine." 2. "I need to have dental checkups every 3 months." 3. "I need to self-monitor my blood pressure at home." 4. "I need to call the health care provider (HCP) if my urine volume decreases or my urine becomes cloudy."

1. "I need to obtain a yearly influenza vaccine." Rationale: Cyclosporine is an immunosuppressant medication. Because of the medication's effects, the client should not receive any vaccinations without first consulting the HCP. The client should report decreased urine output or cloudy urine, which could indicate kidney rejection or infection, respectively. The client must be able to self-monitor blood pressure to check for the side effect of hypertension. The client needs meticulous oral care and dental cleaning every 3 months to help prevent gingival hyperplasia.

10.) The clinic nurse is performing an admission assessment on a client. The nurse notes that the client is taking azelaic acid (Azelex). Because of the medication prescription, the nurse would suspect that the client is being treated for: 1. Acne 2. Eczema 3. Hair loss 4. Herpes simplex

1. Acne Rationale: Azelaic acid is a topical medication used to treat mild to moderate acne. The acid appears to work by suppressing the growth of Propionibacterium acnes and decreasing the proliferation of keratinocytes. Options 2, 3, and 4 are incorrect.

What interventions are used for someone with antisocial behavior?

ASSIGN THE MOST SKILLED NURSE TO THIS MANIPULATIVE PATIENT (no float nurse), set firm consistent limits, explain consequences of breaking rules, work consistently with staff to prevent splitting, and give positive feedback for appropriate behavior.

Sleep pattern disturbance Is included in what part of the RN Process

Assessment/Rn Dx

Laxatives/Stool Softeners Med Names

Cascara, Dulcolax, Colace, MOM

The SSRIs tend to be more popular than TCAs because they have fewer Side effects SSRIs

Cause Less sedation and fewer hypotensive side effects

NSAIDS Med Names

Motrin, Indocin, Naprosyn

Antipyschotic drugs are useful in the management of

Pyschotic illnesses

Zoloft

Selective serotonin reuptake inhibitor (SSRI) drug.can reduce your sex drive. its been nicknamed "so soft".

What are hallucinations? Ex?

Sensory impressions without external stimuli. Ex: Detox= seeing pink elephants (absolutely not there but seeing).

Decrease drug dependence and assess status

The parents of a patient receiving methylphenidate (Ritalin) express concern that the health care provider has suggested the child have a "holiday" from the drug. The nurse explains that the drug-free holiday is designed to: a. Reduce the risk of drug toxicity b. Allow the child's "normal" behavior to return c. Decrease drug dependence and assess status d. Prevent hypertensive crisis

-"tyline"

Tricyclic antidepressents

which of the following are not allowed by the Joint Commission

U, IU, trailing zero,Qd abreviations of most

treatment of Bethanechol chloride (Urecholine)

atropine sulfate subc or IV

before intiating thrombolytic therapy an important vital signs is

blood pressure

a laboratory test is influenced by tetracycline is

blood urea nitrogen BUN- Nephrotoxicity

Antineoplastic: Antitumor Uses

cancer

Biguanides

decrease glucose production by the liver

Cytoprotective Agents Uses

duodenal ulcer

Ataxia from ototoxicity reflects damage from what cranial nerve

eighth

Electrolytes Side Effects

electrolyte imbalances

-"zocin"

peripherally acting anti-adrenergic HTN take at bedtime

BIPOLAR DISORDER

once known as Manic depression- Shift from emotions of extreme depression to extreme rage & agitation.

Sucralfate (Carafate), gastric protectant, should be given

one hour before meals & at bedtime; time is given to allow a protective coating over the ulcer before food intake stimulates gastric acid production & mechanical irritation

Carbonic Anhydrase Inhibitors Uses

open-angle glaucoma

The Usual dose of dronabinol marinol is

5-7mg

Antineoplastic: Antimetabolites Med Names

5-FU, Methotrexate

Antibiotics: Macrolide Uses

acute infections, acne, URI, prophylaxis before dental procedures if allergic to PCN

. The nurse evaluates the effectiveness of client education as it relates to anaphylaxis therapy. Which of the following might indicate that teaching was successful? (Select all that apply.)

"I will carry an EpiPen to administer when I experience a hypersensitivity episode to peanuts." "I will seek emergency medical attention immediately if a single auto-injection of epinephrine fails to bring relief."

H2 receptor antagonists suppress secretion of GI acid, alleviate symptoms of heartburn and assist in preventing complications of peptic ulcer disease. These meds have an ending of

"tidine"

Beta Blockers

**** DO NOT give with ASTHMA, BRONCHIAL CONSTRICTIVE DISEASE!!!**** "LOL" Antenolol, metoprolol. labetolol Antihypertensive. Blocks beta-adrenergic receptors in heart, decreases excitability of the heart, reduces caridac workload and O2 consumption. Lowers BP. Check BP before giving (hypotension) *Orthostatic Hypotension * Do not D/C abruptly (2wks) *Blocks normal signs of Hypoglycemia (sweating, tachycardia). Monitor Blood glucose!!

The Right to education includes

*Client recieves correct information about the drug and how it relates to situation *Possible Side effects *Laboratory Monitoring

Salicylism signs

tinnitus dizziness hyperpnea psychological disturbances

How do you apply Crotamiton (Eurax) used for scabies?

-Massage the med into the skin from the chin downward. -apply a second application in 24 hours -followed by a cleansing bath 48 hours after the second applications -if needed treatment can be repeated in 7 days

Antihypertensives: Angiotensin II Receptor Blockers Med Names

-sartan

normal direct bilirubin level

0-0.3 mg/dL

A pt. is prescribed glycopyrrolate-rubinol 0.1mg IMThe drugs is available in a solution of 0.2mg/mL The nurse administers

0.1mg/0.2mg = 0.5mL

Digoxin Therapeutic Levels

0.5-2

therapeutic Digoxin (Lanoxin) level

0.5-2 ng/mL

maintenance serum levels of lithium are

0.6- 1.2 mEq/L

Azithromycin (Zithromax) given for chlamydial infection is given

1 hour before meals or 2 hours after

an insulin vile in current use can be kept at room temp for

1 month

31.) A community health nurse visits a client at home. Prednisone 10 mg orally daily has been prescribed for the client and the nurse reinforces teaching for the client about the medication. Which statement, if made by the client, indicates that further teaching is necessary? 1. "I can take aspirin or my antihistamine if I need it." 2. "I need to take the medication every day at the same time." 3. "I need to avoid coffee, tea, cola, and chocolate in my diet." 4. "If I gain more than 5 pounds a week, I will call my doctor."

1. "I can take aspirin or my antihistamine if I need it." Rationale: Aspirin and other over-the-counter medications should not be taken unless the client consults with the health care provider (HCP). The client needs to take the medication at the same time every day and should be instructed not to stop the medication. A slight weight gain as a result of an improved appetite is expected, but after the dosage is stabilized, a weight gain of 5 lb or more weekly should be reported to the HCP. Caffeine-containing foods and fluids need to be avoided because they may contribute to steroid-ulcer development.

77.) Phenytoin (Dilantin), 100 mg orally three times daily, has been prescribed for a client for seizure control. The nurse reinforces instructions regarding the medication to the client. Which statement by the client indicates an understanding of the instructions? 1. "I will use a soft toothbrush to brush my teeth." 2. "It's all right to break the capsules to make it easier for me to swallow them." 3. "If I forget to take my medication, I can wait until the next dose and eliminate that dose." 4. "If my throat becomes sore, it's a normal effect of the medication and it's nothing to be concerned about."

1. "I will use a soft toothbrush to brush my teeth." Rationale: Phenytoin (Dilantin) is an anticonvulsant. Gingival hyperplasia, bleeding, swelling, and tenderness of the gums can occur with the use of this medication. The client needs to be taught good oral hygiene, gum massage, and the need for regular dentist visits. The client should not skip medication doses, because this could precipitate a seizure. Capsules should not be chewed or broken and they must be swallowed. The client needs to be instructed to report a sore throat, fever, glandular swelling, or any skin reaction, because this indicates hematological toxicity.

100.) Saquinavir (Invirase) is prescribed for the client who is human immunodeficiency virus seropositive. The nurse reinforces medication instructions and tells the client to: 1. Avoid sun exposure. 2. Eat low-calorie foods. 3. Eat foods that are low in fat. 4. Take the medication on an empty stomach.

1. Avoid sun exposure. Rationale: Saquinavir (Invirase) is an antiretroviral (protease inhibitor) used with other antiretroviral medications to manage human immunodeficiency virus infection. Saquinavir is administered with meals and is best absorbed if the client consumes high-calorie, high-fat meals. Saquinavir can cause photosensitivity, and the nurse should instruct the client to avoid sun exposure.

Antineoplastic: Antimetabolites Uses

acute lymphatic leukemia, cancer of colon, breast, pancreas

145.) A nurse has a prescription to give a client albuterol (Proventil HFA) (two puffs) and beclomethasone dipropionate (Qvar) (nasal inhalation, two puffs), by metered-dose inhaler. The nurse administers the medication by giving the: 1. Albuterol first and then the beclomethasone dipropionate 2. Beclomethasone dipropionate first and then the albuterol 3. Alternating a single puff of each, beginning with the albuterol 4. Alternating a single puff of each, beginning with the beclomethasone dipropionate

1. Albuterol first and then the beclomethasone dipropionate Rationale: Albuterol is a bronchodilator. Beclomethasone dipropionate is a glucocorticoid. Bronchodilators are always administered before glucocorticoids when both are to be given on the same time schedule. This allows for widening of the air passages by the bronchodilator, which then makes the glucocorticoid more effective.

26.) Glimepiride (Amaryl) is prescribed for a client with diabetes mellitus. A nurse reinforces instructions for the client and tells the client to avoid which of the following while taking this medication? 1. Alcohol 2. Organ meats 3. Whole-grain cereals 4. Carbonated beverages

1. Alcohol Rationale: When alcohol is combined with glimepiride (Amaryl), a disulfiram-like reaction may occur. This syndrome includes flushing, palpitations, and nausea. Alcohol can also potentiate the hypoglycemic effects of the medication. Clients need to be instructed to avoid alcohol consumption while taking this medication. The items in options 2, 3, and 4 do not need to be avoided.

156.) A nurse is reviewing the laboratory results for a client receiving tacrolimus (Prograf). Which laboratory result would indicate to the nurse that the client is experiencing an adverse effect of the medication? 1. Blood glucose of 200 mg/dL 2. Potassium level of 3.8 mEq/L 3. Platelet count of 300,000 cells/mm3 4. White blood cell count of 6000 cells/mm3

1. Blood glucose of 200 mg/dL Rationale: A blood glucose level of 200 mg/dL is elevated above the normal range of 70 to 110 mg/dL and suggests an adverse effect. Other adverse effects include neurotoxicity evidenced by headache, tremor, insomnia; gastrointestinal (GI) effects such as diarrhea, nausea, and vomiting; hypertension; and hyperkalemia.

33.) The home health care nurse is visiting a client who was recently diagnosed with type 2 diabetes mellitus. The client is prescribed repaglinide (Prandin) and metformin (Glucophage) and asks the nurse to explain these medications. The nurse should reinforce which instructions to the client? Select all that apply. 1. Diarrhea can occur secondary to the metformin. 2. The repaglinide is not taken if a meal is skipped. 3. The repaglinide is taken 30 minutes before eating. 4. Candy or another simple sugar is carried and used to treat mild hypoglycemia episodes. 5. Metformin increases hepatic glucose production to prevent hypoglycemia associated with repaglinide. 6. Muscle pain is an expected side effect of metformin and may be treated with acetaminophen (Tylenol).

1. Diarrhea can occur secondary to the metformin. 2. The repaglinide is not taken if a meal is skipped. 3. The repaglinide is taken 30 minutes before eating. 4. Candy or another simple sugar is carried and used to treat mild hypoglycemia episodes. Rationale: Repaglinide is a rapid-acting oral hypoglycemic agent that stimulates pancreatic insulin secretion that should be taken before meals, and that should be withheld if the client does not eat. Hypoglycemia is a side effect of repaglinide and the client should always be prepared by carrying a simple sugar with her or him at all times. Metformin is an oral hypoglycemic given in combination with repaglinide and works by decreasing hepatic glucose production. A common side effect of metformin is diarrhea. Muscle pain may occur as an adverse effect from metformin but it might signify a more serious condition that warrants health care provider notification, not the use of acetaminophen.

193.) Sodium hypochlorite (Dakin's solution) is prescribed for a client with a leg wound containing purulent drainage. The nurse is assisting in developing a plan of care for the client and includes which of the following in the plan? 1. Ensure that the solution is freshly prepared before use. 2. Soak a sterile dressing with solution and pack into the wound. 3. Allow the solution to remain in the wound following irrigation. 4. Apply the solution to the wound and on normal skin tissue surrounding the wound.

1. Ensure that the solution is freshly prepared before use. Rationale: Dakin solution is a chloride solution that is used for irrigating and cleaning necrotic or purulent wounds. It can be used for packing necrotic wounds. It cannot be used to pack purulent wounds because the solution is inactivated by copious pus. It should not come into contact with healing or normal tissue, and it should be rinsed off immediately if used for irrigation. Solutions are unstable and the nurse must ensure that the solution has been prepared fresh before use. **Eliminate options 2 and 3 first because they are comparable or alike. It makes sense to ensure that the solution is freshly prepared; therefore, select option 1**

112.) A hospitalized client is started on phenelzine sulfate (Nardil) for the treatment of depression. The nurse instructs the client to avoid consuming which foods while taking this medication? Select all that apply. 1. Figs 2. Yogurt 3. Crackers 4. Aged cheese 5 Tossed salad 6. Oatmeal cookies

1. Figs 2. Yogurt 4. Aged cheese Rationale: Phenelzine sulfate (Nardil) is a monoamine oxidase inhibitor(MAOI). The client should avoid taking in foods that are high in tyramine. Use of these foods could trigger a potentially fatal hypertensive crisis. Some foods to avoid include yogurt, aged cheeses, smoked or processed meats, red wines, and fruits such as avocados, raisins, and figs.

91.) Cyclobenzaprine (Flexeril) is prescribed for a client to treat muscle spasms, and the nurse is reviewing the client's record. Which of the following disorders, if noted in the client's record, would indicate a need to contact the health care provider regarding the administration of this medication? 1. Glaucoma 2. Emphysema 3. Hyperthyroidism 4. Diabetes mellitus

1. Glaucoma Rationale: Because this medication has anticholinergic effects, it should be used with caution in clients with a history of urinary retention, angle-closure glaucoma, and increased intraocular pressure. Cyclobenzaprine hydrochloride should be used only for short-term 2- to 3-week therapy.

125.) A nurse is preparing to administer digoxin (Lanoxin), 0.125 mg orally, to a client with heart failure. Which vital sign is most important for the nurse to check before administering the medication? 1. Heart rate 2. Temperature 3. Respirations 4. Blood pressure

1. Heart rate Rationale: Digoxin is a cardiac glycoside that is used to treat heart failure and acts by increasing the force of myocardial contraction. Because bradycardia may be a clinical sign of toxicity, the nurse counts the apical heart rate for 1 full minute before administering the medication. If the pulse rate is less than 60 beats/minute in an adult client, the nurse would withhold the medication and report the pulse rate to the registered nurse, who would then contact the health care provider.

167.) A nurse prepares to reinforce instructions to a client who is taking allopurinol (Zyloprim). The nurse plans to include which of the following in the instructions? 1. Instruct the client to drink 3000 mL of fluid per day. 2. Instruct the client to take the medication on an empty stomach. 3. Inform the client that the effect of the medication will occur immediately. 4. Instruct the client that, if swelling of the lips occurs, this is a normal expected response.

1. Instruct the client to drink 3000 mL of fluid per day. Rationale: Allopurinol (Zyloprim) is an antigout medication used to decrease uric acid levels. Clients taking allopurinol are encouraged to drink 3000 mL of fluid a day. A full therapeutic effect may take 1 week or longer. Allopurinol is to be given with or immediately following meals or milk to prevent gastrointestinal irritation. If the client develops a rash, irritation of the eyes, or swelling of the lips or mouth, he or she should contact the health care provider because this may indicate hypersensitivity.

Rn needs to be aware Client returning from surgery recieved spinal anesthesia needs to be kept in which position for few hours (4)

Flat Bed

113.) A nurse is reinforcing discharge instructions to a client receiving sulfisoxazole. Which of the following would be included in the plan of care for instructions? 1. Maintain a high fluid intake. 2. Discontinue the medication when feeling better. 3. If the urine turns dark brown, call the health care provider immediately. 4. Decrease the dosage when symptoms are improving to prevent an allergic response.

1. Maintain a high fluid intake. Rationale: Each dose of sulfisoxazole should be administered with a full glass of water, and the client should maintain a high fluid intake. The medication is more soluble in alkaline urine. The client should not be instructed to taper or discontinue the dose. Some forms of sulfisoxazole cause the urine to turn dark brown or red. This does not indicate the need to notify the health care provider.

43.) A histamine (H2)-receptor antagonist will be prescribed for a client. The nurse understands that which medications are H2-receptor antagonists? Select all that apply. 1. Nizatidine (Axid) 2. Ranitidine (Zantac) 3. Famotidine (Pepcid) 4. Cimetidine (Tagamet) 5. Esomeprazole (Nexium) 6. Lansoprazole (Prevacid)

1. Nizatidine (Axid) 2. Ranitidine (Zantac) 3. Famotidine (Pepcid) 4. Cimetidine (Tagamet) Rationale: H2-receptor antagonists suppress secretion of gastric acid, alleviate symptoms of heartburn, and assist in preventing complications of peptic ulcer disease. These medications also suppress gastric acid secretions and are used in active ulcer disease, erosive esophagitis, and pathological hypersecretory conditions. The other medications listed are proton pump inhibitors. H2-receptor antagonists medication names end with -dine. Proton pump inhibitors medication names end with -zole.

12.) A nurse is caring for a client who is receiving an intravenous (IV) infusion of an antineoplastic medication. During the infusion, the client complains of pain at the insertion site. During an inspection of the site, the nurse notes redness and swelling and that the rate of infusion of the medication has slowed. The nurse should take which appropriate action? 1. Notify the registered nurse. 2. Administer pain medication to reduce the discomfort. 3. Apply ice and maintain the infusion rate, as prescribed. 4. Elevate the extremity of the IV site, and slow the infusion.

1. Notify the registered nurse. Rationale: When antineoplastic medications (Chemotheraputic Agents) are administered via IV, great care must be taken to prevent the medication from escaping into the tissues surrounding the injection site, because pain, tissue damage, and necrosis can result. The nurse monitors for signs of extravasation, such as redness or swelling at the insertion site and a decreased infusion rate. If extravasation occurs, the registered nurse needs to be notified; he or she will then contact the health care provider.

18.) The nurse is reviewing the history and physical examination of a client who will be receiving asparaginase (Elspar), an antineoplastic agent. The nurse consults with the registered nurse regarding the administration of the medication if which of the following is documented in the client's history? 1. Pancreatitis 2. Diabetes mellitus 3. Myocardial infarction 4. Chronic obstructive pulmonary disease

1. Pancreatitis Rationale: Asparaginase (Elspar) is contraindicated if hypersensitivity exists, in pancreatitis, or if the client has a history of pancreatitis. The medication impairs pancreatic function and pancreatic function tests should be performed before therapy begins and when a week or more has elapsed between administration of the doses. The client needs to be monitored for signs of pancreatitis, which include nausea, vomiting, and abdominal pain. The conditions noted in options 2, 3, and 4 are not contraindicated with this medication.

30.) A client with diabetes mellitus visits a health care clinic. The client's diabetes mellitus previously had been well controlled with glyburide (DiaBeta) daily, but recently the fasting blood glucose level has been 180 to 200 mg/dL. Which medication, if added to the client's regimen, may have contributed to the hyperglycemia? 1. Prednisone 2. Phenelzine (Nardil) 3. Atenolol (Tenormin) 4. Allopurinol (Zyloprim)

1. Prednisone Rationale: Prednisone may decrease the effect of oral hypoglycemics, insulin, diuretics, and potassium supplements. Option 2, a monoamine oxidase inhibitor, and option 3, a β-blocker, have their own intrinsic hypoglycemic activity. Option 4 decreases urinary excretion of sulfonylurea agents, causing increased levels of the oral agents, which can lead to hypoglycemia.

152.) Intravenous heparin therapy is prescribed for a client. While implementing this prescription, a nurse ensures that which of the following medications is available on the nursing unit? 1. Protamine sulfate 2. Potassium chloride 3. Phytonadione (vitamin K ) 4. Aminocaproic acid (Amicar)

1. Protamine sulfate Rationale: The antidote to heparin is protamine sulfate; it should be readily available for use if excessive bleeding or hemorrhage occurs. Potassium chloride is administered for a potassium deficit. Vitamin K is an antidote for warfarin sodium. Aminocaproic acid is the antidote for thrombolytic therapy.

102.) A client with human immunodeficiency virus is taking nevirapine (Viramune). The nurse should monitor for which adverse effects of the medication? Select all that apply. 1. Rash 2. Hepatotoxicity 3. Hyperglycemia 4. Peripheral neuropathy 5. Reduced bone mineral density

1. Rash 2. Hepatotoxicity Rationale: Nevirapine (Viramune) is a non-nucleoside reverse transcriptase inhibitors (NRTI) that is used to treat HIV infection. It is used in combination with other antiretroviral medications to treat HIV. Adverse effects include rash, Stevens-Johnson syndrome, hepatitis, and increased transaminase levels. Hyperglycemia, peripheral neuropathy, and reduced bone density are not adverse effects of this medication.

239.) Which of the following precautions will the nurse specifically take during the administration of ribavirin (Virazole) to a child with respiratory syncytial virus (RSV)? 1. Wearing goggles 2. Wearing a gown 3. Wearing a gown and a mask 4. Handwashing before administration

1. Wearing goggles Rationale: Some caregivers experience headaches, burning nasal passages and eyes, and crystallization of soft contact lenses as a result of administration of ribavirin. Specific to this medication is the use of goggles. A gown is not necessary. A mask may be worn. Handwashing is to be performed before and after any child contact.

Urinary tract spasms are commonly treated with which of the following drugs?

Flavoxate -Urispas

72.) Cinoxacin (Cinobac), a urinary antiseptic, is prescribed for the client. The nurse reviews the client's medical record and should contact the health care provider (HCP) regarding which documented finding to verify the prescription? Refer to chart. 1. Renal insufficiency 2. Chest x-ray: normal 3. Blood glucose, 102 mg/dL 4. Folic acid (vitamin B6) 0.5 mg, orally daily

1. Renal insufficiency Rationale: Cinoxacin should be administered with caution in clients with renal impairment. The dosage should be reduced, and failure to do so could result in accumulation of cinoxacin to toxic levels. Therefore the nurse would verify the prescription if the client had a documented history of renal insufficiency. The laboratory and diagnostic test results are normal findings. Folic acid (vitamin B6) may be prescribed for a client with renal insufficiency to prevent anemia.

92.) In monitoring a client's response to disease-modifying antirheumatic drugs (DMARDs), which findings would the nurse interpret as acceptable responses? Select all that apply. 1. Symptom control during periods of emotional stress 2. Normal white blood cell counts, platelet, and neutrophil counts 3. Radiological findings that show nonprogression of joint degeneration 4. An increased range of motion in the affected joints 3 months into therapy 5. Inflammation and irritation at the injection site 3 days after injection is given 6. A low-grade temperature upon rising in the morning that remains throughout the day

1. Symptom control during periods of emotional stress 2. Normal white blood cell counts, platelet, and neutrophil counts 3. Radiological findings that show nonprogression of joint degeneration 4. An increased range of motion in the affected joints 3 months into therapy Rationale: Because emotional stress frequently exacerbates the symptoms of rheumatoid arthritis, the absence of symptoms is a positive finding. DMARDs are given to slow progression of joint degeneration. In addition, the improvement in the range of motion after 3 months of therapy with normal blood work is a positive finding. Temperature elevation and inflammation and irritation at the medication injection site could indicate signs of infection.

A young woman makes an appointment to see a physician at the clinic. She complains of tiredness, weight gain, muscle aches and pains, and constipation. The physician will likely order: 1. T3 and T4 serum level laboratory tests. 2. glucose tolerance test. 3. cerebral computed tomography (CT) scan. 4. adrenocortical stimulating test.

1. T3 and T4 serum level laboratory tests. These complaints are strongly suggestive of thyroid disorder; T3 and T4 laboratory tests are the most useful diagnostic tests.

149.) A client taking fexofenadine (Allegra) is scheduled for allergy skin testing and tells the nurse in the health care provider's office that a dose was taken this morning. The nurse determines that: 1. The client should reschedule the appointment. 2. A lower dose of allergen will need to be injected. 3. A higher dose of allergen will need to be injected. 4. The client should have the skin test read a day later than usual.

1. The client should reschedule the appointment. Rationale: Fexofenadine is an antihistamine, which provides relief of symptoms caused by allergy. Antihistamines should be discontinued for at least 3 days (72 hours) before allergy skin testing to avoid false-negative readings. This client should have the appointment rescheduled for 3 days after discontinuing the medication.

221.) A nurse is reviewing the health care provider's prescriptions for an adult client who has been admitted to the hospital following a back injury. Carisoprodol (Soma) is prescribed for the client to relieve the muscle spasms; the health care provider has prescribed 350 mg to be administered four times a day. When preparing to give this medication, the nurse determines that this dosage is: 1. The normal adult dosage 2. A lower than normal dosage 3. A higher than normal dosage 4. A dosage requiring further clarification

1. The normal adult dosage Rationale: The normal adult dosage for carisoprodol is 350 mg orally three or four times daily.

21.) A nurse is assisting with caring for a client with cancer who is receiving cisplatin. Select the adverse effects that the nurse monitors for that are associated with this medication. Select all that apply. 1. Tinnitus 2. Ototoxicity 3. Hyperkalemia 4. Hypercalcemia 5. Nephrotoxicity 6. Hypomagnesemia

1. Tinnitus 2. Ototoxicity 5. Nephrotoxicity 6. Hypomagnesemia Rationale: Cisplatin is an alkylating medication. Alkylating medications are cell cycle phase-nonspecific medications that affect the synthesis of DNA by causing the cross-linking of DNA to inhibit cell reproduction. Cisplatin may cause ototoxicity, tinnitus, hypokalemia, hypocalcemia, hypomagnesemia, and nephrotoxicity. Amifostine (Ethyol) may be administered before cisplatin to reduce the potential for renal toxicity.

213.) A client is admitted to the hospital with complaints of back spasms. The client states, "I have been taking two or three aspirin every 4 hours for the past week and it hasn't helped my back." Aspirin intoxication is suspected. Which of the following complaints would indicate aspirin intoxication? 1. Tinnitus 2. Constipation 3. Photosensitivity 4. Abdominal cramps

1. Tinnitus Rationale: Mild intoxication with acetylsalicylic acid (aspirin) is called salicylism and is commonly experienced when the daily dosage is higher than 4 g. Tinnitus (ringing in the ears) is the most frequently occurring effect noted with intoxication. Hyperventilation may occur because salicylate stimulates the respiratory center. Fever may result because salicylate interferes with the metabolic pathways involved with oxygen consumption and heat production. Options 2, 3, and 4 are incorrect.

3.) Salicylic acid is prescribed for a client with a diagnosis of psoriasis. The nurse monitors the client, knowing that which of the following would indicate the presence of systemic toxicity from this medication? 1. Tinnitus 2. Diarrhea 3. Constipation 4. Decreased respirations

1. Tinnitus Rationale: Salicylic acid is absorbed readily through the skin, and systemic toxicity (salicylism) can result. Symptoms include tinnitus, dizziness, hyperpnea, and psychological disturbances. Constipation and diarrhea are not associated with salicylism.

8.) A client with severe acne is seen in the clinic and the health care provider (HCP) prescribes isotretinoin. The nurse reviews the client's medication record and would contact the (HCP) if the client is taking which medication? 1. Vitamin A 2. Digoxin (Lanoxin) 3. Furosemide (Lasix) 4. Phenytoin (Dilantin)

1. Vitamin A Rationale: Isotretinoin is a metabolite of vitamin A and can produce generalized intensification of isotretinoin toxicity. Because of the potential for increased toxicity, vitamin A supplements should be discontinued before isotretinoin therapy. Options 2, 3, and 4 are not contraindicated with the use of isotretinoin.

24.) A nurse is reinforcing teaching for a client regarding how to mix regular insulin and NPH insulin in the same syringe. Which of the following actions, if performed by the client, indicates the need for further teaching? 1. Withdraws the NPH insulin first 2. Withdraws the regular insulin first 3. Injects air into NPH insulin vial first 4. Injects an amount of air equal to the desired dose of insulin into the vial

1. Withdraws the NPH insulin first Rationale: When preparing a mixture of regular insulin with another insulin preparation, the regular insulin is drawn into the syringe first. This sequence will avoid contaminating the vial of regular insulin with insulin of another type. Options 2, 3, and 4 identify the correct actions for preparing NPH and regular insulin.

195.) A nurse is caring for a client who is taking metoprolol (Lopressor). The nurse measures the client's blood pressure (BP) and apical pulse (AP) immediately before administration. The client's BP is 122/78 mm/Hg and the AP is 58 beats/min. Based on this data, which of the following is the appropriate action? 1. Withhold the medication. 2. Notify the registered nurse immediately. 3. Administer the medication as prescribed. 4. Administer half of the prescribed medication.

1. Withhold the medication. Rationale: Metoprolol (Lopressor) is classified as a beta-adrenergic blocker and is used in the treatment of hypertension, angina, and myocardial infarction. Baseline nursing assessments include measurement of BP and AP immediately before administration. If the systolic BP is below 90 mm/Hg and the AP is below 60 beats/min, the nurse should withhold the medication and document this action. Although the registered nurse should be informed of the client's vital signs, it is not necessary to do so immediately. The medication should not be administered because the data is outside of the prescribed parameters for this medication. The nurse should not administer half of the medication, or alter any dosages at any point in time.

234.) A hospitalized client is started on phenelzine sulfate (Nardil) for the treatment of depression. At lunchtime, a tray is delivered to the client. Which food item on the tray will the nurse remove? 1. Yogurt 2. Crackers 3. Tossed salad 4. Oatmeal cookies

1. Yogurt Rationale: Phenelzine sulfate is a monoamine oxidase inhibitor (MAOI). The client should avoid taking in foods that are high in tyramine. These foods could trigger a potentially fatal hypertensive crisis. Foods to avoid include yogurt, aged cheeses, smoked or processed meats, red wines, and fruits such as avocados, raisins, or figs.

Nursing interventions for a patient receiving enoxaparin (Lovenox) may include: (Select all that apply) 1. teaching the patient or family to give subcutaneous injections at home 2. teaching the patient or family not take any OTC drugs without first consulting with the health care provider 3. teaching the patient to observe for unexplained bleeding such as pink, red, or dark brown urine or blood gums 4. teaching the patient to monitor for the development of DVT 5. teaching the importance of drinking grapefruit juice daily

1. teaching the patient or family to give subcutaneous injections at home 2. teaching the patient or family not take any OTC drugs without first consulting with the health care provider 3. teaching the patient to observe for unexplained bleeding such as pink, red, or dark brown urine or blood gums 4. teaching the patient to monitor for the development of DVT

1. Walter, teenage patient is admitted to the hospital because of acetaminophen (Tylenol) overdose. Overdoses of acetaminophen can precipitate life-threatening abnormalities in which of the following organs? a. Lungs b. Liver c. Kidney d. Adrenal Glands

1.Answer B. Acetaminophen is extensively metabolized by pathways in the liver. Toxic doses of acetaminophen deplete hepatic glutathione, resulting in accumulation of the intermediate agent, quinine, which leads to hepatic necrosis. Prolonged use of acetaminophen may result in an increased risk of renal dysfunction, but a single overdose does not precipitate life-threatening problems in the respiratory system, renal system, or adrenal glands.

1. An infection in a central venous access device is not eliminated by giving antibiotics through the catheter. How would bacterial glycocalyx contribute to this? a. It protects the bacteria from antibiotic and immunologic destruction. b. Glycocalyx neutralizes the antibiotic rendering it ineffective. c. It competes with the antibiotic for binding sites on the microbe. d. Glycocalyx provides nutrients for microbial growth.

1.Answer C. Glycocalyx is a viscous polysaccharide or polypeptide slime that covers microbes. It enhances adherence to surfaces, resists phagocytic engulfment by the white blood cells, and prevents antibiotics from contacting the microbe. Glycocalyx does not have the effects in options B-D.

6.) The burn client is receiving treatments of topical mafenide acetate (Sulfamylon) to the site of injury. The nurse monitors the client, knowing that which of the following indicates that a systemic effect has occurred? 1.Hyperventilation 2.Elevated blood pressure 3.Local pain at the burn site 4.Local rash at the burn site

1.Hyperventilation Rationale: Mafenide acetate is a carbonic anhydrase inhibitor and can suppress renal excretion of acid, thereby causing acidosis. Clients receiving this treatment should be monitored for signs of an acid-base imbalance (hyperventilation). If this occurs, the medication should be discontinued for 1 to 2 days. Options 3 and 4 describe local rather than systemic effects. An elevated blood pressure may be expected from the pain that occurs with a burn injury.

Dantrolene sodium (Dantrium) given for spasticity, what is the oral maintenance dosage?

100 mg two to four times daily

1g to mg

1000 mg

For maintence control of seizures in the adult, the usual dose of phenytoin is

100mg TID

11. A nurse is preparing the client's morning NPH insulin dose and notices a clumpy precipitate inside the insulin vial. The nurse should: a. draw up and administer the dose b. shake the vial in an attempt to disperse the clumps c. draw the dose from a new vial d. warm the bottle under running water to dissolve the clump

11. C. The nurse should always inspect the vial of insulin before use for solution changes that may signify loss of potency. NPH insulin is normally uniformly cloudy. Clumping, frosting, and precipitates are signs of insulin damage. In this situation, because potency is questionable, it is safer to discard the vial and draw up the dose from a new vial.

11. The primary complication of a central venous access device (CVAD) is: a. Thrombus formation in the vein. b. Pain and discomfort. c. Infection. d. Occlusion of the catheter as the result of an intra-lumen clot.

11.Answer C. A foreign body in a blood vessel increases the risk of infection. Catheters that come outside the body have an even higher risk of infection. Most infections are caused by skin bacteria. Other infective organisms include yeasts and fungi. Options 1 and 4 are complications of a CVAD but are not the primary problem. Once placed, these lines do not cause pain and discomfort.

11. Mandy, a patient calls the clinic today because he is taking atrovastatin (Lipitor) to treat his high cholesterol and is having pain in both of his legs. You instruct him to: a. Stop taking the drug and make an appointment to be seen next week. b. Continue taking the drug and make an appointment to be seen next week. c. Stop taking the drug and come to the clinic to be seen today. d. Walk for at least 30 minutes and call if symptoms continue.

11.Answer C. Muscle aches, soreness, and weakness may be early signs of myopathy such as rhabdomyolysis associated with the HMG-CoA reducatase class of antilipemic agents. This patient will need an immediate evaluation to rule out myopathy. Additional doses may exacerbate the problem. Exercise will not reverse myopathy and delays diagnosis.

12. A client with histoplasmosis has an order for ketoconazole (Nizoral). The nurse teaches the client to do which of the following while taking this medication? a. take the medication on an empty stomach b. take the medication with an antacid c. avoid exposure to sunlight d. limit alcohol to 2 ounces per day

12. C. The client should be taught that ketoconazole is an antifungal medication. It should be taken with food or milk. Antacids should be avoided for 2 hours after it is taken because gastric acid is needed to activate the medication. The client should avoid concurrent use of alcohol, because the medication is hepatotoxic. The client should also avoid exposure to sunlight, because the medication increases photosensitivity.

12. Which of the following adverse effects is associated with levothyroxine (Synthroid) therapy? a. Tachycardia b. Bradycardia c. Hypotension d. Constipation

12.Answer A. Levothyroxine, especially in higher doses, can induce hyperthyroid-like symptoms including tachycardia. An agent that increases the basal metabolic rate would not be expected to induce a slow heart rate. Hypotension would be a side effect of bradycardia. Constipation is a symptom of hypothyroid disease.

12. Nurse Blessy is doing some patient education related to a patient's central venous access device. Which of the following statements will the nurse make to the patient? a. "These type of devices are essentially risk free." b. "These devices seldom work for more than a week or two necessitating replacement." c. "The dressing should only the changed by your doctor." d. "Heparin in instilled into the lumen of the catheter to decrease the risk of clotting."

12.Answer D. A solution containing heparin is used to reduce catheter clotting and maintain patency. The concentration of heparin used depends on the patient's age, comorbidities, and the frequency of catheter access/flushing. Although patients have few complications, the device is not risk free. Patients may develop infection, catheter clots, vascular obstruction, pneumothorax, hemothorax, or mechanical problems (catheter breakage). Strict adherence to protocol enhances the longevity of central access devices. They routinely last weeks to months and sometimes years. The patient will be taught how to perform dressing changes at home.

A pt is prescribed oral morphine sulfate 12mg- the dose available is 10mg/ml the RN administers

12/10=1.2

13. A nurse has taught a client taking a xanthine bronchodilator about beverages to avoid. The nurse determines that the client understands the information if the client chooses which of the following beverages from the dietary menu? a. chocolate milk b. cranberry juice c. coffee d. cola

13. B. Cola, coffee, and chocolate contain xanthine and should be avoided by the client taking a xanthine bronchodilator. This could lead to an increased incidence of cardiovascular and central nervous system side effects that can occur with the use of these types of bronchodilators.

13. The chemotherapeutic DNA alkylating agents such as nitrogen mustards are effective because they: a. Cross-link DNA strands with covalent bonds between alkyl groups on the drug and guanine bases on DNA. b. Have few, if any, side effects. c. Are used to treat multiple types of cancer. d. Are cell cycle-specific agents.

13.Answer A. Alkylating agents are highly reactive chemicals that introduce alkyl radicals into biologically active molecules and thereby prevent their proper functioning, replication, and transcription. Alkylating agents have numerous side effects including alopecia, nausea, vomiting, and myelosuppression. Nitrogen mustards have a broad spectrum of activity against chronic lymphocytic leukemia, non-Hodgkin's lymphoma, and breast and ovarian cancer, but they are effective chemotherapeutic agents because of DNA cross-linkage. Alkylating agents are noncell cycle-specific agents.

13. Which of the following adverse effects is specific to the biguanide diabetic drug metformin (Glucophage) therapy? a. Hypoglycemia b. GI distress c. Lactic acidosis d. Somulence

13.Answer C. Lactic acidosis is the most dangerous adverse effect of metformin administration with death resulting in approximately 50 percent of individuals who develop lactic acidosis while on this drug. Metformin does not induce insulin production; thus, administration does not result in hypoglycemic events. Some nausea, vomiting, and diarrhea may develop but is usually not severe. NVD is not specific for metformin. Metformin does not induce sleepiness.

A pt has recived a Dx of myasthenia gravis and begins a regimen of ambenonium. The nursing assessment is important becouse the dose of the drug

IS frequently increased or decreased early in therapy- helps to determine therapuetic levels.

14. A client is taking famotidine (Pepcid) asks the home care nurse what would be the best medication to take for a headache. The nurse tells the client that it would be best to take: a. aspirin (acetylsalicylic acid, ASA) b. ibuprofen (Motrin) c. acetaminophen (Tylenol) d. naproxen (Naprosyn)

14. C. The client is taking famotidine, a histamine receptor antagonist. This implies that the client has a disorder characterized by gastrointestinal (GI) irritation. The only medication of the ones listed in the options that is not irritating to the GI tract is acetaminophen. The other medications could aggravate an already existing GI problem.

14. Hormonal agents are used to treat some cancers. An example would be: a. Thyroxine to treat thyroid cancer. b. ACTH to treat adrenal carcinoma. c. Estrogen antagonists to treat breast cancer. d. Glucagon to treat pancreatic carcinoma.

14.Answer C. Estrogen antagonists are used to treat estrogen hormone-dependent cancer, such as breast carcinoma. A well-known estrogen antagonist used in breast cancer therapy is tamoxifen (Nolvadex). This drug, in combination with surgery and other chemotherapeutic drugs reduces breast cancer recurrence by 30 percent. Estrogen antagonists can also be administered to prevent breast cancer in women who have a strong family history of the disease. Thyroxine is a natural thyroid hormone. It does not treat thyroid cancer. ACTH is an anterior pituitary hormone, which stimulates the adrenal glands to release glucocorticoids. It does not treat adrenal cancer. Glucagon is a pancreatic alpha cell hormone, which stimulates glycogenolysis and gluconeogenesis. It does not treat pancreatic cancer.

14. The most serious adverse effect of tricyclic antidepressant (TCA) overdose is: a. Seizures. b. Hyperpyrexia. c. Metabolic acidosis. d. Cardiac arrhythmias.

14.Answer D. Excessive ingestion of TCAs result in life-threatening wide QRS complex tachycardia. TCA overdose can induce seizures, but they are typically not life-threatening. TCAs do not cause an elevation in body temperature. TCAs do not cause metabolic acidosis.

15. A nurse is planning dietary counseling for the client taking triamterene (Dyrenium). The nurse plans to include which of the following in a list of foods that are acceptable? a. baked potato b. bananas c. oranges d. pears canned in water

15. D. Triamterene is a potassium-sparing diuretic, and clients taking this medication should be cautioned against eating foods that are high in potassium, including many vegetables, fruits, and fresh meats. Because potassium is very water-soluble, foods that are prepared in water are often lower in potassium.

15. The nurse is aware that the following solutions is routinely used to flush an IV device before and after the administration of blood to a patient is: a. 0.9 percent sodium chloride b. 5 percent dextrose in water solution c. Sterile water d. Heparin sodium

15.Answer A. 0.9 percent sodium chloride is normal saline. This solution has the same osmolarity as blood. Its use prevents red cell lysis. The solutions given in options 2 and 3 are hypotonic solutions and can cause red cell lysis. The solution in option 4 may anticoagulate the patient and result in bleeding.

15. Chemotherapeutic agents often produce a certain degree of myelosuppression including leukopenia. Leukopenia does not present immediately but is delayed several days to weeks because: a. The patient's hemoglobin and hematocrit are normal. b. Red blood cells are affected first. c. Folic acid levels are normal. d. The current white cell count is not affected by chemotherapy.

15.Answer D. The time required to clear circulating cells before the effect that chemotherapeutic drugs have on precursor cell maturation in the bone marrow becomes evident. Leukopenia is an abnormally low white blood cell count. Answers A-C pertain to red blood cells.

normal platelet count

150,000 - 400,000 mm

Usual adult dose of NARDIL

15mgTID

16. Currently, there is no way to prevent myelosuppression. However, there are medications available to elicit a more rapid bone marrow recovery. An example is: a. Epoetin alfa (Epogen, Procrit). b. Glucagon. c. Fenofibrate (Tricor). d. Lamotrigine (Lamictal).

16.Answer A. Epoetin alfa (Epogen, Procrit) is a recombinant form of endogenous erythropoietin, a hematopoietic growth factor normally produced by the kidney that is used to induce red blood cell production in the bone marrow and reduce the need for blood transfusion. Glucagon is a pancreatic alpha cell hormone, which cause glycogenolysis and gluconeogenesis. Fenofibrate (Tricor) is an antihyperlipidemic agent that lowers plasma triglycerides. Lamotrigine (Lamictal) is an anticonvulsant.

16. Cris asks the nurse whether all donor blood products are cross-matched with the recipient to prevent a transfusion reaction. Which of the following always require cross-matching? a. packed red blood cells b. platelets c. plasma d. granulocytes

16.Answer A. Red blood cells contain antigens and antibodies that must be matched between donor and recipient. The blood products in options 2-4 do not contain red cells. Thus, they require no cross-match.

17. A month after receiving a blood transfusion an immunocompromised male patient develops fever, liver abnormalities, a rash, and diarrhea. The nurse would suspect this patient has: a. Nothing related to the blood transfusion. b. Graft-versus-host disease (GVHD). c. Myelosuppression. d. An allergic response to a recent medication.

17.Answer B. GVHD occurs when white blood cells in donor blood attack the tissues of an immunocompromised recipient. This process can occur within a month of the transfusion. Options 1 and 4 may be a thought, but the nurse must remember that immunocompromised transfusion recipients are at risk for GVHD.

18. A nurse is providing instructions to a client regarding quinapril hydrochloride (Accupril). The nurse tells the client: a. to take the medication with food only b. to rise slowly from a lying to a sitting position c. to discontinue the medication if nausea occurs d. that a therapeutic effect will be noted immediately

18. B. Accupril is an angiotensin-converting enzyme (ACE) inhibitor. It is used in the treatment of hypertension. The client should be instructed to rise slowly from a lying to sitting position and to permit the legs to dangle from the bed momentarily before standing to reduce the hypotensive effect. The medication does not need to be taken with meals. It may be given without regard to food. If nausea occurs, the client should be instructed to take a noncola carbonated beverage and salted crackers or dry toast. A full therapeutic effect may be noted in 1 to 2 weeks.

18. Serotonin release stimulates vomiting following chemotherapy. Therefore, serotonin antagonists are effective in preventing and treating nausea and vomiting related to chemotherapy. An example of an effective serotonin antagonist antiemetic is: a. ondansetron (Zofran). b. fluoxetine (Prozac). c. paroxetine (Paxil). d. sertraline (Zoloft).

18.Answer A. Chemotherapy often induces vomiting centrally by stimulating the chemoreceptor trigger zone (CTZ) and peripherally by stimulating visceral afferent nerves in the GI tract. Ondansetron (Zofran) is a serotonin antagonist that bocks the effects of serotonin and prevents and treats nausea and vomiting. It is especially useful in single-day highly emetogenic cancer chemotherapy (for example, cisplatin). The agents in options 2-4 are selective serotonin reuptake inhibitors. They increase the available levels of serotonin.

18. Jonas comes into the local blood donation center. He says he is here to donate platelets only today. The nurse knows this process is called: a. Directed donation. b. Autologous donation. c. Allogenic donation. d. Apheresis.

18.Answer D. The process of apheresis involves removal of whole blood from a donor. Within an instrument that is essentially designed as a centrifuge, the components of whole blood are separated. One of the separated portions is then withdrawn, and the remaining components are retransfused into the donor. Directed donation is collected from a blood donor other than the recipient, but the donor is known to the recipient and is usually a family member or friend. Autologous donation is the collection and reinfusion of the patient's own blood. Allogenic donation is collected from a blood donor other than the recipient.

The physian orders acetaminophen elixir 180 mg orally- acetaminophen elixir is available in a 120mg/mL solution. The RN administers?

180/120= 1.5 mL

19. Methotrexate, the most widely used antimetabolite in cancer chemotherapy does not penetrate the central nervous system (CNS). To treat CNS disease this drug must be administered: a. Intravenously. b. Subcutaneously. c. Intrathecally. d. By inhalation.

19.Answer C. With intrathecal administration chemotherapy is injected through the theca of the spinal cord and into the subarachnoid space entering into the cerebrospinal fluid surrounding the brain and spinal cord. The methods in options A, B, and D are ineffective because the medication cannot enter the CNS.

LP is 53 years old and recieving nalidxic acid(neg GRAM ) for a chronic UTI The Dose of NegGram for long term use is

1g bid

The nurse instructs a patient about how insulin affects blood glucose. Arrange the events in sequence. 1. Beta cells are stimulated to release insulin. 2. Glucose enters the bloodstream. 3. Glycogen is converted to glucose by alpha cells (glycogenesis). 4. Glycogen is stored in the liver. 5. Insulin transports glucose to muscle cells.

2 1 5 4 3

what is included in the triple therapy for Helicobacter pylori infection

2 antibacterial drugs & a proton pump inhibitor like Esomeprazole (Nexium)

7.) Isotretinoin is prescribed for a client with severe acne. Before the administration of this medication, the nurse anticipates that which laboratory test will be prescribed? 1. Platelet count 2. Triglyceride level 3. Complete blood count 4. White blood cell count

2. Triglyceride level Rationale: Isotretinoin can elevate triglyceride levels. Blood triglyceride levels should be measured before treatment and periodically thereafter until the effect on the triglycerides has been evaluated. Options 1, 3, and 4 do not need to be monitored specifically during this treatment.

Insulin: Combination Peak

2-12h

After several days of IV medication,JA's medication is changed to the oral form. Oral doses of phenytoin are generally scheduled:

2-3 times daily

PH is taking MAOI for chronic anxiety and fear PH begins taking phenelzine(nardil) This medication is generally scheduled to be taken

2-3times daily

Insulin: Fast Acting Peak

2-4h

183.) A client who received a kidney transplant is taking azathioprine (Imuran), and the nurse provides instructions about the medication. Which statement by the client indicates a need for further instructions? 1. "I need to watch for signs of infection." 2. "I need to discontinue the medication after 14 days of use." 3. "I can take the medication with meals to minimize nausea." 4. "I need to call the health care provider (HCP) if more than one dose is missed."

2. "I need to discontinue the medication after 14 days of use." Rationale: Azathioprine is an immunosuppressant medication that is taken for life. Because of the effects of the medication, the client must watch for signs of infection, which are reported immediately to the HCP. The client should also call the HCP if more than one dose is missed. The medication may be taken with meals to minimize nausea.

194.) A nurse provides instructions to a client regarding the use of tretinoin (Retin-A). Which statement by the client indicates the need for further instructions? 1. "Optimal results will be seen after 6 weeks." 2. "I should apply a very thin layer to my skin." 3. "I should wash my hands thoroughly after applying the medication." 4. "I should cleanse my skin thoroughly before applying the medication."

2. "I should apply a very thin layer to my skin." Rationale: Tretinoin is applied liberally to the skin. The hands are washed thoroughly immediately after applying. Therapeutic results should be seen after 2 to 3 weeks but may not be optimal until after 6 weeks. The skin needs to be cleansed thoroughly before applying the medication.

A patient has come to the doctor's office after finding out that her blood glucose level was 135 mg/dL. She states that she had not eaten before the test and was told to come and see her doctor. She asks you if she has diabetes. The nurse responds: 1. "Having a fasting serum glucose that high certainly indicates diabetes." 2. "That test indicates that we need to do more tests that are specific for diabetes." 3. "How do you feel? Do you have any other signs of diabetes?" 4. "Do you have a family history of diabetes, stroke, or heart disease? We need to know before making a diagnosis."

2. "That test indicates that we need to do more tests that are specific for diabetes The nurse needs to answer the patient's question in a way that gives information and is not misleading. Although 135 is high, there may be a nonpathologic explanation. More tests should be done to evaluate the patient.

The patient receiving heparin therapy asks how the "blood thinner" works. The best response by the nurse would be: 1. "heparin makes the blood less thick" 2. "heparin does not thin the blood but prevents clots from forming as easily in the blood vessels" 3. "heparin decreases the number of platelets so that blood clots more slowly" 4. "heparin dissolves the clot"

2. "heparin does not thin the blood but prevents clots form forming as easily in the blood vessels"

78.) A client is taking phenytoin (Dilantin) for seizure control and a sample for a serum drug level is drawn. Which of the following indicates a therapeutic serum drug range? 1. 5 to 10 mcg/mL 2. 10 to 20 mcg/mL 3. 20 to 30 mcg/mL 4. 30 to 40 mcg/mL

2. 10 to 20 mcg/mL Rationale: The therapeutic serum drug level range for phenytoin (Dilantin) is 10 to 20 mcg/mL. ** A helpful hint may be to remember that the theophylline therapeutic range and the acetaminophen (Tylenol) therapeutic range are the same as the phenytoin (Dilantin) therapeutic range.**

215.) A client with rheumatoid arthritis is taking acetylsalicylic acid (aspirin) on a daily basis. Which medication dose should the nurse expect the client to be taking? 1. 1 g daily 2. 4 g daily 3. 325 mg daily 4. 1000 mg daily

2. 4 g daily Rationale: Aspirin may be used to treat the client with rheumatoid arthritis. It may also be used to reduce the risk of recurrent transient ischemic attack (TIA) or brain attack (stroke) or reduce the risk of myocardial infarction (MI) in clients with unstable angina or a history of a previous MI. The normal dose for clients being treated with aspirin to decrease thrombosis and MI is 300 to 325 mg/day. Clients being treated to prevent TIAs are usually prescribed 1.3 g/day in two to four divided doses. Clients with rheumatoid arthritis are treated with 3.6 to 5.4 g/day in divided doses. **Eliminate options 1 and 4 because they are alike**

64.) Nalidixic acid (NegGram) is prescribed for a client with a urinary tract infection. On review of the client's record, the nurse notes that the client is taking warfarin sodium (Coumadin) daily. Which prescription should the nurse anticipate for this client? 1. Discontinuation of warfarin sodium (Coumadin) 2. A decrease in the warfarin sodium (Coumadin) dosage 3. An increase in the warfarin sodium (Coumadin) dosage 4. A decrease in the usual dose of nalidixic acid (NegGram)

2. A decrease in the warfarin sodium (Coumadin) dosage Rationale: Nalidixic acid can intensify the effects of oral anticoagulants by displacing these agents from binding sites on plasma protein. When an oral anticoagulant is combined with nalidixic acid, a decrease in the anticoagulant dosage may be needed.

74.) A client with myasthenia gravis is receiving pyridostigmine (Mestinon). The nurse monitors for signs and symptoms of cholinergic crisis caused by overdose of the medication. The nurse checks the medication supply to ensure that which medication is available for administration if a cholinergic crisis occurs? 1. Vitamin K 2. Atropine sulfate 3. Protamine sulfate 4. Acetylcysteine (Mucomyst)

2. Atropine sulfate Rationale: The antidote for cholinergic crisis is atropine sulfate. Vitamin K is the antidote for warfarin (Coumadin). Protamine sulfate is the antidote for heparin, and acetylcysteine (Mucomyst) is the antidote for acetaminophen (Tylenol).

86.) A nurse is reinforcing discharge instructions to a client receiving baclofen (Lioresal). Which of the following would the nurse include in the instructions? 1. Restrict fluid intake. 2. Avoid the use of alcohol. 3. Stop the medication if diarrhea occurs. 4. Notify the health care provider if fatigue occurs.

2. Avoid the use of alcohol. Rationale: Baclofen is a central nervous system (CNS) depressant. The client should be cautioned against the use of alcohol and other CNS depressants, because baclofen potentiates the depressant activity of these agents. Constipation rather than diarrhea is an adverse effect of baclofen. It is not necessary to restrict fluids, but the client should be warned that urinary retention can occur. Fatigue is related to a CNS effect that is most intense during the early phase of therapy and diminishes with continued medication use. It is not necessary that the client notify the health care provider if fatigue occurs.

9.) The nurse is applying a topical corticosteroid to a client with eczema. The nurse would monitor for the potential for increased systemic absorption of the medication if the medication were being applied to which of the following body areas? 1. Back 2. Axilla 3. Soles of the feet 4. Palms of the hands

2. Axilla Rationale: Topical corticosteroids can be absorbed into the systemic circulation. Absorption is higher from regions where the skin is especially permeable (scalp, axilla, face, eyelids, neck, perineum, genitalia), and lower from regions in which permeability is poor (back, palms, soles).

199.) A nurse is applying a topical glucocorticoid to a client with eczema. The nurse monitors for systemic absorption of the medication if the medication is being applied to which of the following body areas? 1. Back 2. Axilla 3. Soles of the feet 4. Palms of the hands

2. Axilla Rationale: Topical glucocorticoids can be absorbed into the systemic circulation. Absorption is higher from regions where the skin is especially permeable (scalp, axillae, face, eyelids, neck, perineum, genitalia), and lower from regions where penetrability is poor (back, palms, soles). **Eliminate options 3 and 4 because these body areas are similar in terms of skin characteristics**

123.) A nurse is planning to administer amlodipine (Norvasc) to a client. The nurse plans to check which of the following before giving the medication? 1. Respiratory rate 2. Blood pressure and heart rate 3. Heart rate and respiratory rate 4. Level of consciousness and blood pressure

2. Blood pressure and heart rate Rationale: Amlodipine is a calcium channel blocker. This medication decreases the rate and force of cardiac contraction. Before administering a calcium channel blocking agent, the nurse should check the blood pressure and heart rate, which could both decrease in response to the action of this medication. This action will help to prevent or identify early problems related to decreased cardiac contractility, heart rate, and conduction. **amlodipine is a calcium channel blocker, and this group of medications decreases the rate and force of cardiac contraction. This in turn lowers the pulse rate and blood pressure.**

224.) Neuroleptic malignant syndrome is suspected in a client who is taking chlorpromazine. Which medication would the nurse prepare in anticipation of being prescribed to treat this adverse effect related to the use of chlorpromazine? 1. Protamine sulfate 2. Bromocriptine (Parlodel) 3. Phytonadione (vitamin K) 4. Enalapril maleate (Vasotec)

2. Bromocriptine (Parlodel) Rationale: Bromocriptine is an antiparkinsonian prolactin inhibitor used in the treatment of neuroleptic malignant syndrome. Vitamin K is the antidote for warfarin (Coumadin) overdose. Protamine sulfate is the antidote for heparin overdose. Enalapril maleate is an antihypertensive used in the treatment of hypertension.

20.) The client with metastatic breast cancer is receiving tamoxifen. The nurse specifically monitors which laboratory value while the client is taking this medication? 1. Glucose level 2. Calcium level 3. Potassium level 4. Prothrombin time

2. Calcium level Rationale: Tamoxifen may increase calcium, cholesterol, and triglyceride levels. Before the initiation of therapy, a complete blood count, platelet count, and serum calcium levels should be assessed. These blood levels, along with cholesterol and triglyceride levels, should be monitored periodically during therapy. The nurse should assess for hypercalcemia while the client is taking this medication. Signs of hypercalcemia include increased urine volume, excessive thirst, nausea, vomiting, constipation, hypotonicity of muscles, and deep bone and flank pain.

Insulin: Combination Duration

24h

When administering a timed released medication to a pt , the RN must be aware that

IT SHOULD NOT BE CRUSHED

170.) Atenolol hydrochloride (Tenormin) is prescribed for a hospitalized client. The nurse should perform which of the following as a priority action before administering the medication? 1. Listen to the client's lung sounds. 2. Check the client's blood pressure. 3. Check the recent electrolyte levels. 4. Assess the client for muscle weakness.

2. Check the client's blood pressure. Rationale: Atenolol hydrochloride is a beta-blocker used to treat hypertension. Therefore the priority nursing action before administration of the medication is to check the client's blood pressure. The nurse also checks the client's apical heart rate. If the systolic blood pressure is below 90 mm Hg or the apical pulse is 60 beats per minute or lower, the medication is withheld and the registered nurse and/or health care provider is notified. The nurse would check baseline renal and liver function tests. The medication may cause weakness, and the nurse would assist the client with activities if weakness occurs. **Beta-blockers have "-lol" at the end of the medication name**

63.) A client with coronary artery disease complains of substernal chest pain. After checking the client's heart rate and blood pressure, a nurse administers nitroglycerin, 0.4 mg, sublingually. After 5 minutes, the client states, "My chest still hurts." Select the appropriate actions that the nurse should take. Select all that apply. 1. Call a code blue. 2. Contact the registered nurse. 3. Contact the client's family. 4. Assess the client's pain level. 5. Check the client's blood pressure. 6. Administer a second nitroglycerin, 0.4 mg, sublingually.

2. Contact the registered nurse. 4. Assess the client's pain level. 5. Check the client's blood pressure. 6. Administer a second nitroglycerin, 0.4 mg, sublingually. Rationale: The usual guideline for administering nitroglycerin tablets for a hospitalized client with chest pain is to administer one tablet every 5 minutes PRN for chest pain, for a total dose of three tablets. The registered nurse should be notified of the client's condition, who will then notify the health care provider as appropriate. Because the client is still complaining of chest pain, the nurse would administer a second nitroglycerin tablet. The nurse would assess the client's pain level and check the client's blood pressure before administering each nitroglycerin dose. There are no data in the question that indicate the need to call a code blue. In addition, it is not necessary to contact the client's family unless the client has requested this.

2. The physician orders penicillin for a patient with streptococcal pharyngitis. The nurse administers the drug as ordered, and the patient has an allergic reaction. The nurse checks the medication order sheet and finds that the patient is allergic to penicillin. Legal responsibility for the error is: a. only the nurse's—she should have checked the allergies before administering the medication. b. only the physician's—she gave the order, the nurse is obligated to follow it. c. only the pharmacist's—he should alert the floor to possible allergic reactions. d. the pharmacist, physician, and nurse are all liable for the mistake

2. D. The physician, nurse, and pharmacist all are licensed professionals and share responsibility for errors.

84.) Baclofen (Lioresal) is prescribed for the client with multiple sclerosis. The nurse assists in planning care, knowing that the primary therapeutic effect of this medication is which of the following? 1. Increased muscle tone 2. Decreased muscle spasms 3. Increased range of motion 4. Decreased local pain and tenderness

2. Decreased muscle spasms Rationale: Baclofen is a skeletal muscle relaxant and central nervous system depressant and acts at the spinal cord level to decrease the frequency and amplitude of muscle spasms in clients with spinal cord injuries or diseases and in clients with multiple sclerosis. Options 1, 3, and 4 are incorrect.

209.) A client with multiple sclerosis is receiving diazepam (Valium), a centrally acting skeletal muscle relaxant. Which of the following would indicate that the client is experiencing a side effect related to this medication? 1. Headache 2. Drowsiness 3. Urinary retention 4. Increased salivation

2. Drowsiness Rationale: Incoordination and drowsiness are common side effects resulting from this medication. Options 1, 3, and 4 are incorrect.

133.) A nurse is monitoring a client receiving desmopressin acetate (DDAVP) for adverse effects to the medication. Which of the following indicates the presence of an adverse effect? 1. Insomnia 2. Drowsiness 3. Weight loss 4. Increased urination

2. Drowsiness Rationale: Water intoxication (overhydration) or hyponatremia is an adverse effect to desmopressin. Early signs include drowsiness, listlessness, and headache. Decreased urination, rapid weight gain, confusion, seizures, and coma also may occur in overhydration. **Recall that this medication is used to treat diabetes insipidus to eliminate weight loss and increased urination.**

240.) A client with Parkinson's disease has been prescribed benztropine (Cogentin). The nurse monitors for which gastrointestinal (GI) side effect of this medication? 1. Diarrhea 2. Dry mouth 3. Increased appetite 4. Hyperactive bowel sounds

2. Dry mouth Rationale: Common GI side effects of benztropine therapy include constipation and dry mouth. Other GI side effects include nausea and ileus. These effects are the result of the anticholinergic properties of the medication. **Eliminate options 1 and 4 because they are comparable or alike. Recall that the medication is an anticholinergic, which causes dry mouth**

108.) A nurse is performing a follow-up teaching session with a client discharged 1 month ago who is taking fluoxetine (Prozac). What information would be important for the nurse to gather regarding the adverse effects related to the medication? 1. Cardiovascular symptoms 2. Gastrointestinal dysfunctions 3. Problems with mouth dryness 4. Problems with excessive sweating

2. Gastrointestinal dysfunctions Rationale: The most common adverse effects related to fluoxetine include central nervous system (CNS) and gastrointestinal (GI) system dysfunction. This medication affects the GI system by causing nausea and vomiting, cramping, and diarrhea. Options 1, 3, and 4 are not adverse effects of this medication.

41.) The client has been taking omeprazole (Prilosec) for 4 weeks. The ambulatory care nurse evaluates that the client is receiving optimal intended effect of the medication if the client reports the absence of which symptom? 1. Diarrhea 2. Heartburn 3. Flatulence 4. Constipation

2. Heartburn Rationale: Omeprazole is a proton pump inhibitor classified as an antiulcer agent. The intended effect of the medication is relief of pain from gastric irritation, often called heartburn by clients. Omeprazole is not used to treat the conditions identified in options 1, 3, and 4.

67 y/o pt treated for hisptoplasmosis recieving amohotericin B Med generally admin which route

IV

Morphine

IV for chest pain Check Respirations

131.) The nurse is reinforcing medication instructions to a client with breast cancer who is receiving cyclophosphamide (Neosar). The nurse tells the client to: 1. Take the medication with food. 2. Increase fluid intake to 2000 to 3000 mL daily. 3. Decrease sodium intake while taking the medication. 4. Increase potassium intake while taking the medication.

2. Increase fluid intake to 2000 to 3000 mL daily. Rationale: Hemorrhagic cystitis is a toxic effect that can occur with the use of cyclophosphamide. The client needs to be instructed to drink copious amounts of fluid during the administration of this medication. Clients also should monitor urine output for hematuria. The medication should be taken on an empty stomach, unless gastrointestinal (GI) upset occurs. Hyperkalemia can result from the use of the medication; therefore the client would not be told to increase potassium intake. The client would not be instructed to alter sodium intake.

162.) Carbamazepine (Tegretol) is prescribed for a client with a diagnosis of psychomotor seizures. The nurse reviews the client's health history, knowing that this medication is contraindicated if which of the following disorders is present? 1. Headaches 2. Liver disease 3. Hypothyroidism 4. Diabetes mellitus

2. Liver disease Rationale: Carbamazepine (Tegretol) is contraindicated in liver disease, and liver function tests are routinely prescribed for baseline purposes and are monitored during therapy. It is also contraindicated if the client has a history of blood dyscrasias. It is not contraindicated in the conditions noted in the incorrect options.

89.) A nurse is reviewing the laboratory studies on a client receiving dantrolene sodium (Dantrium). Which laboratory test would identify an adverse effect associated with the administration of this medication? 1. Creatinine 2. Liver function tests 3. Blood urea nitrogen 4. Hematological function tests

2. Liver function tests Rationale: Dose-related liver damage is the most serious adverse effect of dantrolene. To reduce the risk of liver damage, liver function tests should be performed before treatment and periodically throughout the treatment course. It is administered in the lowest effective dosage for the shortest time necessary. **Eliminate options 1 and 3 because these tests both assess kidney function.**

161.) A nurse is caring for a client with severe back pain, and codeine sulfate has been prescribed for the client. Which of the following would the nurse include in the plan of care while the client is taking this medication? 1. Restrict fluid intake. 2. Monitor bowel activity. 3. Monitor for hypertension. 4. Monitor peripheral pulses.

2. Monitor bowel activity. Rationale: While the client is taking codeine sulfate, an opioid analgesic, the nurse would monitor vital signs and monitor for hypotension. The nurse should also increase fluid intake, palpate the bladder for urinary retention, auscultate bowel sounds, and monitor the pattern of daily bowel activity and stool consistency (codeine can cause constipation). The nurse should monitor respiratory status and initiate breathing and coughing exercises. In addition, the nurse monitors the effectiveness of the pain medication.

204.) A client receives a dose of edrophonium (Enlon). The client shows improvement in muscle strength for a period of time following the injection. The nurse interprets that this finding is compatible with: 1. Multiple sclerosis 2. Myasthenia gravis 3. Muscular dystrophy 4. Amyotrophic lateral sclerosis

2. Myasthenia gravis Rationale: Myasthenia gravis can often be diagnosed based on clinical signs and symptoms. The diagnosis can be confirmed by injecting the client with a dose of edrophonium . This medication inhibits the breakdown of an enzyme in the neuromuscular junction, so more acetylcholine binds to receptors. If the muscle is strengthened for 3 to 5 minutes after this injection, it confirms a diagnosis of myasthenia gravis. Another medication, neostigmine (Prostigmin), also may be used because its effect lasts for 1 to 2 hours, providing a better analysis. For either medication, atropine sulfate should be available as the antidote.

169.) Insulin glargine (Lantus) is prescribed for a client with diabetes mellitus. The nurse tells the client that it is best to take the insulin: 1. 1 hour after each meal 2. Once daily, at the same time each day 3. 15 minutes before breakfast, lunch, and dinner 4. Before each meal, on the basis of the blood glucose level

2. Once daily, at the same time each day Rationale: Insulin glargine is a long-acting recombinant DNA human insulin used to treat type 1 and type 2 diabetes mellitus. It has a 24-hour duration of action and is administered once a day, at the same time each day.

47.) A client has been taking isoniazid (INH) for 2 months. The client complains to a nurse about numbness, paresthesias, and tingling in the extremities. The nurse interprets that the client is experiencing: 1. Hypercalcemia 2. Peripheral neuritis 3. Small blood vessel spasm 4. Impaired peripheral circulation

2. Peripheral neuritis Rationale: A common side effect of the TB drug INH is peripheral neuritis. This is manifested by numbness, tingling, and paresthesias in the extremities. This side effect can be minimized by pyridoxine (vitamin B6) intake. Options 1, 3, and 4 are incorrect.

50.) A nurse has given a client taking ethambutol (Myambutol) information about the medication. The nurse determines that the client understands the instructions if the client states that he or she will immediately report: 1. Impaired sense of hearing 2. Problems with visual acuity 3. Gastrointestinal (GI) side effects 4. Orange-red discoloration of body secretions

2. Problems with visual acuity Rationale: Ethambutol causes optic neuritis, which decreases visual acuity and the ability to discriminate between the colors red and green. This poses a potential safety hazard when a client is driving a motor vehicle. The client is taught to report this symptom immediately. The client is also taught to take the medication with food if GI upset occurs. Impaired hearing results from antitubercular therapy with streptomycin. Orange-red discoloration of secretions occurs with rifampin (Rifadin).

186.) A nurse prepares to administer sodium polystyrene sulfonate (Kayexalate) to a client. Before administering the medication, the nurse reviews the action of the medication and understands that it: 1. Releases bicarbonate in exchange for primarily sodium ions 2. Releases sodium ions in exchange for primarily potassium ions 3. Releases potassium ions in exchange for primarily sodium ions 4. Releases sodium ions in exchange for primarily bicarbonate ions

2. Releases sodium ions in exchange for primarily potassium ions Rationale: Sodium polystyrene sulfonate is a cation exchange resin used in the treatment of hyperkalemia. The resin either passes through the intestine or is retained in the colon. It releases sodium ions in exchange for primarily potassium ions. The therapeutic effect occurs 2 to 12 hours after oral administration and longer after rectal administration.

Route for drug absorption that has the greatest bioavailability

IV- fastest

48.) A client is to begin a 6-month course of therapy with isoniazid (INH). A nurse plans to teach the client to: 1. Drink alcohol in small amounts only. 2. Report yellow eyes or skin immediately. 3. Increase intake of Swiss or aged cheeses. 4. Avoid vitamin supplements during therapy.

2. Report yellow eyes or skin immediately. Rationale: INH is hepatotoxic, and therefore the client is taught to report signs and symptoms of hepatitis immediately (which include yellow skin and sclera). For the same reason, alcohol should be avoided during therapy. The client should avoid intake of Swiss cheese, fish such as tuna, and foods containing tyramine because they may cause a reaction characterized by redness and itching of the skin, flushing, sweating, tachycardia, headache, or lightheadedness. The client can avoid developing peripheral neuritis by increasing the intake of pyridoxine (vitamin B6) during the course of INH therapy for TB.

216.) A nurse is caring for a client with gout who is taking Colcrys (colchicine). The client has been instructed to restrict the diet to low-purine foods. Which of the following foods should the nurse instruct the client to avoid while taking this medication? 1. Spinach 2. Scallops 3. Potatoes 4. Ice cream

2. Scallops Rationale: Colchicine is a medication used for clients with gout to inhibit the reabsorption of uric acid by the kidney and promote excretion of uric acid in the urine. Uric acid is produced when purine is catabolized. Clients are instructed to modify their diet and limit excessive purine intake. High-purine foods to avoid or limit include organ meats, roe, sardines, scallops, anchovies, broth, mincemeat, herring, shrimp, mackerel, gravy, and yeast.

114.) A postoperative client requests medication for flatulence (gas pains). Which medication from the following PRN list should the nurse administer to this client? 1. Ondansetron (Zofran) 2. Simethicone (Mylicon) 3. Acetaminophen (Tylenol) 4. Magnesium hydroxide (milk of magnesia, MOM)

2. Simethicone (Mylicon) Rationale: Simethicone is an antiflatulent used in the relief of pain caused by excessive gas in the gastrointestinal tract. Ondansetron is used to treat postoperative nausea and vomiting. Acetaminophen is a nonopioid analgesic. Magnesium hydroxide is an antacid and laxative.

172.) A nurse provides dietary instructions to a client who will be taking warfarin sodium (Coumadin). The nurse tells the client to avoid which food item? 1. Grapes 2. Spinach 3. Watermelon 4. Cottage cheese

2. Spinach Rationale: Warfarin sodium is an anticoagulant. Anticoagulant medications act by antagonizing the action of vitamin K, which is needed for clotting. When a client is taking an anticoagulant, foods high in vitamin K often are omitted from the diet. Vitamin K-rich foods include green, leafy vegetables, fish, liver, coffee, and tea.

188.) The nurse should anticipate that the most likely medication to be prescribed prophylactically for a child with spina bifida (myelomeningocele) who has a neurogenic bladder would be: 1. Prednisone 2. Sulfisoxazole 3. Furosemide (Lasix) 4. Intravenous immune globulin (IVIG)

2. Sulfisoxazole Rationale: A neurogenic bladder prevents the bladder from completely emptying because of the decrease in muscle tone. The most likely medication to be prescribed to prevent urinary tract infection would be an antibiotic. A common prescribed medication is sulfisoxazole. Prednisone relieves allergic reactions and inflammation rather than preventing infection. Furosemide promotes diuresis and decreases edema caused by congestive heart failure. IVIG assists with antibody production in immunocompromised clients.

147.) A client with a prescription to take theophylline (Theo-24) daily has been given medication instructions by the nurse. The nurse determines that the client needs further information about the medication if the client states that he or she will: 1. Drink at least 2 L of fluid per day. 2. Take the daily dose at bedtime. 3. Avoid changing brands of the medication without health care provider (HCP) approval. 4. Avoid over-the-counter (OTC) cough and cold medications unless approved by the HCP.

2. Take the daily dose at bedtime. Rationale: The client taking a single daily dose of theophylline, a xanthine bronchodilator, should take the medication early in the morning. This enables the client to have maximal benefit from the medication during daytime activities. In addition, this medication causes insomnia. The client should take in at least 2 L of fluid per day to decrease viscosity of secretions. The client should check with the physician before changing brands of the medication. The client also checks with the HCP before taking OTC cough, cold, or other respiratory preparations because they could cause interactive effects, increasing the side effects of theophylline and causing dysrhythmias.

45.) A client has a prescription to take guaifenesin (Humibid) every 4 hours, as needed. The nurse determines that the client understands the most effective use of this medication if the client states that he or she will: 1. Watch for irritability as a side effect. 2. Take the tablet with a full glass of water. 3. Take an extra dose if the cough is accompanied by fever. 4. Crush the sustained-release tablet if immediate relief is needed.

2. Take the tablet with a full glass of water. Rationale: Guaifenesin is an expectorant. It should be taken with a full glass of water to decrease viscosity of secretions. Sustained-release preparations should not be broken open, crushed, or chewed. The medication may occasionally cause dizziness, headache, or drowsiness as side effects. The client should contact the health care provider if the cough lasts longer than 1 week or is accompanied by fever, rash, sore throat, or persistent headache.

214.) A health care provider initiates carbidopa/levodopa (Sinemet) therapy for the client with Parkinson's disease. A few days after the client starts the medication, the client complains of nausea and vomiting. The nurse tells the client that: 1. Taking an antiemetic is the best measure to prevent the nausea. 2. Taking the medication with food will help to prevent the nausea. 3. This is an expected side effect of the medication and will decrease over time. 4. The nausea and vomiting will decrease when the dose of levodopa is stabilized.

2. Taking the medication with food will help to prevent the nausea. Rationale: If carbidopa/levodopa is causing nausea and vomiting, the nurse would tell the client that taking the medication with food will prevent the nausea. Additionally, the client should be instructed not to take the medication with a high-protein meal because the high-protein will affect absorption. Antiemetics from the phenothiazine class should not be used because they block the therapeutic action of dopamine. **eliminate options 3 and 4 because they are comparable or alike**

b. Blocking angiotensin II from AT1 receptors

A client is prescribed losartan (Cozaar). The nurse teaches the client that an angiotensin II receptor blocker (ARB) acts by doing what? a. Inhibiting angiotensin-converting enzyme b. Blocking angiotensin II from AT1 receptors c. Preventing the release of angiotensin I d. Promoting the release of aldosterone

57.) A nurse is monitoring a client who is taking propranolol (Inderal LA). Which data collection finding would indicate a potential serious complication associated with propranolol? 1. The development of complaints of insomnia 2. The development of audible expiratory wheezes 3. A baseline blood pressure of 150/80 mm Hg followed by a blood pressure of 138/72 mm Hg after two doses of the medication 4. A baseline resting heart rate of 88 beats/min followed by a resting heart rate of 72 beats/min after two doses of the medication

2. The development of audible expiratory wheezes Rationale: Audible expiratory wheezes may indicate a serious adverse reaction, bronchospasm. β-Blockers may induce this reaction, particularly in clients with chronic obstructive pulmonary disease or asthma. Normal decreases in blood pressure and heart rate are expected. Insomnia is a frequent mild side effect and should be monitored.

28.) The health care provider (HCP) prescribes exenatide (Byetta) for a client with type 1 diabetes mellitus who takes insulin. The nurse knows that which of the following is the appropriate intervention? 1. The medication is administered within 60 minutes before the morning and evening meal. 2. The medication is withheld and the HCP is called to question the prescription for the client. 3. The client is monitored for gastrointestinal side effects after administration of the medication. 4. The insulin is withdrawn from the Penlet into an insulin syringe to prepare for administration.

2. The medication is withheld and the HCP is called to question the prescription for the client. Rationale: Exenatide (Byetta) is an incretin mimetic used for type 2 diabetes mellitus only. It is not recommended for clients taking insulin. Hence, the nurse should hold the medication and question the HCP regarding this prescription. Although options 1 and 3 are correct statements about the medication, in this situation the medication should not be administered. The medication is packaged in prefilled pens ready for injection without the need for drawing it up into another syringe.

165.) The client has been on treatment for rheumatoid arthritis for 3 weeks. During the administration of etanercept (Enbrel), it is most important for the nurse to assess: 1. The injection site for itching and edema 2. The white blood cell counts and platelet counts 3. Whether the client is experiencing fatigue and joint pain 4. A metallic taste in the mouth and a loss of appetite

2. The white blood cell counts and platelet counts Rationale: Infection and pancytopenia are adverse effects of etanercept (Enbrel). Laboratory studies are performed before and during treatment. The appearance of abnormal white blood cell counts and abnormal platelet counts can alert the nurse to a potential life-threatening infection. Injection site itching is a common occurrence following administration of the medication. In early treatment, residual fatigue and joint pain may still be apparent. A metallic taste and loss of appetite are not common signs of side effects of this medication.

83.) The client has been on treatment for rheumatoid arthritis for 3 weeks. During the administration of etanercept (Enbrel), it is most important for the nurse to check: 1. The injection site for itching and edema 2. The white blood cell counts and platelet counts 3. Whether the client is experiencing fatigue and joint pain 4. A metallic taste in the mouth, with a loss of appetite

2. The white blood cell counts and platelet counts Rationale: Infection and pancytopenia are side effects of etanercept (Enbrel). Laboratory studies are performed before and during drug treatment. The appearance of abnormal white blood cell counts and abnormal platelet counts can alert the nurse to a potentially life-threatening infection. Injection site itching is a common occurrence following administration. A metallic taste with loss of appetite are not common signs of side effects of this medication.

126.) A nurse is caring for a client who has been prescribed furosemide (Lasix) and is monitoring for adverse effects associated with this medication. Which of the following should the nurse recognize as a potential adverse effect Select all that apply. 1. Nausea 2. Tinnitus 3. Hypotension 4. Hypokalemia 5. Photosensitivity 6. Increased urinary frequency

2. Tinnitus 3. Hypotension 4. Hypokalemia Rationale: Furosemide is a loop diuretic; therefore, an expected effect is increased urinary frequency. Nausea is a frequent side effect, not an adverse effect. Photosensitivity is an occasional side effect. Adverse effects include tinnitus (ototoxicity), hypotension, and hypokalemia and occur as a result of sudden volume depletion.

51.) Cycloserine (Seromycin) is added to the medication regimen for a client with tuberculosis. Which of the following would the nurse include in the client-teaching plan regarding this medication? 1. To take the medication before meals 2. To return to the clinic weekly for serum drug-level testing 3. It is not necessary to call the health care provider (HCP) if a skin rash occurs. 4. It is not necessary to restrict alcohol intake with this medication.

2. To return to the clinic weekly for serum drug-level testing Rationale: Cycloserine (Seromycin) is an antitubercular medication that requires weekly serum drug level determinations to monitor for the potential of neurotoxicity. Serum drug levels lower than 30 mcg/mL reduce the incidence of neurotoxicity. The medication must be taken after meals to prevent gastrointestinal irritation. The client must be instructed to notify the HCP if a skin rash or signs of central nervous system toxicity are noted. Alcohol must be avoided because it increases the risk of seizure activity.

82.) A client is receiving meperidine hydrochloride (Demerol) for pain. Which of the following are side effects of this medication. Select all that apply. 1. Diarrhea 2. Tremors 3. Drowsiness 4. Hypotension 5. Urinary frequency 6. Increased respiratory rate

2. Tremors 3. Drowsiness 4. Hypotension Rationale: Meperidine hydrochloride is an opioid analgesic. Side effects include respiratory depression, drowsiness, hypotension, constipation, urinary retention, nausea, vomiting, and tremors.

135.) A nurse reinforces medication instructions to a client who is taking levothyroxine (Synthroid). The nurse instructs the client to notify the health care provider (HCP) if which of the following occurs? 1. Fatigue 2. Tremors 3. Cold intolerance 4. Excessively dry skin

2. Tremors Rationale: Excessive doses of levothyroxine (Synthroid) can produce signs and symptoms of hyperthyroidism. These include tachycardia, chest pain, tremors, nervousness, insomnia, hyperthermia, heat intolerance, and sweating. The client should be instructed to notify the HCP if these occur. Options 1, 3, and 4 are signs of hypothyroidism.

b. Activated partial thromboplastin time (aPTT) of 120 seconds

A client is receiving an intravenous heparin drip. Which laboratory value will require immediate action by the nurse? a. Platelet count of 150,000 b. Activated partial thromboplastin time (aPTT) of 120 seconds c. INR of 1.0 d. Blood urea nitrogen (BUN) level of 12 mg/dL

14.) The client with acute myelocytic leukemia is being treated with busulfan (Myleran). Which laboratory value would the nurse specifically monitor during treatment with this medication? 1. Clotting time 2. Uric acid level 3. Potassium level 4. Blood glucose level

2. Uric acid level Rationale: Busulfan (Myleran) can cause an increase in the uric acid level. Hyperuricemia can produce uric acid nephropathy, renal stones, and acute renal failure. Options 1, 3, and 4 are not specifically related to this medication.

68.) Bethanechol chloride (Urecholine) is prescribed for a client with urinary retention. Which disorder would be a contraindication to the administration of this medication? 1. Gastric atony 2. Urinary strictures 3. Neurogenic atony 4. Gastroesophageal reflux

2. Urinary strictures Rationale: Bethanechol chloride (Urecholine) can be harmful to clients with urinary tract obstruction or weakness of the bladder wall. The medication has the ability to contract the bladder and thereby increase pressure within the urinary tract. Elevation of pressure within the urinary tract could rupture the bladder in clients with these conditions.

The patient has been admitted with hyperglycemic hyperosmolar nonketotic syndrome (HHNKS). Her blood glucose level is very high (880 mg/dL on admission). The physician believes that her condition is to the result of large amounts of glucose solutions administered intravenously during kidney dialysis. The nurse would anticipate that the patient would exhibit: 1. a fruity breath and high level of ketones in her urine. 2. severe dehydration and hypernatremia caused by the hyperglycemia. 3. exactly the same symptoms and signs as diabetic ketoacidosis. 4. Kussmaul's respirations, nausea, and vomiting.

2. severe dehydration and hypernatremia caused by the hyperglycemia. IV solutions containing glucose will bypass the digestive system, so there is no trigger for the pancreas to release insulin, but there is just enough insulin to prevent the breakdown of fatty acids and the formation of ketones.

A patient has started clopidogrel (Plavix) after experiencing a TIA (transient ischemic attack). The desired therapeutic effects of this drug will be: 1. anti-inflammatory and antipyretic effects 2. to reduce the risk of a stroke from a blood clot 3. analgesic as well as clot-dissolving effects 4. to stop clots from becoming emboli.

2. to reduce the risk of a stroke from a blood clot Rationale: Antiplatelet drugs such as clopidogerel are given to inhibit platelet aggreagtion and thus reduce the risk of thrombus formation. Antiplatelet drugs do no exert antiinflammatory, antipyretic or analgesic effects. The antiplatelet and anticoagulant drugs do not prevent emboli formation. Thrombolytics dissolve existing blood clots.

55.) A client who is receiving digoxin (Lanoxin) daily has a serum potassium level of 3.0 mEq/L and is complaining of anorexia. A health care provider prescribes a digoxin level to rule out digoxin toxicity. A nurse checks the results, knowing that which of the following is the therapeutic serum level (range) for digoxin? 1. 3 to 5 ng/mL 2. 0.5 to 2 ng/mL 3. 1.2 to 2.8 ng/mL 4. 3.5 to 5.5 ng/mL

2.) 0.5 to 2 ng/mL Rationale: Therapeutic levels for digoxin range from 0.5 to 2 ng/mL. Therefore, options 1, 3, and 4 are incorrect.

The primary care provider prescribed 2.5 mg of bethanechol subQ. The drug is available in a solution of 5 mg/mL. The nurse should administer

2.5mg/5mg/mL = 0.05mL

2. Central venous access devices are beneficial in pediatric therapy because: a. They don't frighten children. b. Use of the arms is not restricted. c. They cannot be dislodged. d. They are difficult to see.

2.Answer B. The child can move his extremities and function in a normal fashion. This lessens stress associated with position restriction and promotes normal activity. Fear may not be eliminated. All lines can be dislodged. Even small catheters can be readily seen.

2. A contraindication for topical corticosteroid usage in a male patient with atopic dermatitis (eczema) is: a. Parasite infection. b. Viral infection. c. Bacterial infection. d. Spirochete infection.

2.Answer B. Topical agents produce a localized, rather than systemic effect. When treating atopic dermatitis with a steroidal preparation, the site is vulnerable to invasion by organisms. Viruses, such as herpes simplex or varicella-zoster, present a risk of disseminated infection. Educate the patient using topical corticosteroids to avoid crowds or people known to have infections and to report even minor signs of an infection. Topical corticosteroid usage results in little danger of concurrent infection with these agents.

IV Needle size,Location,Angle of admin

20-22 common sizes dependant upon needs location arms Med Ac hands etc Angle no angle required

20. Methotrexate is a folate antagonist. It inhibits enzymes required for DNA base synthesis. To prevent harm to normal cells, a fully activated form of folic acid known as leucovorin (folinic acid citrovorum factor) can be administered. Administration of leucovorin is known as: a. Induction therapy. b. Consolidation therapy. c. Pulse therapy. d. Rescue therapy.

20.Answer B. Leucovorin is used to save or "rescue" normal cells from the damaging effects of chemotherapy allowing them to survive while the cancer cells die. Therapy to rapidly reduce the number of cancerous cells is the induction phase. Consolidation therapy seeks to complete or extend the initial remission and often uses a different combination of drugs than that used for induction. Chemotherapy is often administered in intermittent courses called pulse therapy. Pulse therapy allows the bone marrow to recover function before another course of chemotherapy is given.

96.) The nurse is caring for a postrenal transplant client taking cyclosporine (Sandimmune, Gengraf, Neoral). The nurse notes an increase in one of the client's vital signs, and the client is complaining of a headache. What is the vital sign that is most likely increased? 1. Pulse 2. Respirations 3. Blood pressure 4. Pulse oximetry

3. Blood pressure Rationale: Hypertension can occur in a client taking cyclosporine (Sandimmune, Gengraf, Neoral), and because this client is also complaining of a headache, the blood pressure is the vital sign to be monitoring most closely. Other adverse effects include infection, nephrotoxicity, and hirsutism. Options 1, 2, and 4 are unrelated to the use of this medication.

20. A child is admitted with a serious infection. After two days of antibiotics, he is severely neutropenic. The physician orders granulocyte transfusions for the next four days. The mother asks the nurse why? The nurse responds: a. "This is the only treatment left to offer the child." b. "This therapy is fast and reliable in treating infections in children." c. "The physician will have to explain his rationale to you." d. "Granulocyte transfusions replenish the low white blood cells until the body can produce its own."

20.Answer D. Granulocyte (neutrophil) replacement therapy is given until the patient's blood values are normal and he is able to fight the infection himself. Options 1 and 3 are not therapeutic responses. The treatment in option 2 takes days and is not always able to prevent morbidity and mortality.

The physician orders celecoxib celebrex 200 mg orally the nurse has celecoxib 100 mg tablets. How much shou,d the RN administer

200/100=2 tablets

21. A male Patient is undergoing chemotherapy may also be given the drug allopurinol (Zyloprim, Aloprim). Allopurinol inhibits the synthesis of uric acid. Concomitant administration of allopurinol prevents: a. Myelosuppression. b. Gout and hyperuricemia. c. Pancytopenia. d. Cancer cell growth and replication

21.Answer B. Prevent uric acid nephropathy, uric acid lithiasis, and gout during cancer therapy since chemotherapy causes the rapid destruction of cancer cells leading to excessive purine catabolism and uric acid formation. Allopurinol can induce myelosuppression and pancytopenia. Allopurinol does not have this function.

21. A neighbor tells nurse Maureen he has to have surgery and is reluctant to have any blood product transfusions because of a fear of contracting an infection. He asks the nurse what are his options. The nurse teaches the person that the safest blood product is: a. An allogenic product. b. A directed donation product. c. An autologous product. d. A cross-matched product.

21.Answer C. This process is the collection and reinfusion of the patient's own blood. It is recommended by the American Medical Association's Council on Scientific Affairs as the safest product since it eliminates recipient incompatibility and infection. The product in option 1 is collected from a blood donor other than the recipient. The process in option 2 is also collected from a blood donor other than the recipient, but the donor is known to the recipient and is usually a family member or friend. Cross-matching significantly enhances compatibility. It does not detect infection.

22. Superficial bladder cancer can be treated by direct instillation of the antineoplastic antibiotic agent mitomycin (Mutamycin). This process is termed: a. Intraventricular administration. b. Intravesical administration. c. Intravascular administration. d. Intrathecal administration.

22.Answer B. Medications administered intravesically are instilled into the bladder. Intraventricular administration involves the ventricles of the brain. Intravascular administration involves blood vessels. Intrathecal administration involves the fluid surrounding the brain and spinal cord.

22. A severely immunocompromised female patient requires a blood transfusion. To prevent GVHD, the physician will order: a. Diphenhydramine hydrochloride (Benadryl). b. The transfusion to be administered slowly over several hours. c. Irradiation of the donor blood. d. Acetaminophen (Tylenol).

22.Answer C. This process eliminates white blood cell functioning, thus, preventing GVHD. Diphenhydramine HCl is an antihistamine. It's use prior to a blood transfusion decreases the likelihood of a transfusion reaction. Option 2 will not prevent GVHD. Use of acetaminophen prevents and treats the common side effects of blood administration caused by the presence of white blood cells in the transfusion product: fever, headache, and chills.

23. Louie who is to receive a blood transfusion asks the nurse what is the most common type of infection he could receive from the transfusion. The nurse teaches him that approximately 1 in 250,000 patients contract: a. Human immunodeficiency disease (HIV). b. Hepatitis C infection. c. Hepatitis B infection. d. West Nile viral disease.

23.Answer C. Hepatitis B is the most common infection spread via blood transfusion. Donors are screened by a questionnaire that includes symptoms. The donated blood is also tested for infection. The risk of infection with the agents in options 2 and 3 has decreased to approximately 1 in 2 million secondary to donor questioning and donor blood testing. The incidence of West Nile viral transmission is unknown, but donor infection is still relatively rare.

23. The most common dose-limiting toxicity of chemotherapy is: a. Nausea and vomiting. b. Bloody stools. c. Myelosuppression. d. Inability to ingest food orally due to stomatitis and mucositis.

23.Answer C. The overall goal of cancer chemotherapy is to give a dose large enough to be lethal to the cancer cells, but small enough to be tolerable for normal cells. Unfortunately, some normal cells are affected including the bone marrow. Myelosuppression limits the body's ability to prevent and fight infection, produce platelets for clotting, and manufacture red blood cells for oxygen portage. Even though the effects in options a, b, and d are uncomfortable and distressing to the patient, they do not have the potential for lethal outcomes that myelosuppression has.

24. Chemotherapy induces vomiting by: a. Stimulating neuroreceptors in the medulla. b. Inhibiting the release of catecholamines. c. Autonomic instability. d. Irritating the gastric mucosa.

24.Answer A. Vomiting (emesis) is initiated by a nucleus of cells located in the medulla called the vomiting center. This center coordinates a complex series of events involving pharyngeal, gastrointestinal, and abdominal wall contractions that lead to expulsion of gastric contents. Catecholamine inhibition does not induce vomiting. Chemotherapy does not induce vomiting from autonomic instability. Chemotherapy, especially oral agents, may have an irritating effect on the gastric mucosa, which could result in afferent messages to the solitary tract nucleus, but these pathways do not project to the vomiting center.

24. A male patient with blood type AB, Rh factor positive needs a blood transfusion. The Transfusion Service (blood bank) sends type O, Rh factor negative blood to the unit for the nurse to infuse into this patient. The nurse knows that: a. This donor blood is incompatible with the patient's blood. b. Premedicating the patient with diphenhydramine hydrochloride (Benadryl) and acetaminophen (Tylenol) will prevent any transfusion reactions or side effects. c. This is a compatible match. d. The patient is at minimal risk receiving this product since it is the first time he has been transfused with type O, Rh negative blood.

24.Answer C. Type O, Rh negative blood has none of the major antigens and is safely administered to patients of all blood types. It is also known as the universal donor. Premedicating with these agents will not prevent a major transfusion reaction if the blood type and Rh factors of the donor blood are incompatible with the recipient's blood.

25. Myeloablation using chemotherapeutic agents is useful in cancer treatment because: a. It destroys the myelocytes (muscle cells). b. It reduces the size of the cancer tumor. c. After surgery, it reduces the amount of chemotherapy needed. d. It destroys the bone marrow prior to transplant.

25.Answer A. Myelo comes from the Greek word myelos, which means marrow. Ablation comes from the Latin word ablatio, which means removal. Thus, myeloablative chemotherapeurtic agents destroy the bone marrow. This procedure destroys normal bone marrow as well as the cancerous marrow. The patient's bone marrow will be replaced with a bone marrow transplant. Myelocytes are not muscle cells Tumors are solid masses typically located in organs. Surgery may be performed to reduce tumor burden and require less chemotherapy afterward.

25. Dr. Rodriguez orders 250 milliliters of packed red blood cells (RBC) for a patient. This therapy is administered for treatment of: a. Thrombocytopenia. b. Anemia. c. Leukopenia. d. Hypoalbuminemia.

25.Answer B. A red blood cell transfusion is used to correct anemia in patients in which the low red blood cell count must be rapidly corrected. RBC transfusion will not correct a low platelet count. RBC transfusion will not correct a low white blood cell count. Packed RBCs contain very little plasma and, thus, only a small amount of albumin. This amount will not correct low albumin levels.

how many mL of drug should pt recieve per dose and per 24hrs.

250 ordered/250mg per 5mL on hand ((5ml per 6hrs)) 6x4=24hrs

Usual dose of tetracycline is

250-500 q6h

Naproxen -naprosyn oral suspension 250 mg is prescribed -the dose on hand is oral suspension 125mg/5mL How much should the RN administer

250/125x5mL= 10mL

26. A female patient needs a whole blood transfusion. In order for transfusion services (the blood bank) to prepare the correct product a sample of the patient's blood must be obtained for: a. A complete blood count and differential. b. A blood type and cross-match. c. A blood culture and sensitivity. d. A blood type and antibody screen.

26.Answer B. This is needed to utilize the correct type of donor blood and to match the donor product with the patient. Incompatible matches would result in severe adverse events and possible death. The tests in options 1 and 3 are unnecessary. The test in option 4 is utilized to determine the patient's blood type and presence of antibodies to blood antigens. It does not determine donor blood compatibility with the patient.

26. Anticipatory nausea and vomiting associated with chemotherapy occurs: a. Within the first 24 hours after chemotherapy. b. 1-5 days after chemotherapy. c. Before chemotherapy administration. d. While chemotherapy is being administered.

26.Answer C. Nausea and vomiting (N&V) are common side effects of chemotherapy. Some patients are able to trigger these events prior to actually receiving chemotherapy by anticipating, or expecting, to have these effects. N&V occurring post-chemotherapeutic administration is not an anticipatory event but rather an effect of the drug. N&V occurring during the administration of chemotherapy is an effect of the drug.

27. A male patient needs to receive a unit of whole blood. What type of intravenous (IV) device should the nurse consider starting? a. A small catheter to decrease patient discomfort b. The type of IV device the patient has had in the past, which worked well c. A large bore catheter d. The type of device the physician prefers

27.Answer C. Large bore catheters prevent damage to blood components and are less likely to develop clotting problems than a small bore catheter. The nurse should determine the correct device without asking the patient what type has been used before or asking the physician which type he prefers and start the IV.

27. Medications bound to protein have the following effect: a. Enhancement of drug availability. b. Rapid distribution of the drug to receptor sites. c. The more drug bound to protein, the less available for desired effect. d. Increased metabolism of the drug by the liver.

27.Answer C. Only an unbound drug can be distributed to active receptor sites. Therefore, the more of a drug that is bound to protein, the less it is available for the desired drug effect. Less drug is available if bound to protein. Distribution to receptor sites is irrelevant since the drug bound to protein cannot bind with a receptor site. Metabolism would not be increased. The liver will first have to remove the drug from the protein molecule before metabolism can occur. The protein is then free to return to circulation and be used again.

28. Some drugs are excreted into bile and delivered to the intestines. Prior to elimination from the body, the drug may be absorbed. This process is known as: a. Hepatic clearance. b. Total clearance. c. Enterohepatic cycling. d. First-pass effect.

28.Answer C. Drugs and drug metabolites with molecular weights higher than 300 may be excreted via the bile, stored in the gallbladder, delivered to the intestines by the bile duct, and then reabsorbed into the circulation. This process reduces the elimination of drugs and prolongs their half-life and duration of action in the body. Hepatic clearance is the amount of drug eliminated by the liver. Total clearance is the sum of all types of clearance including renal, hepatic, and respiratory. First-pass effect is the amount of drug absorbed from the GI tract and then metabolized by the liver thus, reducing the amount of drug making it into circulation.

28. Dr. Smith orders a gram of human salt poor albumin product for a patient. The product is available in a 50 milliliter vial with a concentration of 25 percent. What dosage will the nurse administer? a. The nurse should use the entire 50 milliliter vial. b. The nurse should determine the volume to administer from the physician. c. This concentration of product should not be used. d. The nurse will administer 4 milliliters.

28.Answer D. A 25 percent solution contains one quarter of a gram per milliliter. Thus, the nurse will administer 4 milliliters to provide a complete gram of albumin. The volume in option 1 would provide 12.5 grams of albumin. The nurse should determine the volume. It is unnecessary to seek the answer from the physician. A 25 percent solution is an acceptable product and can safely be used.

A patient is receiving a thrombolytic agent, alteplase (Activase), follwing an acute myocardial infraction. Which condition is most likely attributed to thrombolytic therapy with this agent? 1. Skin rash with urticaria 2. Wheezing with labored respirations 3. Brusing and epistaxis 4. Temperature elevation of 100.8F

3. Bruising and epistaxis Rationale: Thrombolytic agents dissolve existing clots rapidly and continue to have effects for 2 to 4 days. All forms of bleeding must be monitored and reported immediately. Skin rash with urticara, wheezing with labored respirations, and temperature elevation of 100.8F are not symptoms of adverse effects directly attributed to thrombolytic therapy.

29. An adult patient has been taking a drug (Drug A) that is highly metabolized by the cytochrome p-450 system. He has been on this medication for 6 months. At this time, he is started on a second medication (Drug B) that is an inducer of the cytochrome p-450 system. You should monitor this patient for: a. Increased therapeutic effects of Drug A. b. Increased adverse effects of Drug B. c. Decreased therapeutic effects of Drug A. d. Decreased therapeutic effects of Drug B.

29.Answer C. Drug B will induce the cytochrome p-450 enzyme system of the liver thus, increasing the metabolism of Drug A. Therefore, Drug A will be broken down faster and exert decreased therapeutic effects. Drug A will be metabolized faster, thus reducing, not increasing its therapeutic effect. Inducing the cytochrome p-450 system will not increase the adverse effects of Drug B. Drug B induces the cytochrome p-450 system but is not metabolized faster. Thus, the therapeutic effects of Drug B will not be decreased.

128.) A nurse is providing instructions to an adolescent who has a history of seizures and is taking an anticonvulsant medication. Which of the following statements indicates that the client understands the instructions? 1. "I will never be able to drive a car." 2. "My anticonvulsant medication will clear up my skin." 3. "I can't drink alcohol while I am taking my medication." 4. "If I forget my morning medication, I can take two pills at bedtime."

3. "I can't drink alcohol while I am taking my medication." Rationale: Alcohol will lower the seizure threshold and should be avoided. Adolescents can obtain a driver's license in most states when they have been seizure free for 1 year. Anticonvulsants cause acne and oily skin; therefore a dermatologist may need to be consulted. If an anticonvulsant medication is missed, the health care provider should be notified.

106.) Fluoxetine (Prozac) is prescribed for the client. The nurse reinforces instructions to the client regarding the administration of the medication. Which statement by the client indicates an understanding about administration of the medication? 1. "I should take the medication with my evening meal." 2. "I should take the medication at noon with an antacid." 3. "I should take the medication in the morning when I first arise." 4. "I should take the medication right before bedtime with a snack."

3. "I should take the medication in the morning when I first arise." Rationale: Fluoxetine hydrochloride is administered in the early morning without consideration to meals. **Eliminate options 1, 2, and 4 because they are comparable or alike and indicate taking the medication with an antacid or food.**

197.) Collagenase (Santyl) is prescribed for a client with a severe burn to the hand. The nurse provides instructions to the client regarding the use of the medication. Which statement by the client indicates an accurate understanding of the use of this medication? 1. "I will apply the ointment once a day and leave it open to the air." 2. "I will apply the ointment twice a day and leave it open to the air." 3. "I will apply the ointment once a day and cover it with a sterile dressing." 4. "I will apply the ointment at bedtime and in the morning and cover it with a sterile dressing."

3. "I will apply the ointment once a day and cover it with a sterile dressing." Rationale: Collagenase is used to promote debridement of dermal lesions and severe burns. It is usually applied once daily and covered with a sterile dressing.

164.) A client receives a prescription for methocarbamol (Robaxin), and the nurse reinforces instructions to the client regarding the medication. Which client statement would indicate a need for further instructions? 1. "My urine may turn brown or green." 2. "This medication is prescribed to help relieve my muscle spasms." 3. "If my vision becomes blurred, I don't need to be concerned about it." 4. "I need to call my doctor if I experience nasal congestion from this medication."

3. "If my vision becomes blurred, I don't need to be concerned about it." Rationale: The client needs to be told that the urine may turn brown, black, or green. Other adverse effects include blurred vision, nasal congestion, urticaria, and rash. The client needs to be instructed that, if these adverse effects occur, the health care provider needs to be notified. The medication is used to relieve muscle spasms.

The patient, newly diagnosed with hypothyroidism, seems very anxious to begin her drug regimen. The nurse's instructions include: 1. "Be certain that no dose is skipped." 2. "If a dose is skipped one day, double the dose the next day." 3. "Know the signs and symptoms of hyperthyroidism." 4. "You will be able to notice the benefits of thyroid replacement therapy right away."

3. "Know the signs and symptoms of hyperthyroidism." Her enthusiasm may lead her to overdose on the thyroid replacement pills. She needs to be aware of the proper prescription and the reasons for following the prescribed dosage.

The patient asks about his lab test, which showed a high level of TSH and a low level of T4. You explain: 1. "It means that you have an inconsistency in your thyroid tests, and you will need more testing." 2. "I am sorry. You will have to ask your doctor about your lab results. We are not allowed to discuss them." 3. "The TSH is sending a message to your thyroid gland to increase production, but your thyroid isn't doing that." 4. "That means that you will have to go on hormone therapy for the rest of your life."

3. "The TSH is sending a message to your thyroid gland to increase production, but your thyroid isn't doing that." The test determines if the problem is in the pituitary or in the thyroid. In this case the high TSH is coming from the pituitary as it should but the thyroid is not responding.

11.) The health care provider has prescribed silver sulfadiazine (Silvadene) for the client with a partial-thickness burn, which has cultured positive for gram-negative bacteria. The nurse is reinforcing information to the client about the medication. Which statement made by the client indicates a lack of understanding about the treatments? 1. "The medication is an antibacterial." 2. "The medication will help heal the burn." 3. "The medication will permanently stain my skin." 4. "The medication should be applied directly to the wound."

3. "The medication will permanently stain my skin." Rationale: Silver sulfadiazine (Silvadene) is an antibacterial that has a broad spectrum of activity against gram-negative bacteria, gram-positive bacteria, and yeast. It is applied directly to the wound to assist in healing. It does not stain the skin.

42.) A client with a peptic ulcer is diagnosed with a Helicobacter pylori infection. The nurse is reinforcing teaching for the client about the medications prescribed, including clarithromycin (Biaxin), esomeprazole (Nexium), and amoxicillin (Amoxil). Which statement by the client indicates the best understanding of the medication regimen? 1. "My ulcer will heal because these medications will kill the bacteria." 2. "These medications are only taken when I have pain from my ulcer." 3. "The medications will kill the bacteria and stop the acid production." 4. "These medications will coat the ulcer and decrease the acid production in my stomach."

3. "The medications will kill the bacteria and stop the acid production." Rationale: Triple therapy for Helicobacter pylori infection usually includes two antibacterial drugs and a proton pump inhibitor. Clarithromycin and amoxicillin are antibacterials. Esomeprazole is a proton pump inhibitor. These medications will kill the bacteria and decrease acid production.

141.) The nurse has reinforced instructions to a client who has been prescribed cholestyramine (Questran). Which statement by the client indicates a need for further instructions? 1. "I will continue taking vitamin supplements." 2. "This medication will help lower my cholesterol." 3. "This medication should only be taken with water." 4. "A high-fiber diet is important while taking this medication."

3. "This medication should only be taken with water." Rationale: Cholestyramine (Questran) is a bile acid sequestrant used to lower the cholesterol level, and client compliance is a problem because of its taste and palatability. The use of flavored products or fruit juices can improve the taste. Some side effects of bile acid sequestrants include constipation and decreased vitamin absorption. **Note the closed-ended word "only" in option 3**

191.) A child is brought to the emergency department for treatment of an acute asthma attack. The nurse prepares to administer which of the following medications first? 1. Oral corticosteroids 2. A leukotriene modifier 3. A β2 agonist 4. A nonsteroidal anti-inflammatory

3. A β2 agonist Rationale: In treating an acute asthma attack, a short acting β2 agonist such as albuterol (Proventil HFA) will be given to produce bronchodilation. Options 1, 2, and 4 are long-term control (preventive) medications.

2.) Oral iron supplements are prescribed for a 6-year-old child with iron deficiency anemia. The nurse instructs the mother to administer the iron with which best food item? 1. Milk 2. Water 3. Apple juice 4. Orange juice

4. Orange juice Rationale: Vitamin C increases the absorption of iron by the body. The mother should be instructed to administer the medication with a citrus fruit or a juice that is high in vitamin C. Milk may affect absorption of the iron. Water will not assist in absorption. Orange juice contains a greater amount of vitamin C than apple juice.

120.) A client is taking lansoprazole (Prevacid) for the chronic management of Zollinger-Ellison syndrome. The nurse advises the client to take which of the following products if needed for a headache? 1. Naprosyn (Aleve) 2. Ibuprofen (Advil) 3. Acetaminophen (Tylenol) 4. Acetylsalicylic acid (aspirin)

3. Acetaminophen (Tylenol) Rationale: Zollinger-Ellison syndrome is a hypersecretory condition of the stomach. The client should avoid taking medications that are irritating to the stomach lining. Irritants would include aspirin and nonsteroidal antiinflammatory drugs (ibuprofen). The client should be advised to take acetaminophen for headache. **Remember that options that are comparable or alike are not likely to be correct. With this in mind, eliminate options 1 and 2 first.**

207.) A client is suspected of having myasthenia gravis, and the health care provider administers edrophonium (Enlon) to determine the diagnosis. After administration of this medication, which of the following would indicate the presence of myasthenia gravis? 1. Joint pain 2. A decrease in muscle strength 3. An increase in muscle strength 4. Feelings of faintness, dizziness, hypotension, and signs of flushing in the client

3. An increase in muscle strength Rationale: Edrophonium is a short-acting acetylcholinesterase inhibitor used as a diagnostic agent. When a client with suspected myasthenia gravis is given the medication intravenously, an increase in muscle strength would be seen in 1 to 3 minutes. If no response occurs, another dose is given over the next 2 minutes, and muscle strength is again tested. If no increase in muscle strength occurs with this higher dose, the muscle weakness is not caused by myasthenia gravis. Clients receiving injections of this medication commonly demonstrate a drop of blood pressure, feel faint and dizzy, and are flushed.

3. James Perez, a nurse on a geriatric floor, is administering a dose of digoxin to one of his patients. The woman asks why she takes a different pill than her niece, who also has heart trouble. James replies that as people get older, liver and kidney function decline, and if the dose is as high as her niece's, the drug will tend to: a. have a shorter half-life. b. accumulate. c. have decreased distribution. d. have increased absorption.

3. B. The decreased circulation to the kidney and reduced liver function tend to allow drugs to accumulate and have toxic effects.

174.) A client with portosystemic encephalopathy is receiving oral lactulose (Chronulac) daily. The nurse assesses which of the following to determine medication effectiveness? 1. Lung sounds 2. Blood pressure 3. Blood ammonia level 4. Serum potassium level

3. Blood ammonia level Rationale: Lactulose is a hyperosmotic laxative and ammonia detoxicant. It is used to prevent or treat portosystemic encephalopathy, including hepatic precoma and coma. It also is used to treat constipation. The medication retains ammonia in the colon (decreases the blood ammonia concentration), producing an osmotic effect. It promotes increased peristalsis and bowel evacuation, expelling ammonia from the colon.

c. Administer the medication into subcutaneous tissue.

A nurse is preparing to administer enoxaparin sodium (Lovenox) to a client for prevention of deep vein thrombosis. What is an essential nursing intervention? a. Draw up the medication in a syringe with a 22-gauge, 1-½ inch needle. b. Utilize the Z-track method to inject the medication. c. Administer the medication into subcutaneous tissue. d. Rub the administration site after injecting.

49.) A client has been started on long-term therapy with rifampin (Rifadin). A nurse teaches the client that the medication: 1. Should always be taken with food or antacids 2. Should be double-dosed if one dose is forgotten 3. Causes orange discoloration of sweat, tears, urine, and feces 4. May be discontinued independently if symptoms are gone in 3 months

3. Causes orange discoloration of sweat, tears, urine, and feces Rationale: Rifampin should be taken exactly as directed as part of TB therapy. Doses should not be doubled or skipped. The client should not stop therapy until directed to do so by a health care provider. The medication should be administered on an empty stomach unless it causes gastrointestinal upset, and then it may be taken with food. Antacids, if prescribed, should be taken at least 1 hour before the medication. Rifampin causes orange-red discoloration of body secretions and will permanently stain soft contact lenses.

16.) The clinic nurse is reviewing a teaching plan for the client receiving an antineoplastic medication. When implementing the plan, the nurse tells the client: 1. To take aspirin (acetylsalicylic acid) as needed for headache 2. Drink beverages containing alcohol in moderate amounts each evening 3. Consult with health care providers (HCPs) before receiving immunizations 4. That it is not necessary to consult HCPs before receiving a flu vaccine at the local health fair

3. Consult with health care providers (HCPs) before receiving immunizations Rationale: Because antineoplastic medications lower the resistance of the body, clients must be informed not to receive immunizations without a HCP's approval. Clients also need to avoid contact with individuals who have recently received a live virus vaccine. Clients need to avoid aspirin and aspirin-containing products to minimize the risk of bleeding, and they need to avoid alcohol to minimize the risk of toxicity and side effects.

150.) A client complaining of not feeling well is seen in a clinic. The client is taking several medications for the control of heart disease and hypertension. These medications include a β-blocker, digoxin (Lanoxin), and a diuretic. A tentative diagnosis of digoxin toxicity is made. Which of the following assessment data would support this diagnosis? 1. Dyspnea, edema, and palpitations 2. Chest pain, hypotension, and paresthesia 3. Double vision, loss of appetite, and nausea 4. Constipation, dry mouth, and sleep disorder

3. Double vision, loss of appetite, and nausea Rationale: Double vision, loss of appetite, and nausea are signs of digoxin toxicity. Additional signs of digoxin toxicity include bradycardia, difficulty reading, visual alterations such as green and yellow vision or seeing spots or halos, confusion, vomiting, diarrhea, decreased libido, and impotence. **gastrointestinal (GI) and visual disturbances occur with digoxin toxicity**

85.) A nurse is monitoring a client receiving baclofen (Lioresal) for side effects related to the medication. Which of the following would indicate that the client is experiencing a side effect? 1. Polyuria 2. Diarrhea 3. Drowsiness 4. Muscular excitability

3. Drowsiness Rationale: Baclofen is a central nervous system (CNS) depressant and frequently causes drowsiness, dizziness, weakness, and fatigue. It can also cause nausea, constipation, and urinary retention. Clients should be warned about the possible reactions. Options 1, 2, and 4 are not side effects.

138.) A daily dose of prednisone is prescribed for a client. A nurse reinforces instructions to the client regarding administration of the medication and instructs the client that the best time to take this medication is: 1. At noon 2. At bedtime 3. Early morning 4. Anytime, at the same time, each day

3. Early morning Rationale: Corticosteroids (glucocorticoids) should be administered before 9:00 AM. Administration at this time helps minimize adrenal insufficiency and mimics the burst of glucocorticoids released naturally by the adrenal glands each morning. **Note the suffix "-sone," and recall that medication names that end with these letters are corticosteroids.**

244.) A client has a prescription for valproic acid (Depakene) orally once daily. The nurse plans to: 1. Administer the medication with an antacid. 2. Administer the medication with a carbonated beverage. 3. Ensure that the medication is administered at the same time each day. 4. Ensure that the medication is administered 2 hours before breakfast only, when the client's stomach is empty.

3. Ensure that the medication is administered at the same time each day. Rationale: Valproic acid is an anticonvulsant, antimanic, and antimigraine medication. It may be administered with or without food. It should not be taken with an antacid or carbonated beverage because these products will affect medication absorption. The medication is administered at the same time each day to maintain therapeutic serum levels. **Use general pharmacology guidelines to assist in eliminating options 1 and 2. Eliminate option 4 because of the closed-ended word "only."**

122.) A client who has begun taking fosinopril (Monopril) is very distressed, telling the nurse that he cannot taste food normally since beginning the medication 2 weeks ago. The nurse provides the best support to the client by: 1. Telling the client not to take the medication with food 2. Suggesting that the client taper the dose until taste returns to normal 3. Informing the client that impaired taste is expected and generally disappears in 2 to 3 months 4. Requesting that the health care provider (HCP) change the prescription to another brand of angiotensin-converting enzyme (ACE) inhibitor

3. Informing the client that impaired taste is expected and generally disappears in 2 to 3 months Rationale: ACE inhibitors, such as fosinopril, cause temporary impairment of taste (dysgeusia). The nurse can tell the client that this effect usually disappears in 2 to 3 months, even with continued therapy, and provide nutritional counseling if appropriate to avoid weight loss. Options 1, 2, and 4 are inappropriate actions. Taking this medication with or without food does not affect absorption and action. The dosage should never be tapered without HCP approval and the medication should never be stopped abruptly.

243.) A hospitalized client is having the dosage of clonazepam (Klonopin) adjusted. The nurse should plan to: 1. Weigh the client daily. 2. Observe for ecchymosis. 3. Institute seizure precautions. 4. Monitor blood glucose levels.

3. Institute seizure precautions. Rationale: Clonazepam is a benzodiazepine used as an anticonvulsant. During initial therapy and during periods of dosage adjustment, the nurse should initiate seizure precautions for the client. Options 1, 2, and 4 are not associated with the use of this medication.

d. Serum glucose (sugar)

A nurse teaching a client who has diabetes mellitus and is taking hydrochlorothiazide 50 mg/day. The teaching should include the importance of monitoring which levels? a. Hemoglobin and hematocrit b. Blood urea nitrogen (BUN) c. Arterial blood gases d. Serum glucose (sugar)

Who is a good roommate for an autistic child?

A person in a coma.

37.) The client has begun medication therapy with pancrelipase (Pancrease MT). The nurse evaluates that the medication is having the optimal intended benefit if which effect is observed? 1. Weight loss 2. Relief of heartburn 3. Reduction of steatorrhea 4. Absence of abdominal pain

3. Reduction of steatorrhea Rationale: Pancrelipase (Pancrease MT) is a pancreatic enzyme used in clients with pancreatitis as a digestive aid. The medication should reduce the amount of fatty stools (steatorrhea). Another intended effect could be improved nutritional status. It is not used to treat abdominal pain or heartburn. Its use could result in weight gain but should not result in weight loss if it is aiding in digestion.

111.) A client arrives at the health care clinic and tells the nurse that he has been doubling his daily dosage of bupropion hydrochloride (Wellbutrin) to help him get better faster. The nurse understands that the client is now at risk for which of the following? 1. Insomnia 2. Weight gain 3. Seizure activity 4. Orthostatic hypotension

3. Seizure activity Rationale: Bupropion does not cause significant orthostatic blood pressure changes. Seizure activity is common in dosages greater than 450 mg daily. Bupropion frequently causes a drop in body weight. Insomnia is a side effect, but seizure activity causes a greater client risk.

95.) The nurse is reviewing the results of serum laboratory studies drawn on a client with acquired immunodeficiency syndrome who is receiving didanosine (Videx). The nurse interprets that the client may have the medication discontinued by the health care provider if which of the following significantly elevated results is noted? 1. Serum protein 2. Blood glucose 3. Serum amylase 4. Serum creatinine

3. Serum amylase Rationale: Didanosine (Videx) can cause pancreatitis. A serum amylase level that is increased 1.5 to 2 times normal may signify pancreatitis in the client with acquired immunodeficiency syndrome and is potentially fatal. The medication may have to be discontinued. The medication is also hepatotoxic and can result in liver failure.

187.) A clinic nurse prepares to administer an MMR (measles, mumps, rubella) vaccine to a child. How is this vaccine best administered? 1. Intramuscularly in the deltoid muscle 2. Subcutaneously in the gluteal muscle 3. Subcutaneously in the outer aspect of the upper arm 4. Intramuscularly in the anterolateral aspect of the thigh

3. Subcutaneously in the outer aspect of the upper arm Rationale: The MMR vaccine is administered subcutaneously in the outer aspect of the upper arm. The gluteal muscle is most often used for intramuscular injections. The MMR vaccine is not administered by the intramuscular route.

58.) Isosorbide mononitrate (Imdur) is prescribed for a client with angina pectoris. The client tells the nurse that the medication is causing a chronic headache. The nurse appropriately suggests that the client: 1. Cut the dose in half. 2. Discontinue the medication. 3. Take the medication with food. 4. Contact the health care provider (HCP).

3. Take the medication with food. Rationale: Isosorbide mononitrate is an antianginal medication. Headache is a frequent side effect of isosorbide mononitrate and usually disappears during continued therapy. If a headache occurs during therapy, the client should be instructed to take the medication with food or meals. It is not necessary to contact the HCP unless the headaches persist with therapy. It is not appropriate to instruct the client to discontinue therapy or adjust the dosages.

229.) A client who is taking lithium carbonate (Lithobid) is scheduled for surgery. The nurse informs the client that: 1. The medication will be discontinued a week before the surgery and resumed 1 week postoperatively. 2. The medication is to be taken until the day of surgery and resumed by injection immediately postoperatively. 3. The medication will be discontinued 1 to 2 days before the surgery and resumed as soon as full oral intake is allowed. 4. The medication will be discontinued several days before surgery and resumed by injection in the immediate postoperative period.

3. The medication will be discontinued 1 to 2 days before the surgery and resumed as soon as full oral intake is allowed. Rationale: The client who is on lithium carbonate must be off the medication for 1 to 2 days before a scheduled surgical procedure and can resume the medication when full oral intake is prescribed after the surgery. **lithium carbonate is an oral medication and is not given as an injection**

80.) A nurse is caring for a client who is taking phenytoin (Dilantin) for control of seizures. During data collection, the nurse notes that the client is taking birth control pills. Which of the following information should the nurse provide to the client? 1. Pregnancy should be avoided while taking phenytoin (Dilantin). 2. The client may stop taking the phenytoin (Dilantin) if it is causing severe gastrointestinal effects. 3. The potential for decreased effectiveness of the birth control pills exists while taking phenytoin (Dilantin). 4. The increased risk of thrombophlebitis exists while taking phenytoin (Dilantin) and birth control pills together.

3. The potential for decreased effectiveness of the birth control pills exists while taking phenytoin (Dilantin). Rationale: Phenytoin (Dilantin) enhances the rate of estrogen metabolism, which can decrease the effectiveness of some birth control pills. Options 1, 2, are 4 are not accurate.

103.) A nurse is caring for a hospitalized client who has been taking clozapine (Clozaril) for the treatment of a schizophrenic disorder. Which laboratory study prescribed for the client will the nurse specifically review to monitor for an adverse effect associated with the use of this medication? 1. Platelet count 2. Cholesterol level 3. White blood cell count 4. Blood urea nitrogen level

3. White blood cell count Rationale: Hematological reactions can occur in the client taking clozapine and include agranulocytosis and mild leukopenia. The white blood cell count should be checked before initiating treatment and should be monitored closely during the use of this medication. The client should also be monitored for signs indicating agranulocytosis, which may include sore throat, malaise, and fever. Options 1, 2, and 4 are unrelated to this medication.

The nurse instructs the patient is scheduled to have a radioactive iodine uptake test to: 1. watch for any signs of bleeding or swelling from the biopsy site. 2. avoid contact with others until notified otherwise. 3. wash hands with soap and water after each urination for 24 hours after the test. 4. this test demonstrates the effectiveness of the pituitary gland on the thyroid gland.

3. wash hands with soap and water after each urination for 24 hours after the test. Radiation dose is small and will not harm others.

If a psych nurse floated to a cancer unit what type of patient should she receive?

A person who is in the acceptance stage of their disease.

3. How can central venous access devices (CVADs) be of value in a patient receiving chemotherapy who has stomatitis and severe diarrhea? a. The chemotherapy can be rapidly completed allowing the stomatitis and diarrhea to resolve. b. Crystalloid can be administered to prevent dehydration. c. Concentrated hyperalimentation fluid can be administered through the CVAD. d. The chemotherapy dose can be reduced.

3.Answer C. In patients unable to take oral nutrition, parenteral hyperalimentation is an option for providing nutritional support. High concentrations of dextrose, protein, minerals, vitamins, and trace elements can be provided. Dosing is not affected with options a and d. Crystalloid can provide free water but has very little nutritional benefits. Hyperalimentation can provide free water and considerable nutritional benefits.

3. In infants and children, the side effects of first generation over-the-counter (OTC) antihistamines, such as diphenhydramine (Benadryl) and hydroxyzine (Atarax) include: a. Reye's syndrome. b. Cholinergic effects. c. Paradoxical CNS stimulation. d. Nausea and diarrhea.

3.Answer C. Typically, first generation OTC antihistamines have a sedating effect because of passage into the CNS. However, in some individuals, especially infants and children, paradoxical CNS stimulation occurs and is manifested by excitement, euphoria, restlessness, and confusion. For this reason, use of first generation OTC antihistamines has declined, and second generation product usage has increased. Reye's syndrome is a systemic response to a virus. First generation OTC antihistamines do not exhibit a cholinergic effect. Nausea and diarrhea are uncommon when first generation OTC antihistamines are taken.

When a pt is prescribed alendronate-fosamax for osteoporosis the nurse administers

30 minutes prior to breakfast

30. A female patient's central venous access device (CVAD) becomes infected. Why would the physician order antibiotics to be given through the line rather than through a peripheral IV line? a. To prevent infiltration of the peripheral line b. To reduce the pain and discomfort associated with antibiotic administration in a small vein c. To lessen the chance of an allergic reaction to the antibiotic d. To attempt to sterilize the catheter and prevent having to remove it

30.Answer D. Microorganisms that infect CVADs are often coagulase-negative staphylococci, which can be eliminated by antibiotic administration through the catheter. If unsuccessful in eliminating the microorganism, the CVAD must be removed. CVAD use lessens the need for peripheral IV lines and, thus, the risk of infiltration. In this case however, the antibiotics are given to eradicate microorganisms from the CVAD. CVAD use has this effect, but in this case, the antibiotics are given through the CVAD to eliminate the infective agent. The third option would not occur

Mild intoxication w/ acetylsalicylic acid (aspirin) is called salicylism and is experienced when daily dose is higher than

4 g or 4000 mg

61.) A home health care nurse is visiting a client with elevated triglyceride levels and a serum cholesterol level of 398 mg/dL. The client is taking cholestyramine (Questran). Which of the following statements, if made by the client, indicates the need for further education? 1. "Constipation and bloating might be a problem." 2. "I'll continue to watch my diet and reduce my fats." 3. "Walking a mile each day will help the whole process." 4. "I'll continue my nicotinic acid from the health food store."

4. "I'll continue my nicotinic acid from the health food store." Rationale: Nicotinic acid, even an over-the-counter form, should be avoided because it may lead to liver abnormalities. All lipid-lowering medications also can cause liver abnormalities, so a combination of nicotinic acid and cholestyramine resin is to be avoided. Constipation and bloating are the two most common side effects. Walking and the reduction of fats in the diet are therapeutic measures to reduce cholesterol and triglyceride levels.

87.) A client with acute muscle spasms has been taking baclofen (Lioresal). The client calls the clinic nurse because of continuous feelings of weakness and fatigue and asks the nurse about discontinuing the medication. The nurse should make which appropriate response to the client? 1. "You should never stop the medication." 2. "It is best that you taper the dose if you intend to stop the medication." 3. "It is okay to stop the medication if you think that you can tolerate the muscle spasms." 4. "Weakness and fatigue commonly occur and will diminish with continued medication use."

4. "Weakness and fatigue commonly occur and will diminish with continued medication use." Rationale: The client should be instructed that symptoms such as drowsiness, weakness, and fatigue are more intense in the early phase of therapy and diminish with continued medication use. The client should be instructed never to withdraw or stop the medication abruptly, because abrupt withdrawal can cause visual hallucinations, paranoid ideation, and seizures. It is best for the nurse to inform the client that these symptoms will subside and encourage the client to continue the use of the medication.

163.) A client with trigeminal neuralgia tells the nurse that acetaminophen (Tylenol) is taken on a frequent daily basis for relief of generalized discomfort. The nurse reviews the client's laboratory results and determines that which of the following indicates toxicity associated with the medication? 1. Sodium of 140 mEq/L 2. Prothrombin time of 12 seconds 3. Platelet count of 400,000 cells/mm3 4. A direct bilirubin level of 2 mg/dL

4. A direct bilirubin level of 2 mg/dL Rationale: In adults, overdose of acetaminophen (Tylenol) causes liver damage. Option 4 is an indicator of liver function and is the only option that indicates an abnormal laboratory value. The normal direct bilirubin is 0 to 0.4 mg/dL. The normal platelet count is 150,000 to 400,000 cells/mm3. The normal prothrombin time is 10 to 13 seconds. The normal sodium level is 135 to 145 mEq/L.

In the administration of a drug such as levothyroxine (Synthroid), the nurse should teach the client: A) That therapy typically lasts about 6 months. B) That weekly laboratory tests for T4 levels will be required. C) To report weight loss, anxiety, insomnia, and palpitations. D) That the drug may be taken every other day if diarrhea occurs.

C) To report weight loss, anxiety, insomnia, and palpitations.

181.) A client is taking ticlopidine hydrochloride (Ticlid). The nurse tells the client to avoid which of the following while taking this medication? 1. Vitamin C 2. Vitamin D 3. Acetaminophen (Tylenol) 4. Acetylsalicylic acid (aspirin)

4. Acetylsalicylic acid (aspirin) Rationale: Ticlopidine hydrochloride is a platelet aggregation inhibitor. It is used to decrease the risk of thrombotic strokes in clients with precursor symptoms. Because it is an antiplatelet agent, other medications that precipitate or aggravate bleeding should be avoided during its use. Therefore, aspirin or any aspirin-containing product should be avoided.

242.) A client who was started on anticonvulsant therapy with clonazepam (Klonopin) tells the nurse of increasing clumsiness and unsteadiness since starting the medication. The client is visibly upset by these manifestations and asks the nurse what to do. The nurse's response is based on the understanding that these symptoms: 1. Usually occur if the client takes the medication with food 2. Are probably the result of an interaction with another medication 3. Indicate that the client is experiencing a severe untoward reaction to the medication 4. Are worse during initial therapy and decrease or disappear with long-term use

4. Are worse during initial therapy and decrease or disappear with long-term use Rationale: Drowsiness, unsteadiness, and clumsiness are expected effects of the medication during early therapy. They are dose related and usually diminish or disappear altogether with continued use of the medication. It does not indicate that a severe side effect is occurring. It is also unrelated to interaction with another medication. The client is encouraged to take this medication with food to minimize gastrointestinal upset. **Eliminate options 2 and 3 first because they are comparable or alike and because of the word "severe" in option 3**

178.) Methylergonovine (Methergine) is prescribed for a client with postpartum hemorrhage caused by uterine atony. Before administering the medication, the nurse checks which of the following as the important client parameter? 1. Temperature 2. Lochial flow 3. Urine output 4. Blood pressure

4. Blood pressure Rationale: Methylergonovine is an ergot alkaloid used for postpartum hemorrhage. It stimulates contraction of the uterus and causes arterial vasoconstriction. Ergot alkaloids are avoided in clients with significant cardiovascular disease, peripheral disease, hypertension, eclampsia, or preeclampsia. These conditions are worsened by the vasoconstrictive effects of the ergot alkaloids. The nurse would check the client's blood pressure before administering the medication and would follow agency protocols regarding withholding of the medication. Options 1, 2, and 3 are items that are checked in the postpartum period, but they are unrelated to the use of this medication.

237.) A client who is on lithium carbonate (Lithobid) will be discharged at the end of the week. In formulating a discharge teaching plan, the nurse will instruct the client that it is most important to: 1. Avoid soy sauce, wine, and aged cheese. 2. Have the lithium level checked every week. 3. Take medication only as prescribed because it can become addicting. 4. Check with the psychiatrist before using any over-the-counter (OTC) medications or prescription medications.

4. Check with the psychiatrist before using any over-the-counter (OTC) medications or prescription medications. Rationale: Lithium is the medication of choice to treat manic-depressive illness. Many OTC medications interact with lithium, and the client is instructed to avoid OTC medications while taking lithium. Lithium is not addicting, and, although serum lithium levels need to be monitored, it is not necessary to check these levels every week. A tyramine-free diet is associated with monoamine oxidase inhibitors.

155.) Mycophenolate mofetil (CellCept) is prescribed for a client as prophylaxis for organ rejection following an allogeneic renal transplant. Which of the following instructions does the nurse reinforce regarding administration of this medication? 1. Administer following meals. 2. Take the medication with a magnesium-type antacid. 3. Open the capsule and mix with food for administration. 4. Contact the health care provider (HCP) if a sore throat occurs.

4. Contact the health care provider (HCP) if a sore throat occurs. Rationale: Mycophenolate mofetil should be administered on an empty stomach. The capsules should not be opened or crushed. The client should contact the HCP if unusual bleeding or bruising, sore throat, mouth sores, abdominal pain, or fever occurs because these are adverse effects of the medication. Antacids containing magnesium and aluminum may decrease the absorption of the medication and therefore should not be taken with the medication. The medication may be given in combination with corticosteroids and cyclosporine. **neutropenia can occur with this medication**

132.) The client with non-Hodgkin's lymphoma is receiving daunorubicin (DaunoXome). Which of the following would indicate to the nurse that the client is experiencing a toxic effect related to the medication? 1. Fever 2. Diarrhea 3. Complaints of nausea and vomiting 4. Crackles on auscultation of the lungs

4. Crackles on auscultation of the lungs Rationale: Cardiotoxicity noted by abnormal electrocardiographic findings or cardiomyopathy manifested as congestive heart failure is a toxic effect of daunorubicin. Bone marrow depression is also a toxic effect. Nausea and vomiting are frequent side effects associated with the medication that begins a few hours after administration and lasts 24 to 48 hours. Fever is a frequent side effect, and diarrhea can occur occasionally. The other options, however, are not toxic effects. **keep in mind that the question is asking about a toxic effect and think: ABCs—airway, breathing, and circulation**

117.) A nurse has given the client taking ethambutol (Myambutol) information about the medication. The nurse determines that the client understands the instructions if the client immediately reports: 1. Impaired sense of hearing 2. Distressing gastrointestinal side effects 3. Orange-red discoloration of body secretions 4. Difficulty discriminating the color red from green

4. Difficulty discriminating the color red from green Rationale: Ethambutol causes optic neuritis, which decreases visual acuity and the ability to discriminate between the colors red and green. This poses a potential safety hazard when driving a motor vehicle. The client is taught to report this symptom immediately. The client is also taught to take the medication with food if gastrointestinal upset occurs. Impaired hearing results from antitubercular therapy with streptomycin. Orange-red discoloration of secretions occurs with rifampin (Rifadin).

159.) A nurse is caring for a client receiving morphine sulfate subcutaneously for pain. Because morphine sulfate has been prescribed for this client, which nursing action would be included in the plan of care? 1. Encourage fluid intake. 2. Monitor the client's temperature. 3. Maintain the client in a supine position. 4. Encourage the client to cough and deep breathe.

4. Encourage the client to cough and deep breathe. Rationale: Morphine sulfate suppresses the cough reflex. Clients need to be encouraged to cough and deep breathe to prevent pneumonia. **ABCs—airway, breathing, and circulation**

236.) A client is being treated for depression with amitriptyline hydrochloride. During the initial phases of treatment, the most important nursing intervention is: 1. Prescribing the client a tyramine-free diet 2. Checking the client for anticholinergic effects 3. Monitoring blood levels frequently because there is a narrow range between therapeutic and toxic blood levels of this medication 4. Getting baseline postural blood pressures before administering the medication and each time the medication is administered

4. Getting baseline postural blood pressures before administering the medication and each time the medication is administered Rationale: Amitriptyline hydrochloride is a tricyclic antidepressant often used to treat depression. It causes orthostatic changes and can produce hypotension and tachycardia. This can be frightening to the client and dangerous because it can result in dizziness and client falls. The client must be instructed to move slowly from a lying to a sitting to a standing position to avoid injury if these effects are experienced. The client may also experience sedation, dry mouth, constipation, blurred vision, and other anticholinergic effects, but these are transient and will diminish with time.

15.) The client with small cell lung cancer is being treated with etoposide (VePesid). The nurse who is assisting in caring for the client during its administration understands that which side effect is specifically associated with this medication? 1. Alopecia 2. Chest pain 3. Pulmonary fibrosis 4. Orthostatic hypotension

4. Orthostatic hypotension Rationale: A side effect specific to etoposide is orthostatic hypotension. The client's blood pressure is monitored during the infusion. Hair loss occurs with nearly all the antineoplastic medications. Chest pain and pulmonary fibrosis are unrelated to this medication.

175.) A nurse notes that a client is receiving lamivudine (Epivir). The nurse determines that this medication has been prescribed to treat which of the following? 1. Pancreatitis 2. Pharyngitis 3. Tonic-clonic seizures 4. Human immunodeficiency virus (HIV) infection

4. Human immunodeficiency virus (HIV) infection Rationale: Lamivudine is a nucleoside reverse transcriptase inhibitor and antiviral medication. It slows HIV replication and reduces the progression of HIV infection. It also is used to treat chronic hepatitis B and is used for prophylaxis in health care workers at risk of acquiring HIV after occupational exposure to the virus. **Note the letters "-vir" in the trade name for this medication**

192.) A nurse is collecting medication information from a client, and the client states that she is taking garlic as an herbal supplement. The nurse understands that the client is most likely treating which of the following conditions? 1. Eczema 2. Insomnia 3. Migraines 4. Hyperlipidemia

4. Hyperlipidemia Rationale: Garlic is an herbal supplement that is used to treat hyperlipidemia and hypertension. An herbal supplement that may be used to treat eczema is evening primrose. Insomnia has been treated with both valerian root and chamomile. Migraines have been treated with feverfew.

73.) A client with myasthenia gravis is suspected of having cholinergic crisis. Which of the following indicate that this crisis exists? 1. Ataxia 2. Mouth sores 3. Hypotension 4. Hypertension

4. Hypertension Rationale: Cholinergic crisis occurs as a result of an overdose of medication. Indications of cholinergic crisis include gastrointestinal disturbances, nausea, vomiting, diarrhea, abdominal cramps, increased salivation and tearing, miosis, hypertension, sweating, and increased bronchial secretions.

76.) Carbidopa-levodopa (Sinemet) is prescribed for a client with Parkinson's disease, and the nurse monitors the client for adverse reactions to the medication. Which of the following indicates that the client is experiencing an adverse reaction? 1. Pruritus 2. Tachycardia 3. Hypertension 4. Impaired voluntary movements

4. Impaired voluntary movements Rationale: Dyskinesia and impaired voluntary movement may occur with high levodopa dosages. Nausea, anorexia, dizziness, orthostatic hypotension, bradycardia, and akinesia (the temporary muscle weakness that lasts 1 minute to 1 hour, also known as the "on-off phenomenon") are frequent side effects of the medication.

225.) A nursing student is assigned to care for a client with a diagnosis of schizophrenia. Haloperidol (Haldol) is prescribed for the client, and the nursing instructor asks the student to describe the action of the medication. Which statement by the nursing student indicates an understanding of the action of this medication? 1. It is a serotonin reuptake blocker. 2. It inhibits the breakdown of released acetylcholine. 3. It blocks the uptake of norepinephrine and serotonin. 4. It blocks the binding of dopamine to the postsynaptic dopamine receptors in the brain.

4. It blocks the binding of dopamine to the postsynaptic dopamine receptors in the brain. Rationale: Haloperidol acts by blocking the binding of dopamine to the postsynaptic dopamine receptors in the brain. Imipramine hydrochloride (Tofranil) blocks the reuptake of norepinephrine and serotonin. Donepezil hydrochloride (Aricept) inhibits the breakdown of released acetylcholine. Fluoxetine hydrochloride (Prozac) is a potent serotonin reuptake blocker.

200.) A client is seen in the clinic for complaints of skin itchiness that has been persistent over the past several weeks. Following data collection, it has been determined that the client has scabies. Lindane is prescribed, and the nurse is asked to provide instructions to the client regarding the use of the medication. The nurse tells the client to: 1. Apply a thick layer of cream to the entire body. 2. Apply the cream as prescribed for 2 days in a row. 3. Apply to the entire body and scalp, excluding the face. 4. Leave the cream on for 8 to 12 hours and then remove by washing.

4. Leave the cream on for 8 to 12 hours and then remove by washing. Rationale: Lindane is applied in a thin layer to the entire body below the head. No more than 30 g (1 oz) should be used. The medication is removed by washing 8 to 12 hours later. Usually, only one application is required.

119.) A client with diabetes mellitus who has been controlled with daily insulin has been placed on atenolol (Tenormin) for the control of angina pectoris. Because of the effects of atenolol, the nurse determines that which of the following is the most reliable indicator of hypoglycemia? 1. Sweating 2. Tachycardia 3. Nervousness 4. Low blood glucose level

4. Low blood glucose level Rationale: β-Adrenergic blocking agents, such as atenolol, inhibit the appearance of signs and symptoms of acute hypoglycemia, which would include nervousness, increased heart rate, and sweating. Therefore, the client receiving this medication should adhere to the therapeutic regimen and monitor blood glucose levels carefully. Option 4 is the most reliable indicator of hypoglycemia.

205.) A nurse is assisting in preparing to administer acetylcysteine (Mucomyst) to a client with an overdose of acetaminophen (Tylenol). The nurse prepares to administer the medication by: 1. Administering the medication subcutaneously in the deltoid muscle 2. Administering the medication by the intramuscular route in the gluteal muscle 3. Administering the medication by the intramuscular route, mixed in 10 mL of normal saline 4. Mixing the medication in a flavored ice drink and allowing the client to drink the medication through a straw

4. Mixing the medication in a flavored ice drink and allowing the client to drink the medication through a straw Rationale: Because acetylcysteine has a pervasive odor of rotten eggs, it must be disguised in a flavored ice drink. It is consumed preferably through a straw to minimize contact with the mouth. It is not administered by the intramuscular or subcutaneous route. **Knowing that the medication is a solution that is also used for nebulization treatments will assist you to select the option that indicates an oral route**

17.) The client with ovarian cancer is being treated with vincristine (Oncovin). The nurse monitors the client, knowing that which of the following indicates a side effect specific to this medication? 1. Diarrhea 2. Hair loss 3. Chest pain 4. Numbness and tingling in the fingers and toes

4. Numbness and tingling in the fingers and toes Rationale: A side effect specific to vincristine is peripheral neuropathy, which occurs in almost every client. Peripheral neuropathy can be manifested as numbness and tingling in the fingers and toes. Depression of the Achilles tendon reflex may be the first clinical sign indicating peripheral neuropathy. Constipation rather than diarrhea is most likely to occur with this medication, although diarrhea may occur occasionally. Hair loss occurs with nearly all the antineoplastic medications. Chest pain is unrelated to this medication.

222.) A nurse has administered a dose of diazepam (Valium) to a client. The nurse would take which important action before leaving the client's room? 1. Giving the client a bedpan 2. Drawing the shades or blinds closed 3. Turning down the volume on the television 4. Per agency policy, putting up the side rails on the bed

4. Per agency policy, putting up the side rails on the bed Rationale: Diazepam is a sedative-hypnotic with anticonvulsant and skeletal muscle relaxant properties. The nurse should institute safety measures before leaving the client's room to ensure that the client does not injure herself or himself. The most frequent side effects of this medication are dizziness, drowsiness, and lethargy. For this reason, the nurse puts the side rails up on the bed before leaving the room to prevent falls. Options 1, 2, and 3 may be helpful measures that provide a comfortable, restful environment, but option 4 is the one that provides for the client's safety needs.

184.) A nurse preparing a client for surgery reviews the client's medication record. The client is to be nothing per mouth (NPO) after midnight. Which of the following medications, if noted on the client's record, should the nurse question? 1. Cyclobenzaprine (Flexeril) 2. Alendronate (Fosamax) 3. Allopurinol (Zyloprim) 4. Prednisone

4. Prednisone Rationale: Prednisone is a corticosteroid that can cause adrenal atrophy, which reduces the body's ability to withstand stress. Before and during surgery, dosages may be temporarily increased. Cyclobenzaprine is a skeletal muscle relaxant. Alendronate is a bone-resorption inhibitor. Allopurinol is an antigout medication.

189.) Prostaglandin E1 is prescribed for a child with transposition of the great arteries. The mother of the child asks the nurse why the child needs the medication. The nurse tells the mother that the medication: 1. Prevents hypercyanotic (blue or tet) spells 2. Maintains an adequate hormone level 3. Maintains the position of the great arteries 4. Provides adequate oxygen saturation and maintains cardiac output

4. Provides adequate oxygen saturation and maintains cardiac output Rationale: A child with transposition of the great arteries may receive prostaglandin E1 temporarily to increase blood mixing if systemic and pulmonary mixing are inadequate to maintain adequate cardiac output. Options 1, 2, and 3 are incorrect. In addition, hypercyanotic spells occur in tetralogy of Fallot. **Use the ABCs—airway, breathing, and circulation—to answer the question. The correct option addresses circulation**

13.) The client with squamous cell carcinoma of the larynx is receiving bleomycin intravenously. The nurse caring for the client anticipates that which diagnostic study will be prescribed? 1. Echocardiography 2. Electrocardiography 3. Cervical radiography 4. Pulmonary function studies

4. Pulmonary function studies Rationale: Bleomycin is an antineoplastic medication (Chemotheraputic Agents) that can cause interstitial pneumonitis, which can progress to pulmonary fibrosis. Pulmonary function studies along with hematological, hepatic, and renal function tests need to be monitored. The nurse needs to monitor lung sounds for dyspnea and crackles, which indicate pulmonary toxicity. The medication needs to be discontinued immediately if pulmonary toxicity occurs. Options 1, 2, and 3 are unrelated to the specific use of this medication.

70.) Oxybutynin chloride (Ditropan XL) is prescribed for a client with neurogenic bladder. Which sign would indicate a possible toxic effect related to this medication? 1. Pallor 2. Drowsiness 3. Bradycardia 4. Restlessness

4. Restlessness Rationale: Toxicity (overdosage) of this medication produces central nervous system excitation, such as nervousness, restlessness, hallucinations, and irritability. Other signs of toxicity include hypotension or hypertension, confusion, tachycardia, flushed or red face, and signs of respiratory depression. Drowsiness is a frequent side effect of the medication but does not indicate overdosage.

44.) A client is receiving acetylcysteine (Mucomyst), 20% solution diluted in 0.9% normal saline by nebulizer. The nurse should have which item available for possible use after giving this medication? 1. Ambu bag 2. Intubation tray 3. Nasogastric tube 4. Suction equipment

4. Suction equipment Rationale: Acetylcysteine can be given orally or by nasogastric tube to treat acetaminophen overdose, or it may be given by inhalation for use as a mucolytic. The nurse administering this medication as a mucolytic should have suction equipment available in case the client cannot manage to clear the increased volume of liquefied secretions.

166.) Alendronate (Fosamax) is prescribed for a client with osteoporosis. The client taking this medication is instructed to: 1. Take the medication at bedtime. 2. Take the medication in the morning with breakfast. 3. Lie down for 30 minutes after taking the medication. 4. Take the medication with a full glass of water after rising in the morning.

4. Take the medication with a full glass of water after rising in the morning. Rationale: Precautions need to be taken with administration of alendronate to prevent gastrointestinal side effects (especially esophageal irritation) and to increase absorption of the medication. The medication needs to be taken with a full glass of water after rising in the morning. The client should not eat or drink anything for 30 minutes following administration and should not lie down after taking the medication.

99.) The client with acquired immunodeficiency syndrome and Pneumocystis jiroveci infection has been receiving pentamidine isethionate (Pentam 300). The client develops a temperature of 101° F. The nurse does further monitoring of the client, knowing that this sign would most likely indicate: 1. The dose of the medication is too low. 2. The client is experiencing toxic effects of the medication. 3. The client has developed inadequacy of thermoregulation. 4. The result of another infection caused by leukopenic effects of the medication.

4. The result of another infection caused by leukopenic effects of the medication. Rationale: Frequent side effects of this medication include leukopenia, thrombocytopenia, and anemia. The client should be monitored routinely for signs and symptoms of infection. Options 1, 2, and 3 are inaccurate interpretations.

Ibuprofen (Motrin) given to an older adult w/ rheumatoid arthritis or osteoarthritis , what is the normal oral dose?

400-800 mg three or four times a day 1200-3200 mg/day

Insulin: Slow Acting Onset

4h

The nurse is preparing to administer an anticonvulsant for status epilepticus. The PCP prescibed lorazepam-ativanmg IV. The drug is availble in a dosage of mg/mL the Rn administers

4mg/2mg/mL= 2mL

Controlled substances are grouped in schedules/categories what is the Number of schedules?

5

Allergic effects occur in what percentage of persons recieving penicillin compounds

5-10%

121.) A client who is taking hydrochlorothiazide (HydroDIURIL, HCTZ) has been started on triamterene (Dyrenium) as well. The client asks the nurse why both medications are required. The nurse formulates a response, based on the understanding that: 1. Both are weak potassium-losing diuretics. 2. The combination of these medications prevents renal toxicity. 3. Hydrochlorothiazide is an expensive medication, so using a combination of diuretics is cost-effective. 4. Triamterene is a potassium-sparing diuretic, whereas hydrochlorothiazide is a potassium-losing diuretic.

4. Triamterene is a potassium-sparing diuretic, whereas hydrochlorothiazide is a potassium-losing diuretic. Rationale: Potassium-sparing diuretics include amiloride (Midamor), spironolactone (Aldactone), and triamterene (Dyrenium). They are weak diuretics that are used in combination with potassium-losing diuretics. This combination is useful when medication and dietary supplement of potassium is not appropriate. The use of two different diuretics does not prevent renal toxicity. Hydrochlorothiazide is an effective and inexpensive generic form of the thiazide classification of diuretics. **It is especially helpful to remember that hydrochlorothiazide is a potassium-losing diuretic and triamterene is a potassium-sparing diuretic**

104.) Disulfiram (Antabuse) is prescribed for a client who is seen in the psychiatric health care clinic. The nurse is collecting data on the client and is providing instructions regarding the use of this medication. Which is most important for the nurse to determine before administration of this medication? 1. A history of hyperthyroidism 2. A history of diabetes insipidus 3. When the last full meal was consumed 4. When the last alcoholic drink was consumed

4. When the last alcoholic drink was consumed Rationale: Disulfiram is used as an adjunct treatment for selected clients with chronic alcoholism who want to remain in a state of enforced sobriety. Clients must abstain from alcohol intake for at least 12 hours before the initial dose of the medication is administered. The most important data are to determine when the last alcoholic drink was consumed. The medication is used with caution in clients with diabetes mellitus, hypothyroidism, epilepsy, cerebral damage, nephritis, and hepatic disease. It is also contraindicated in severe heart disease, psychosis, or hypersensitivity related to the medication.

When the Type 1 diabetic patient asks why his 7 AM insulin has been changed from NPH insulin to 70/30 premixed insulin, the nurse explains that 70/30 insulin: 1. is absorbed more rapidly into the bloodstream. 2. has no peak action time and lasts all day. 3. makes insulin administration easier and safer. 4. give a bolus of rapid-acting insulin to prevent hyperglycemia after breakfast. the morning meal.

4. give a bolus of rapid-acting insulin to prevent hyperglycemia after breakfast. 70/30 insulin is 30% rapid-acting and 70% intermediate-acting insulin. The rapid action of the 7 AM premixed insulin prevents hyperglycemia after the morning meal.

The type 1 diabetic patient has an insulin order for NPH insulin, 35 U, to be given at 7 AM. The patient is also NPO for laboratory work that will not be drawn until 10 AM. The nurse should: 1. give the insulin as ordered. 2. give the insulin with a small snack. 3. inform the charge nurse. 4. hold the insulin until after the blood draw.

4. hold the insulin until after the blood Holding the insulin for the NPO order is appropriate. The patient will not be getting food until after the blood draw, so will not need the insulin until then. Giving the insulin as ordered will create a possibility of hypoglycemia before the blood is drawn. Giving a snack to a patient who is NPO is inappropriate.

The nurse is drawing up a teaching plan for a patient who has type 1 diabetes. The doctor has ordered two types of insulin, 10 U of regular insulin and 35 U of NPH insulin. The proper procedure is to: 1. draw up the insulins in two separate syringes so that there can be no confusion. 2. draw up the regular insulin before drawing up the NPH insulin. 3. inject air into the NPH insulin, draw it up to 35 U, then inject air into the clear regular insulin and withdraw to 45 U. 4. inject 35 U air into the NPH insulin, inject 10 U air into the regular insulin, withdraw 10 U of the regular insulin, and withdraw 35 U of the NPH insulin.

4. inject 35 U air into the NPH insulin, inject 10 U air into the regular insulin, withdraw 10 U of the regular insulin, and withdraw 35 U of the NPH insulin. When drawing up two insulins, the vials are injected with air and the regular insulin is drawn first. This slow and time-consuming activity has been greatly reduced with the advent of premixed insulins.

4. Reye's syndrome, a potentially fatal illness associated with liver failure and encephalopathy is associated with the administration of which over-the-counter (OTC) medication? a. acetaminophen (Tylenol) b. ibuprofen (Motrin) c. aspirin d. brompheniramine/psudoephedrine (Dimetapp)

4.Answer C. Virus-infected children who are given aspirin to manage pain, fever, and inflammation are at an increased risk of developing Reye's syndrome. Use of acetaminophen has not been associated with Reye's syndrome and can be safely given to patients with fever due to viral illnesses. Ibuprofen adverse effects include GI irritation and bleeding, and in toxic doses, both renal and hepatic failure are reported. However, ibuprofen has not been associated with the onset of Reye's disease. Brompheniramine/psudoephedrine contains a first generation OTC antihistamine and a decongestant. Neither agent has been associated with the development of Reye's syndrome.

4. Some central venous access devices (CVAD) have more than one lumen. These multi lumen catheters: a. Have an increased risk of infiltration. b. Only work a short while because the small bore clots off. c. Are beneficial to patient care but are prohibitively expensive. d. Allow different medications or solutions to be administered simultaneously.

4.Answer D. A multilumen catheter contains separate ports and means to administer agents. An agent infusing in one port cannot mix with an agent infusing into another port. Thus, agents that would be incompatible if given together can be given in separate ports simultaneously.

185.) Which of the following herbal therapies would be prescribed for its use as an antispasmodic? Select all that apply. 1.Aloe 2.Kava 3.Ginger 4.Chamomile 5.Peppermint oil

4.Chamomile 5.Peppermint oil Rationale: Chamomile has a mild sedative effect and acts as an antispasmodic and anti-inflammatory. Peppermint oil acts as an antispasmodic and is used for irritable bowel syndrome. Topical aloe promotes wound healing. Aloe taken orally acts as a laxative. Kava has an anxiolytic, sedative, and analgesic effect. Ginger is effective in relieving nausea.

Oral trihexyphenidyl 4 mg is ordered. The drug is available as an elixir w/ a strength of 2mg/5mL the Nurse should administer

4/2 x 5 =10 mL

Methdaone Treatment programs can be effective in helping the opioid addicted person withdrawl- which of the following is the recomended maintenence dose of methadone

40-120mg/d

5. The most important instructions a nurse can give a patient regarding the use of the antibiotic Ampicillin prescribed for her are to a. call the physician if she has any breathing difficulties. b. take it with meals so it doesn't cause an upset stomach. c. take all of the medication prescribed even if the symptoms stop sooner. d. not share the pills with anyone else.

5. C. Frequently patients do not complete an entire course of antibiotic therapy, and the bacteria are not destroyed.

what size needle is used for heparin or enoxaparin subc?

5/8 inch

Order Zithromax for Mild moderate Strep, How many mL of suspension are req for 1st dose and each of the daily doses for next 4days.

500 mg max daily dose/200mg per 5ml hand=12.5mL 1st Load dose 250 mg normal daily dose/200mg per5ml hand=6.25mL remaining 4 days

6. A male patient needs a percutaneously inserted central catheter (PICC) for prolonged IV therapy. He knows it can be inserted without going to the operating room. He mentions that, "at least the doctor won't be wearing surgical garb, will he?" How will the nurse answer the patient? a. "You are correct. It is a minor procedure performed on the unit and does not necessitate surgical attire." b. "To decrease the risk of infection, the doctor inserting the PICC will wear a cap, mask, and sterile gown and gloves." c. "It depends on the doctor's preference." d. "Most doctors only wear sterile gloves, not a cap, mask, or sterile gown."

6.Answer C. Strict aseptic technique including the use of cap, mask, and sterile gown and gloves is require when placing a central venous line including a PICC. Options A, B, and D are incorrect statements. They increase the risk of infection.

6. A 13-month-old child recently arrived in the United States from a foreign country with his parents and needs childhood immunizations. His mother reports that he is allergic to eggs. Upon further questioning, you determine that the allergy to eggs is anaphylaxis. Which of the following vaccines should he not receive? a. Hepatitis B b. inactivated polio c. diphtheria, acellular pertussis, tetanus (DTaP) d. mumps, measles, rubella (MMR)

6.Answer D. The measles portion of the MMR vaccine is grown in chick embryo cells. The current MMR vaccine does not contain a significant amount of egg proteins, and even children with dramatic egg allergies are extremely unlikely to have an anaphylactic reaction. However, patients that do respond to egg contact with anaphylaxis should be in a medically controlled setting where full resuscitation efforts can be administered if anaphylaxis results. The vaccines in options a,b and c do not contain egg protein.

Aspirin 650 mg oral is prescribed On hand is aspirin 325mg tablets. The RN should administer?

650/325= 2 tablets

7. The cell and Coombs classification system categorizes allergic reactions and is useful in describing and classifying patient reactions to drugs. Type I reactions are immediate hypersensitivity reactions and are mediated by: a. immunoglobulin E (IgE). b. immunoglobulin G (IgG). c. immunoglobulin A (IgA). d. immunoglobulin M (IgM).

7.Answer A. IgE, the least common serum immunoglobulin (Ig) binds very tightly to receptors on basophils and mast cells and is involved in allergic reactions. Binding of the allergen to the IgE on the cells results in the release of various pharmacological mediators that result in allergic symptoms. IgG is the major Ig (75 percent of serum Ig is IgG). Most versatile Ig because it is capable of carrying out all of the functions of Ig molecules. IgG is the only class of Ig that crosses the placenta. It is an opsonin, a substance that enhances phagocytosis. IgA, the second most common serum Ig is found in secretions (tears, saliva, colostrum, and mucus). It is important in local (mucosal) immunity. IgM, the third most common serum Ig, is the first Ig to be made by the fetus and the first Ig to be made by a virgin B cell when it is stimulated by antigen. IgM antibodies are very efficient in leading to the lysis of microorganisms.

7. A male patient is to receive a percutaneously inserted central catheter (PICC). He asks the nurse whether the insertion will hurt. How will the nurse reply? a. "You will have general anesthesia so you won't feel anything." b. "It will be inserted rapidly, and any discomfort is fleeting." c. "The insertion site will be anesthetized. Threading the catheter through the vein is not painful." d. "You will receive sedation prior to the procedure."

7.Answer C. Pain related to PICC insertion occurs with puncture of the skin. When inserting PICC lines, the insertion site is anesthetized so no pain is felt. The patient will not receive general anesthesia or sedation. Statement 2 is false. Unnecessary pain should be prevented.

Bulk forming laxatives should be mixed in at least _____ounces of fluid

8 ounces

Insulin: Slow Acting Peak

8-20h

8. When counseling a patient who is starting to take MAO (monoamine oxidase) inhibitors such as Nardil for depression, it is essential that they be warned not to eat foods containing tyramine, such as: a. Roquefort, cheddar, or Camembert cheese. b. grape juice, orange juice, or raisins. c. onions, garlic, or scallions. d. ground beef, turkey, or pork.

8. A. Monoamine oxidase inhibitors react with foods high in the amino acid tyramine to cause dangerously high blood pressure. Aged cheeses are all high in this amino acid the other foods are not.

8. What volume of air can safely be infused into a patient with a central venous access device (CVAD)? a. It is dependent on the patient's weight and height. b. Air entering the patient through a CVAD will follow circulation to the lungs where it will be absorbed and cause no problems. c. It is dependent on comorbidities such as asthma or chronic obstructive lung disease. d. None.

8.Answer B. Any air entering the right heart can lead to a pulmonary embolus. All air should be purged from central venous lines none should enter the patient.

Septra Tablets- e scored tablets contains 80mg trimethoprim and 400 mg sulfamethoxazole. How many tablets should pt take per 24 hrs?

80mg ordered/400mg on hand =2 tablets per 12 hrs =4daily

When assessing the patient experiencing the onset of symptoms of type 1 diabetes, which question should the nurse ask? a. "Have you lost any weight lately?" b. "Do you crave fluids containing sugar?" c. "How long have you felt anorexic?" d. "Is your urine unusually dark-colored?"

A Rationale: Weight loss occurs because the body is no longer able to absorb glucose and starts to break down protein and fat for energy. The patient is thirsty but does not necessarily crave sugar- containing fluids. Increased appetite is a classic symptom of type 1 diabetes. With the classic symptom of polyuria, urine will be very dilute.

An increased susceptibility to infections

A 17-year-old male has started valproic acid (Depakene) for treatment of bipolar disorder. While he is taking this drug, he should be carefully monitored for: a. Unusual abdominal pain, especially in the upper quadrant areas b. An increased susceptibility to infections c. Lethargy or confusion d. Unusual bleeding or bruising

b. "These factors may put you at higher risk for myopathy."

A 70-year-old client who is taking several cardiac antidysrhythmic medications has been prescribed simvastatin (Zocor) 80 mg/day. What is essential information for the nurse to teach the client? a. "This dose may lower your cholesterol too much." b. "These factors may put you at higher risk for myopathy." c. "You should not take this drug with cardiac medications." d. "This combination will cause you to have nausea and vomiting."

b. Hepatic disease

A client diagnosed with hypercholesterolemia is prescribed lovastatin (Mevacor). The nurse is reviewing the client's history and would contact the health care provider about which of these conditions in the client's history? a. Chronic pulmonary disease b. Hepatic disease c. Leukemia d. Renal disease

c. Hyperlipidemia

A client has a serum cholesterol level of 265 mg/dL, triglyceride level of 235 mg/dL, and LDL of 180 mg/dL. What do these serum levels indicate? a. Hypolipidemia b. Normolipidemia c. Hyperlipidemia d. Alipidemia

c. Thrombolytic agent

A client is admitted to the emergency department with an acute myocardial infarction. Which drug category does the nurse expect to be given to the client early for the prevention of tissue necrosis following blood clot blockage in a coronary or cerebral artery? a. Anticoagulant agent b. Antiplatelet agent c. Thrombolytic agent d. Low-molecular-weight heparin (LMWH)

b. warfarin (Coumadin)

A client is being changed from an injectable anticoagulant to an oral anticoagulant. Which anticoagulant does the nurse realize is administered orally? a. enoxaparin sodium (Lovenox) b. warfarin (Coumadin) c. bivalirudin (Angiomax) d. lepirudin (Refludan)

a. Relaxes the arterial walls within the skeletal muscles b. May cause hypotension, chest pain, and palpitations

A client is diagnosed with peripheral arterial disease (PAD). He is prescribed isoxsuprine (Vasodilan). The nurse acknowledges that isoxsuprine does what? (Select all that apply.) a. Relaxes the arterial walls within the skeletal muscles b. May cause hypotension, chest pain, and palpitations c. Increases the rigidity of arteriosclerotic blood vessels d. May increase intermittent claudication e. May lead to hypertension and bradycardia f. Commonly causes an adverse effect of rhabdomyolysis

b. Assess lung sounds before and after administration. c. Assess blood pressure before and after administration. d. Maintain accurate intake and output record.

A client is ordered furosemide (Lasix) to be given via intravenous push. What interventions should the nurse perform? (Select all that apply.) a. Administer at a rate no faster than 20 mg/min. b. Assess lung sounds before and after administration. c. Assess blood pressure before and after administration. d. Maintain accurate intake and output record. e. Monitor ECG continuously. f. Insert an arterial line for continuous blood pressure monitoring.

d. "Wear protective clothing and sunscreen while on this medication."

A client is prescribed Thalitone (chlorthalidone). What is the most important information the nurse should teach the client? a. "Do not drink more than 10 ounces of fluid a day while on this medication." b. "Take this medication on an empty stomach." c. "Take this medication before bed each night." d. "Wear protective clothing and sunscreen while on this medication."

b. Respiratory assessment

A client is prescribed a noncardioselective beta1 blocker. What nursing intervention is a priority for this client? a. Assessment of blood glucose levels b. Respiratory assessment c. Orthostatic blood pressure assessment d. Teaching about potential tachycardia

d. Subcutaneously

A client is prescribed dalteparin (Fragmin). LMWH is administered via which route? a. Intravenously b. Intramuscularly c. Intradermally d. Subcutaneously

a. A longer half-life than heparin

A client is prescribed enoxaparin (Lovenox). The nurse knows that low-molecular-weight heparin (LMWH) has what kind of half-life? a. A longer half-life than heparin b. A shorter half-life than heparin c. The same half-life as heparin d. A four-times shorter half-life than heparin

c. Muscle pain.

A client is prescribed ezetimibe (Zetia). Which assessment finding will require immediate action by the nurse? a. Headache. b. Slight nausea. c. Muscle pain. d. Fatigue.

b. "You may experience headaches with this medication."

A client is prescribed gemfibrozil (Lopid) for treatment of hyperlipidemia type IV. What is important for the nurse to teach the client? a. "Take aspirin before the medication if you experience facial flushing." b. "You may experience headaches with this medication." c. "You will need to have weekly blood drawn to assess for hyperkalemia." d. "Cholesterol levels will need to be assessed daily for one week."

b. "It usually takes about 3 days to achieve a therapeutic effect for warfarin, so the heparin is continued until the warfarin is therapeutic."

A client is started on warfarin (Coumadin) therapy while still receiving intravenous heparin. The client questions the nurse about the risk for bleeding. How should the nurse respond? a. "Your concern is valid. I will call the doctor to discontinue the heparin." b. "It usually takes about 3 days to achieve a therapeutic effect for warfarin, so the heparin is continued until the warfarin is therapeutic." c. "Because of your valve replacement, it is especially important for you to be anticoagulated. The heparin and warfarin together are more effective than one alone." d. "Because you are now up and walking, you have a higher risk of blood clots and therefore need to be on both medications."

a. Evaluate digoxin levels.

A client is taking digoxin (Lanoxin) 0.25 mg and furosemide (Lasix) 40 mg. When the nurse enters the room, the client states, "There are yellow halos around the lights." Which action will the nurse take? a. Evaluate digoxin levels. b. Withhold the furosemide c. Administer potassium. d. Document the findings and reassess in 1 hour.

a. "I take aspirin daily for headaches."

A client is taking enoxaparin (Lovenox) daily. Which client statement requires additional monitoring? a. "I take aspirin daily for headaches." b. "I take ibuprofen (Motrin) at least once a week for joint pain." c. "Whenever I have a fever, I take acetaminophen (Tylenol)." d. "I take my medicine first thing in the morning."

b. Hypokalemia

A client is taking hydrochlorothiazide 50 mg/day and digoxin 0.25 mg/day. What type of electrolyte imbalance does the nurse expect to occur? a. Hypocalcemia b. Hypokalemia c. Hyperkalemia d. Hypermagnesemia

d. Liver enzymes

A client is taking lovastatin (Mevacor). Which serum level is most important for the nurse to monitor? a. Blood urea nitrogen b. Complete blood count c. Cardiac enzymes d. Liver enzymes

b. Elevated INR range

A client is taking warfarin 5 mg/day for atrial fibrillation. The client's international normalized ration (INR) is 3.8. The nurse would consider the INR to be what? a. Within normal range b. Elevated INR range c. Low INR range d. Low average INR range

a. "Apply the patch to a nonhairy area of the upper torso or arm."

A client is to be discharged home with a transdermal nitroglycerin patch. Which instruction will the nurse include in the client's teaching plan? a. "Apply the patch to a nonhairy area of the upper torso or arm." b. "Apply the patch to the same site each day." c. "If you have a headache, remove the patch for 4 hours and then reapply." d. "If you have chest pain, apply a second patch next to the first patch."

c. Monitor blood pressure continuously.

A client is to begin treatment for short-term management of heart failure with milrinone lactate (Primacor). What is the priority nursing action? a. Administer digoxin via IV infusion with the Primacor. b. Administer Lasix (furosemide) via IV infusion after the Primacor. c. Monitor blood pressure continuously. d. Maintain an infusion of lactated Ringers with Primacor infusion.

b. abciximab (ReoPro)

A client is to undergo a coronary angioplasty. The nurse acknowledges that which drug is used primarily for preventing reocclusion of coronary arteries following a coronary angioplasty? a. clopidogrel (Plavix) b. abciximab (ReoPro) c. warfarin (Coumadin) d. streptokinase

b. Administer vitamin K.

A client who has been taking warfarin (Coumadin) is admitted with coffee-ground emesis. What is the nurse's primary action? a. Administer vitamin E. b. Administer vitamin K. c. Administer protamine sulfate. d. Administer calcium gluconate.

b. Teach the client of potential drug interactions with anticoagulants.

A client who is taking warfarin (Coumadin) requests an aspirin for headache relief. What is the nurse's best response? a. Administer 650 mg of acetylsalicylic acid (ASA) and reassess pain in 30 minutes. b. Teach the client of potential drug interactions with anticoagulants. c. Explain to the client that ASA is contraindicated and administer ibuprofen as ordered. d. Explain that the headache is an expected side effect and will subside shortly.

a. protamine sulfate

A client who received heparin begins to bleed, and the physician calls for the antidote. The nurse knows that which is the antidote for heparin? a. protamine sulfate b. vitamin K c. aminocaproic acid d. vitamin C

b. "Increasing fluid and fiber in your diet can help prevent the side effect of constipation."

A client who takes clonidine (Catapres) is to be discharged to home. Which instruction will the nurse include when teaching this client? a. "Your blood pressure should be checked by a health care provider at least once a year." b. "Increasing fluid and fiber in your diet can help prevent the side effect of constipation." c. "Intense exercise or prolonged standing is not a problem with clonidine as it can be with other antihypertensive agents." d. "If you are having difficulty with the common side effect of drooling, notify your health care provider so your dosage can be adjusted."

c. Decreased aldosterone

A client with hyperaldosteronism is prescribed spironolactone (Aldactone). What assessment finding would the nurse evaluate as a positive outcome? a. Decreased potassium level b. Decreased crackles in the lung bases c. Decreased aldosterone d. Decreased ankle edema

c. Stage 1 hypertension

A client's blood pressure (BP) is 145/90. According to the guidelines for determining hypertension, the nurse realizes that the client's BP is at which stage? a. Normal b. Prehypertension c. Stage 1 hypertension d. Stage 2 hypertension

b. It is the desired level of HDL.

A client's high-density lipoprotein (HDL) is 60 mg/dL. What does the nurse acknowledge concerning this level? a. It is lower than the desired level of HDL. b. It is the desired level of HDL. c. It is higher than the desired level of HDL. d. It is a much lower HDL level than desired.

c. Continue to monitor the client.

A client's recently drawn serum lidocaine drug level is 3.0 mcg/mL. What is the nurse's priority intervention? a. Increase the lidocaine infusion. b. Decrease the lidocaine infusion. c. Continue to monitor the client. d. Stop the IV drip for 1 hour.

a. Administer ordered dose of digoxin.

A client's serum digoxin level is drawn, and it is 0.4 ng/mL. What is the nurse's priority action? a. Administer ordered dose of digoxin. b. Hold future digoxin doses. c. Administer potassium. d. Call the health care provider.

What is agoraphobia?

A fear of open spaces or leaving their home. They fail to interact with others and are never w/o symptoms.

c. The fact that Lasix has shown efficacy in treating persons with renal insufficiency.

A nurse admits a client diagnosed with pneumonia. The client has a history of chronic renal insufficiency, and the health care provider orders furosemide (Lasix) 40 mg twice a day. What is most important to include in the teaching plan for this client? a. That the medication will have to be monitored very carefully owing to the client's diagnosis of pneumonia. b. The fact that Lasix has been proven to decrease symptoms with pneumonia. c. The fact that Lasix has shown efficacy in treating persons with renal insufficiency. d. That the medication will need to be given at a higher than normal dose owing to the client's medical problems.

c. A decrease in arterial pH

A nurse is caring for a client receiving acetazolamide (Diamox). Which assessment finding will require immediate nursing intervention? a. A decrease in bicarbonate level b. An increase in urinary output c. A decrease in arterial pH d. An increase in PaO2

c. ECG

A nurse is caring for a client who has been started on ibutilide (Corvert). Which assessment is a priority for this client? a. Blood pressure measurement b. BUN and creatinine c. ECG d. Lung sounds

c. gemfibrozil (Lopid)

A nurse is caring for a client with elevated triglyceride levels who is unresponsive to HMG-CoA reductase inhibitors. What medication will the nurse administer? a. cholestyramine (Questran) b. colestipol (Colestid) c. gemfibrozil (Lopid) d. simvastatin (Zocor)

d. Client stating that pain is 0 out of 10

A nurse is monitoring a client with angina for therapeutic effects of nitroglycerin. Which assessment finding indicates that the nitroglycerin has been effective? a. Blood pressure 120/80 mm Hg b. Heart rate 70 beats per minute c. ECG without evidence of ST changes d. Client stating that pain is 0 out of 10

At 5 PM a patient who is taking NPH insulin develops hunger, shakiness, and sweating. A nurse assesses the medication administration record (MAR) and should recognize that the patient's symptoms are related to an injection of NPH insulin at which of these times? A) 2 AM B) 8 AM C) 1 PM D) 3 PM

A) 2 AM The patient is exhibiting symptoms of hypoglycemia at 5 PM. NPH has a peak action of 8 to 10 hours after administration. Based on the duration of action of NPH insulin, the patient's hypoglycemic symptoms are from the 8 AM injection of NPH insulin. An injection of NPH insulin at 2 AM, 1 PM, or 3 PM would not cause hypoglycemic symptoms based on the average duration of action from NPH insulin.

A patient is scheduled to start taking insulin glargine (Lantus). On the care plan a nurse should include which of these outcomes related to the therapeutic effects of the medication? A) Blood glucose control for 24 hours B) Mealtime coverage of blood glucose C) Less frequent blood glucose monitoring D) Peak effect achieved in 2 to 4 hours

A) Blood glucose control for 24 hours Insulin glargine is administered as a once-daily subcutaneous injection for patients who have type 1 diabetes. It is used for basal insulin coverage, not mealtime coverage. It has a prolonged duration up to 24 hours with no peaks. Blood glucose monitoring is still an essential component to achieve tight glycemic control.

The nurse receives a lab report indicating that the phenytoin (Dilantin) level for the patient she saw in the clinic yesterday is 16 mcg/mL. Which intervention is most appropriate? A) Continue as planned since the level is within normal limits. B) Tell the patient to hold today's dose and return to the clinic. C) Consult the prescriber to recommend an increased dose. D) Have the patient call 911 and meet the patient in the emergency department.

A) Continue as planned since the level is within normal limits

Which of these instructions should the nurse provide when teaching a patient to mix regular insulin and NPH insulin in the same syringe? A) Draw up the clear regular insulin first, followed by the cloudy NPH insulin. B) It is not necessary to rotate the NPH insulin vial when it is mixed with regular insulin. C) The order of drawing up insulin does not matter as long as the insulin is refrigerated. D) Rotate each day subcutaneous injection sites among the arm, thigh, and abdomen.

A) Draw up the clear regular insulin first, followed by the cloudy NPH insulin. To ensure a consistent response, only NPH insulin is appropriate for mixing with a short-acting insulin. Unopened vials of insulin should be refrigerated; current vials can be kept at room temperature for up to 1 month. Drawing up the regular insulin into the syringe first prevents accidental mixture of NPH insulin into the vial of regular insulin, which could alter the pharmacokinetics of subsequent doses taken out of the regular insulin vial. NPH insulin is a cloudy solution, and it should always be gently rotated to evenly disperse the particles before loading the syringe. Subcutaneous injections should be made using one region of the body (e.g., the abdomen or thigh) and rotated within that region for 1 month.

Which agent below is most likely to cause serious respiratory depression as a potential adverse reaction? A) Morphine (Duramorph) B) Pentazocine (Talwin) C) Hydrocodone (Lortab) D) Nalmefene (Revex)

A) Morphine (Duramorph) Morphine is a strong opioid agonist and as such has the highest likelihood of respiratory depression. Pentazocine, a partial agonist, and hydrocodone, a moderate to strong agonist, may cause respiratory depression but not as often and serious as morphine. Nalmefene is an opioid antagonist and would be used to reverse respiratory depression with opioids.

A patient who has type 2 diabetes has a glycated hemoglobin (HbA1c) result of 10%. A nurse should make which of these changes to the nursing care plan? A) Refer to a diabetic educator, there is poor glycemic control. B) Glycemic control is adequate, no changes are needed. C) Hypoglycemia is a risk, teach the patient the symptoms. D) Instruct the patient to limit activity and weekly exercise.

A) Refer to a diabetic educator, there is poor glycemic control.

The nurse is conducting discharge teaching related to a new prescription for phenytoin (Dilantin). Which statements are appropriate to include in the teaching for this patient and his family? Select all that apply. A) "Be sure to call the clinic if you or your family notice increased anxiety or agitation." B) "You may have some mild sedation. Do not drive until you know how this drug will affect you." C) "This drug may cause easy bruising. If you notice this, call the clinic immediately." D) "It is very important to have good oral hygiene and visit your dentist regularly." E) "You may continue to have wine with your evening meals but only in moderation."

A, B, D Patients receiving an antiepileptic drug are at increased risk for suicidal thoughts and behavior beginning early in their treatment. The U.S. Food and Drug Administration (FDA) advises that patients, families, and caregivers be informed of the signs that may precede suicidal behavior and be encouraged to report these immediately. Mild sedation can occur in patients taking phenytoin even at therapeutic levels. Carbamazepine(Tegretol), not phenytoin, increases the risk for hematologic effects, such as easy bruising. Phenytoin causes gingival hyperplasia in about 20% of patients who take it. Dental hygiene is important. Patients receiving phenytoin should avoid alcohol and other central nervous system depressants because they have an additive depressant effect.

21. The client scheduled for electroconvulsive therapy tells the nurse, "I'm so afraid. What will happen to me during the treatment?" Which of the following statements is most therapeutic for the nurse to make? A. "You will be given medicine to relax you during the treatment." B. "The treatment will produce a controlled grand mal seizure." C. The treatment might produce nausea and headache. d. You can expect to be sleepy and confused for a time after the treatment.

A. The patient will receive medication that relaxes skeletal muscles and produces mild sedation.

-"pril"

ACE inhibitor HTN CHF

"pril" is the ending for drugs that are

ACE inhibitors

Remeron

Indication: major depression Class: anti-depressant Labs: CBC, hepatic Side Effects: drowsiness, constipation, weight gain Precautions: assess mental status, monitor weight, monitor for seizures

Nitro

Vasodilator Angina (1 tab 3x, 5 min apart) Keep med dry, cool, only effective for 6 mo. (Tingling under tongue = good) *ORTHOSTATIC HYPOTENSION*

c. A 47-year-old client with anuria

Which client would the nurse need to assess first if the client is receiving mannitol (Osmitrol)? a. A 67-year-old client with type 1 diabetes mellitus b. A 21-year-old client with a head injury c. A 47-year-old client with anuria d. A 55-year-old client receiving cisplatin to treat ovarian cancer

The Rn notes that the Cl taking doxycycline vibramycin are jaundiced and lethargic. What lab tests results would be most specific for the RN to review

ALT SGPt Alanie aminotransferase -Specific to Dx liver disease or impairment

The teaching plan for a diabetic is focused on smoking cessation and control of hypertension for the avoidance of microvascular complications, such as (select all that apply): 1. macular degeneration. 2. end-stage renal disease (ESRD). 3. coronary artery disease (CAD). 4. peripheral vascular disease (PVD). 5. cerebrovascular accident (CVA).

ANS: 1, 2 Macular degeneration and ESRD are both microvascular complications. CAD, PVD, and CVA are all macrovascular complications.

When the type 2 diabetic patient says, "Why in the world are they looking at my hemoglobin? I thought my problem was with my blood sugar." The nurse responds that the level of hemoglobin A1c: 1. shows how a high glucose level can cause a significant drop in the hemoglobin level. 2. shows what the glucose level has done for the last 3 months. 3. indicates a true picture of the patient's nutritional state. 4. reflects the effect of high glucose levels on the ability to produce red blood cells.

ANS: 2 By analyzing the amount of glucose bound to the hemoglobin, the level of blood glucose can be evaluated for the last 3 months, because the glucose stays bound to the hemoglobin for the life of the red blood cell (RBC).

The home health nurse is assessing a type 1 diabetic patient who has been controlled for 6 months. The nurse is surprised and concerned about a blood glucose reading of 52. This episode of hypoglycemia is probably caused by the patient's having: 1. taken a new form of birth control pill this morning. 2. used large amounts of sugar substitute in her tea this morning. 3. had a 2-hour long exercise class at the spa this morning. 4. underdosed herself with insulin this morning.

ANS: 3 Excessive exercise used up the glucose that was made available by the insulin taken by the patient. The patient now has too much insulin for the available glucose and has become hypoglycemic.

The patient with type 2 diabetes shows a blood sugar reading of 72 at 6 AM. Based on the reading of 72, the nurse should: 1. notify the charge nurse of the reading. 2. give regular insulin per sliding scale. 3. give him cup of milk. 4. administer the oral hyperglycemic tablet.

ANS: 3 milk The patient is hypoglycemic and needs an immediate source of glucose, such as milk or orange juice. The oral hyperglycemic agent will not work quickly enough. Notifying the charge nurse can be done later. Giving insulin per sliding scale would lower the blood sugar level.

When a newly diagnosed type 2 diabetes mellitus patient asks the nurse why she has to take a pill instead of insulin, you reply that in type 2 diabetes, the body makes insulin but: 1. overweight and underactive people simply cannot use the insulin produced. 2. metabolism is slowed in some people so they have to take a pill to speed up their metabolism. 3. sometimes the autoimmune system works against the action of the insulin. 4. the cells become resistant to the action of insulin. Pills are given to increase the sensitivity.

ANS: 4 Type 2 diabetes mellitus is a disease in which the cells become resistant to the action of insulin and the blood glucose level rises. Oral hyperglycemic agents make the cells more sensitive.

-"cycline"

antibiotic

-"floxacin"

antibiotic

Atropine sulfate is what type of med

anticholinergic

-"pium"

anticholinergic bronchodilator

physostigmine

anticholinergics antidote

-"arin"

anticoagulant

Read labels of food and over-the-counter drugs.

Which of the following would be a priority component of the teaching plan for a patient prescribed * phenelzine (Nardil) for treatment of depression? a. Headache may occur. b. Hyperglycemia may occur. c. Read labels of food and over-the-counter drugs. d. Monitor blood pressure for hypotension.

Amiodarone (Cordorone)

antidysrhythmic prolongs repolarization relaxes smooth muscle

Which of the following must be frequently monitored for the CL taking fluconazole

AST ALT BUN

Which of the following adverse reactions, if observed in a pt prescribed phenytoin, would indicate

ATAXIA- lack of voluntary control of movement

The groups of urinary antiseptics include which of the following? SELECT ALL THAT APPLY

AZTREONAM METHANAMINE QUINALONES NITROFURANTION INCORRECT Phenothiazide

Adenosine (Adenocard)

antidysrhythmic slows conduction time through AV node rapid IVP 1-2 sec

-"azine"

antiemetic

-"zine"

antihistamine

What teaching points would the nurse include when educationg the pt who will begin taking risendronate-actonel

After taking the drug remain upright for at least 30 minutes

A Contraindication for prophalytic treatment of TB is

Alcoholism

Bone-Reabsorption Inhibitors: Biophosphonates Med Names

Alendronate (fosamax), Risedronate (actonel), Ibandronate (boniva)

The uses of Diphenhydramine (Benadryl)

antihistamine antitussive (cough suppressant) antidyskinetic sedative-hypnotic

-"statin"

antilipemic lower chol. lower triglycerides take in the evening

The nursing care plan for a client receiving epoetin alfa (Epogen, Procrit) should include careful monitoring for symptoms of:

Angina, or a change in level of consciousness.

Irbesartan (Avapro) given for Hypertension is what type of med?

Angiotension II type 1 receptor antagonist

Which possible S/E trimethoprim/sulfamethoxazole should the RN advise the CL

Anorexia Crystalluria Photosensitivity decreased WBC/Platelets

36. The nurse is teaching the client with insulin-dependent diabetes the signs of hypoglycemia. Which of the following signs is associ- ated with hypoglycemia? A. Tremulousness B. Slow pulse C. Nausea D. Flushed skin

Answer A is correct. Tremulousness (a state of trembling or quivering) is an early sign of hypoglycemia. Answers B,C, and D are incorrect because they are symptoms of hyperglycemia.

A patient arrives at the emergency department complaining of back pain. He reports taking at least 3 acetaminophen tablets every three hours for the past week without relief. Which of the following symptoms suggests acetaminophen toxicity? A. Tinnitus. B. Diarrhea. C. Hypertension. D. Hepatic damage.

Answer: D Acetaminophen in even modestly large doses can cause serious liver damage that may result in death. Immediate evaluation of liver function is indicated with consideration of N-acetylcysteine administration as an antidote. Tinnitus is associated with ASPIRIN overdose, not acetaminophen. Diarrhea and hypertension are not associated with acetaminophen.

paroxetine, Paxil

Antidepressant. Weight gain. pack it on ____

A nurse assesses a patient who is taking pramlintide (Symlin) with mealtime insulin. Which of these findings should require immediate follow-up by the nurse? A) Skin rash B) Sweating C) Itching D) Pedal edema

B) Sweating Pramlintide is a new type of antidiabetic medication that is used as a supplement to mealtime insulin in type 1 and 2 diabetes. Hypoglycemia, which is manifested by sweating, tremors, and tachycardia, is the adverse reaction of most concern. Skin rash, itching, and edema are not adverse effects of pramlintide.

Which of the following statements about asprin is TRUE?

Asprin should generally be avoided for 1-2 weeks before and after surgery

Mr. Jessie Ray, a newly admitted patient, has a seizure disorder which is being treated with medication. Which of the following drugs would the nurse question if ordered for him? A. Phenobarbitol, 150 mg hs B. Amitriptylene (Elavil), 10 mg QID. C. Valproic acid (Depakote), 150 mg BID D. Phenytoin (Dilantin), 100 mg TID

B. Elavil is an antidepressant that lowers the seizure threshold, so would not be appropriate for this patient. The other medications are anti-seizure drugs.

what is recommended shcedule for taking chloroquine in prep for a visit to a country infested w malaria

antimalrial drug- before,during,after visit

Antipyretic Action

antiprostaglandin activity in hypothalamus

Antidepressants: MAOIs Nursing Considerations

Avoid foods containing tyramine Monitor Output Takes 4 wks to work Do not take with cold medications or CNS stimulates

which of the foollowing statements would the RN be cerain to include in a teaching plan for the pt taking NSAID

Avoid the use of aspirin or other salicylates

A patient with newly diagnosed type 2 diabetes mellitus asks the nurse what "type 2" means in relation to diabetes. The nurse explains to the patient that type 2 diabetes differs from type 1 diabetes primarily in that with type 2 diabetes a. the patient is totally dependent on an outside source of insulin. b. there is decreased insulin secretion and cellular resistance to insulin that is produced. c. the immune system destroys the pancreatic insulin-producing cells. d. the insulin precursor that is secreted by the pancreas is not activated by the liver.

B Rationale: In type 2 diabetes, the pancreas produces insulin, but the insulin is insufficient for the body's needs or the cells do not respond to the insulin appropriately. The other information describes the physiology of type 1 diabetes.

A nurse caring for a patient who has diabetic ketoacidosis recognizes which of these characteristics in the patient? (Select all that apply.) A) Occurs mainly in type 2 diabetes patients B) Altered fat metabolism leading to ketones C) Arterial blood pH of 7.35 to 7.45 D) Sudden onset, triggered by acute illness E) Plasma osmolality of 300 to 320 milliosmoles/L

B D E

A 20-year-old client presents to the clinic with complaints of breast tenderness, nausea, vomiting, and absence of menses for 2 months. She has a history of a seizure disorder well controlled with carbamazepine (Tegretol). She tells the nurse that she has been taking her oral contraceptives as directed, but she wonders if she might be pregnant. The nurse's best response to her concern should be which of the following? A. "You can't be pregnant if you have been taking your oral contraceptives correctly." B. "Carbamazepine can decrease the effectiveness of oral contraceptive drugs, so we need to do a pregnancy test." C. "There is no need to worry. Oral contraceptives are very effective." D. "Taking antiseizure drugs with oral contraceptives significantly decreases your risk of getting pregnant."

B. "Carbamazepine can decrease the effectiveness of oral contraceptive drugs, so we need to do a pregnancy test.

The Nurse should encourage a cl with constipation to ingest which of the following-SELECT ALL APPLY

BRAN GRAINS FRUIT WATER

Which of the following is a centrally acting muscle relaxant SELECT ALL THAT APPLY

Baclofen Orphenadrine cyclobenzaprine

Tylenol

Bad for Liver - DO NOT GIVE TO ALCOHOLICS

You are knowledgable about the drug and or food interactions with phenelzine (nardil) Which of the following types of drugs /foods are known to interact with Nardil SELECT ALL THAT APPLY

Beer Cheese Cold Medications Incorrect answers are pork, citrus

c. Assess blood pressure.

Before the nurse administers isosorbide mononitrate (Imdur), what is a priority nursing assessment? a. Assess serum electrolytes. b. Measure blood urea nitrogen and creatinine. c. Assess blood pressure. d. Monitor level of consciousness.

JT has anxiety over recent terrorist attacks The health care provider prescribed diazepam(valium) 5mg BID Valium is classified as an

Benzodiazepine

Anticholinergic agents are used to decrease these EPS Examples include

Benzotropine (cogentin) and tirhexyphenidyl (artane)

-"terol"

Beta 2 agonist bronchodilator sympathomimetic

-"alol"

Beta blocker angina HTN Adverse Effects (4 Bs) B- bradycardia B- BP decreased B- BG masked B- Bronchoconstriction

Lithium

Bipolar Disorder Decrease Na+ (risk of toxicity) Increase Na+ levels (causes sodium diuresis) Toxic Level > 2 mEq/L

Antithyroid Agents Nursing Considerations

Bitter Taste May cause burning in mouth Give with meals Check CBC

The nurse should closely monitor the client receiving Dextran 70 (Macrodex) for:

Bleeding

Thrombolytics Side Effects

Bleeding, bradycardia, dysrhythmias

TCAs

Block or inhibit the reuptake of NE and serotonin (or 5-HT) Are less dangerous than MAOI's. Sedation is a reported complaint at beginning of therapy.

All the following statements True about pharmocokinetics of penicillin derivatives amoxacillin and cloxacillin EXCEPT

Both drugs absorbed well from GI

Colchicine given for gout is used w/ caution in pts w/

old age debilitated pts pts w/ cardiac disease renal disease GI disease

Contraindication for taking fluphenazine include which of following: Select all tha APPLY

Blood Dyscrasias (increased/Decreased RBC) Hepatic Dysfunction Incorrect answers Neuromuscular Pain Subcortical Brain Damage Narrow Angle Glaucoma

The nurse explains to the client that the development of pernicious anemia is caused by:

Blood Loss

Assessment is essential w/ Cl taking MAOIs frequent monitoring of which of the following is required

Blood Pressure

As a result of EK long term use of phenytoin, which laboratory test results would be monitored?

Blood Sugar-Hypoglycemia

While pt is taking meperidine frequent monitoring of _____ is required

Blood pressure

Antibiotics: Cephalosporins Side Effects

Bone Marrow Depression, Superinfections, Rash

What is the difference between borderline personality disorder and antisocial behavior?

Borderline PD patients hurt/abuse themselves.

Levothyroxine (Synthroid) needs to be taken

on an empty stomach

A patient is recieving a cyrstalloid infusion (lactated ringer;s) for treatement of hypovolemic shock . Because of concerns for fluid volume overload, the nurse will frequently monitor (select all that apply)

Breath sounds, daily weight, level of consciousness.

Aminophylline

Bronchodilator - Decreases SOB Therapeutic levels 10-20 mcg/dl TOXIC > 20 *CANT MIX WITH ANYTHING*

During a clinic visit 3 months following a diagnosis of type 2 diabetes, the patient reports following a reduced-calorie diet. The patient has not lost any weight and did not bring the glucose-monitoring record. The nurse will plan to obtain a(n) a. fasting blood glucose level. b. urine dipstick for glucose. c. glycosylated hemoglobin level. d. oral glucose tolerance test.

C Rationale: The glycosylated hemoglobin (Hb A1C) test shows the overall control of glucose over 90 to 120 days. A fasting blood level indicates only the glucose level at one time. Urine glucose testing is not an accurate reflection of blood glucose level and does not reflect the glucose over a prolonged time. Oral glucose tolerance testing is done to diagnose diabetes, but is not used for monitoring glucose control once diabetes has been diagnosed.

The healthcare provider prescribes naproxen (Naproxen) twice daily for a client with osteoarthritis of the hands. The client tells the nurse that the drug does not seem to be effective after three weeks. Which is the best response for the nurse to provide? A) The frequency of the dosing is necessary to increase the effectiveness. B) Therapeutic blood levels of this drug are reached in 4 to 6 weeks. C) Another type of nonsteroidal antiinflammatory drug may be indicated. D) Systemic corticosteroids are the next drugs of choice for pain relief

C) Another type of nonsteroidal antiinflammatory drug may be indicated.

drugs with 1/2 life 24-30hrs would probably be adminsiterd on a dose schedule of

once daily

naloxone

opiod (morphine, codeine) antidote

11. A nurse is preparing the client's morning NPH insulin dose and notices a clumpy precipitate inside the insulin vial. The nurse should: A. draw up and administer the dose B. shake the vial in an attempt to disperse the clumps C. draw the dose from a new vial D. warm the bottle under running water to dissolve the clump

C. The nurse should always inspect the vial of insulin before use for solution changes that may signify loss of potency. NPH insulin is normally uniformly cloudy. Clumping, frosting, and precipitates are signs of insulin damage. In this situation, because potency is questionable, it is safer to discard the vial and draw up the dose from a new vial.

1. Which of the following statements made by a client taking phenytoin indicates understanding of the nurse's teaching? A. "I will increase the dose if my seizures don't stop." B. "I don't need to contact my health care provider before taking an over-the-counter cold remedy." C. "I will take good care of my teeth and see my dentist regularly." D. "I cannot take this drug with food."

C. "I will take good care of my teeth and see my dentist regularly."

Anticholinergics Uses

opthalmic exam, motion sickness, pre-operative

Which information should be given to the client taking phenytoin (Dilantin)? A. Taking the medication with meals will increase its effectiveness. B. The medication can cause sleep disturbances C. More frequent dental appointments will be needed for special gum care. D. The medication decreases the effects of oral contra- ceptives.

C. More frequent dental appointments will be needed for special gum care. Gingival hyperplasia is a side effect of phenytoin. The client will need more frequent dental visits. Answers A, B, and D do not apply to the medication; therefore, they are incorrect.

The nurse is creating a pain management plan for a client with a previous history of substance abuse. Which of the following should be included in this plan? a.) Ask the physician to prescribe short-acting analgesics. b.) Ask the physician to prescribe a medication similar to the one the client abused. c.) Ask the physician to prescribe all analgesics for the oral route. d.) Keep a dose of Narcan at the bedside.

C.) Ask the physician to prescribe all analgesics for the oral route. Extended-release and long-acting analgesics are recommended for clients with a history of abuse. Specific interventions should avoid analgesics similar to the abused drug, utilize long-acting analgesics, avoid Narcan, and administer medications through the oral route.

A pt given Ticlopidine (Ticlid) needs to be monitored for neutropenia development thus a nurse should monitor

CBC count

Prior to the administration of sargramostim (Leukine), the nurse should assess:

CBC.

Diuretics Uses

CHF, renal disease

The Rn planning to instruct Hispanic American Female about drug regimen presribed for newly Dx HTN when developing the plan the RN actions should be based on which of the following most relevant information

CL lives in an extended hispanic family setting in which men are the decision makers

Pt has recieved local anesthesia in throat for bronch, Rn should intervene when

CNA offers pt glass of water

When administering opioid analgesics to an elderly pt the RN must monitor the pt closely for

CONFUSION

M.U.C.H.

Ca Channel blockers M-meal, give with meal U- Under 100 systolic BP, hold C- Ca++, needs to be monitored H- HTN, used for HTN

A Rn is having difficulty reading a physicians order, the RN knows the Physician is very busy, does not like to be called what should RN do?

Call MD clarify order

Antineoplastic: Hormonal Agents Uses

Cancer

Which of the following statements is true about anticonvulsant drug therapy is

Carbamazepine

-"line"

bronchodilators COPD bronchospasms

main SE of etoposide (Toposar) given to pt w/ small cell lung cancer

orthostatic hypotension

Antibiotics: Cephalosporins Med Names

Ceclor, Ancef, Keflex, Rocephin, Mefoxin

Antianginals Nursing Considerations

Check expiration date Teach when to take medication May take q 5 min x 3 doses Wet with saliva and place under tongue

PH is having difficulty selecting foods from his menu. Tyramine rich foods to be avoided include

Cheese - Chocolate- Raisins

After spending the day treating his crops with insectisides, a 65 year old farmer presents to the emergency room with blurred vision, severe abdominal cramping, wheezing, and bradycardia. You determine that he is experiencing a______________ and should be treated with __________.

Cholinergic Crisis, Atropine

Bone-Reabsorption Inhibitors: Biophosphonates Uses

osteoprosis, paget's disease

What is Dysthmia?

Chronic depression that is less severe than major depression and can occur at all ages.

d. To suppress platelet aggregation

Cilostazol (Pletal) is being prescribed for a client with coronary artery disease. The nurse knows that which is the major purpose for antiplatelet drug therapy? a. To dissolve the blood clot b. To decrease tissue necrosis c. To inhibit hepatic synthesis of vitamin K d. To suppress platelet aggregation

Contraindications for the use of laxatives/cathartics include all of the following EXCEPT

Cirrhosis

what opioid analgesic is given to pts SPECIFICALLY after craniotomy & why?

Codeine sulfate bc it does not alter respirations or mask neurological signs as do other opioids

Mag Sulfate - Anticonvulsant

Common given in OB Deep Tendon Reflexes be evaluated SE: Resp Arrest

Asprin,NSAID's and acetaminophen act by:

Competitively blocking the neuronal terminals

Which of the following findings requires that the RN immediatly contacts the primary health care provider

Complaints of Breakthrough Pain

Antidiarheals are contraindicated in Cl with all of the following EXCEPT

Congestive Heart Failure

The nurse assesses for toxic affects of the drug- which of the following is an adverse reaction

Constricted Pupils

Lorazepam is an anxiolytic drug - however it may be prescribed for other clinical problems. Which of the following may it be prescribed-Select All that APPLY

Correct Anxiety Status Epilepticus PreOperative Sadation Incorrect answers -Manage Scitzophrenia -Depression and Delusions

Antibiotics: Aminoglycosides Side Effects

ototoxicity, nephrotoxicty, nausea, vomiting, diarrhea

The nurse teaches the client and caregivers to immediately report to the health care provider which of the following symptoms of adverse reaction to oprelvekin (Neumega)?

Cough; difficulty breathing

What are the symptoms of a panic attack?

Crying, flushed face, trembling hands, diaphoretic, trouble breathing, heart is fluttering, feels as if going crazy, increased HR, BP, and respirations.

Methanamine -mandelamine should not be used with sulfonamides becouse of the risk for:

Crystalluria

A college student who has type 1 diabetes normally walks each evening as part of an exercise regimen. The student now plans to take a swimming class every day at 1:00 PM. The clinic nurse teaches the patient to a. delay eating the noon meal until after the swimming class. b. increase the morning dose of neutral protamine Hagedorn (NPH) insulin on days of the swimming class. c. time the morning insulin injection so that the peak occurs while swimming. d. check glucose level before, during, and after swimming.

D Rationale: The change in exercise will affect blood glucose, and the patient will need to monitor glucose carefully to determine the need for changes in diet and insulin administration. Because exercise tends to decrease blood glucose, patients are advised to eat before exercising. Increasing the morning NPH or timing the insulin to peak during exercise may lead to hypoglycemia, especially with the increased exercise.

A type I diabetic patient comes to the clinic for a follow-up appointment. The patient is taking NPH insulin, 30 units every day. A nurse notes that the patient is also taking metoprolol (Lopressor). What education should the nurse provide to the patient? A) "You need to increase your insulin to allow for the agonist effects of metoprolol." B) "Metoprolol may potentiate the effects of the insulin, so the dose should be reduced." C) "Metoprolol has no effects on diabetes mellitus or on your insulin requirements." D) "Metoprolol may mask signs of hypoglycemia, so you need to monitor your blood glucose closely."

D) "Metoprolol may mask signs of hypoglycemia, so you need to monitor your blood glucose closely."

The nurse is assessing a patient receiving valproic acid (Depakene) for potential adverse effects associated with this drug. Which item represents the most common problem with this drug? A) Increased risk for infection B) Reddened, swollen gums C) Nausea, vomiting, and indigestion D) Central nervous system depression

D) Central nervous system depression Valproic acid is generally well tolerated. It does not cause hematologic effects resulting in increased risk for infection nor does it cause gingival hyperplasia. It causes minimal sedation. Gastrointestinal effects, which include nausea, vomiting, and indigestion, are the most common problems but tend to subside with use and can be lessened by giving with food.

A patient takes oxycodone (OxyContin), 40 mg PO twice daily, for the management of chronic pain. Which intervention should be added to the plan of care to minimize the gastrointestinal adverse effects? A) Take an antacid with each dose. B) Eat foods high in lactobacilli. C) Take the medication on an empty stomach. D) Increase fluid and fiber in the diet.

D) Increase fluid and fiber in the diet. Narcotic analgesics reduce intestinal motility, leading to constipation. Increasing fluid and fiber in the diet can help manage this adverse effect.

A patient who is newly diagnosed with type 1 diabetes asks a nurse, "How does insulin normally work in my body?" The nurse explains that normal insulin has which of these actions in the body? A) Stimulates the pancreas to reabsorb glucose B) Promotes synthesis of amino acids into glucose C) Stimulates the liver to convert glycogen to glucose D) Promotes the passage of glucose into cells for energy

D) Promotes the passage of glucose into cells for energy Insulin is a hormone that promotes the passage of glucose into cells, where it is metabolized for energy. Insulin does not stimulate the pancreas to reabsorb glucose or synthesize amino acids into glucose. It does not stimulate the liver to convert glycogen into glucose.

Antihypertensives: Alpha 1 Adrenergic Blockers Side Effects

reflex tachycardia, orthostac hypotension, nasal congestion

Which of the following regimens offers the best blood glucose control for persons with type 1 diabetes? a. A single anti-diabetes drugs b. Once daily insulin injections c. A combination of oral anti-diabetic medications d. Three or four injections per day of different types of insulin.

D. Three or four injections per day of different types of insulin. Because persons with type 1 diabetes do not produce insulin, they require insulin and cannot be treated with oral anti-diabetic drugs. Several injections of insulin per day, calibrated to respond to measured blood glucose levels, offer the best blood glucose control and may prevent or postpone the retinal, renal, and neurological complications of diabetes.

Attention Disorder Agents Side Effects

restlessness, insomnia, tachycardia, palitations

Which of the following drugs tends to be more toxic in elderly CL than of middle aged?

DEMEROL

What are the signs and symptoms of a cocaine/crack user?

DILATED PUPILS, TACHYCARDIA, ELEVATED BP (HEART BLOWS UP!), nose bleeds, nasal congestion, sniffing, impaired movement, seizures, insomnia, and anorexia. (OVERDOSE= TACHYCARDIA AND INCREASED BP).

Mydriatics

DILATES (Atropine) Dont use in pts with gaucoma (build up IOP)

Exenatide (Byetta) is an incretin mimetic used only for

DM type 2

before storing unused stable solutions from opern vials in refrigerator, the Rn should write the following info on bottle

DTI

Specific Nursing interventions w RT would include monitoring ALL of the following EXCEPT

Daily ECG

Digoxin toxicity

Dc Dig and K sparring diuretics Treat dysrhythmias with phenytoin Bradycardia with atropine

An expected action to opiates is to

Decrease Intestinal Motility

Lidocaine

Decrease cardiac irritability Titate IV Use with V. TACH

Warfarin (Coumadin) & Levothyroxine (Synthroid) what dose needs to be changed when in use with the other?

Decrease the Warfarin (Coumadin) dose bc Levothyroxine (Synthroid) enhances the effects of Warfarin ( Coumadin)

Elderly Cl frequently require a reduction in opiod dosage to avoid severe side effects-Reasons for this include which of the following

Decreased excretion of drug Decreased metabolism of drug Polypharmacy ALL OF ABOVE!

The Rn knows that food ( ) absorbtion of the drug by 50% therefore rn should instruct pt to

Decreases absorption, Take medication on an empty stomach

Site easily accessible but NOt suitable for repeated injections or injections of ml or more

Deltoid

What are the 5 stages of grieving?

Denial, Anger, Bargaining, Depression, and Acceptance.

The nurse weighs the client receiving dobutamine (Dobutrex) therapy daily. The rationale for this is to:

Determine drug dosage.

What are the S&S of neuroleptic malignant syndrome?

Diaphoresis, incoherent speech, disorientation, muscle rigidity, fever (101), hypertension, tachycardia, incontinence, tremor, increased salivation, and increased respirations. (present after receiving an antipsychotic medication).

What factors increase retained lithium?

Diarrhea, diuretics or dehydration, low salt diets, or high fevers/strenuous exercise.

Major teaching w/ Alendronate (Fosamax)

Do not eat or drink anything for 30 min after meds; do not lie down; take med w/ full glass of water after rising in the morning

Give Ritalin in the morning. do not give Ritalin after 4pm.

Do not give Ritalin after ?

Which following statements are correct?

Drugs are absorbed faster in acidic fluid The very young have decreased gastric acidity Liquid drugs are more rapid available in GI absorbtion than of solids

c. To administer phentolamine (Regitine)

During assessment of a client diagnosed with pheochromocytoma, the nurse auscultates a blood pressure of 210/110 mm Hg. What is the nurse's best action? a. To ask the client to lie down and rest b. To assess the client?s dietary intake of sodium and fluid c. To administer phentolamine (Regitine) d. To administer nitroprusside (Nipride)

which intervention is of highest priority for RN working w client who has Herpes Zoster shingles and who has recently began drug therapy with acyclovir

Encourage Fl intake of 2500-300 mL daily unless otherwise contrindicared

Antibiotics: Fluoroquinolones Uses

E. coli, Pseudomonas, S. aureus

Fluphenazine Prolixin commonly produces effects of

EPS

The nurse assesses RT for all of the following S/E of compazine EXCEPT

EXCESSIVE SALIVATION

which of the following adverse reaction would the nurse expect after the administration of atropine as part of a pt. preoperative medication regimen

Enhanced secretions of the Upper Respiratory tract

Primary purpose of federal legislation r/t drug standards

Ensure Safety

Bronchodilators

Epinephrine (adrenalin) *Careful with Cardiac Pts*

A Cl is recieving an AI drug and begins to wheeze the rn anticipates initial administration of what drug

Epinephrine -treatment for anaphalaxis

Lidocaine is frequently used for: what type anesthesia

Local anesthesia

Antianxiety Uses

anxiety disorders, manic episodes, panic attacks

What should the nurse do if after placing the mannitol w/ crystal formation does not dissolve in warm water?

return to pharm; do not use

Vasoconstrictors

Ex. epinephrine, ephedrine, isoprote N- nervousness A- anxious S- BG increased C- cardiac arrest A- allergic reaction R- resp broncodilator

What are 6 common side effects of antipsychotics?

Extrapyramidal sx, orthostatic hypotension, constipation, urinary retention, confusion, sexual dysfunction.

Cholesterol absorption inhibitor

Ezetimibe take 1hr before or 4hrs after another antilipemic

I Glargine (Lantus)

Long Acting Onset 1 hr give at bedtime don't mix with other insulin

The atypical antipyschotics marketed in the US since 1990 have a weak affinty for the D2 receptors; thus these agents cause

Fewer EPS

Piroxicam - feldene is a NSAID- Its advantage over othe agents is which of the following

Long Half Life

Which nursing assessment would be least important when monitoring a CL recieving meperidine

Fluid intake

factors commonly resulting in non adherance with a drug therapy plan includes:

Forgetfullness Knowledge defecit Side Effects Language culture barriers Finances

To best monitor for therapeutic effects from filgrastim (Neupogen) should include:

Frequent observations for infection

The Nurse recomends that a cl w dirahea avoid which of the following SELECT ALL THAT APPLY

Fried Foods Raw Vegtables Milk Products

Metoclopramide (Reglan) is an antiemetic as well as a GI stimulant; bc it is a GI stimulant , this med is not given to pts w/

GI obstruction hemorrhage perforation

When a pt is recieving an NSAID the RN must monitor the pt for

GI symptoms which can be serious and sometimes fatal

Metronidazole primarily used for treatment of disorders caused by organisms in which of the following

GI tract

NSAIDS Side Effects

GI upset, dizziness, headaches, bleeding, fluid retention

Antihypertensives: Ace Inhibitors Side Effects

GI upset, orthostatic hypotension, dizziness

Which drugs are frequently prescribed to control seizures

Long acting barbituates

Antibiotics: Sulfonamides Med Names

Gantrisin, Bactrim, Azulfidine

Aminoglycosides

Gentamycin, Kanamycin sulfate, Neomycin sulate, Steptomycin Sulfate OTOTOXICITY KIDNEY DAMAGE

Antibiotics: Aminoglycosides Med Names

Gentamycin, Neomycin, Streptomycin, Tobramycin

EK has been taking PHENYTOIN for 20 years. He has not reported seizure activity while taking the maintence dose. You would assess the Cl. for which common side effect of the drug.

Ginigival Hyperplasia

Antineoplastic: Vinca Alkaloids Nursing Considerations

Give antiemetic before administration Check reflexes Given with Zyloprim to decrease uric acid

Atropine like drugs are contraindicated in clients with

Glaucoma

The RN monitors for which common adverse reactions when administering naproxen-naprosyn to a pt.

HA ,DYSPEPCIA

What is important to remember with patients on lithium?

HIKE= Hydration. If have diarrhea could be from elevated lithium level from dehydration as a result of hiking.

Ergotamine tartrate (Cafergot) is used

to stop an ongoing migraine attack OR treat cluster headaches

Your assessment of a cl for S/E of a fluoroquinalone would include which of the following SELECT ALL THAT APPLY

Headache and Rash Syncope and Visual Disturbances Peripheral Neuritis

Chloroquinalone affects all of the following lab tests Except!

Hemoglobin

Anticoagulant Coumadin Side Effects

Hemorrhage, alopecia

adverse effect of Cyclophosphamide for breast cancer

Hemorrhagic cystitis

Anticoagulants

Heparin (Antecdote - Protamine Sulfate) Coumadin (Antecdote - Vit K.) Bleeding precautions PPT - Heparin PT/INR - Coumadin

Itraconazole (Sporanox) given for fungal infection of the hands needs to be monitored for what adverse effect?

Hepatitis

Serious Adv.Effect of isoniazid is

Hepatotoxicity

Probenecid given for gout to inhibit reabsorption of uric acid by the kidney and promote excretion of uric acid needs to avoid what type of food intake?

High purine foods like - organ meats -sardines -scallops -anchovies -broth -mincemeat -herring -shrimp -gravy -yeast

What factors decrease the amount of lithium through enhanced excretion?

High salt intake, high intake of sodium bicarbonate, and pregnancy.

-"tidine"

Histamine H2 receptor agonist suppress secretion of gastric acid GERD antiulcer

What is the best way to respond to a patient who is using neologisms, clang association, or word salad?

Honesty is the best response. If you don't understand then tell them you don't understand but you'd like to help.

The nurse giving Humulin R 20 U at 7 AM is aware that this drug will peak in: 1. 15 minutes. 2. 30 minutes. 3. 1 hour. 4. 2 hours.

Humulin R has its onset in about 15 minutes, but its peak is in 2 hours.

What characteristics are present with anorexia nervosa?

INTENSE FEAR OF BECOMING FAT, WEIGHT DECREASE BY AT LEAST 25% FROM ORIGINAL BODY WT, body image disturbance (draw fat person even though skinny), no known physical illness, absence of periods, fine hair (lanugo), excessive exercising, cook a lot but don't eat any, low self-esteem, and may think they don't have a problem.

Antihypertensives: Angiotensin II Receptor Blockers Uses

Hypertension, heart failure, MI, diabetic neuropathy, stroke prevention

What are the S&S of delerium tremens?

Hyperthermia (105), disorientation, delusions, paranoia, and frightening visual and tactile hallucinations. (tachycardia, tachypnea, hypertension, dilated pupils, and diaphoresis).

Antidiabetic Agents: Side Effects

Hypoglycemia, allergic skin reactions, GI upset

after receiving penicillin for days-Cl reports muscle weakness,numbness,tingling, weak pulses,irregular heart rate decreased bowl sounds- rn should review labs for

Hypokalemia

Overdose (Toxicity) of oxybutnin chloride

Hypotension or Hypertension Confusion Tachycardia Flushed or red face Signs of Resp Depression CNS excitation like -nervousness -hallucinations -irritability

Antihypertensives: Angiotensin II Receptor Blockers Side Effects

Hypotension, dizziness, GI distress

Salicylate intoxication(salicylism) is characterized by which of the following symptoms

Hypoventilation

Health teaching for RT includes all of the following EXCEPT

If the medication is stopped the depressive symptoms will reappear

Which is drug of choice for treatment of enuresis in children

Imipramin-Trofranil

Cl teaching for antihelmintic

Importance of hygeine

The nurse evaluates the effectiveness of dapamine therapy for a patient in shock. Which of the following may indicate the treatment is successful (select all that apply)

Improved urine output and Increased blood pressure

Antibiotics: Cephalosporins Uses

tonsillitis, otitis media, peri-operative prophylaxis, meningitis

Oxymetholone

Indication: anemia Class: androgen (increase production of erythropoietin) Labs: hepatic Side Effects: acne, allergic reaction, depression, mood changes, anxiety Precautions: do not take if pregnant

Metoprolol (Lopressor)

Indication: angina, hypertension, prevention and decreased mortality of MI Class: beta blocker Labs: glucose, hepatic, K, cholesterol, uric acid Side Effects: fatigue, weakness, bradycardia, hypotension, pulmonary edema Precautions: hold for pulse less than 60 and BP less than 100, monitor I/O, assess angina, do not stop all at once

Prazepam

Indication: anxiety, conscious sedation, seizures, alcohol withdrawl Class: benzodiazepines Labs: hepatic, renal, CMB Side effects: dizziness, lethargy, drowsiness Precautions: monitor vitals, physical dependence for long term use, assess mental status, assess pt with alcohol withdrawl fo symptoms Reversal: Flumazenil

Diazepam (Valium)

Indication: anxiety, conscious sedation, seizures, alcohol withdrawl Class: benzodiazepines Labs: hepatic, renal, CMB Side effects: dizziness, lethargy, drowsiness Precautions: monitor vitals, physical dependence for long term use, assess mental status, assess pt with alcohol withdrawl for symptoms Reversal: Flumazenil

Alprazolam (Xanax)

Indication: anxiety, panic disorders, anxiety caused by depression Class: benzodiazapien Labs: hematocrit may decrease, renal function, liver function Side effects: dizziness, drowsiness, depression, dependance Precuations: monitor risk to fall, monitor drowsiness, monitor anxiety Reversal: Flumanizil

Ativan (Lorazepam)

Indication: anxiety, preoperative sedation, seizures Class: benzodiazepine Labs: renal, hepatic, hematological Side Effects: drowsiness, apnea, cardiac arrest, dependence, tolerance Precautions: assess mental status, assess seizures, monitor for dependence/tolerance

Amiodarone

Indication: arrhythmias, tachycardia, ventricular fibrillation, cardiopulomanry resuscitation Rate: Direct -- push, 300mg first does then 150mg next dose Intermittent -- over 10 min Continuous --- 1 mg/min for first 6 hr then decrease to 0.5mg/min Concentration: direct ---- 50 mg/ml Intermittent ---- 1.5mg/ml Continuous ---- 1.8 mg/ml

Betamethasone (Celestone)

Indication: asthma, neoplastic disease, renal disease, respiratory disease, pre-term labor at 36wk Class: gluccocorticoid Labs: WBC, glucose (increase), Na (increase), potassium(decrease) Side effects: depression, euphoria, decreased healing, hypertension Precuations: rinse mouth if inhaled, monitor condition used for

Amitriptyline (Elvail)

Indication: depression Class: antidepressant Labs: liver, WBC, glucose Side effects: suicide, arrythmias, hypotension, blurred vision, lethargy, dry mouth Precuations: monitor BP and pulse before and after dose, monitor for suicide

Fluoxetine (Prozac)

Indication: depression, panic disorder, OCD Class: antidepressant Labs: CBC, hepatic, renal Side effects: seizures, suicidal thoughts, headache, insomnia, diarrhea, sweating Precautions: suicide, mood changes, changes in mental status (serotonin syndrome)

Bupropion (Wellbutrin)

Indication: depression, quit smoking aid Class: antidepressant Labs: liver, renal Side effects: seizure, suicide, agitation, headache, tremor, N/V Precuations: mental status, mood changes

Furosemide (Lasix)

Indication: edema caused by HF, liver disease, or renal failure, hypertension Class: loop diuretic Labs: Na, K, glucose, renal, Mg Side effects: tinnitus, hypokalemia, low mag, hypovolemia, hyponatremia, dehydration, hyperglycemia Precautions: monitor fluid status, weight, I/O, lung sounds, BP, pulse

Bumetanide (Bumes)

Indication: fluid retention, high blood pressure, heart failure Class: loop diuretic Labs: Na, K, Mg, renal, liver Side effects: dehydration, low K, low Na, metabolic alkalosis, muscle cramps Precuations: fluid status, lung sounds, location of edema, monitor BP and pulse, monitor for tinnitus

Tolnaftate

Indication: fungal skin infection Class: antifungal topical Labs: N/A Side Effects: rash Precautions: N/A

Doxorubicin (Adiamycin)

Indication: in combination treatment for tumors Class: cancer drug Labs: CBC, platelets, WBC, renal, hepatic, uric acid Side Effects: red color urine, diarrhea, alopecia, vomiting, anemia, leucopenia, thrombocytopenia Precautions: monitor vitals, monitor for bone marrow depression, monitor for bleeding, monitor I/O, monitor for signs of toxicity (ST depression, tachycardia, dyspnea), monitor injection site

Amoxicillin/clvulanate (Augmentin)

Indication: infection Class: anti-infective Labs: liver Side effects: seizure, diarrhea, rash, C-Diff Precuations: assess vital signs, appearance of any infection, monitor bowel function, monitor for signs of anaphlaxis

Erythromycin

Indication: infection, if penicillin can't be used Class: anti-infective Labs: hepatic Side effects: prolonged QRS, arrythmias, N/V, itching Precautions: monitor infection, culture and sensitivity

Temazepam

Indication: insomnia Class: benzodiazepien Labs: hepatic, renal, CBC Side effects: dizziness, lethargy, drowsiness, suicidal thoughts, behavioral changes Precautions: monitor vitals, physical dependence for long term use, assess mental status, assess pt with alcohol withdrawl fo symptoms Reversal: Flumazenil

Lisinopril

Indication: management of hypertension and CHF Class: ACE Lab to monitor: Monitor BUN, creatine, and electrolyte levels, (K) Monitor Na levels Monitor CBC periodically hematocrit or hemoglobin may decrease Monitor AST, ALT, serum bilirubin, uric acid, glucose Assess urine protein Assessments: Monitor BP and pulse frequently during therapy Assess for angioedema (dyspnea, facial swelling) Monitor weight Monitor for signs of hypotension Side effects: cough, hypotension, taste disturbance, agranuloctes, angioedema Excessive hypotension may occur with the use of other anti-hypertensives and diuretics Response decrease with the use of NSAID Risk of hyperkalemia with use of potassium supplements or angiogenesis II receptor antagonists Precautions/Special considerations: Initiate therapy at 5mg/day for those on diuretics Initial therapy at 2.5 mg/day for patients with hyponatremia For patients with difficulty swallowing pharamcists may compound oral suspension

Propoxyphene

Indication: pain Class: opiod Labs: hepatic, amylase, lipase Side Effects: dizziness, weakness, nausea, hypotension Precautions: monitor pain, vitals, bowel function Reversal: Narcan

Dilantin

Indication: prevention of seizures Class: anti-arrhythmic Labs: CBC, Ca, albumin, hepatic Side Effects: ataxia, suicide, anemia, hypotension, rash Precautions: monitor for mood changes, assess for hypersensitivity, oral hygiene, monitor seizures, monitor vitals Always monitor levels

Atrovastatin (Lipitor)

Indication: primary prevention to decrease risk MI/Stroke, lowers cholesterol Class: lipid lowering agent Lab to monitor: serum cholesterol, triglycerides, liver function tests, if develop muscle tenderness monitor CK levels if > 10 D/C therapy Specific patient teaching: Instruct to take as directed Ingest no more than 200 ml/day of grapefruit juice Advise of diet restriction Notify health care professional if unexplained muscle pain, tenderness, or weakness occurs accompanied by fevers Emphasize importance of follow up care Side effects: dizziness, headache, rhinitis, chest pain, abdominal cramps, constipation, heartburn, rhabdomyolisis, rashes, RUQ pain - liver Drug-Drug: Risk of sympathy increase with use of amiodarone, niacin, nelfinavir Increase levels of hormonal contraceptives Increase bleeding risk with Warfarin Levels may be increased with use of Erythromycin Grape fruit may increase blood levels Precautions/Special considerations: Obtain dietary history in regards to fat particularly Administer with food May be administered at any time Avoid large amounts of grapefruit Monitor for rhabdo and upper right quadrant pain

Clopidogrel (Plavix)

Indication: reduction of atherosclerosis events Class: anti-platelet Lab to monitor: Bleeding time CBC Platelet count Bilirubin, hepatic enzymes, cholesterol, non protein nitrogen, uric acid (may increase) Side effects: depression, cough, GI bleeding, Neutropenia, fever, chest pain, hypertension, edema Drug-Drug: Aspirin, heparin, warfarin may increase bleeding risks NSAID may decrease absorption Omeprazole or fluvoxamine may decrease effects Precautions/Special considerations: Assess for symptoms of stroke, MI, vascular disease during therapy Monitor for signs of thrombocytopenia, anemia, renal dysfunction, fever D/C 5-7 days before surgery

Haloperidol (Haldol)

Indication: schizophrenia, chronic psychotic disorders, manic episodes Class: anti-psychotic Labs: CBC, liver, WBC Side effects: seizures, blurred vision, constipation, hypotension, respiratory depression, confusion Precautions: monitor mental status, monitor BP, make sure meds are taken, monitor for fever/respiratory distress/tachycardia/hypotension and report immediately

Gentamicin

Indication: staph infections Class: amino glycoside, anti-infective Labs: renal, hepatic, calcium, gent levels Side effects: ototoxicity, nephrotoxicity Precautions: infection monitor, test hearing, monitor I/O, monitor neurological function Levels: Peak 30 min after dose, troph right before next dose

Vancomycin

Indication: super bugs Class: anti-infective Labs: renal, CBC, levels in blood Side Effects: nephrotoxicity, ototoxicity, anaphylaxis, hypotension Precautions: monitor urinary output and color, monitor hearing, monitor vitals, monitor infection

Promethazine

Indication: vomiting and nausea Class: anti-metic Labs: glucose Side Effects: confusion, sedation, bradycardia Precautions: monitor vitals, monitor N/V

Ceftriazone (Rocephin)

Indications: treatment for skin infection, urinary infections, respiratory tract infections, meningitis, septicemia, intra abdominal infections, otitis media, lyme disease Class-Therapeutic: anti-infectives Lab to monitor: may increase serum AST, ALT, LDH, BUN, and creatine Monitor WBC Assessments: assess infection Monitor for any signs of anaphylaxis Monitor injection sight Monitor bowel function Side effects: seizures, C-Diff, Steven Johnson Syndrome (skin condition), anaphylaxis, serum sickness Can cause disulfiram reaction Use of loop diuretics or nephrotoxic agents may increase risk of nephrotoxicity Precautions/Special considerations: use cautiously if patient has renal impairment Do not use if have an allergy to cephlasporins or penicillins Should not be administered with Ca containing solutions Reconsiture IM dose with sterile water for injection or 0.9% NaCl Inject deep into a well develop muscle mass, massage well

Levofloxacin (Levaquin)

Indications: treatment of the following bacterial infections: urinary tract, gonorrhea, gynecological infections, respiratory tract, skin, bone/joint, GI, post exposure treatment Class-Therapeutic: anti-infectives Lab to monitor: liver function tests Monitor AST, ALT, bilirubin Monitor serum glucose Monitor prothrombin time Assessments: observe for S/S anaphylaxis Monitor bowel function Assess infection Side effects: seizures, arrhythmias, C-Diff, hepatotoxicity, anaphylaxis, Steven-Johnson Syndrome Decreased absorption if used with antacids Probenecid decrease renal elimination Risk for tendon rupture if used with steroids Prolong QT interval V-tach/V-fib Precautions/Special considerations: dosing changes in renal impaired patients Use cautiously with CNS disorders Can be administered with disregards to food Discard unused IV solution Diluted solution is stable for 72 hours room temp and 14 days refrigerated Must measure QT interval prior to giving

Albuterol

Indications: used as a bronchodialator to control and prevent reversible air way obstruction caused by asthma or COPDClass-Therapeutic: bronchodialators Lab to monitor: Serum Potassium levels Assessments: lung sounds, pulse, BP before administration and during peak If have sputum note color, consistency, and amount Observe for any wheezing Monitor pulmonary function tests before intiating and periodically through therapy Side effects: Paradoxacal bronchospams Chest pain, paplpatations, tremor Concurrent with other adrenergic agents Cardiovascular effects increase with pateints on tricyclic antidepressants May decrease serum digoxin can also cause hypokalemia which can lead to digoxin toxicity Caffeine may increase adverse effects Precautions/Special considerations: extended release tablets should be swallowed whole not crushed Shake inhaler well and allow 1 minute between each puff

Balanced Anesthesia comprised of

Inhaled Gas Premedicated 1Hr prior Narcotic Analgesic/Anticholinergic Hypnotic night prior Muscle relaxant

Usually food has what affect on drug dissolution/absorbtion

Interferes counteracts

Antacids Nursing Consideration

Interferes with absorption of antibiotics, iron preps, INH, oral contraceptives Monitor bowel functions

What is a compulsion? EX?

It is an action in response to an obsession. EX: Washing hands repetitively, counting things, rechecking things 20 times.

Warfarin

Lab to monitor: Monitor PT, INR, and clotting factors want PT 1.3-1.5 and INR 2.5-3.0 Monitor hepatic function and CBC Monitor stool and urine for occult blood Assessments: Assess for signs of bleeding and hemorrhage Monitor Blood Pressure Geri: patients over 60 exhibit greater than expected PT/INR response monitor for side effects at lower therapeutic range Specific patient teaching: Take medication as directed take missed dose as soon as remembered do not double doses Limit intake of food high in vitamin K Report any symptoms of unusual bleeding or bruising, tarry stools, pain, temperature change in area of the body Don't drink alcohol Carry card stating take this medication Emphasize the importance of frequent lab test to monitor coagulation factor

Amiodarone

Lab to monitor: liver and thyroid functions before and every 6 months during therapy Monitor AST, ALT, and alkane phosphatase Side effects: ARD, pulmonary fibrosis, CHF, bradycardia, dizziness, headache, neuropathy, tremor, ataxia, photosensitivity, hypotension, N/V, constipation Drug-Drug: Macrolides, antifungal increase risk of QT prolongation Increase levels of class I antiarrythmics Increase activity of Warfarin Increase risk of bradyarrhythmias, sinus arrest, or AV heart block with beta blockers of calcium channel blockers Precautions/Special considerations: Monitor ECG continuously Assess for signs of pulmonary toxicity (crackles, decreased breath sounds, friction rub, fatigue) Assess for s/s of ARDS (dyspnea, rales/crackles bilaterally) Monitor B/P Opthalmic exams should be performed before and during therapy May be administered with food and in divided doses if GI irritation occurs

Heparin

Lab to monitor: monitor aPTT and hematocrit periodically during therapy Monitor platelet count 2-3 days throughout therapy Monitor potassium, AST, ALT Assessments: Assess for signs of bleeding and hemorrhage Assess for evidence of additional thrombus Monitor for hypersensitivity reaction If sub-cut assess injection sight for hematomas, ecchymosed, or inflammation To prevent clot formation in intermittent infusion inject heparin dilute solution after each medication injection Specific patient teaching: Advise patient to report any signs of unusual bleeding or bruising Instruct patient not to take medications containing aspirin or NSAIDs while on heparin During therapy should use soft tooth brushes and electric razors avoiding things that can causes injury Patient should carry a medical card saying they take heparin

The benefits of methadone over other opioids is (are)

Less dependancy -Daily Dosing A and C Only

Two DIS-advantages assoc w/ use of generic drugs include

Less extense of testing Variation in response (tablets not 100 med/ may contain fillers)

The Nurse explains to the PT that some opiods may be used as a part of the pre-operative medication regimen to:

Lessen anxiety and sedate the Pt

You are knowledgable about drug interactions with phenytoin.Which of the following drugs are NOT known to alter the action of phenytoin.

Laxatives

What interventions would be appropriate for a manic patient?

Lithium (.6-1.2), NO GROUPS, high protein/calorie diet to help increase wt (mobile food is impt and NO Caffeine), Treadmill=good activity bc helps with energy (NO PUZZLES), place the patient in a private room or with another manic pt, and make sure to meet their physical needs (hygiene).

Zafirlukast (Accolate) is a leukotriene receptor antagonists used in prophylaxis and long term treatment of bronchial asthma; what lab test is done before admin?

Liver function tests

When an immediate drug response is desired a large initial dose is given to rapidly achieve an MEC in the plasma this is called:

Loading Dose

What is characteristic of schizophrenia?

Loss of boundaries, inability to trust, withdrawn or peculiar behavior, indifferent/aloof, love/hate feelings, personality changes, confused chaotic thoughts, retreat to fantasy world, auditory hallucinations and delusions, hypersensitivity to sound & smell, difficulty relating to others, negativism, religiosity, lack of social awareness, and disorganized.

When administering an opthalmic solution, the drug is instilled into the _____

Lower Conjuctival sac

Sulfonamide derivatives for treatment of second and third degree burns

Mafenide Acetate-sulfamylon

What should you give to a patient experiencing alcohol withdrawal to prevent seizures?

Magnesium sulfate IV

Communication on of the domains of culture include

Maintenence of eye contact

what do blood dyscrasias include?

aplastic anemia agranulocytosis thrombocytopenia thrombophlebitis dysrhythmias dermatological effects

Antitussive Agents Nursing Considerations

Monitor Cough Avoid alcohol

Antipyretic Nursing Considerations

Monitor LFT & ALT Aspirin contraindicated under 21 yo due to risk of Reye's syndrome

Antineoplastic: Antitumor Nursing Considerations

Monitor VS Give antiemetic before administration

Attention Disorder Agents Nursing Consideration

Monitor growth rate Monitor liver enzymes Give in am

Antineoplastic: Antimetabolites Nursing Consideration

Monitor hematopoietic function Good mouth care Discuss body image changes

Anticholinergics Nursing Considerations

Monitor output Contraindicated with glaucoma Give 30 min before meals, at bedtime or 2 hours after meals

Cl taking Azithromycin Rn should apply which interventions

Monitor periodic LFT's Tell CL to report Hearing loss Instruct CL to report evidence of superinfection Teach Cl to take oral drug 1hr AC 2hr PC Avoid antacids from 2 hrs prior to 2hrs after taking Azithromycin admin.

The nurse recognizes that opioid analgesics exert their action by interacting with a variety of opioid receptors. Drugs such as morphine act by activating

Mu and kappa

What is the effect of Nalidixic acid (Neg gram) and warfarin sodium (Coumadin)?

Nalidixic acid intensifies the effects of the oral anticoagulant.

The opiod antagonist used to treat an overdose of morphine like substance is

Naloxone(NARCAN)

a. "I will check my blood pressure daily and take my medication when it is over 140/90."

Which statement indicates that the client needs additional instruction about antihypertensive treatment? a. "I will check my blood pressure daily and take my medication when it is over 140/90." b. "I will include rest periods during the day to help me tolerate the fatigue my medicine may cause." c. "I will change my position slowly to prevent feeling dizzy." d. "I will not mow my lawn until I see how this medication makes me feel."

MAOIs

N-nardil P-parmate- M-marplan avoid foods with high tyramine

Side effects related to peripherally acting muscle relaxants include SELECT ALL THAT APPLY

NAUSEA,DIPLOPIA,DROWSINESS

The drug used to treat myasthenia gravis by increasing muscle strength is

NEOSTIGMINE- PROSTIGMIN

What's important to know about MAOIs?

NO Foods with tyramine should be consumed during therapy and for 2 weeks after treatment is discontinued. May cause hypotension.

What types of meaningful socialization would be good for a depressed patient?

NO GROUP ACTIVITIES bc they hate to be in groups. NO competitive activities bc may decrease self esteem more. They will do better with puzzles, music, or activities/hobbies of their choice. These should help increase their self esteem.

If a person has a delusion of being GOD, should you challenge them and tell them that he is not GOD he is Joe, etc.?

NO. Challenging a firmly fixed delusion serves no therapeutic purpose, but it could antagonize the patient.

Insulin: Intermediate Acting Names

NPH Humulin N

what characteristics are associated w celecoxib-celebrex

NSAID-Relieves pain and inflammation w/ out causing GI distress

Depakote - Anticonvulsant

No carbonated beverages

Antineoplastic: Hormonal Agents Med Names

Nolvadex, Testosterone

Andrenergics Med Names

Norepinephrine, Dopamine, Epinephrine, Dobutamine

When is the best time to administer oral medications if food interferes with absorbtion

On an empty stomach

Electrolytes Med Names

Os-Cal, Slow Mag, K-Dur, Sodium Chloride

What must you do immediately if someone is presenting with neuroleptic malignant syndrome?

Page the house physician and hold all antipsychotic meds. Provide 1 on 1 observation. Closely monitor vital signs, establish IV access for fluids or give oral fluids. Adminsiter an antipyretic because he/she is at risk for SEIZURES d/t hyperthermia (TEMPERATURE REGULATION BLANKET).

206.) A client is receiving baclofen (Lioresal) for muscle spasms caused by a spinal cord injury. The nurse monitors the client, knowing that which of the following is a side effect of this medication? 1. Muscle pain 2. Hypertension 3. Slurred speech 4. Photosensitivity

Rationale: Side effects of baclofen include drowsiness, dizziness, weakness, and nausea. Occasional side effects include headache, paresthesia of the hands and feet, constipation or diarrhea, anorexia, hypotension, confusion, and nasal congestion. Paradoxical central nervous system excitement and restlessness can occur, along with slurred speech, tremor, dry mouth, nocturia, and impotence. **Option 3 is most closely associated with a neurological disorder**

In which of the following areas are muscarinic receptors located? CHOOSE TWO

Parasympathetic Postganglionic Effector Sites Skeletal Muscle

The Rn assess for which most common side effect in a cl w tb taking INH

Parathesia in limbs numbness tingling

After drug administration, The highest plasma/serum concentrations of a drug at spec time is called:

Peak Level

In combo of agents metronidazole commonly used for Hpylori assoc w/ reoccurent

Peptic Ulcers

In canada OTC preparations are admin by what groups of the respective provinces?

Pharmacy acts

A commonly prescribed urinary analgesics is

Phenazopyridine hyrdochloride-Pyridium

Which of the following drugs is used to treat glaucoma

Pilocarpine

Ds has been Dx w Schitzophrenia-The health care provider prescribed fluphenazine prolixin Fluohenazine Prolixin is classafied as

PiperaZINE Phenothiazine

What should the nurse do innitially if there are crystal formations in the mannitol solution?

Place the vial in warm water **bc it will quickly dissolve if the container is placed in warm water then cooled to body temp before admin

The CL will be free from hyperactivity is included in which phase RN process

Planning

The CL will recieve adequate nutritional support through enteral feedings is included in which phase the RN process

Planning

Factors that most commonly affect drug actions are

Poor circulation pain stress hunger fasting

All of the following substances can be taken to decrease urine PH EXCEPT

Potassium Chloride

Antianxiety Nursing Considerations

Potential for addiction/overdose, Avoid Alcohol, Monitor LFT

Factors that affect CL adhereance with medications include:

Poverty Trust in health care providers Access to healthcare

Which of the following drugs is used to treat an overdose of organophosphate pesticides that cause paralysis

Pralidoxime Chloride

d. "I should sit or lie down after I take a nitroglycerin tablet to prevent dizziness."

Which statement indicates to the nurse that the client understands sublingual nitroglycerin medication instructions? a. "I will take up to five doses every 3 minutes for chest pain." b. "I can chew the tablet for the quickest effect." c. "I will keep the tablets locked in a safe place until I need them." d. "I should sit or lie down after I take a nitroglycerin tablet to prevent dizziness."

What is the rationale for warming an otic solution that has been refrigerated before instilling the drops into the Pt's Ear.

Prevent dizziness- nausea

. The client care plan of a client receiving dopamine should include having phentolamine (Regitine) available for use. The purpose of this drug is to:

Prevent necrosis if the IV extravasates.

A Cl with pneumonia being treated with amoxil what assessment data would be best indicator to the Rn of the effectiveness of drug therapy

Pulse Ox

Urinary Antispasmodics

Pyridium (cystitis) - Orange/Red urine

What is characteristic of the depressive part of bipolar disorder?

Previous manic episodes, Mood: dysphoric, depressive, and despairing), decrease interest in pleasure, negative views, fatigue, decreased appetite, constipation, insomnia, libido, suicidal preoccupation, and agitation or movement retardation.

Anticholinergics Med Names

Pro-Banthine, Atropine, Scopace

When administering eardrops to a 7 year old child, the nurse should do the following SELECT ALL THAT APPLY

Pull down/back on the auricle Tilt head to unaffected side Instill medicationat room temperature

Client is taking dronabinol marinol for nauseas and vomiting caused by CA chemotherapy-IT is recomended that this drug be administered

Q3-Q4 H PRN

What state law controls drug administration by nurses

RN practice Act

Thiopental an iv anesthetic barbiturate administed, intended side effect would be

Rapid, Loss unconciousness

182.) A client with angina pectoris is experiencing chest pain that radiates down the left arm. The nurse administers a sublingual nitroglycerin tablet to the client. The client's pain is unrelieved, and the nurse determines that the client needs another nitroglycerin tablet. Which of the following vital signs is most important for the nurse to check before administering the medication? 1. Temperature 2. Respirations 3. Blood pressure 4. Radial pulse rate

Rationale: Nitroglycerin acts directly on the smooth muscle of the blood vessels, causing relaxation and dilation. As a result, hypotension can occur. The nurse would check the client's blood pressure before administering the second nitroglycerin tablet. Although the respirations and apical pulse may be checked, these vital signs are not affected as a result of this medication. The temperature also is not associated with the administration of this medication.

What should be done immediately for someone experiencing a panic attack?

Reassure them that they are safe and that you'll stay with them until their symptoms resolve (most last for 10 mins although they can last for than an hour). Reassure them that symptoms won't harm them. COACH them to take slow deep breaths, inhaling through nose and exhaling gently through her mouth with pursed lips. Relaxation techniques will calm their body and mind, easing their symptoms.

A pt is prescribed phenytoin for a recurrent convulsive disorder the nurse informs the pt that the most common adverse reactions are

Related to CNS

Implications of HIPPA include

Release of information only to client Private area for pharm consults Client signs for copy of privacy statement

What is echolalia? Ex?

Repeating exactly what someone else said to you. Ex: "It's time for your shower", "It's time for your shower".

The patient in hypovolemic shock is prescribed an infusion of lactated Ringers's The nurse recognizes the function of this fluid in the treatment of shock is to (select all that apply)

Replace fluid and promote urine output Draw water into cells Maintain vascular volume

The client in shock is prescribed an infusion of lactated Ringer's solution. The nurse recognizes that the function of this fluid in the treatment of shock is to:

Replace fluid, and promote urine output.

complications result from spinal anesthesia

Respiratory Distress Headache

After administering an opioid analgesic to a cl the RN would make which priority to follow up assessments in addition to pain relief

Respiratory Rate , LOC

For the client taking a diuretic, a combination such as triamterene and hydrochlorothiazide may be prescribed. The nurse realizes that this combination is ordered for which purpose? a. To decrease the serum potassium level b. To increase the serum potassium level c. To decrease the glucose level d. To increase the glucose level

b. To increase the serum potassium level

Anticonvulsants Side Effects

Respiratory depression, aplastic anemia, gingival hypertrophy, ataxia

Your Health teaching plan for the Cl taking phenytoin would include which of the following SELECT ALL APPLY

Restrict fluids while taking phenytoin Correct answers Urine may be harmless pink or reddish brown color Alcoholic Beverages are not recommended Drug may have teratogenic effect of fetus Avoid Aspirin while taking phenytoin

Urinary analgesics are used to relieve all of the following EXCEPT

Retention

Attention Disorder Agents Med Names

Ritalin, Dexedrine

Becouse of the high incidence of sleep disorders, the most frequently presribed drugs are:

Sedative/Hypnotics

Pt refuses an essential heart medication, what does the RN do next?

Seek reason from pt.

Which of the following is are true about seizures and anticonvulsant use during pregnancy (SELECT ALL APPLY

Seizures increase 25% in epileptic women Many Anticonvulasants have teratogenic properties Anitconvulsant use increases the loss of folic acid InCORRECT ANSWERS Anticonvulsant use increases the effects of vit K Valproic Acid causes major malformations in 40-80% of infants of pregnant females

Antihypertensives: Alpha 1 Adrenergic Blockers Actions

Selective blockade of alpha-1 reception in peripheral blood vessels

Which of the following is true about relenza & tamiflu

Should be taken w/in 48 hrs of flu symptoms

physiological affects NOT related to DESIRED effects (predicatable or associated with the use of a drug) is called:

Side effects

Heparin

Side effects: bleeding, anemia, thrombocytopenia, alopecia, hypersensitivity Risk of bleeding increases with the use of drugs that affect platelet function like Aspirin, NSAID, and some penicillins Concurrent use of thrombolytics increase risk of bleeding Digoxin may decrease effects Precautions/Special considerations: When intermittent IV therapy is used draw aPTT levels 30 min before each dose and during therapy if continuous monitor every 4 hours. For subcut draw blood 4-6 hr after injection Do not confuse hepatin sodium injection vials with heparing flush vials Inform other health care providers that patient is on heparin Reversal: protamine sulfate

St. John's wort

Taking ____ with an MAOI could result in hypertensive crisis; patients should always consult with their health care provider before taking any medications or OTC drugs/herbal remedies.

Antithyroid Agents Med Names

Tapazole, SSKI

The client receiving chemotherapy is prescribed oprelvekin (Neumega) as part of the treatment regimen. The nurse explains that the function of this drug is to:

Stimulate platelet production.

Antidiabetic Agents: Actions

Stimulates insulin release from beta cells in pancreas

Specific Rn interventions for female client taking tetracycline include

Storing drug away from light Monitoring lab results Obtain specimen for C&S Advising CL to use adtl. contraceptives

Digitalis

Strengthens cardiac muscle contraction Antiarrhythmic *CANT MIX WITH ANYTHING* TOXICITY: Pulse (slows - Check Apical Pulse before giving) Photosensitivity Puke (vomiting, nausea) Yellow-Green Halos Light Flashes

Thrombolytics Med Names

Strepokinase, Urokinase, Tissue Plasminogen Activator (TPA)

Which is an example of Primary source information

Subjective date from pt

You are scheduled to provide a general anesthetic to a pt recieving treatment with echothiophate eye drops.Which of the following drugs should be avoided in the perioperative period?

Succinylcholine____________ Mivacurium______________ Atracurium A&B----------------------CORRECT ANSWER B&C ALL OF ABOVE

Which of the following adverse reactions would the nurse suspect in a pt reciving prolonged treatment with an AB drug

Superinfection

The most severe adverse extrapyramidal reaction is

Tardive Dyskinesia

Anticholinergics Side Effects

blurred vision, dry mouth, urinary retention, change in heart rate

Carbonic Anhydrase Inhibitors Side Effects

blurred vision, lethargy, depression

b. Weigh the client before administration.

The client is receiving tirofiban (Aggrastat). What is an essential nursing intervention for this client? a. Have protamine sulfate available in case of an overdose. b. Weigh the client before administration. c. Have vitamin K available in case of an overdose. d. Assess intake and output.

Antibiotics: Macrolide Nursing Consderations

Take 1 h before or 2-3 h after meals Monitor LFT Take w full glass of water

Cytoprotective Agents Nursing Considerations

Take 1h before food Give 2h becofe or after other meds

specific Rn interventions for a CL taking ceftazidime would include which following

Take C&S Assess Allergic rxn Monitor I/O

Penicillines

Take all 10 days

Cardiac Glycosides Nursing Consideration

Take apical pulse Notify physician if adult <60, child <90-110 Monitor potassium level (3.5-5)

Antihyperlipidemic: Nursing Considerations

Take at bedtime or 30 mins after meals Administer 1 h before or 4-6h after other meds

Antihypertensives

Take med at same time every day avoid saunas, hot tubs

Iron Preparations Nursing Considerations

Take on empty stomach Vit C increases absorption Monitor Hct and Hgb

Immunosuppressants Nursing Considerations

Take once daily in am Used with adrenal corticosteroids Monitor renal and liver function tests Take w milk or juice Stay away from sick ppl Good Handwashing

The Rn is admin tetracycline to client what would be appropriate teaching

Take sunscreen precautions- photosensitivity tooth discoloration

An elderly patient diagnosed with iron-deficency anemia will be taking ferrous sulfate (feosol, others) orally. the nurse will teach required administration guidlines to the patient including (select all that apply)

Take the tablets on an empty stomach if possible increase fluid intake and increase dietary fiber while taking this medication if liquid prepartions are used, dilute with water or juice and sip through a straw placed in back of the mouth.

Antibiotics Teaching

Take until gone Do culture and sensitivity first Encourage fluids Check expiration date

Antibiotics: Cephalosporins Nursing Consideration

Take w food Cross allergy with PCN Avoid alcohol Obtain Culture and Sensitivity before first dose

NSAIDS Nursing Considerations

Take w food or after meals Monitor liver and renal functions Use cautiously with aspirin allergy Check for bleeding

H2-Receptor Blockers Nursing Considerations

Take w meals and hs Smoking decreases effectiveness Monitor LFT and CBC

Rn interventions concearning drug therapy include the following EXCEPT

Teach client to wait week after occurence of S/S to see if the disappear

A pt. with glaucoma is prescribed pilocarpine eye drops. One adverse reaction that the nurse will expect with the use of this drug is?

Temporary Loss of Visual Acuity

Which of the following instructions would be included in a teaching plan for the pt prescribed an opthalmic solution

Temporary sting/burning may be felt at the time the drug is instilled

Cl has developed Vag candidiasis the Rn should know that which med is approp treatment for this condition

Terconazole

b. Notify the health care provider.

The client taking Methyldopa (Aldomet) has elevated liver function tests. What is the nurse's best action? a. Document the finding and continue care. b. Notify the health care provider. c. Immediately stop the medication. d. Change the client's diet.

d. "I can take up to five tablets at 3-minute intervals for chest pain if necessary."

Which statement made by the client demonstrates a need for further instruction regarding the use of nitroglycerin? a. "If I get a headache, I should keep taking nitroglycerin and use Tylenol for pain relief." b. "I should keep my nitroglycerin in a cool, dry place." c. "I should change positions slowly to avoid getting dizzy." d. "I can take up to five tablets at 3-minute intervals for chest pain if necessary."

b. "It's best to keep it in its original container away from heat and light."

The client asks the nurse how nitroglycerin should be stored while traveling. What is the nurse's best response? a. "You can protect it from heat by placing the bottle in an ice chest." b. "It's best to keep it in its original container away from heat and light." c. "You can put a few tablets in a resealable bag and carry it in your pocket." d. "It's best to lock them in the glove compartment to keep them away from heat and light."

a. It causes an alkaline urine, which facilitates the elimination of uric acid.

The client asks the nurse why the health care provider prescribed acetazolamide (Diamox), a diuretic, to treat gout. What is the nurse's best response? a. It causes an alkaline urine, which facilitates the elimination of uric acid. b. It increases alkalinity of urine, thus decreasing the formation of uric acid. c. It causes an acid urine, which facilitates the elimination of uric acid. d. It decreases alkalinity of urine, thus decreasing the formation of uric acid.

b. "Dalteparin is a low-molecular-weight heparin that is more predictable in its effect and has a lower risk of bleeding."

The client asks what the difference is between dalteparin (Fragmin) and heparin. What is the nurse's best response? a. "There is no real difference. Dalteparin is preferred because it is less expensive." b. "Dalteparin is a low-molecular-weight heparin that is more predictable in its effect and has a lower risk of bleeding." c. "I'm not sure why some health care providers choose dalteparin and some heparin. You should ask your doctor." d. "The only difference is that heparin dosing is based on the client's weight."

b. Hyperkalemia

The client has been receiving spironolactone (Aldactone) 50 mg/day for heart failure. The nurse should closely monitor the client for which condition? a. Hypokalemia b. Hyperkalemia c. Hypoglycemia d. Hypermagnesemia

a. Increase the serum digoxin sensitivity level

The client is also taking a diuretic that decreases her potassium level. The nurse expects that a low potassium level (hypokalemia) could have what effect on the digoxin? a. Increase the serum digoxin sensitivity level b. Decrease the serum digoxin sensitivity level c. Not have any effect on the serum digoxin sensitivity level d. Cause a low average serum digoxin sensitivity level

Antineoplastic: Antitumor Side Effects

bone marrow suppression, alopecia, stomatitis

a. It is in the high (elevated) range.

The client's serum digoxin level is 3.0 ng/mL. What does the nurse know about this serum digoxin level? a. It is in the high (elevated) range. b. It is in the low (decreased) range. c. It is within the normal range. d. It is in the low average range.

4. When counseling a patient who is starting to take MAO (monoamineoxidase) inhibitors such as Nardil for depression, it is essential that they be warned not to eat foods containing tyramine, such as 1) Roquefort, cheddar, or Camembert cheese. 2) grape juice, orange juice, or raisins. 3) onions, garlic, or scallions. 4) ground beef, turkey, or pork.

The correct answer is (1). Monoamine oxidase inhibitors react with foods high in the amino acid tyramine to cause dangerously high blood pressure. Aged cheeses are all high in this amino acid the other foods are not.

3. Tom R., a newly admitted patient, has a seizure disorder which is being treated with medication. Which of the following drugs would the nurse question if ordered for him? 1) Phenobarbitol, 150 mg hs 2) Amitriptylene (Elavil), 10 mg QID. 3) Valproic acid (Depakote), 150 mg BID 4) Phenytoin (Dilantin), 100 mg TID

The correct answer is (2). Elavil is an antidepressant that lowers the seizure threshold, so would not be appropriate for this patient. The other medications are anti-seizure drugs.

9. Frank Jones, a nurse on a geriatric floor, is administering a dose of digoxin to one of his patients. The woman asks why she takes a different pill than her niece, who also has heart trouble. Frank replies that as people get older, liver and kidney function decline, and if the dose is as high as her niece's, the drug will tend to 1) have a shorter half-life. 2) accumulate. 3) have decreased distribution. 4) have increased absorption.

The correct answer is (2). The decreased circulation to the kidney and reduced liver function tend to allow drugs to accumulate and have toxic effects.

7. Mrs. Garvey has been dealing with uterine cancer for several months. Pain management is the primary focus of her current admission to your oncology unit. Her vital signs on admission are BP 110/64, pulse 78, respirations 18,and temperature 99.2 F. Morphine sulfate 6mg IV, q 4 hours, prn has been ordered. During your assessment after lunch, your findings are: BP 92/60,pulse 66, respirations 10, and temperature 98.8. Mrs. Garvey is crying and tells you she is still experiencing severe pain. Your action should be to 1) give her the next ordered dose of MS. 2) give her a back rub, put on some light music, and dim the lights in the room. 3) report your findings to the RN, requesting an alternate medication order be obtained from the physician. 4) call her daughter to come and sit with her.

The correct answer is (3). Morphine sulfate depresses the respiratory center. When the rate is less than 10, the MD should be notified.

2. The nurse is administering Augmentin to her patient with a sinus infection. Which is the best way for her to insure that she is giving it to the right patient? 1) Call the patient by name 2) Read the name of the patient on the patient's door 3) Check the patient's wristband 4) Check the patient's room number on the unit census list.

The correct answer is (3). The correct way to identify a patient before giving a medication is to check the name on the medication administration record with the patient's identification band. The nurse should also ask the patient to state their name. The name on the door or the census list are not sufficient proof of identification. Calling the patient by name is not as effective as having the patient state their name patients may not hear well or understand what the nurse is saying, and may respond to a name which is not their own.

5. The nurse is administering an antibiotic to her pediatric patient. She checks the patient's armband and verifies the correct medication by checking the physician's order, medication kardex, and vial. Which of the following is not considered one of the five "rights" of drug administration? 1) Right dose 2) Right route 3) Right frequency 4) Right time

The correct answer is (3). The five rights of medication administration are right drug, right dose, right route, right time, right patient. Frequency is not included.

10. The nursery nurse is putting erythromycin ointment in the newborn's eyes to prevent infection. She places it in the following area of the eye: 1) under the eyelid. 2) on the cornea. 3) in the lower conjunctival sac. 4) by the optic disc.

The correct answer is (3). The ointment is placed in the lower conjunctival sac so it will not scratch the eye itself and will get well distributed.

1. The physician orders penicillin for a patient with streptococcal pharyngitis. The nurse administers the drug as ordered, and the patient has an allergic reaction. The nurse checks the medication order sheet and finds that the patient is allergic to penicillin. Legal responsibility for the error is 1) only the nurse's—she should have checked the allergies before administering the medication. 2) only the physician's—she gave the order, the nurse is obligated to follow it. 3) only the pharmacist's—he should alert the floor to possible allergic reactions. 4) the pharmacist, physician, and nurse are all liable for the mistake.

The correct answer is (4). The physician, nurse, and pharmacist all are licensed professionals and share responsibility for errors.

b. The beta blocker should NOT be abruptly stopped; the dose should be tapered down.

The health care provider is planning to discontinue a client's beta blocker. What instruction should the nurse give the client regarding the beta blocker? a. The beta blocker should be abruptly stopped when another cardiac drug is prescribed. b. The beta blocker should NOT be abruptly stopped; the dose should be tapered down. c. The beta blocker dose should be maintained while taking another antianginal drug. d. Half the beta blocker dose should be taken for the next several weeks.

c. Decrease heart rate and decrease myocardial contractility.

The nurse acknowledges that beta blockers are as effective as antianginals because they do what? a. Increase oxygen to the systemic circulation. b. Maintain heart rate and blood pressure. c. Decrease heart rate and decrease myocardial contractility. d. Decrease heart rate and increase myocardial contractility.

a. Diuretic

The nurse acknowledges that the first-line drug for treating this client's blood pressure might be which drug? a. Diuretic b. Alpha blocker c. ACE inhibitor d. Alpha/beta blocker

a. Hypokalemia

The nurse acknowledges that which condition could occur when taking furosemide? a. Hypokalemia b. Hyperkalemia c. Hypoglycemia d. Hypermagnesemia

d. "I should use a soft toothbrush for dental hygiene."

The nurse evaluates that the client understood discharge teaching regarding warfarin (Coumadin) based on which statement? a. "I will double my dose if I forget to take it the day before." b. "I should keep taking ibuprofen for my arthritis." c. "I should decrease the dose if I start bruising easily." d. "I should use a soft toothbrush for dental hygiene."

c. Administer 2 mEq potassium chloride per kilogram per day IV.

The nurse is assessing a client who is taking furosemide (Lasix). The client's potassium level is 3.4 mEq/L, chloride is 90 mmol/L, and sodium is 140 mEq/L. What is the nurse's primary intervention? a. Mix 40 mEq of potassium in 250 mL D5W and infuse rapidly. b. Administer Kayexalate. c. Administer 2 mEq potassium chloride per kilogram per day IV. d. Administer PhosLo, two tablets three times per day.

c. Beta blockers and ACE inhibitors

The nurse is aware that which group(s) of antihypertensive drugs are less effective in African-American clients? a. Diuretics b. Calcium channel blockers and vasodilators c. Beta blockers and ACE inhibitors d. Alpha blockers

d. Get up slowly from a sitting to a standing position.

The nurse is caring for a client with hypertension who is prescribed Clonidine transdermal preparation. What is the correct information to teach this client? a. Change the patch daily at the same time. b. Remove the patch before taking a shower or bath. c. Do not take other antihypertensive medications while on this patch. d. Get up slowly from a sitting to a standing position.

b. Heart rate 58 beats per minute

The nurse is monitoring a client taking digoxin (Lanoxin) for treatment of heart failure. Which assessment finding indicates a therapeutic effect of the drug? a. Heart rate 110 beats per minute b. Heart rate 58 beats per minute c. Urinary output 40 mL/hr d. Blood pressure 90/50 mm Hg

d. spironolactone (Aldactone)

The nurse is reviewing a medication history on a client taking an ACE inhibitor. The nurse plans to contact the health care provider if the client is also taking which medication? a. docusate sodium (Colace) b. furosemide (Lasix) c. morphine sulfate d. spironolactone (Aldactone)

b. "Take this medication at the same time each day."

The nurse is reviewing instructions for a client taking an HMG-CoA reductase inhibitor (statin). What information is essential for the nurse to include? a. "Take this medication on an empty stomach." b. "Take this medication at the same time each day." c. "Take this medication with breakfast." d. "Take this medication with an antacid."

a. 150 to 200 mg/dL

The nurse knows that the client's cholesterol level should be within which range? a. 150 to 200 mg/dL b. 200 to 225 mg/dL c. 225 to 250 mg/dL d. Greater than 250 mg/dL

b. hydrochlorothiazide

The nurse knows that which diuretic is most frequently combined with an antihypertensive drug? a. chlorthalidone b. hydrochlorothiazide c. bendroflumethiazide d. potassium-sparing diuretic

b. Administer aspirin 30 minutes before nicotinic acid.

The nurse plans which intervention to decrease the flushing reaction of niacin? a. Administer niacin with an antacid. b. Administer aspirin 30 minutes before nicotinic acid. c. Administer diphenhydramine hydrochloride (Benadryl) with niacin. d. Apply cold compresses to the head and neck.

b. homocysteine

The nurse realizes that which is the laboratory test ordered to determine the presence of the amino acid that can contribute to cardiovascular disease and stroke? a. antidiuretic hormone b. homocysteine c. ceruloplasmin d. cryoglobulin

b. To administer digoxin immune FAB

The nurse reviews a client's laboratory values and finds a digoxin level of 10 ng/mL and a serum potassium level of 5.9 mEq/L. What is the nurse's primary intervention? a. To administer atropine b. To administer digoxin immune FAB c. To administer epinephrine d. To administer Kayexalate

c. Client is on oral contraceptives.

The nurse reviews the history for a client taking atorvastatin (Lipitor). What will the nurse act on immediately? a. Client takes medications with grape juice. b. Client takes herbal therapy including kava kava. c. Client is on oral contraceptives. d. Client was started on penicillin for a respiratory infection.

A time response curve that evaluates three parameters of drug action, does NOT include:

Theraputic Range Includes: Duration Onset Peak Trough

In the administration of a drug such as levothyroxine (Synthroid), the nurse must teach the client: (Select all that apply.) a.) Therapy could take three weeks or longer. b.) Periodic lab tests for T4 levels are required. c.) Report weight loss, anxiety, insomnia, and palpitations. d.) Jaundice

Therapy could take three weeks or longer. Periodic lab tests for T4 levels are required. Report weight loss, anxiety, insomnia, and palpitations. A,B,C

SSRI

These should not be started within 14 days of taking MAOI. use with a MAOI can cause serotonin sydrome (autonomic hyperactivity, hyperthermia, rigidity, diaphoresis, and neuroleptic malignant syndrome, hypertensive crisis.

How do you know therapy for agoraphobia is working?

They are out shopping or initiating conversation!

How do you know when the treatment for an antisocial person has been effective?

They show remorse for taking advantage of others.

What are the S&S of alcohol withdrawal and when do they usually begin?

They usually begin 3 to 36 hours after the last drink. The patient may present with tremulousness, motor hyperactivity, anxiety, and hyperalertness, irritability, agitation, diaphoresis, tachycardia, hypertension, pounding HA, abdominal cramping, anorexia, nausea, vomiting, or diarrhea. A tonic-clonic (Grand Mal) seizure is a common but serious complication also.

Antipsychotic Agents Med Names

Thorazine-low, Trilafon-medium, Haldol-high

The reason for the clients position after spinal anethesia would be

To decrease leakage of spinal fluids

JT asks what BID means your response would be

Twice daily

Your Cl had her ears irrigated. To determine the results of this irrigation,visualization of which structure is required?

Tympanic Membrane

Antidiabetic Agents: Uses

Type 2 diabetes

Current authoratative source for drug standards

U.S Pharmacopia/Natl. Formulary

RT is an acutely manic cl prescribed lithium for 1st time Nursing Interventions associated with lithium Carbonate in the management of bipolar disorders include all of the following EXCEPT

Understanding the drug is most effective in the depressive phase

In an adult, gently pull the auricle in what direction before instilling ear drops

Up,Back

The Nurse assesses PH for Side effects of the drug Which of the following is NOT a side effect of MAOIs

Urinary Retention

Your health teaching plan for LP would include all of the following EXCEPT

Urine May Turn Orange

A Cl is recieving both Selegiline (eldepryl) and MAOI and meperidine Demerol -what instructions should the RN provide to the Unlicensed Assistive Personel UAP

Use auto BP to measure BP

Antidiarrheals: Nursing Considerations

Use before chemo When used with viral infections may cause Rye's syndrome

Based on your knowledge of analgesia, which factor is MOST relevant to the relief of chronic pain

Use of drugs w/ long half lifes

a CL has been prescribed to take both tetracycline and sulfonamide drugs- when providing teaching what is the priority information should the Rn give the CL r/t adverse drug affects

Use protective measures when exposed to the sun sulfonamides- photosensitivity

What is milieu therapy?

Use the environment/peer pressure to change a person's behaviors. Patients develop the rules/punishments. (A controlling nurse for this type of therapy isn't appropriate).

It may be difficult to assess pain in children- Pain management is more apt to be successful if the RN does which of the following

Uses age appropriate communication skills Uses ouch scale Discusses childs response w. parents ALL OF ABOVE!

Serotonin antagonists atypical antipsychotic are effective for treating which types of scizophrenia

both positive and negative symptoms

What would be appropriate treatment/coping for PTSD?

Valerian root can help with symptoms (takes 1 1/2 weeks to kick in) and avoiding triggers (stress) can help prevent episodes (not watching army movies).

Preffered site for IN injections in infants and children

Vastus lateralis-infants

d. "I will continue my exercise program to help increase my high-density lipoprotein serum levels."

Which statement made by the client indicates understanding about discharge instructions on antihyperlipidemic medications? a. "Antihyperlipidemic medications will replace the other interventions I have been doing to try to decrease my cholesterol." b. "It is important to double my dose if I miss one in order to maintain therapeutic blood levels." c. "I will stop taking the medication if it causes nausea and vomiting." d. "I will continue my exercise program to help increase my high-density lipoprotein serum levels."

What are alcohol dependent patients with a thiamine deficiency at risk for?

WERNICKE-KORSAKOFF SYNDROME (WERNIKE'S ENCEPHALOPATHY AND KORSAKOFF PSYCHOSIS) AN IRREVERSIBLE DEGENERATIVE BRAIN DISORDER.

Baclofen (Lioresal) is a skeletal muscle relaxant and decreases muscle spasms in pts w/ spinal cord

a skeletal muscle relaxant and decreases muscle spasms in pts w/ spinal cord

b. "This combination promotes diuresis but decreases the risk of hypokalemia."

What is the best information for the nurse to provide to the client who is receiving spironolactone (Aldactone) and furosemide (Lasix) therapy? a. "Moderate doses of two different diuretics are more effective than a large dose of one." b. "This combination promotes diuresis but decreases the risk of hypokalemia." c. "This combination prevents dehydration and hypovolemia." d. "Using two drugs increases the osmolality of plasma and the glomerular filtration rate."

b. Continuous blood pressures d. Presence of chest pain

What must the nurse monitor when titrating intravenous nitroglycerin for a client? (Select all that apply.) a. Continuous oxygen saturation b. Continuous blood pressures c. Hourly ECGs d. Presence of chest pain e. Serum nitroglycerin levels f. Visual acuity

a. Assess for reperfusion dysrhythmias.

What nursing intervention is essential for the client receiving alteplase? a. Assess for reperfusion dysrhythmias. b. Monitor liver enzymes. c. Administer vitamin K if bruising is observed. d. Monitor blood pressure and stop the medication if blood pressure drops below 110 systolic.

d. "This medication will work for 24 hours and you will need to change the patch daily."

What statement is the most important for the nurse to include in the teaching plan for a client who has started on a transdermal nitroglycerin patch? a. "This medication works faster than sublingual nitroglycerin works." b. "This medication is the strongest of any nitroglycerin preparation available." c. "This medication should be used only when you are experiencing chest pain." d. "This medication will work for 24 hours and you will need to change the patch daily."

a. Inhibits absorption of dietary cholesterol in the intestines.

When a client is taking ezetimibe (Zetia), she asks the nurse how it works. The nurse should explain that Zetia does what? a. Inhibits absorption of dietary cholesterol in the intestines. b. Binds with bile acids in the intestines to reduce LDL levels. c. Inhibits HMG-CoA reductase, which is necessary for cholesterol production in the liver. d. Forms insoluble complexes and reduces circulating cholesterol in blood.

What is juvenile schizophrenia/autism?

When a person has ritualistic behaviors so everyone must follow the plan of care. They self stimulate and don't like loud noises. To relax an autistic child try to de-pressure the situation.

When should you take off a cooling blanket on someone with a fever?

When their temperature reaches 101 bc they will continue to decrease after removal.

b. Elevated liver function tests

Which assessment finding in a client taking an HMG-CoA reductase inhibitor will the nurse act on immediately? a. Decreased hemoglobin b. Elevated liver function tests c. Elevated HDL d. Elevated LDL

b. Crackles in the lungs

Which assessment finding will alert the nurse to possible toxic effects of amiodarone? a. Heart rate 100 beats per minute b. Crackles in the lungs c. Elevated blood urea nitrogen d. Decreased hemoglobin

After Liquid tetracycline is ordered for 2yr old, Rn should provide which important instruction to LPN who is admin med.

Withhold drug Telephone MD Below 8 year old- tetracycline causes Causes tooth discoloration

Cortisone (Steroids)

Wt gain, Na+ retention, K+ Loss NEVER STOP SUDDENLY

A physician prescribes levothyroxine sodium (Synthroid), 0.15 mg orally daily, for a client with hypothyroidism. The nurse will prepare to administer this medication: a) in the morning to prevent insomnia b) only when the client complains of fatigue and cold intolerance c) at various times during the day to prevent tolerance from occurring d) three times daily in equal doses of 0.5 mg each to ensure consistent serum drug levels

a) in the morning to prevent insomnia Levothyroxine (Synthroid) is a synthetic thyroid hormone that increases cellular metabolism. Levothyroxine should be given in the morning in a single dose to prevent insomnia and should be given at the same time each day to maintain an adequate drug level. Therefore, options B, C, and D are incorrect.

A client is prescribed enoxaparin (Lovenox). The nurse knows that low-molecular-weight heparin (LMWH) has what kind of half-life? a. A longer half-life than heparin b. A shorter half-life than heparin c. The same half-life as heparin d. A four-times shorter half-life than heparin

a. A longer half-life than heparin

The nurse explains that which beta blocker category is preferred for treating hypertension? a. Beta1 blocker b. Beta2 blocker c. Beta1 and beta2 blockers d. Beta2 and beta3 blockers

a. Beta1 blocker

When a newly admitted client is placed on heparin, the nurse acknowledges that heparin is effective for preventing new clot formation in clients who have which disorder(s)? (Select all that apply.) a. Coronary thrombosis b. Acute myocardial infarction c. Deep vein thrombosis (DVT) d. Cerebrovascular accident (CVA) (stroke) e. Venous disorders

a. Coronary thrombosis b. Acute myocardial infarction c. Deep vein thrombosis (DVT) d. Cerebrovascular accident (CVA) (stroke) e. Venous disorders

The nurse acknowledges that the first-line drug for treating this client's blood pressure might be which drug? a. Diuretic b. Alpha blocker c. ACE inhibitor d. Alpha/beta blocker

a. Diuretic

A client who has angina is prescribed nitroglycerin. The nurse reviews which appropriate nursing interventions for nitroglycerin? (Select all that apply.) a. Have client lie down when taking a nitroglycerin sublingual tablet. b. Teach client to repeat taking a tablet in 5 minutes if chest pain persists. c. Apply Transderm-Nitro patch to a hairy area to protect skin from burning. d. Call the health care provider after taking 5 tablets if chest pain persists. e. Warn client against ingesting alcohol while taking nitroglycerin.

a. Have client lie down when taking a nitroglycerin sublingual tablet. b. Teach client to repeat taking a tablet in 5 minutes if chest pain persists. e. Warn client against ingesting alcohol while taking nitroglycerin.

The nurse acknowledges that which condition could occur when taking furosemide? a. Hypokalemia b. Hyperkalemia c. Hypoglycemia d. Hypermagnesemia

a. Hypokalemia

The client's serum digoxin level is 3.0 ng/mL. What does the nurse know about this serum digoxin level? a. It is in the high (elevated) range. b. It is in the low (decreased) range. c. It is within the normal range. d. It is in the low average range.

a. It is in the high (elevated) range.

antidote for acetaminophen (Tylenol)

acetylcysteine (Mucomyst)

A client is diagnosed with peripheral arterial disease (PAD). He is prescribed isoxsuprine (Vasodilan). The nurse acknowledges that isoxsuprine does what? (Select all that apply.) a. Relaxes the arterial walls within the skeletal muscles b. May cause hypotension, chest pain, and palpitations c. Increases the rigidity of arteriosclerotic blood vessels d. May increase intermittent claudication e. May lead to hypertension and bradycardia f. Commonly causes an adverse effect of rhabdomyolysis

a. Relaxes the arterial walls within the skeletal muscles b. May cause hypotension, chest pain, and palpitations

neutropenia

abnormally small number of MATURE white blood cells

Laxatives/Stool Softeners Action

absorb water increasing bulk, lubricate surface of stool, stimulate peristalsis

N-acetylcysteine (mucomyst)

acetaminophen antidote

flumazenil (mucomyst)

acetaminophen antidote

Antihypertensives: Alpha 1 Adrenergic Blockers Nursing Considerations

administer first dose bedtime to avoid fainting Monitor BUN, weight and edema Change position slowly

antacids are given

after meals

Pt to recieve nitrous oxide, which adv rxn is expected

agitation

neutropenia is aka?

agranulocytosis

Major adverse effect of Clozapine (Clozaril)

agranulocytosis monitor WBC count treatment is stopped if WBC under 3000

Antigout Agents: Side Effects

agranulocytosis, GI upset, renal calculi

Antipsychotic Agents Side Effects

akathisia, dyskinesia, dystonia, parkinson's syndrome, tardive dyskinesias, leukpenia

Antihistamines: Uses

allergic rhinitis, allergic reactions to blood

-"caine"

anesthetics

Antihypertensives: Calcium Channel Blockers Uses

angina, hypertension, interstitial cystitis

Antianginals Uses

angina, peri-operative hypoertension, HF

adrenal insufficiency signs

anorexia nausea weakness fatigue hypotension hypoglycemia

protamine sulfate

anoxoparin (lovenox) antidote heparin

Drugs that prevent or inhibit a response are known as

antagonist

Clomipramine (Anafranil)

are used to treat obsessive compulsive behavior. (OCD)

chelation therapy

arsenic antidote lead antidote mercury antidote

NSAIDS Uses

arthritis, mild to moderate pain, fever

phenytoin level of 30 mcg/mL or higher the pt will show signs of

ataxia & slurred speech

what is the antidote for bethanechol chloride (Urecholine); a med that causes a cholinergic reaction

atropine sulfate

What manifestation in a COPD or asthma pt taking Propanolol (Inderal), a beta blocker, show that they have bronchospasm?

audible expiratory wheezes

208.) A client with myasthenia gravis verbalizes complaints of feeling much weaker than normal. The health care provider plans to implement a diagnostic test to determine if the client is experiencing a myasthenic crisis and administers edrophonium (Enlon). Which of the following would indicate that the client is experiencing a myasthenic crisis? 1. Increasing weakness 2. No change in the condition 3. An increase in muscle spasms 4. A temporary improvement in the condition

auto-define "A client with myasthen..." Rationale: Edrophonium (Enlon) is administered to determine whether the client is reacting to an overdose of a medication (cholinergic crisis) or to an increasing severity of the disease (myasthenic crisis). When the edrophonium (Enlon) injection is given and the condition improves temporarily, the client is in myasthenic crisis. This is known as a positive test. Increasing weakness would occur in cholinergic crisis. Options 2 and 3 would not occur in either crisis.

A newly admitted client takes digoxin 0.25 mg/day. The nurse knows that which is the serum therapeutic range for digoxin? a. 0.1 to 1.5 ng/mL b. 0.5 to 2.0 ng/mL c. 1.0 to 2.5 ng/mL d. 2.0 to 4.0 ng/mL

b. 0.5 to 2.0 ng/mL

A client is prescribed losartan (Cozaar). The nurse teaches the client that an angiotensin II receptor blocker (ARB) acts by doing what? a. Inhibiting angiotensin-converting enzyme b. Blocking angiotensin II from AT1 receptors c. Preventing the release of angiotensin I d. Promoting the release of aldosterone

b. Blocking angiotensin II from AT1 receptors

During an admission assessment, the client states that she takes amlodipine (Norvasc). The nurse wishes to determine whether or not the client has any common side effects of a calcium channel blocker. The nurse asks the client if she has which signs and symptoms? (Select all that apply.) a. Insomnia b. Dizziness c. Headache d. Angioedema e. Ankle edema f. Hacking cough

b. Dizziness c. Headache e. Ankle edema

Your patient has been switched from valproic acid (Depakote) to gabepentin (Neurontin). Which of the following is a false statement? a. Gabapentin (Neurontin) is also used for bipolar disorder therapy b. Gabapentin (Neurontin) requires more frequent hepatic monitoring c. Gabapentin (Neurontin) is also used for migraine therapy d. Gabapentin (Neurontin) should not be given concurrently with antacids containing magnesium

b. Gabapentin (Neurontin) requires more frequent hepatic monitoring

When a client first takes a nitrate, the nurse expects which symptom that often occurs? a. Nausea and vomiting b. Headaches c. Stomach cramps d. Irregular pulse rate

b. Headaches

A client's high-density lipoprotein (HDL) is 60 mg/dL. What does the nurse acknowledge concerning this level? a. It is lower than the desired level of HDL. b. It is the desired level of HDL. c. It is higher than the desired level of HDL. d. It is a much lower HDL level than desired.

b. It is the desired level of HDL.

The health care provider is planning to discontinue a client's beta blocker. What instruction should the nurse give the client regarding the beta blocker? a. The beta blocker should be abruptly stopped when another cardiac drug is prescribed. b. The beta blocker should NOT be abruptly stopped; the dose should be tapered down. c. The beta blocker dose should be maintained while taking another antianginal drug. d. Half the beta blocker dose should be taken for the next several weeks.

b. The beta blocker should NOT be abruptly stopped; the dose should be tapered down.

Your patient is taking valproic acid (Depakote). Which of the following is a false statement? a. Valproic acid requires hepatic monitoring b. Valproic acid has the lowest seizure relapse rate when discontinued c. Valproic acid is also used in migraine therapy d. Valproic acid is also used in bipolar disorder therapy

b. Valproic acid has the lowest seizure relapse rate when discontinued

A client is to undergo a coronary angioplasty. The nurse acknowledges that which drug is used primarily for preventing reocclusion of coronary arteries following a coronary angioplasty? a. clopidogrel (Plavix) b. abciximab (ReoPro) c. warfarin (Coumadin) d. streptokinase

b. abciximab (ReoPro)

Antiplatelet agents include all of the following except a. acetylsalicylic acid b. acetaminophen c. ticlopidine (Ticlid) d. dipyridamole (Persantine)

b. acetaminophen

The nurse realizes that which is the laboratory test ordered to determine the presence of the amino acid that can contribute to cardiovascular disease and stroke? a. antidiuretic hormone b. homocysteine c. ceruloplasmin d. cryoglobulin

b. homocysteine

The nurse knows that which diuretic is most frequently combined with an antihypertensive drug? a. chlorthalidone b. hydrochlorothiazide c. bendroflumethiazide d. potassium-sparing diuretic

b. hydrochlorothiazide

Your patient has been stabilized taking only primidone (Mysoline). Which drug besides primidone may be assayed during his stay in the hospital to monitor his therapy? a. pentobarbital b. phenobarbital c. valproic acid d. phenytoin

b. phenobarbital

The nurse teaches the client relaxation techniques and guided imagery as an adjunct to medication for treatment of pain. The nurse explains that the major benefit of these techniques is that they: a.) Are less costly. b.) Allow lower doses of drugs with fewer side effects. c.) Can be used at home or in any environment. d.) Do not require self-injection.

b.) Allow lower doses of drugs with fewer side effects. When used concurrently with medication, non-pharmacologic techniques can allow for lower doses, and possibly fewer drug-related adverse effects. The other options also are advantages to guided imagery and relaxation, but not the major one.

Of what precautions should a client receiving radioactive iodine-131 be made aware? a.) Drink plenty of fluids, especially those high in calcium. b.) Avoid close contact with children or pregnant women for one week after administration of drug. c.) Be aware of the symptoms of tachycardia, increased metabolic rate, and anxiety. d.) Wear a mask if around children or pregnant women.

b.) Avoid close contact with children or pregnant women for one week after administration of drug.

When hydrocortisone use is discontinued, the nurse must recognize the possibility of what side effect, if this drug is stopped abruptly? a.) Development of myxedema b.) Circulatory collapse c.) Development of Cushing's syndrome d.) Development of diabetes insipidus

b.) Circulatory collapse

Parkinsonism signs

tremors mask-like facies rigidity shuffling gait

Nursing intervention for a client receiving opioid analgesics over an extended period of time should include: a.) Referring the client to a drug treatment center. b.) Encouraging increased fluids and fiber in the diet. c.) Monitoring for G.I. bleeding. d.0 Teaching the client to take her own blood pressure.

b.) Encouraging increased fluids and fiber in the diet. Opioids suppress intestinal contractility, increase anal sphincter tone, and inhibit fluids into the intestines, which can lead to constipation. There is nothing to indicate the drug is related to addiction problems. Opioids do not cause GI bleeding.

The client is prescribed ketorolac tromethamine (Toradol) for treatment of pain following a surgical procedure. The nurse should question which of the following drug orders? a.) Toradol 10 mg p.o. b.i.d. b.) Toradol 20 mg p.o. b.i.d c.) Toradol 5 mg p.o. t.i.d. d.) Toradol 20 mg p.o q.i.d

b.) Toradol 20 mg p.o. b.i.d The maximum daily dose of Toradol is 40 mg.

-"barbitol"

barbituates CNS depressant, long acting

Phenobarbital sodium (Luminal) is both a

barbiturate & antiseizure med

best time to take corticosteroids

before 9 am or early morning

Antihypertensives: Angiotensin II Receptor Blockers Action

blocks vasoconstriction and aldosterone effects of aniotensin II

Trimethoprim- sulfamethoxazole (Bactrim) ; most important thing to look for

blood disorders

what is the major adverse effects of carbamazepine (Tegretol)

blood dyscrasias

glucocorticoids elevate

blood glucose levels

A client has heart failure and is prescribed Lasix. The nurse is aware that furosemide (Lasix) is what kind of drug? a. Thiazide diuretic b. Osmotic diuretic c. High-ceiling (loop) diuretic d. Potassium-sparing diuretic

c. High-ceiling (loop) diuretic

What would cause the same client's electrolyte imbalance? a. High dose of digoxin b. Digoxin taken daily c. Hydrochlorothiazide d. Low dose of hydrochlorothiazide

c. Hydrochlorothiazide

A client has a serum cholesterol level of 265 mg/dL, triglyceride level of 235 mg/dL, and LDL of 180 mg/dL. What do these serum levels indicate? a. Hypolipidemia b. Normolipidemia c. Hyperlipidemia d. Alipidemia

c. Hyperlipidemia

An elderly client had abdominal surgery six hours earlier. When the nurse asks the client about pain, the client responds that there is none. The best intervention on the part of the nurse is: a. Administer a PRN dose of IV pain medication as ordered. b. Assist the client into a sitting position in preparation for ambulation. c. Question the client further about discomfort to assess the meaning of pain. d. Assess the abdominal dressing and consult the surgeon about findings.

c. Question the client further about discomfort to assess the meaning of pain.

A client's blood pressure (BP) is 145/90. According to the guidelines for determining hypertension, the nurse realizes that the client's BP is at which stage? a. Normal b. Prehypertension c. Stage 1 hypertension d. Stage 2 hypertension

c. Stage 1 hypertension

Thiazolidinediones

decrease insulin resistance

The client admitted with hepatitis B is prescribed Vicodin tabs 2 for treatment of pain. The appropriate nursing action is to: a.) Administer the drug as ordered. b.) Administer one tablet only. c.) Question the physician about the order. d.) Hold the drug until the physician arrives.

c.) Question the physician about the order. Vicodin is a combination drug of hydrocodone and acetaminophen. Acetaminophen can be hepatotoxic, and is contraindicated in liver disease.

Celecoxib (Celebrex) is added to the treatment regimen of a client with arthritis. The nurse explains that the major advantage of this drug is: a.) The drug is less expensive. b.) The drug has no known side effects. c.) The drug has anti-inflammatory properties. d.) The drug's effectiveness is the same as opioids.

c.) The drug has anti-inflammatory properties. Celecoxib (Celebrex) has anti-inflammatory properties. It is not less expensive, has many side effects, and is less potent than opioids.

Corticosteroid therapy can cause

calcium depletion potassium depletion sodium retention glucose intolerance increased blood glucose levels

cultural woman per mothers requests eats red clay to provide for her unborn fetus- cultural practice she intends to continue- you as an RN should

calculate determine amount clay she eats Do not discourage any cultural practice unless harmful to self others

Anti Impotence Nursing Considerations

can not take with grapefruit juice call PCP if erection lasting longer than 4 h

Antineoplastic: Vinca Alkaloids Uses

cancer

Antifungals: Uses

candidiasis, oral thrush, histoplasmosis

pt w/ Crohn's disease takes infliximab (Remicade) how can the RN determine the effectiveness of treatment?

check frequency & consistency of bowel movements which inflammation in the colon will reduce thus reducing diarrhea

Ferrous sulfate (Feosol) is an iron supplement used to treat anemia what is the common SE?

constipation

Antihyperlipidemic: Side Effects

constipation, fat-soluble, vitamin deficiency

-"asone, -"solone"

corticosteroid suppress inflammation

Guaifenesin (Mucinex) needs to be monitored for _____ that lasts longer than 1 week or is accompanied by fever, rash, sore throat, persistent headache; notify MD

cough

Antitussive Agents Uses

coughs due to URI, COPD

Drugs with simalair actions, such as pennicillins/cephalosporins can result in

cross resistance

amyl nitrate, Na thiosulfate

cyanide antidote

A nurse is teaching a client who has diabetes mellitus and is taking hydrochlorothiazide 50 mg/day. The teaching should include the importance of monitoring which levels? a. Hemoglobin and hematocrit b. Blood urea nitrogen (BUN) c. Arterial blood gases d. Serum glucose (sugar)

d. Serum glucose (sugar)

Captopril (Capoten) has been ordered for a client. The nurse teaches the client that ACE inhibitors have which common side effects? a. Nausea and vomiting b. Dizziness and headaches c. Upset stomach d. Constant, irritating cough

d. Constant, irritating cough

When an elderly client with cancer experiences "breakthrough pain," the nurse should expect that pharmacological treatment will include: a. Initiation of a placebo after every third dose of narcotic. b. More aggressive chemotherapy. c. Giving narcotics every hour. d. Increasing the dose of the narcotic.

d. Increasing the dose of the narcotic.

The safest narcotic choice for an elderly client with acute pain is: a. Meperidine (Demerol). b. Oxycodone. c. Fentanyl transdermal patch. d. Morphine sulfate.

d. Morphine sulfate. Rationale: Morphine is the "gold standard" of narcotics for acute pain. The other choices are incorrect.

The nurse is assessing the client for possible evidence of digitalis toxicity. The nurse acknowledges that which is included in the signs and symptoms for digitalis toxicity? a. Pulse (heart) rate of 100 beats/min b. Pulse of 72 with an irregular rate c. Pulse greater than 60 beats/min and irregular rate d. Pulse below 60 beats/min and irregular rate

d. Pulse below 60 beats/min and irregular rate

The benefits of using an insulin pump include all of the following except: a. By continuously providing insulin they eliminate the need for injections of insulin b. They simplify management of blood sugar and often improve A1C c. They enable exercise without compensatory carbohydrate consumption d. They help with weight loss

d. They help with weight loss Using an insulin pump has many advantages, including fewer dramatic swings in blood glucose levels, increased flexibility about diet, and improved accuracy of insulin doses and delivery; however, the use of an insulin pump has been associated with weight gain.

The client informs the nurse that he has experienced pain in the lower extremities for the past eight months. The nurse recognizes that this pain is classified as: a.) Moderate. b.) Severe. c.) Acute. d.) Chronic.

d.) Chronic. Chronic pain persists longer than six months.

Anticonvulsants Actions

decreases flow of calcium and sodium across neuronal membranes

Antigout Agents: Actions

decreases production and resorption of uric acid

Carbonic Anhydrase Inhibitors Action

decreases production of aqueous humor in ciliary body

lithium toxicity

dehydration can lead to?

Antidepressants: Heterocyclics Uses

depression and smoking cessation

Antidepressants: MAOIs Uses

depression, chronic pain

Antidepressants: SSRIs Uses

depression, obsessive compulsive disorders, bulimia

Antidepressants: Tricyclics Uses

depression, sleep apnea

Antidiarrheals: Uses

diarrhea

common SE of Metformin is

diarrhea

loperamide hydrochloride (Imodium) is used to treat

diarrhea

Antibiotics: Macrolide Side Effects

diarrhea, confusion, hepatotoxicity, superinfections

Antibiotics: Fluoroquinolones Side Effects

diarrhea, decreased WBC and hematocrit, elevated liver enzymes (AST ALT), elevated alkaline phosphatase

Cardiac glycosides

digoxin Heart failure, AFib adverse reaction: Halos anorexia N/V Hold if HR <60 adult, <70 child, <90 infant

most important nursing intervention for Allopurinol (Zyloprim)

drink 3000 mL of fluid daily

Antihyperlipidemic: Uses

elevated cholesterol, reduce incidence of cardiovascular disease

If GI systems occur Oral erythromycin can be given w/meals but it is BEST given

empty stomach w/ full glass of water

important teaching of Allopuinol (Zyloprim) intake

encourage 3000 mL to prevent formations of crytsals in urine

Fludrocortisone acetate (Florinef Acetate) is given for Addison's disease; what is the primary action

enhances the REABSORPTION of sodium & chloride ions in the distal tubules of the kidney ** promotes water retention

Clonidine is an ANTI-hypertensive med that is applied to a hairless intact skin area of the upper arm or torso and changed?

every 7 days

Overdose (toxicity) of bethanechol chloride manifestations

excessive muscarinic stimulation : -salivation - sweating - involuntary urination - defecation - bradycardia - severe hypotension

Isotretinoin (Accutane) needs to be monitored for____ levels bc it can elevate _______ levels.

triglyceride

Norepinephrine (levofed)

given as last effort of saving life S- stimulation of alpa/beta receptors H- hypovolemia must be corrected prior to administration O- output, increases urinary output C- Constricts blood vessels K- Keep eye on patient (monitor every 5-15 min)

Immunosuppressants Side Effects

hepatotxicity, nephrotoxicity, leukonpenia, thrombovytopenia

what diet is prescribed to prevent constipation?

high fiber diet & stool softners & increase fluid intake

hypertensive crisis signs

hypertension occipital headache readiating frontanlly neck stiffness soreness n/v

Antihypertensives: Ace Inhibitors Uses

hypertension, CHF

Antihypertensives: Beta Blockers Uses

hypertension, angina, SVT

Pts taking a thiazide diuretic like hydrochlorothiazide are at risk for

hypokalemia hyperglycemia hypercalcemia hyperlipidemia hyperuricemia

Hypoglycemic Agent: Side Effects

hypotension, bronchospasms

signs of opiod withdrawl

increased temp increased BP abdominal cramping vomiting restlessness

signs of hypercalcemia

increased urine volume excessive thirst n/v constipation hypotonicity of muscles deep bone pain flank pain

Antidiarrheals: Action

increases GI motility, blocks effect of dopamine in chemoreceptor trigger zone

client taking lorazepam(ativan) asks how drug works, proper response would be:

increases action of Inhibitory NT Gamma-amniobutyric-Acid GABA to receptors

Cardiac Glycosides Action

increases force of myocardial contraction, slows rate

Antivirals Action

inhibits DNA and RNA relpications

Antineoplastic: Antimetabolites Actions

inhibits DNA polymerase

H2-Receptor Blockers Action

inhibits action of histamine and gastric acid secretions

Antitubercular Agents Action

inhibits cell and protein synthesis

Antihyperlipidemic: Actions

inhibits cholesterol and triglyceride synthesis

NPH INSULIN

intermediate acting insulin

Iron Preparations Uses

iron-deficiency anemia

Spironolactone (Aldactone) is what type of diuretic?

k sparing diuretic

Antineoplastic: Alkylating Agents Uses

leukemia, multiple myeloma

Antineoplastic: Hormonal Agents Side Effects

leukopenia, bone pain, hypercalcemia

Antithyroid Agents Side Effects

leukopenia, rash, thrombocytopenia

-"lukast"

leukotrenes anti-inflammatory

Niratetes

life threatening if taken with Viagra headache keep in dark container replace every 6 months may be used 5-10min before exercise

Dantrolene sodium (Dantrium) major adverse effect

liver damage; thus monitor liver function tests

-"mide"

loop diuretic High K diet

after investigating Amoxicillin for 10 days the Cl develops diarhea which inclded eight watery stools per day- Rn should anticipate the need for which following

monitor clinical manifestations of metabolic acidosis collect stool specimen for cytotoxin assay to detect C-diff

Ticlopidine (Ticlid) is an antiplatelet med given to prevent thrombotic stroke what is the major adverse effect of Ticlopidine (Ticlid)?

neutropenia

Cholestyramine (Questran) & ________ (Niacin) need to be avoided

nicotinic acid

Anti Impotence Contraindications

nitrates, alpha blockers

Sulfonlureas

promote insulin secretion by the pancreas and may increase tissue response to insulin

Anticoagulant Coumadin Uses

pulmonary embolism, venous thrombosis, prophylaxis after acute MI

Anticoagulant Heparin Uses

pulmonary embolism, venous thrombosis, prophylaxis after acute MI

lispro (Humalog)

rapid acting insulin onset <15 min.

Tyramine

rich in wines and cheese, yogart, beef or chicken liver; these foods are contraindicated in patients on MAO inhibitors

Thrombolytic therapy is contraindicated in what conditions

risk of uncontrolled bleeding severe uncontrolled HTN

Fomepizole (Antizol) given for treatment of known or suspected ethylene glycol (antifreeze) intoxication; The RN recieves the med and it has solidified, what to do?

run the vial under warm water

Postoperative pt requests sleeping pill, which of the following would be used for pt that is having difficulty falling asleep, and nonpharmacologic alt have not been effective?

secoarbital- short term drug used for insomnia- causes pt to awake early.

Antidepressants: Tricyclics Side Effects

sedation, anticholinergic effect, confusion, postural hypotension, urinary retention

Bupropion (Wellbutrin) causes toxicity when levels are greater than 450 mg/day; what do you need to look for when looking for toxicity

seizure activity

early signs of blood disorders

sore throat fever pallor

most important teaching of tetracycline intake

stains teeth; use straw

Order states Give Multi V ii caps Po OD example of what category of order

standing order

Antibiotics: Penicillins Side Effect

stomatitis, diarrhea, allergic reactions, renal and hepatic changes

ACE-I

stop diuretic 24-48 hrs prior to taking ACE-I monitor K levels A- angioedema (treat with epinephrine) C- dry persistent cough E- electrolyte imbalance

Antidiabetic Agents: Nursing Considerations

take before breakfast Monitor glucose levels avoid alcohol

-"ase" -"plase"

thrombolytic dissolve clots

antidote for aminocaproic acid

thrombolytic therapy

Megestrol acetate (Megace), an antineoplastic med is used w/ caution in pts w/

thrombophlebitis

-"thy"

thyroid hormones take in the AM

When adminsitering ear drops, the CL should be sitting with the head tilted towards which side

unaffected side

Tardive dyskinesia signs

uncontrollable involuntar movements of the body & extremities, particulary the tongue

Hyperuricemia can produce

uric acid nephropathy renal stones acute renal failure

Busulfan is given to a pt w/ acute myelocytic leukemia; what needs to be monitored?

uric acid level

adverse result of Desmopressin acetate

water intoxication aka overhydration hyponatremia

Desmopressin is an ANTI-diuretic hormone used in treatment of diabetes insipidus; it causes

water reabsorption


Ensembles d'études connexes

ITIL Service Design Availability Management

View Set

CIE A Level: Control and Coordination

View Set

Ch. 13 Project Planning and Scheduling

View Set

PrepU: Chapter 19-Lung Assessment

View Set

unit 4: frankenstein + romanticism

View Set

types of federalism/ alternatives to federalism

View Set